actuarialbrew.com exam mfe / 3f actuarial models...

464
ActuarialBrew.com Exam MFE / 3F Actuarial Models Financial Economics Segment Solutions 2014, 1 st edition www.ActuarialBrew.com Brewing Better Actuarial Exam Preparation Materials © ActuarialBrew.com 2014

Upload: dinhtuong

Post on 02-Sep-2018

266 views

Category:

Documents


13 download

TRANSCRIPT

Page 1: ActuarialBrew.com Exam MFE / 3F Actuarial Models …actuarialbrew.com/data/documents/MFE-Solutions-2014-1st-ed.pdf · Exam MFE / 3F Actuarial Models Financial Economics Segment Solutions

ActuarialBrew.com

Exam MFE / 3F

Actuarial Models Financial Economics Segment

Solutions 2014, 1st edition

www.ActuarialBrew.com

Brewing Better Actuarial Exam Preparation Materials

© ActuarialBrew.com 2014

Page 2: ActuarialBrew.com Exam MFE / 3F Actuarial Models …actuarialbrew.com/data/documents/MFE-Solutions-2014-1st-ed.pdf · Exam MFE / 3F Actuarial Models Financial Economics Segment Solutions

Please visit our website, www.actuarialbrew.com, for a description of our exam preparation materials.

We also offer: Live Seminars Online Seminar Study Manual

Questions Set of 4 Practice Exams

Flashcards

Our products can be ordered from Actex or the Actuarial Bookstore. Links to order our products can be found on our website, www.actuarialbrew.com.

Please email [email protected] with any content-related questions.

For more general questions about any of our products, please email [email protected].

Page 3: ActuarialBrew.com Exam MFE / 3F Actuarial Models …actuarialbrew.com/data/documents/MFE-Solutions-2014-1st-ed.pdf · Exam MFE / 3F Actuarial Models Financial Economics Segment Solutions

Exam MFE/3F Solutions Overview

© ActuarialBrew.com 2014 Page O-1

Solutions to the Exam MFE/3F Questions

Overview

Solutions to Questions

The Solutions to ActuarialBrew.com’s MFE/3F Questions provide full solutions to the approximately 650 exam-style practice questions contained in the Questions.

If you have access to the online normal distribution calculator or a spreadsheet when working the Questions, the SOA advises students to use 5 decimal places for both the inputs and the outputs of the normal distribution calculator. If you don’t have access to either one, we provide the old printed Normal Distribution Table at the end of the Questions for your convenience. Please note that your results from using the printed table may be different due to rounding.

Practice Questions Each solution has the following key to indicate the question’s degree of difficulty. The more boxes that are filled in, the more difficult the question:

Easy:

Very Difficult:

Table of Contents The MFE/3F Questions cover the entire required course of reading for Exam MFE/3F: Chapter 01: Put-Call Parity and Replication Chapter 02: Comparing Options Chapter 03: Binomial Trees: Part I Chapter 04: Binomial Trees: Part II Chapter 05: Lognormally Distributed Prices Chapter 06: Histograms and Normal Probability Plots Chapter 07: The Black-Scholes Formula Chapter 08: The Greeks and Other Measures Chapter 09: Delta-Hedging Chapter 10: Exotic Options: Part I Chapter 11: Exotic Options: Part II Chapter 12: Monte Carlo Simulation Chapter 13: Volatility Chapter 14: Brownian Motion Chapter 15: The Sharpe Ratio and Itô’s Lemma Chapter 16: The Black-Scholes Equation Chapter 17: The Black Model for Options on Bonds Chapter 18: Binomial Short Rate Models Chapter 19: Continuous-Time Models of Interest Rates

Page 4: ActuarialBrew.com Exam MFE / 3F Actuarial Models …actuarialbrew.com/data/documents/MFE-Solutions-2014-1st-ed.pdf · Exam MFE / 3F Actuarial Models Financial Economics Segment Solutions

Exam MFE/3F Solutions Overview

© ActuarialBrew.com 2014 Page O-2

Errata The errata for the MFE/3F Study Manual and the Solutions to the MFE/3F Questions can be found on our website at www.ActuarialBrew.com. Please let us know about any errata you find by emailing us at [email protected].

Other Products Please visit our website for a description of our other exam preparation products: Live and Online Seminars Study Manual Questions Set of 4 Practice Exams Flashcards

Good Luck!

Page 5: ActuarialBrew.com Exam MFE / 3F Actuarial Models …actuarialbrew.com/data/documents/MFE-Solutions-2014-1st-ed.pdf · Exam MFE / 3F Actuarial Models Financial Economics Segment Solutions

Exam MFE/3F Solutions Chapter 1 – Put-Call Parity and Replication

© ActuarialBrew.com 2014 Page 1.01

Chapter 1 – Solutions

Solution 1.01

C Put-Call Parity

Since the underlying stock does not pay dividends, the value of a European call is equal to the value of the American call:

=

=( , ) ( , )

(50,0.25) 3.48Eur Amer

Eur

C K T C K T

C

We can use put-call parity to solve for the value of the European put option:

-

-

+ = +

+ = +=

00.08(0.25)

( , ) ( , )

3.48 50 50 (50,0.25)

(50,0.25) 2.49

rTEur Eur

Eur

Eur

C K T Ke S P K T

e P

P

Solution 1.02

B Put-Call Parity

The European call premium can be found using put-call parity:

-

- -

+ = - +

+ = - +=

0 0,

0.08(0.5) 0.08(0.25)

( , ) ( ) ( , )

(48,0.5) 48 50 5 4.38

(48,0.5) 3.36

rTEur T Eur

Eur

Eur

C K T Ke S PV Div P K T

C e e

C

Solution 1.03

B Put-Call Parity

We ignore the dividend occurring in 7 months, because it occurs after the call option expires.

The European call premium can be found using put-call parity:

-

- -

+ = - +

+ = - +=

0 0,

0.05(0.5) 0.05(4 /12)

( , ) ( ) ( , )

(80,0.5) 80 75 4 10.37

(80,0.5) 3.41

rTEur T Eur

Eur

Eur

C K T Ke S PV Div P K T

C e e

C

Solution 1.04

C Put-Call Parity

This question may appear tricky since the options are euro-denominated. There is no other currency involved though, so we can use the standard put-call parity formula.

Page 6: ActuarialBrew.com Exam MFE / 3F Actuarial Models …actuarialbrew.com/data/documents/MFE-Solutions-2014-1st-ed.pdf · Exam MFE / 3F Actuarial Models Financial Economics Segment Solutions

Exam MFE/3F Solutions Chapter 1 – Put-Call Parity and Replication

© ActuarialBrew.com 2014 Page 1.02

We can use put-call parity to solve for the risk-free rate of return:

d- -

- -

-

+ = +

+ = +

==

0(1) 0.04(1)

( , ) ( , )

7.53 80 75 10.07

0.932490.0699

rT TEur Eur

r

r

C K T Ke S e P K T

e e

er

Solution 1.05

D Put-Call Parity

The dividend occurs after the expiration of the options, so we can ignore it. Furthermore, when there are no dividends prior to the expiration of an American call option, the option has the same value as a European call option.

We can use put-call parity to solve for the value of the call option:

-

-

+ = +

+ = +=

00.07(0.5)

( , ) ( , )

( , ) 70 75 3.02

( , ) 10.428

rTEur Eur

Eur

Eur

C K T Ke S P K T

C K T e

C K T

Solution 1.06

A Put-Call Parity

Since the options are at-the-money, the strike price is equal to the stock price. We can use put-call parity to find the price of the stock:

-

-

- -

- -

-

-

+ = - +

= - + +

= - + +

= - + +

- += =-

0 0,

0 0,

0.07(0.75) 0.07(0.5)0

0.07(0.75) 0.07(0.5)0 0

0.07(0.5)

0 0.07(0.75)

( , ) ( ) ( , )

( , ) ( , ) ( )

1.34 3

1.34 3

1.34 330.44

1

rTEur T Eur

rTEur Eur T

C K T Ke S PV Div P K T

S C K T P K T Ke PV Div

S Ke e

S S e e

eS

e

Solution 1.07

D Synthetic Stock

We can rearrange put-call parity so that it is a guide for replicating the stock:

-= + + -0 0,( , ) ( ) ( , )rTEur T EurS C K T Ke PV Div P K T

Page 7: ActuarialBrew.com Exam MFE / 3F Actuarial Models …actuarialbrew.com/data/documents/MFE-Solutions-2014-1st-ed.pdf · Exam MFE / 3F Actuarial Models Financial Economics Segment Solutions

Exam MFE/3F Solutions Chapter 1 – Put-Call Parity and Replication

© ActuarialBrew.com 2014 Page 1.03

To replicate the stock, we must purchase the call, sell the put and lend the present value of the strike plus the present value of the dividends. The present value of the strike plus the present value of the dividends is:

- - -+ = + =0.09(0.75) 0.09(0.5)0, ( ) 28 3 29.04rT

TKe PV Div e e

Solution 1.08

D Synthetic T-bills

As an alternative to the method below, we could solve for r and then use it to find the present value of $1,000.

We can rearrange put-call parity so that it is a guide for replicating a T-bill:

d- -= - +0 ( , ) ( , )rT TEur EurKe S e C K T P K T

To create an asset that matures for the strike price of $50, we must purchase d- Te shares of the stock, sell a call option, and buy a put option. The cost of doing this is:

d- -

-

= - +

= - +=

00.07(0.75)

( , ) ( , )

52 6.56 3.6146.3904

rT TEur EurKe S e C K T P K T

e

Since it costs $46.3904 to create an asset that is guaranteed to mature for $50, it must cost 20 times as much to create an asset that is guaranteed to mature for $1,000:

¥ =20 46.3904 927.81

Solution 1.09

A Synthetic Stock

To answer this question, we don’t need to know the risk-free rate of return or the time until the options expire.

We can rearrange put-call parity so that it is a guide for replicating the stock:

-= + + -0 0,( , ) ( ) ( , )rTEur T EurS C K T Ke PV Div P K T

To replicate the stock, we must purchase the call, sell the put and lend the present value of the strike plus the present value of the dividends. We can use the equation above to find the present value of the strike plus the present value of the dividends:

-

-

= + + -

+ = - + =

0,

0,

52 6.01 ( ) 3.87

( ) 52 6.01 3.87 49.86

rTT

rTT

Ke PV Div

Ke PV Div

Solution 1.10

E Currency options

The current exchange rate is in the form of euros per dollar, which is the inverse of our usual form of dollars per euro.

Page 8: ActuarialBrew.com Exam MFE / 3F Actuarial Models …actuarialbrew.com/data/documents/MFE-Solutions-2014-1st-ed.pdf · Exam MFE / 3F Actuarial Models Financial Economics Segment Solutions

Exam MFE/3F Solutions Chapter 1 – Put-Call Parity and Replication

© ActuarialBrew.com 2014 Page 1.04

The exchange rate in dollars per euro is:

= =01

1.041670.96

x

Put-call parity for currency options can be used to find the value of the call option:

--

- -

+ = +

+ = +=

00.06(1) 0.04(1)

( , ) ( , )

(0.94,1) 0.94 1.04167 0.005

(0.94,1) 0.12056

fr TrTEur Eur

Eur

Eur

C K T Ke x e P K T

C e e

C

Solution 1.11

A Currency options

We treat the Swiss franc as the base currency for the first part of this question. But then at the end we must convert its value into dollars.

With the Swiss franc as the base currency, we have the following exchange rate in terms of francs per dollar:

= =01

1.250.80

x

Put-call parity for currency options can be used to find the value of the put option:

--

- -

+ = +

+ = +=

00.06(1) 0.04(1)

( , ) ( , )

0.127 1.15 1.25 (1.15,1)

(1.15,1) 0.00904

fr TrTEur Eur

Eur

Eur

C K T Ke x e P K T

e e P

P

Therefore the cost of the put option is 0.00904 Swiss francs. But the possible choices are all expressed in dollars, so we must convert the value to dollars at the current exchange rate. One Swiss franc costs $0.80, so 0.00904 Swiss francs must cost (in dollars):

¥ =0.00904 0.80 0.00723

Solution 1.12

C Currency options

We treat the Iraqi dinar as the base currency.

Put-call parity for currency options can be used to find interest rate on the dinar:

--

- -

-

+ = +

+ = +

==

00.5 0.08(0.5)

0.5

( , ) ( , )

13.37 198 200 1.27

198 180.057890.19

fr TrTEur Eur

r

r

C K T Ke x e P K T

e e

er

Page 9: ActuarialBrew.com Exam MFE / 3F Actuarial Models …actuarialbrew.com/data/documents/MFE-Solutions-2014-1st-ed.pdf · Exam MFE / 3F Actuarial Models Financial Economics Segment Solutions

Exam MFE/3F Solutions Chapter 1 – Put-Call Parity and Replication

© ActuarialBrew.com 2014 Page 1.05

Solution 1.13

A Options on Bonds

Here’s a quick refresher on compound interest. The annual effective interest rate is denoted by i:

r

n

n i

1 i e1v

1 i1 va

i

+ =

=+-

=

The price of the bond is:

-

--

= +

-= +-

= +=

0.10(15)0 15

0.10(15)0.10(15)

0.10

120 1,000

1120 1,000

1120(7.3867) 223.13021,109.5385

B a e

ee

e

Only one coupon occurs before the expiration of the options, and it occurs one year from now. Using put-call parity, we have:

-

- -

+ = - +

+ = - +=

0 0,

0.10(1.25) 0.10(1)

( , ) ( ) ( , )

150 1,000 1,109.5385 120 (1,000,1.25)

(1,000,1.25) 31.54

rTEur T Eur

Eur

Eur

C K T Ke B PV Coupons P K T

e e P

P

Solution 1.14

E Options on Bonds

The 6-month effective interest rate is:

- =0.08(0.5) 1 4.081%e

Page 10: ActuarialBrew.com Exam MFE / 3F Actuarial Models …actuarialbrew.com/data/documents/MFE-Solutions-2014-1st-ed.pdf · Exam MFE / 3F Actuarial Models Financial Economics Segment Solutions

Exam MFE/3F Solutions Chapter 1 – Put-Call Parity and Replication

© ActuarialBrew.com 2014 Page 1.06

The price of the bond one month after it is issued is:

( )

[ ]

0.08(12) 0.08(1 /12)0 24 4.081%

2411.0481 0.08(12) 0.08(1 /12)

0.04(24)0.08(12) 0.08(1 /12)

0.04

0.08(1 /12)

45 1,000

145 1,000

0.0481

145 1,000

1

45(15.1212) 382.8929

1,063.346

B e e

e e

ee e

e

e

-

-

--

È ˘= +Î ˚È ˘-Í ˙+Í ˙Í ˙Î ˚È ˘-= +Í ˙

-Í ˙Î ˚

= +

=

a

0.08(1 /12)01,070.4587

e¥=

Two coupons occur prior to the expiration of the option, the first of which occurs in 5 months and the second of which occurs in 11 months. Using put-call parity, we have:

-

- - -

+ = - +

+ = - - +=

0 0,

0.08(1) 0.08(5 /12) 0.08(11 /12)

( , ) ( ) ( , )

(950,1) 950 1,070.4587 45 45 25

(950,1) 133.16

rTEur T Eur

Eur

Eur

C K T Ke B PV Coupons P K T

C e e e

C

Solution 1.15

B Options on Bonds

The price of the bond is equal to $1,000. Since the bond price is equal to its par value, its yield must be equal to its coupon rate. Therefore the yield is 7%. Since 7% is the only interest rate provided in the problem, we use 7% as the risk-free interest rate. The 7% interest rate is compounded twice per year since coupons are paid semi-annually. Therefore, the semiannual effective interest rate is 3.5%.

Using put-call parity for bonds, we have:

( )

-

-

= - + -

= - -

=

0 0,

1.5

( ) ( , ) ( , )

351.035 1,000 58.43

1.035955.83

rTT Eur EurKe B PV Coupons P K T C K T

K

K

Solution 1.16

D Exchange Options

The first step is to pick one of the assets to be the underlying asset. You can choose either one. We chose Stock X below, so a call option costs $2.70. If we chose Stock Y to be the underlying asset, then the same option would be a put option.

Page 11: ActuarialBrew.com Exam MFE / 3F Actuarial Models …actuarialbrew.com/data/documents/MFE-Solutions-2014-1st-ed.pdf · Exam MFE / 3F Actuarial Models Financial Economics Segment Solutions

Exam MFE/3F Solutions Chapter 1 – Put-Call Parity and Replication

© ActuarialBrew.com 2014 Page 1.07

Let’s assume that Stock X is the underlying asset and Stock Y is the strike asset. In that case, the option to give up Stock Y for Stock X is a call option:

=( , ,0.5) 2.70Eur t tC X Y

We can now use put-call parity for exchange options:

- -

- = -

È ˘- = - -Î ˚=

, ,

0.06(2 /12) 0.03(0.5)

( , ,0.5) ( , ,0.5) ( ) ( )

2.70 ( , ,0.5) 50 3 51

( , ,0.5) 5.91

P PEur t t Eur t t t T t t T t

Eur t t

Eur t t

C X Y P X Y F X F Y

P X Y e e

P X Y

Solution 1.17

A Exchange Options

We didn’t need to know the risk-free rate of return. That was included in the question as a red herring.

The put option that allows its owner to give up Stock B in exchange for Stock A has Stock B as its underlying asset:

=( , ,1) 11.49Eur t tP B A

Using put-call parity, we can find the value of the call option having Stock B as its underlying asset:

-

- = -

- = -=

, ,

0.05(1)

( , ,1) ( , ,1) ( ) ( )

( , ,1) 11.49 67 70

( , ,1) 5.22

P PEur t t Eur t t t T t t T t

Eur t t

Eur t t

C B A P B A F B F A

C B A e

C B A

We can describe the call option as a put option by switching the underlying asset and the strike asset:

==

( , ,1) ( , ,1)

( , ,1) 5.22Eur t t Eur t t

Eur t t

C B A P A B

P A B

Therefore, the value of a put option giving its owner the right to give up a share of Stock A in exchange for a share of Stock B is $5.22.

Solution 1.18

D Exchange Options

Let’s choose Stock X to be the underlying asset. In that case, Option A is a put option and Option B is a call option. Using put call parity, we have:

- -

- = -

- = -

- = -=

, ,

, ,

0.04(5 /12) 0.04(5 /12)0

0

( , ,5 /12) ( , ,5 /12) ( ) ( )

Option B Option A ( ) ( )

7.76 40

47.89

P PEur t t Eur t t t T t t T t

P Pt T t t T t

C S Y P S Y F X F Y

F X F Y

e Y e

Y

Page 12: ActuarialBrew.com Exam MFE / 3F Actuarial Models …actuarialbrew.com/data/documents/MFE-Solutions-2014-1st-ed.pdf · Exam MFE / 3F Actuarial Models Financial Economics Segment Solutions

Exam MFE/3F Solutions Chapter 1 – Put-Call Parity and Replication

© ActuarialBrew.com 2014 Page 1.08

Solution 1.19

E Exchange Options

Let’s establish two portfolios. Portfolio X consists of 1 share of Stock A and 2 shares of Stock B. Portfolio Y consists of 1 share of Stock C and 1 share of Stock D.

The first call option described in the question has Portfolio X as its underlying asset:

=( , ,1) 10Eur t tC X Y

We can find the prepaid forward prices for both portfolios:

-

-

= + =

= + =

0.04(1),

0.02(1),

( ) 40 2(50) 138.4316

( ) 60 75 133.5149

Pt T t

Pt T t

F X e

F Y e

We can now use put-call parity to find the value of the corresponding put option:

- = -

- = -=

, ,( , ,1) ( , ,1) ( ) ( )

10 ( , ,1) 138.4316 133.5149

( , ,1) 5.0833

P PEur t t Eur t t t T t t T t

Eur t t

Eur t t

C X Y P X Y F X F Y

P X Y

P X Y

We can describe a put option as a call option by switching the underlying asset with the strike asset:

==

( , ,1) ( , ,1)

( , ,1) 5.0833Eur t t Eur t t

Eur t t

P X Y C Y X

C Y X

Therefore, the call option giving its owner the right to acquire Portfolio Y in exchange for Portfolio X has a value of $5.0833.

Solution 1.20

B Exchange Options

This question is easier if we assume that Stock X is the underlying asset.

Let’s assume that Stock X is the underlying asset. The American option is then an American call option. Since Stock X does not pay dividends, the American call option is not exercised prior to maturity. Therefore the American call option has the same value as a European call option:

=( , ,7 /12) 10.22Eur t tC X Y

We are asked to find the value of the corresponding European put option. We can make use of put-call parity:

-

- = -

- = -=

, ,

0.04(7 /12)

( , ,7 /12) ( , ,7 /12) ( ) ( )

10.22 ( , ,7 /12) 100 100

( , ,7 /12) 7.91

P PEur t t Eur t t t T t t T t

Eur t t

Eur t t

C X Y P X Y F X F Y

P X Y e

P X Y

Page 13: ActuarialBrew.com Exam MFE / 3F Actuarial Models …actuarialbrew.com/data/documents/MFE-Solutions-2014-1st-ed.pdf · Exam MFE / 3F Actuarial Models Financial Economics Segment Solutions

Exam MFE/3F Solutions Chapter 1 – Put-Call Parity and Replication

© ActuarialBrew.com 2014 Page 1.09

Solution 1.21

C Options on Currencies

We can use put-call parity to determine the spot exchange rate:

--

- -

+ = +

+ = +=

00.05(0.5) 0.035(0.5)

0

0

( , ) ( , )

0.047 0.89 0.011

0.920

fr TrTEur EurC K T Ke x e P K T

e x e

x

Solution 1.22

B Exchange Options

Neither the time to maturity nor the risk-free rate of return is needed to answer the question. They were provided as red herrings.

The prepaid forward price of Stock A is $22. The prepaid forward price of Stock B is $26. Stock A is the underlying asset, giving rise to the following generalized form of put-call parity:

- = -

- = -- = -

0 0 0 0 0, 0,

0 0 0 0

0 0 0 0

( , , ) ( , , ) ( ) ( )

( , , ) ( , , ) 22 26

( , , ) ( , , ) 4

P PEur Eur T T

Eur Eur

Eur Eur

C A B T P A B T F A F B

C A B T P A B T

C A B T P A B T

Therefore the put price exceeds the call price by $4. By inspection, only Choice B has a put price that is $4 greater than the call price.

Solution 1.23

C Exchange Options, Options on Currencies

To answer this question, let’s use the yen as the base currency. In that case, the U.S. Dollar and the Canadian dollars are assets with prices denominated in the base currency. The U.S. dollar is the underlying asset for the options, and the Canadian dollar is the strike asset:

- -

- - - = -

- = -- =

, ,

0.08(0.5) 0.07(0.5)

( , , ) ( , , ) ( ) ( )

( , ,0.5) ( , ,0.5) 120 105

( , ,0.5) ( , ,0.5) 13.91

P PEur t t Eur t t t T t T

Eur t t Eur t t

Eur t t Eur t t

C S Q T t P S Q T t F S F Q

C S Q P S Q e e

C S Q P S Q

Solution 1.24

A Put-Call Parity

Let each dividend amount be D. The first dividend occurs at the end of 2 months, and the second dividend occurs at the end of 5 months.

Page 14: ActuarialBrew.com Exam MFE / 3F Actuarial Models …actuarialbrew.com/data/documents/MFE-Solutions-2014-1st-ed.pdf · Exam MFE / 3F Actuarial Models Financial Economics Segment Solutions

Exam MFE/3F Solutions Chapter 1 – Put-Call Parity and Replication

© ActuarialBrew.com 2014 Page 1.10

We can use put-call parity to find D:

0 0,

0.05(0.5) 0.05(2 /12) 0.05(5 /12)

0.05(2 /12) 0.05(5 /12) 0.05(0.5)

( , ) ( ) ( , )

2.55 42 40 4.36

[ ] 40 4.36 2.55 421.97108 0.84698

0.4297

rTEur T EurC K T Ke S PV Div P K T

e De De

D e e eD

D

Solution 1.25

D Early Exercise

For each put option, the choice is between having the exercise value now or having a 1-year European put option. Therefore, the decision depends on whether the exercise value is greater than the value of the European put option. The value of each European put option is found using put-call parity:

d

d

- -

- -

+ = +

= + -0

0

( , ) ( , )

( , ) ( , )

rT TEur Eur

rT TEur Eur

C K T Ke S e P K T

P K T C K T Ke S e

The values of each of the 1-year European put options are:

- -

- -

- -

- -

= + - =

= + - =

= + - =

= + - =

0.05(1) 0.09(1)

0.05(1) 0.09(1)

0.05(1) 0.09(1)

0.05(1) 0.09(1)

(25,1) 21.93 25 50 0.01

(50,1) 3.76 50 50 5.62

(75,1) 0.21 75 50 25.86

(100,1) 0.01 100 50 49.44

Eur

Eur

Eur

Eur

P e e

P e e

P e e

P e e

In the third and fourth columns of the table below, we compare the exercise value with the value of the European put options. The exercise value is - 0( ,0)Max K S .

Exercise European K C Value Put

$25.00 $21.93 0 0.01 $50.00 $3.76 0 5.62 $75.00 $0.21 25 25.86 $100.00 $0.01 50 49.44

The exercise value is less than the value of the European put option when the strike price is $75 or less. When the strike price is $100, the exercise value is greater than the value of the European put option. Therefore, it is optimal to exercise the special put option with an exercise price of $100.

Page 15: ActuarialBrew.com Exam MFE / 3F Actuarial Models …actuarialbrew.com/data/documents/MFE-Solutions-2014-1st-ed.pdf · Exam MFE / 3F Actuarial Models Financial Economics Segment Solutions

Exam MFE/3F Solutions Chapter 1 – Put-Call Parity and Replication

© ActuarialBrew.com 2014 Page 1.11

Solution 1.26

B Exchange Options

The first page of the study note, ”Some Remarks on Derivatives Markets,” tells us that “for each share of the stock the amount of dividends paid between time t and time t dt+ is assumed to be S(t ) dtd .” Therefore, the continuously compounded dividend rate for Stock 1 is 7%, and the continuously compounded dividend rate for Stock 2 is 3%.

The claim has the following payoff at time 4:

[ ]1 2(4), (4)Max S S

A portfolio consisting of a share of Stock 2 and the option to exchange Stock 2 for Stock 1 effectively gives its owner the maximum value of the two stocks. If the value of Stock 2 is greater than the value of Stock 1 at time 4, then the owner keeps Stock 2 and allows the exchange option to expire unexercised. If the value of Stock 1 is greater than the value of Stock 2, then the owner exercises the option, giving up Stock 2 for Stock 1.

Since Stock 2 has a continuously compounded dividend rate of 3%, the cost now of a share of Stock 2 at time 4 is:

d-

-

=

= =

00,

0.03(4)0,4 2

( )

( ) 85 75.39

P TT

P

F S e S

F S e

The cost of an exchange option allowing its owner to exchange Stock 2 for Stock 1 at time 4 is $43.

Adding the costs together, we obtain the cost of the claim:

+ =75.39 43 118.39

Solution 1.27

E Reverse Conversion

A T-bill with a current value of ( )0.75 0.5055 r re Xe- -+ can be replicated with a conversion

by purchasing a share of stock, selling a call option, and purchasing a put option:

-

- -

+ = - +

+ = - +=

0, 0

0.75 0.50

( ) ( , ) ( , )

55 50 2.38 8.9856.60

rTT Eur Eur

r r

Ke PV Div S C K T P K T

e Xe

Doing the opposite creates a reverse conversion. Selling a share of stock, buying a call option, and selling a put option is a reverse conversion and provides us with $56.60 now

and an obligation to pay ( )+ 0.2555 rXe in 9 months.

Page 16: ActuarialBrew.com Exam MFE / 3F Actuarial Models …actuarialbrew.com/data/documents/MFE-Solutions-2014-1st-ed.pdf · Exam MFE / 3F Actuarial Models Financial Economics Segment Solutions

Exam MFE/3F Solutions Chapter 1 – Put-Call Parity and Replication

© ActuarialBrew.com 2014 Page 1.12

To borrow $1,000 now, we must sell:

1,000

17.6756.60

= shares of stock.

Solution 1.28

E Minimum of 2 Assets

The European option has a payoff of:

1 217 (1), (1) ,0Max Min S S

We recognize the European option as a put option on the minimum of the two stocks. Let’s call the underlying asset X:

[ ]= 1 2(1) (1), (1)X Min S S

The price of the option is $0.67:

=( ,17,1) 0.67EurP X

We are told that a claim that pays the minimum of (1)X and 17 has a current value of $15.50:

[ ]( ) [ ]( )= =0,1 1 2 0,1(1), (1), 17 (1), 17 15.50P PF Min S S F Min X

We can express the value of the claim as the value of the strike asset minus the value of the put option:

( )( ) -

-

-

= - -

= -

= -

==

, ,

0,1

[ , ] ( ) ( , , )

[ (1),17] 17 0.67

15.50 17 0.67

16.17 170.05

P PT T Eur t tt T t T

P r

r

r

F Min X Q F Q P X Q T t

F Min X e

e

er

Solution 1.29

A Maximum of 2 Assets

The European option has a payoff of:

1 2(1), (1) 22,0Max Max S S

We recognize the European option as a call option on the maximum of the two stocks. Let’s call the underlying asset X:

[ ]= 1 2(1) (1), (1)X Max S S

Page 17: ActuarialBrew.com Exam MFE / 3F Actuarial Models …actuarialbrew.com/data/documents/MFE-Solutions-2014-1st-ed.pdf · Exam MFE / 3F Actuarial Models Financial Economics Segment Solutions

Exam MFE/3F Solutions Chapter 1 – Put-Call Parity and Replication

© ActuarialBrew.com 2014 Page 1.13

The price of the option is $2.83:

=( ,22,1) 2.83EurC X

We are told that a claim that pays the maximum of (1)X and 22 has a current value of $23.34:

[ ]( ) [ ]( )= =0,1 1 2 0,1(1), (1), 22 (1), 22 23.34P PF Max S S F Max X

We can express the value of the claim as the value of the call option plus the value of the strike asset:

( )( ) -

-

-

= - +

= +

= +

==

, ,

0,1 1

[ , ] ( , , ) ( )

[ ,22] 2.83 22

23.34 2.83 22

20.51 220.0701

P PT T Eur t tt T t T

P r

r

r

F Max X Q C X Q T t F Q

F Max X e

e

er

Solution 1.30

D Maximum of 2 Assets

The European option has a payoff of:

1 2(3), (3) 25,0Max Max S S

We recognize the European option as a call option on the maximum of the two stocks. Let’s call the underlying asset X:

[ ]= 1 2(3) (3), (3)X Max S S

We are told that a claim that pays the maximum of (3)X and 25 has a current value of $25.50:

[ ]( ) [ ]( )= =0,3 1 2 0,3(3), (3), 25 (3), 25 25.50P PF Max S S F Max X

We can express the value of the claim as the value of the call option on X plus the value of the strike asset:

( )( ) - ¥ - ¥

- ¥

- ¥

= - +

= +

= +

=

, ,

3 0.08 3 0.080,3 3 3

3 0.083

3 0.083

[ , ] ( , , ) ( )

[ ,25] ( ,25 ,3) 25

25.50 ( ,25 ,3) 19.6657

( ,25 ,3) 5.8343

P PT T Eur t tt T t T

PEur

Eur

Eur

F Max X Q C X Q T t F Q

F Max X C X e e

C X e

C X e

Page 18: ActuarialBrew.com Exam MFE / 3F Actuarial Models …actuarialbrew.com/data/documents/MFE-Solutions-2014-1st-ed.pdf · Exam MFE / 3F Actuarial Models Financial Economics Segment Solutions

Exam MFE/3F Solutions Chapter 1 – Put-Call Parity and Replication

© ActuarialBrew.com 2014 Page 1.14

Solution 1.31

B Maximum of 2 Assets

The European option has a payoff of:

25 2 (5),3 (5) ,0Max Min X Y

We recognize the European option as a put option on the minimum of the two stocks. Let’s call the underlying asset Z:

( )=(5) 2 (5),3 (5)Z Min X Y

The rainbow option pays the minimum of (5)Z and 25, and it has a current value of $12.67:

[ ]( ) [ ]( )= =0,5 0,52 (5), 3Y(5), 25 (5), 25 12.67P PF Min X F Min Z

We can express the value of the claim as the value of the prepaid forward price of the strike asset minus the value of the European put option:

( )( ) - ¥ - ¥

- ¥

- ¥

= - -

= -

= -

=

, ,

5 0.05 5 0.050,5

5 0.05

5 0.05

[ , ] ( ) ( , , )

[ (5),25] 25 ( ,25 ,5)

12.67 19.4700 ( ,25 ,5)

( ,25 ,5) 6.80

P PT T Eur t tt T t T

PEur t

Eur t

Eur t

F Min Z Q F Q P Z Q T t

F Min Z e P Z e

P Z e

P Z e

Solution 1.32

B Put-Call Parity

We can rewrite the payoff of the first option with parentheses in the first Max funtion:

[ ] [ ]- - - -0, (1) ( 1) 0, (1)Max S K Max S K

We can now see that the special option consists of a long position in a call with a strike price of -( 1)K and a short position in a call with a strike price of K:

= - -ˆ( ) ( 1) ( )C K C K C K

Likewise, the second option consists of a long position in a put with a strike price of K and a short position in a put with a strike price of -( 1)K :

= - -ˆ ( ) ( ) ( 1)P K P K P K

Page 19: ActuarialBrew.com Exam MFE / 3F Actuarial Models …actuarialbrew.com/data/documents/MFE-Solutions-2014-1st-ed.pdf · Exam MFE / 3F Actuarial Models Financial Economics Segment Solutions

Exam MFE/3F Solutions Chapter 1 – Put-Call Parity and Replication

© ActuarialBrew.com 2014 Page 1.15

We can add the value of the two special options together and use put-call parity to show that the value of the sum does not depend on K:

[ ] [ ]

0.22

ˆ ˆ( ) ( ) ( 1) ( ) ( ) ( 1)

( 1) ( 1) ( ) ( )

(0) ( 1) (0)

1 0.8025

r r

r

C K P K C K C K P K P K

C K P K P K C K

S K e Ke S

e e

- -

- -

+ = - - + - -= - - - + -

È ˘ È ˘= - - + -Î ˚ Î ˚

= = =

We can now find ˆ (109)P :

+ =

+ =

+ =

= - =

ˆ ˆ( ) ( ) 0.8025ˆ ˆ(109) (109) 0.8025

ˆ0.3 (109) 0.8025ˆ (109) 0.8025 0.3 0.5025

C K P K

C P

P

P

Solution 1.33

A Put-Call Parity

To answer this question, we use put-call parity:

-- = - 20,2( ) ( ) ( )P rC K P K F S Ke

Let’s subtract the value of Set 2 from the value of Set 1 and use put-call parity to simplify:

[ ] [ ][ ] [ ]

- -

-

- - - = - -

- + - =

È ˘ È ˘- + - =Î ˚ Î ˚

- =

2 20,2 0,2

20,2

(48) (54 (48) (54) 11.91 ( 10.53)

(48) (48) (54) (54) 22.44

( ) 48 ( ) 54 22.44

2 ( ) 102 22.44

P r P r

P r

C P P C

C P C P

F S e F S e

F S e

Subtracting Set 4 from Set 3 and again using put-call parity to simplify, we obtain:

[ ] [ ] [ ][ ] [ ] [ ]

[ ][ ]

[ ]

- -

-

- - - = - -

- + - = - -

È ˘ È ˘- + - = - -Î ˚ Î ˚

- = - -

= - -

2 20,2 0,2

20,2

(55) (47 (55) (47) 10.30 (55) (47)

(55) (55) (47) (47) 10.30 (55) (47)

( ) 55 ( ) 47 10.30 (55) (47)

2 ( ) 102 10.30 (55) (47)

22.44 10.30 (55) (47)

(55

P r P r

P r

C P P C P C

C P C P P C

F S e F S e P C

F S e P C

P C

P[ ]- = -) (47) 12.14C

Page 20: ActuarialBrew.com Exam MFE / 3F Actuarial Models …actuarialbrew.com/data/documents/MFE-Solutions-2014-1st-ed.pdf · Exam MFE / 3F Actuarial Models Financial Economics Segment Solutions

Exam MFE/3F Solutions Chapter 1 – Put-Call Parity and Replication

© ActuarialBrew.com 2014 Page 1.16

Solution 1.34

A Currency Options

We didn’t need to know the dollar-denominated or the euro-denominated rates of return. They were included in the question as red herrings.

The first option allows its owner to give up $1.25 in order to get €1.00.

The second option allows its owner to give up $1.00 in order to get €0.80.

Therefore, the value of the second option is 0.80 times the value of the first option:

¥ =0.80 $0.083 $0.0664

The value of the second option in euros is:

¥ =€1$0.0664 €0.0553

$1.20

Solution 1.35

E Currency Options

The first option allows its owner to give up €0.80 in order to get $1.00.

The second option allows its owner to give up €1.00 in order to get $1.25.

Therefore, the value of the second option is 1.25 times the value of the first option:

¥ =1.25 €0.0898 €0.11225

The value of the second option in dollars is:

¥ =$1€0.11225 $0.1321

€0.85

Solution 1.36

B Currency Options

The first option allows its owner to give up €1.00 in order to get 1.60 francs.

The second option allows its owner to give up €0.625 in order to get 1.00 francs.

Therefore, the value of the second option is 0.625 times the value of the first option:

¥ =0.625 0.0918 0.057375francs francs

The value of the second option in euros is:

¥ =€10.057375 €0.0348

1.65francs

francs

Page 21: ActuarialBrew.com Exam MFE / 3F Actuarial Models …actuarialbrew.com/data/documents/MFE-Solutions-2014-1st-ed.pdf · Exam MFE / 3F Actuarial Models Financial Economics Segment Solutions

Exam MFE/3F Solutions Chapter 1 – Put-Call Parity and Replication

© ActuarialBrew.com 2014 Page 1.17

Solution 1.37

D Currency Options

The pound-denominated put allows its owner to give up $1.00 in order to get £0.40.

The dollar-denominated put allows its owner to give up £1.00 in order to get $2.50.

We can find the value of a dollar-denominated call that allows its owner to give up $2.50 in order to get £1.00. The value of this call is 2.5 times the value of the pound-denominated put. Below, the value of this call is expressed in dollars:

Ê ˆ = ¥ ¥ =Á ˜Ë ¯$1 $1

,2.50,1 2.50 £0.0133 $0.079170.42 £0.42

C

We can use put-call parity to find the value of the dollar-denominated put:

- -

- -

Ê ˆ Ê ˆ+ = +Á ˜ Á ˜Ë ¯ Ë ¯

Ê ˆ+ = + Á ˜Ë ¯

Ê ˆ =Á ˜Ë ¯

0.07 0.08$ $

0.07 0.08$

$

1 1 1,2.50,1 2.50 ,2.50,1

0.42 0.42 0.42

1 10.07917 2.50 ,2.50,1

0.42 0.42

1,2.50,1 0.2123

0.42

C e e P

e e P

P

Solution 1.38

B Currency Options

The euro-denominated call allows its owner to give up €0.625 in order to get $1.00.

The dollar-denominated call allows its owner to give up $1.60 in order to get €1.00.

We can find the value of a dollar-denominated put that allows its owner to give up €1.00 in order to get $1.60. The value of this put is 1.6 times the value of the euro-denominated call. Below, the value of this put is expressed in dollars:

Ê ˆ = ¥ ¥ =Á ˜Ë ¯$1 $1

,1.60,1 1.60 €0.08 $0.1828570.7 €0.7

P

We can use put-call parity to find the value of the dollar-denominated call:

- ¥ - ¥

- -

Ê ˆ Ê ˆ+ = +Á ˜ Á ˜Ë ¯ Ë ¯

Ê ˆ + = +Á ˜Ë ¯

Ê ˆ =Á ˜Ë ¯

0.07 0.5 0.08 0.5$ $

0.035 0.04$

$

1 1 1,1.60,0.5 1.60 ,1.60,0.5

0.7 0.7 0.7

1 1,1.60,0.5 1.60 0.182857

0.7 0.7

1,1.60,0.5 0.0104448

0.7

C e e P

C e e

C

The value of the Canadian dollar-denominated call is 0.0104448 Canadian dollars. The answer choices are expressed in euros though, so we must convert this amount into euros:

¥ =€0.7$0.0104448 €0.0073

$1

Page 22: ActuarialBrew.com Exam MFE / 3F Actuarial Models …actuarialbrew.com/data/documents/MFE-Solutions-2014-1st-ed.pdf · Exam MFE / 3F Actuarial Models Financial Economics Segment Solutions

Exam MFE/3F Solutions Chapter 1 – Put-Call Parity and Replication

© ActuarialBrew.com 2014 Page 1.18

The value of the Canadian dollar-denominated call is 0.0073 euros.

Solution 1.39

B Currency Options

The put option gives its owner the right to give up €1 in order to get ¥100.

The call option gives its owner the right to give up €0.01 in order to get ¥1.

Therefore, the value of the call option is 0.01 times the value of the put option:

Ê ˆ = ¥ =Á ˜Ë ¯€1

,0.01, 0.01 ¥1.58 ¥0.0158110

C T

We can convert this value from yen into euros:

€1 €1

,0.01, ¥0.0158 €0.000144110 ¥110

C TÊ ˆ = ¥ =Á ˜Ë ¯

Solution 1.40

C Currency Options

The exchange rate can be expressed in two equivalent ways:

1

$0.008 ¥125 or

¥ $1

The dollar-denominated call allows its owner to give up $0.008 in order to get ¥1.00.

The yen-denominated call allows its owner to give up ¥125.00 in order to get $1.00.

We can find the value of a yen-denominated put that allows is owner to give up $1.00 in order to get ¥125.00. The value of this put is 125 time the value of the dollar-denominated call. Below, the value of this put is expressed in yen:

( ) = ¥ ¥ =¥¥125

125,125,1 125 $0.00054 ¥8.4375$1

P

We can use put-call parity to find the value of the yen-denominated call:

( ) ( )( )( )

- -

- -

+ = +

+ = +

=

0.01 0.06¥ ¥

0.01 0.06¥

¥

125,125,1 125 125 125,125,1

125,125,1 125 125 8.4375

125,125,1 2.4018

C e e P

C e e

C

Page 23: ActuarialBrew.com Exam MFE / 3F Actuarial Models …actuarialbrew.com/data/documents/MFE-Solutions-2014-1st-ed.pdf · Exam MFE / 3F Actuarial Models Financial Economics Segment Solutions

Exam MFE/3F Solutions Chapter 2 – Comparing Options

© ActuarialBrew.com 2014 Page 2.01

Chapter 2 – Solutions

Solution 2.01

E Bounds on Option Prices

This question was written in terms of euros to make it look more confusing, but since there is no other currency involved, we don’t have to worry about exchange rates.

Unless arbitrage is available, it must be the case that:

- -È ˘- £ -Î ˚( )

,0, ( ) ( , , )P r T tt T t Eur tMax F S Ke C S K T t

In fact, the inequality above is not satisfied since:

- -

-

- > -

- >>

( ),

0.19(0.5)

( ) ( , , )

200 198 1819.944 18

P r T tt T t Eur tF S Ke C S K T t

e

The call option has too low of a price relative to the forward prices of the underlying asset and the strike asset. Therefore, we can earn arbitrage profits by buying the right side of the inequality and selling the left side of the inequality.

Time 0.5

Transaction Time 0 £0.5 198S >0.5 198S

Buy call –18 0 -0.5 198S

Sell stock 200 - 0.5S - 0.5S

Lend the present value of the strike -- 0.19(0.5)198e 198 198

Total 1.944 - 0.5198 S 0

The ending price of the stock is €199, so the cash flow at time 0 is 1.94, while the cash flow at time 0.5 is 0. The accumulated value of the arbitrage profits is:

=0.19(0.5)1.944 2.14e

Solution 2.02

C Comparing Options With Different Strikes and Maturities

The solution below is lengthy, making this problem look harder than it really is. The solution is thorough to assist those that might have trouble with this question.

The strike price grows at the interest rate:

-= fi ≥0.17(0.25) ( )206.60 198 r T tT te K K e

Page 24: ActuarialBrew.com Exam MFE / 3F Actuarial Models …actuarialbrew.com/data/documents/MFE-Solutions-2014-1st-ed.pdf · Exam MFE / 3F Actuarial Models Financial Economics Segment Solutions

Exam MFE/3F Solutions Chapter 2 – Comparing Options

© ActuarialBrew.com 2014 Page 2.02

Therefore, to preclude arbitrage it should be the case that:

≥(0.75) (0.5)P P

But the longer option costs less than the shorter option, indicating that arbitrage is possible:

<3.35 4.03

The arbitrageur buys the longer option for $3.35 and sells the shorter option for $4.03. The difference of $0.68 is lent at the risk-free rate of return.

The 6-month option

After 6 months, the stock price is $196. Therefore, the shorter option is exercised against the arbitrageur. The arbitrageur borrows $198 and uses it to buy the stock.

As a result, at the end of 9 months the arbitrageur owns the stock and must repay the borrowed funds. This position results in the following cash flow at the end of 9 months:

- = - = - = -0.17(0.25)0.75 0.75198 206.60 205 206.60 1.60S e S

The 9-month option

The stock price at the end of 9 months is less than $206.60, so the 9-month put option has the following cash flow at the end of 9 months:

- = - =0.75206.60 206.60 205 1.60S

Payoff table

We don’t necessarily recommend constructing the entire payoff table during the exam, but we’ve included it below to assist you in understanding this problem.

Below is the payoff table for the strategy.

Time 0.75

<0.75 206.60S ≥0.75 206.60S

Transaction Time 0 <0.5 198S ≥0.5 198S <0.5 198S ≥0.5 198S

Sell (0.5)P 4.03 -0.75 206.60S 0 -0.75 206.60S 0

Buy (0.75)P –3.35 - 0.75206.60 S - 0.75206.60 S 0 0

Total 0.68 0 - 0.75206.60 S -0.75 206.60S 0

The cash flow of the two options nets to zero after 9 months, since the stock price path makes the first column applicable.

The accumulated value of the arbitrage strategy is the difference between the cost of the two options, accumulated for 9 months:

- =0.17(0.75)(4.03 3.35) 0.7725e

Page 25: ActuarialBrew.com Exam MFE / 3F Actuarial Models …actuarialbrew.com/data/documents/MFE-Solutions-2014-1st-ed.pdf · Exam MFE / 3F Actuarial Models Financial Economics Segment Solutions

Exam MFE/3F Solutions Chapter 2 – Comparing Options

© ActuarialBrew.com 2014 Page 2.03

Solution 2.03

E Comparing Options With Different Strikes and Maturities

The solution below is lengthy, making this problem look harder than it really is. The solution is thorough to assist those that might have trouble with this question.

The strike price grows at the interest rate:

-= fi £0.19(0.25) ( )207.63 198 r T tT te K K e

Therefore, to preclude arbitrage it should be the case that:

≥(0.75) (0.5)C C

But the longer option costs less than the shorter option, indicating that arbitrage is possible:

<22.00 23.50

The arbitrageur buys the longer option for $22.00 and sells the shorter option for $23.50. The difference of $1.50 is lent at the risk-free rate of return.

The 6-month option

After 6 months, the stock price is $278. Therefore, the shorter option is exercised against the arbitrageur. The arbitrageur borrows a share of stock and sells it for $198.

As a result, at the end of 9 months the arbitrageur owes the stock and has the accumulated value of the $198. This position results in the following cash flow at the end of 9 months:

- + = - + = - + =0.19(0.25)0.75 0.75198 207.63 205 207.63 2.63S e S

The 9-month option

The stock price at the end of 9 months is less than $207.63, so the 9-month call option expires worthless.

Payoff table

We don’t necessarily recommend constructing the entire payoff table during the exam, but we’ve included it below to assist you in understanding this problem.

Below is the payoff table for the strategy.

Page 26: ActuarialBrew.com Exam MFE / 3F Actuarial Models …actuarialbrew.com/data/documents/MFE-Solutions-2014-1st-ed.pdf · Exam MFE / 3F Actuarial Models Financial Economics Segment Solutions

Exam MFE/3F Solutions Chapter 2 – Comparing Options

© ActuarialBrew.com 2014 Page 2.04

Time 0.75

<0.75 207.63S ≥0.75 207.63S

Transaction Time 0 <0.5 198S ≥0.5 198S <0.5 198S ≥0.5 198S

Sell (0.5)C 23.50 0 - +0.75 207.63S 0 - +0.75 207.63S

Buy (0.75)C –22.00 0 0 -0.75 207.63S -0.75 207.63S

Total 1.50 0 - +0.75 207.63S -0.75 207.63S 0

After 9 months, the net cash flow from the two options is:

- + = - + =0.75 207.63 205 207.63 2.63S

Adding this to the accumulated value of the $1.50 obtained at the outset, we have the accumulated arbitrage profits at the end of 9 months:

+ =0.19(0.75)1.50 2.63 4.36e

Solution 2.04

C Comparing Options With Different Strikes and Maturities

The option has the total return portfolio as its underlying asset, so it should be the case that:

≥( , ,1.5) ( , ,1.0)t t t tP Port K P Port K

But the longer option costs less than the shorter option, indicating that arbitrage is possible:

<7.00 7.85

The arbitrageur buys the longer option for $7.00 and sells the shorter option for $7.85. The difference of $0.85 is lent at the risk-free rate of return.

The 1-year option

After 1 year, the stock price is $60. This means that the total return portfolio has a value of:

= =0.03(1)1.0 60 61.827Port e

The strike price is:

= =0.06(1)1 61.2147 65.000K e

Therefore, the 1-year put option is exercised against the arbitrageur. The arbitrageur borrows $65 and uses it to buy the total return portfolio.

Page 27: ActuarialBrew.com Exam MFE / 3F Actuarial Models …actuarialbrew.com/data/documents/MFE-Solutions-2014-1st-ed.pdf · Exam MFE / 3F Actuarial Models Financial Economics Segment Solutions

Exam MFE/3F Solutions Chapter 2 – Comparing Options

© ActuarialBrew.com 2014 Page 2.05

As a result, at the end of 1.5 years the arbitrageur owns the total return portfolio and must repay the borrowed funds. This position results in the following cash flow at the end of 1.5 years:

- = - = - =0.06(0.5) 0.03(1.5)1.5 1.565 66.980 66 66.980 2.058Port e Port e

The 1.5-year option

The stock price at the end of 1.5 years is $66. This means that the total return portfolio has a value of:

=0.03(1.5)66 69.038e

The strike price is:

= =0.06(1.5)1.5 61.2147 66.980K e

Therefore, the 1.5-year put option expires unexercised. This position results in zero cash flow at the end of 1.5 years.

Net accumulated cash flows

Buying the longer put option and selling the shorter put option results in a cash flow of 0.85 at time 0.

The 1-year option resulted in a cash flow of 2.058 at time 1.5 years.

The 1.5 year option resulted in a cash flow of 0 at time 1.5 years.

The accumulated net cash flows are:

+ + =0.06(1.5)0.85 2.058 0 2.99e

Solution 2.05

A Proposition 2

The prices of the options violate Proposition 2:

- £ -2 1 2 1( ) ( )Eur EurP K P K K K

because:

-- > -

>

0.09(1)8.75 4 (55 50)4.75 4.57

e

Arbitrage is available using a put bull spread:

Buy (50)P

Sell (55)P

Page 28: ActuarialBrew.com Exam MFE / 3F Actuarial Models …actuarialbrew.com/data/documents/MFE-Solutions-2014-1st-ed.pdf · Exam MFE / 3F Actuarial Models Financial Economics Segment Solutions

Exam MFE/3F Solutions Chapter 2 – Comparing Options

© ActuarialBrew.com 2014 Page 2.06

Here’s a tip for remembering which option to buy and which option to sell. Notice that Proposition 2 fails because the 55-strike put premium is too high relative to the 50-strike put premium. To earn arbitrage profits, we sell the asset that is priced “too high” and buy the asset that is priced “too low.” This is a recurring theme in arbitrage strategies.

This strategy produces the following payoff table:

Time 1

Transaction Time 0 <1 50S £ £150 55S < 155 S

Buy (50)P –4.00 - 150 S 0 0

Sell (55)P 8.75 - - 1(55 )S - - 1(55 )S 0

Total 4.75 –5.00 - - 1(55 )S 0

If the final stock price is $48, then the accumulated arbitrage profits are:

= - =0.094.75 5.00 0.19733X e

If the final stock price is $52, then the accumulated arbitrage profits are:

= - - =0.094.75 (55 52) 2.19733Y e

The ratio of X to Y is:

= =0.197330.09

2.19733XY

Solution 2.06

C Proposition 3

The prices of the options violate Proposition 3:

-- £

- -3 22 1

2 1 3 2

( ) ( )( ) ( ) P K P KP K P KK K K K

because:

- ->- -

>

7 3 11 755 50 61 55

4 45 6

Page 29: ActuarialBrew.com Exam MFE / 3F Actuarial Models …actuarialbrew.com/data/documents/MFE-Solutions-2014-1st-ed.pdf · Exam MFE / 3F Actuarial Models Financial Economics Segment Solutions

Exam MFE/3F Solutions Chapter 2 – Comparing Options

© ActuarialBrew.com 2014 Page 2.07

Arbitrage is available using an asymmetric butterfly spread:

Buy l of the 50-strike options

Sell 1 of the 55-strike options

Buy l-(1 ) of the 61-strike options

where:

l - -= = =- -

3 2

3 1

61 55 661 50 11

K KK K

In the payoff table below, we have scaled the strategy up by multiplying by 11:

Time 1

Transaction Time 0 <1 50S £ £150 55S £ £155 61S < 161 S

Buy 6 of (50)P –6(3.00) - 16(50 )S 0.00 0.00 0.00

Sell 11 of (55)P 11(7.00) - - 111(55 )S - - 111(55 )S 0.00 0.00

Buy 5 of (61)P –5(11.00) - 15(61 )S - 15(61 )S - 15(61 )S 0.00

Total 4.00 0.00 -16 300S - 1305 5S 0.00

If the final stock price is $52, then the accumulated arbitrage profits are:

= + - = + - =0.11 0.1114.00 6 300 4.00 6(52) 300 16.4651X e S e

If the final stock price is $60, then the accumulated arbitrage profits are:

= + - = + - =0.11 0.1114.00 305 5 4.00 305 5(60) 9.4651Y e S e

The ratio of X to Y is:

= =16.46511.74

9.4651XY

Solution 2.07

E Proposition 2

The prices of Option A and Option B violate Proposition 2:

-- £ -1 2 2 1( ) ( ) ( ) rtEur EurC K C K K K e

because:

-- > -

>

0.07(1)14 9.25 (55 50)

4.75 4.66

e

Page 30: ActuarialBrew.com Exam MFE / 3F Actuarial Models …actuarialbrew.com/data/documents/MFE-Solutions-2014-1st-ed.pdf · Exam MFE / 3F Actuarial Models Financial Economics Segment Solutions

Exam MFE/3F Solutions Chapter 2 – Comparing Options

© ActuarialBrew.com 2014 Page 2.08

Arbitrage is available using a call bear spread:

Buy (55)C

Sell (50)C

This strategy produces the following payoff table:

Time 1

Transaction Time 0 <1 50S £ £150 55S < 155 S

Buy (55)C –9.25 0.00 0.00 -1 55S

Sell (50)C 14.00 0.00 - -1( 50)S - -1( 50)S

Total 4.75 0.00 - -1( 50)S –5.00

If the final stock price is $52, then the accumulated arbitrage profits are:

= - - = - - = - =0.07 0.07 0.0714.75 ( 50) 4.75 (52 50) 4.75 2 3.0944X e S e e

If the final stock price is $60, then the accumulated arbitrage profits are:

= - =0.074.75 5 0.0944Y e

The ratio of X to Y is:

= =3.094432.77

0.0944XY

Solution 2.08

D Proposition 1

The prices of Option A and Option B violate Proposition 1:

≥2 1( ) ( )P K P K

This is because:

<<

(127) (120)10 12

P P

Therefore, arbitrage profits can be earned by buying the 127-strike put and selling the 120-strike put. This is a put bear spread.

Page 31: ActuarialBrew.com Exam MFE / 3F Actuarial Models …actuarialbrew.com/data/documents/MFE-Solutions-2014-1st-ed.pdf · Exam MFE / 3F Actuarial Models Financial Economics Segment Solutions

Exam MFE/3F Solutions Chapter 2 – Comparing Options

© ActuarialBrew.com 2014 Page 2.09

Solution 2.09

C Proposition 3

The prices of the options violate Proposition 3:

-- ≥

- -2 31 2

2 1 3 2

( ) ( )( ) ( ) C K C KC K C KK K K K

This is because:

- -<- -

- -<- -

<

(45) (50) (50) (53)50 45 53 50

7 5 5 350 45 53 50

2 25 3

C C C C

Arbitrage is available using an asymmetric butterfly spread:

Buy l of the 45-strike options

Sell 1 of the 50-strike options

Buy l-(1 ) of the 53-strike options

where:

l - -= = =- -

3 2

3 1

53 50 353 45 8

K KK K

In the payoff table below, we have scaled the strategy up by multiplying by 8:

Time 1

Transaction Time 0 <1 45S £ £145 50S £ £150 53S < 153 S

Buy 3 of (45)C –3(7.00) 0.00 -13( 45)S -13( 45)S -13( 45)S

Sell 8 of (50)C 8(5.00) 0.00 0.00 - -18( 50)S - -18( 50)S

Buy 5 of (53)C –5(3.00) 0.00 0.00 0.00 -15( 53)S

Total 4.00 0.00 -13( 45)S - 1265 5S 0.00

Page 32: ActuarialBrew.com Exam MFE / 3F Actuarial Models …actuarialbrew.com/data/documents/MFE-Solutions-2014-1st-ed.pdf · Exam MFE / 3F Actuarial Models Financial Economics Segment Solutions

Exam MFE/3F Solutions Chapter 2 – Comparing Options

© ActuarialBrew.com 2014 Page 2.10

The cash flows at time 1 for each of the possible stock prices are in the table below:

Answer Choice Stock Price Time 1 Cash Flow

A 40 0.00

B 46 - = - =13( 45) 3(46 45) 3.00S

C 50 - = - =13( 45) 3(50 45) 15.00S

D 52 - = - =1265 5 265 5(52) 5.00S

E 55 0.00

The highest cash flow at time 1, $15.00, occurs if the final stock price is $50. This results in the highest arbitrage profits since the time 0 cash flow is the same for each future stock price.

Solution 2.10

A Bid-Ask Prices

This question might seem a bit unfair since bid and ask prices are not discussed in the assigned reading in the Derivatives Markets textbook. On the other hand, Problem 9.16 at the end of the second edition of Derivatives Markets Chapter 9 uses bid and ask prices. Someone writing an exam question just might think that it is a clever idea for an exam question.

The arbitrageur must pay the ask price to purchase assets. The arbitargeur receives the bid price when selling assets.

Let’s write put-call parity as follows:

-= + -0 ( , ) ( , )rTEur EurS C K T Ke P K T

This shows that the purchase of a share of stock can be replicated by:

buying a call option

lending the present value of the strike price

selling a put option

Let’s see if arbitrage profits can be earned by selling the stock and then replicating the purchase of the stock. The cash flows at time 0 are broken out below:

-

-

-0.059

Sell Stock 99.75Buy Call 19.75

Lend 90 84.8436Sell Put 5.00 Net Cash Flow 0.1564

e

Selling the stock and replicating the purchase of the stock results in a time 0 cash flow of 0.1564. The cash flow at the end of the year will be zero regardless of the stock price. Therefore, the time 0 cash flow of 0.1564 is an arbitrage profit.

Page 33: ActuarialBrew.com Exam MFE / 3F Actuarial Models …actuarialbrew.com/data/documents/MFE-Solutions-2014-1st-ed.pdf · Exam MFE / 3F Actuarial Models Financial Economics Segment Solutions

Exam MFE/3F Solutions Chapter 2 – Comparing Options

© ActuarialBrew.com 2014 Page 2.11

At this point, we have answered the question, but for thoroughness, we also consider whether arbitrage profits can be earned by buying the stock and replicating the sale of the stock.

The sale of a share of stock can be replicated by doing the opposite of the actions needed to replicate the purchase of a share of stock. The sale of a share of stock is replicated by:

selling a call option

borrowing the present value of the strike price

buying a put option

Let’s see if arbitrage profits can be earned by buying the stock and then replicating the sale of the stock.

The cash flows at time 0 are broken out below:

-

-

--

0.06

Buy Stock 100.00Sell Call 19.50

Borrow 90 84.7588Buy Put 5.10 Net Cash Flow 0.8412

e

Buying the stock and replicating the sale of the stock results in a time 0 cash flow that is negative, so it does not produce arbitrage profits.

Solution 2.11

C Application of Option Pricing Concepts

We are told that the price of a European call option with a strike price of $54.08 has a value of $10.16. The payoff of the call option at time 2 is:

0, (2) 54.08Max S

Once we express the payoff of the single premium deferred annuity in terms of the expression above, we will be able to obtain the price of the annuity.

The payoff at time 2 is:

2

2

(2)Time 2 Payoff (1 %) , 1.04

501

(1 %) (2), 50 1.04501

(1 %) (2), 54.08501

(1 %) (2) 54.08, 0 54.0850

SP y Max

P y Max S

P y Max S

P y Max S

Page 34: ActuarialBrew.com Exam MFE / 3F Actuarial Models …actuarialbrew.com/data/documents/MFE-Solutions-2014-1st-ed.pdf · Exam MFE / 3F Actuarial Models Financial Economics Segment Solutions

Exam MFE/3F Solutions Chapter 2 – Comparing Options

© ActuarialBrew.com 2014 Page 2.12

The current value of a payoff of 0, (2) 54.08Max S at time 2 is $10.16. The current

value of $54.08 at time 2 is:

- ¥ =0.06 254.08 47.9647e

Therefore, the current value of the payoff is:

{ }= - ¥ +1Current value of payoff (1 %) 10.16 47.9647

50P y

For the company to break even on the contract, the current value of the payoff must be equal to the single premium of P:

1(1 %) 10.16 47.9647

50(1 %) 1.16249 1

% 13.978%

P y P

y

y

Solution 2.12

C Early Exercise

Early exercise should not occur if the interest on the strike price exceeds the value of the dividends obtained through early exercise:

No early exercise if: - -- >( ), ( )r T t

t TK Ke PV div

The present value of the dividends is:

-= + =0.10(0.75), ( ) 3 2 4.86t TPV div e

The interest cost of paying the strike price early is shown in the rightmost column below:

Option Strike Price T - -- ( 0)r TK Ke

A 40 1.50 5.57

B 50 1.50 6.96

C 50 1.00 4.76

D 52 1.00 4.95

E 59 0.75 4.26

Only Option C and Option E have interest on the strike price that is less than the present value of the dividends of $4.86. Option E is not in the money though, because its strike price exceeds the stock price of $58, so it is not optimal to exercise Option E. Therefore, Option C is the only option for which early exercise might be optimal.

Page 35: ActuarialBrew.com Exam MFE / 3F Actuarial Models …actuarialbrew.com/data/documents/MFE-Solutions-2014-1st-ed.pdf · Exam MFE / 3F Actuarial Models Financial Economics Segment Solutions

Exam MFE/3F Solutions Chapter 2 – Comparing Options

© ActuarialBrew.com 2014 Page 2.13

Solution 2.13

B Comparing Options With Different Strikes and Maturities

The underlying asset is the total return portfolio.

The strike price grows at the interest rate:

-= fi £0.08(0.25) ( )5,101 5,000 r T tT te K K e

Therefore, to preclude arbitrage it should be the case that:

≥(0.75) (0.5)C C

But the longer option costs less than the shorter option, indicating that arbitrage is possible:

<470 473

The arbitrageur buys the longer option for $470 and sells the shorter option for $473. The difference of $3 is lent at the risk-free rate of return.

The 6-month option

After 6 months, the stock price is $49. Therefore, the value of the total return portfolio is:

¥ ¥ =0.5(0.06)100 49 5,049.23e

Since the strike price of the shorter option is $5,000, the shorter option is exercised against the arbitrageur. The arbitrageur sells the total return portfolio short for $5,000 and lends the $5,000 at the risk-free rate.

After 9 months, the stock price is $51. Therefore, the value of the total return portfolio is:

¥ ¥ = ¥ ¥ =0.06(0.75) 0.06(0.75)0.75100 100 51 5,334.74e S e

At the end of 9 months the arbitrageur owes the total return porfolio and owns the accumulated value of the $5,000. The position results in the following cash flow at the end of 9 months:

- + = -0.08(0.25)5,334.74 5,000 233.74e

The 9-month option

The strike price of the 9-month option is $5,101, so the 9-month call option has the following cash flow at the end of 9 months:

0.06(0.75)0.75100 5,101 5,334.74 5,101.00 233.74e S

The net cash flow

Since the 6-month option and the 9-month options have offsetting cash flows at the end of 9 months, the accumulated value of the arbitrage strategy is the difference between the cost of the two options, accumulated for 9 months:

- =0.08(0.75)(473 470) 3.19e

Page 36: ActuarialBrew.com Exam MFE / 3F Actuarial Models …actuarialbrew.com/data/documents/MFE-Solutions-2014-1st-ed.pdf · Exam MFE / 3F Actuarial Models Financial Economics Segment Solutions

Exam MFE/3F Solutions Chapter 2 – Comparing Options

© ActuarialBrew.com 2014 Page 2.14

Solution 2.14

E Bounds on Option Prices

Let’s begin by noting the prepaid forward price of the stock and the present value of the strike price:

d- - - ¥ -

- - -

= = =

= =

( ) 0.08 (0.5) 0.04,

( ) 0.08(0.5)

( )

100 96.08

P T tt T

r T t

F S e S e S Se

Ke e

For the European call option, the bounds are:

- -

- -

È ˘- £ - £Î ˚È ˘- £ - £Î ˚

( ), ,

0.04 0.04

0, ( ) ( , , ) ( )

0, 96.08 ( , , )

P r T t Pt T Eur t t T

Eur t

Max F S Ke C S K T t F S

Max Se C S K T t Se

This describes Graph I, so the European call option corresponds to Graph I.

For the American call option, the bounds are:

[ ]

- -

-

È ˘- - £ - £Î ˚È ˘- - £ - £Î ˚

- £ - £

( ),

0.04

0, ( ) , ( , , )

0, 96.08, 100 ( , , )

0, 100 ( , , )

P r T tAmer t tt T

Amer t

Amer t

Max F S Ke S K C S K T t S

Max Se S C S K T t S

Max S C S K T t S

This describes Graph II, so the American call option corresponds to Graph II.

For the European put option, the bounds are:

- - - -

-

È ˘- £ - £Î ˚È ˘- £ - £Î ˚

( ) ( ),

0.04

0, ( ) ( , , )

0, 96.08 ( , , ) 96.08

r T t P r T tEur tt T

Eur t

Max Ke F S P S K T t Ke

Max Se P S K T t

This describes Graph IV, so the European put option corresponds to Graph IV.

For the American put option, the bounds are:

[ ]

- -

-

È ˘- - £ - £Î ˚È ˘- - £ - £Î ˚

- £ - £

( ),

0.04

0, ( ), ( , , )

0, 96.08 , 100 ( , , ) 100

0, 100 ( , , ) 100

r T t PAmer tt T

Amer t

Amer t

Max Ke F S K S P S K T t K

Max Se S P S K T t

Max S P S K T t

This describes Graph III, so the American put option corresponds to Graph III.

Page 37: ActuarialBrew.com Exam MFE / 3F Actuarial Models …actuarialbrew.com/data/documents/MFE-Solutions-2014-1st-ed.pdf · Exam MFE / 3F Actuarial Models Financial Economics Segment Solutions

Exam MFE/3F Solutions Chapter 2 – Comparing Options

© ActuarialBrew.com 2014 Page 2.15

Solution 2.15

C Propositions 2 and 3

Choice A is true. Proposition 2 can be used to determine the following relationship for the first derivative of a call price:

- £ -- £--- £-

fi - £

£ +

1 2 2 1

1 2

2 1

2 1

2 1

( ) ( )

( ) ( )1

( ) ( )1

1

0 1

C K C K K K

C K C KK K

C K C KK K

dCdK

dCdK

Choice B is true, because Proposition 2 implies that the first derivative of a put price is positive, and the first derivative of a call option is negative.

Choice C is false because Proposition 3 implies that the second derivative of the call price is positive:

-- ≥- -

-- £- -

- -£ -- -

fi £

2 31 2

2 1 3 2

3 22 1

2 1 3 2

3 2 2 1

3 2 2 12

2

( ) ( )( ) ( )

( ) ( )( ) ( )

( ) ( ) ( ) ( )0

0

C K C KC K C KK K K K

C K C KC K C KK K K K

C K C K C K C KK K K K

d C

dK

Choice D and Choice E are true because the second derivative of a European put price is equal to the second derivative of a European call price. We can show this by taking the derivative of both sides of the put-call parity expression:

d- -

-

+ = +

+ =

=2 2

2 2

rT Tt

rT

C Ke S e P

dC dPe

dK dK

d C d P

dK dK

Page 38: ActuarialBrew.com Exam MFE / 3F Actuarial Models …actuarialbrew.com/data/documents/MFE-Solutions-2014-1st-ed.pdf · Exam MFE / 3F Actuarial Models Financial Economics Segment Solutions

Exam MFE/3F Solutions Chapter 2 – Comparing Options

© ActuarialBrew.com 2014 Page 2.16

Solution 2.16

A Option Payoffs

A butterfly spread involves options with 3 different strike prices. Part one of the investor’s position is a symmetric butterfly spread with strike prices $2, $3 and $4. Part two of the investor’s position is a symmetric butterfly spread with strike prices $4, $5 and $6. The payoff when buying a butterfly spread is never less than zero. In this case, each butterfly spread was sold, which means the payoffs are less than zero. The payoff of the butterfly spread looks the same whether the position is composed of calls or puts. Part one of the position’s payoff is shown as the bold line below at the left, and part two of the position’s payoff is shown as the bold line below at the right.

1 2 3 4 5 6 7Stock Price

Payoff

1 2 3 4 5 6 7Stock Price

Payoff

When combined, the investor’s position looks like Choice A.

Solution 2.17

D Comparing Options With Different Strikes and Maturities

The prices of the options decrease as time to maturity increases. Therefore, if the strike price increases at a rate that is less than the risk-free rate, then arbitrage is available. Option B expires 0.5 years after Option A, so let’s accumulate Option A’s strike price for 0.5 years at the risk-free rate:

¥ =0.06 0.550 51.5227e

Since the strike price of Option B is $52, which is greater than $51.5227, arbitrage is not indicated by the prices of Option A and Option B.

Option C expires 0.5 years after Option B, so let’s accumulate Option B’s strike price for 0.5 years at the risk-free rate:

¥ =0.06 0.552 53.5836e

Since the strike price of Option C is $53, the strike price grows from time 1 to time 1.5 at a rate that is less than the risk-free rate of return. Consequently, arbitrage can be earned by purchasing Option C and selling Option B (i.e., buy low and sell high).

The arbitrageur buys the 2-year option for $7.50 and sells the 1.5-year option for $7.70 The difference of $0.20 is lent at the risk-free rate of return.

Page 39: ActuarialBrew.com Exam MFE / 3F Actuarial Models …actuarialbrew.com/data/documents/MFE-Solutions-2014-1st-ed.pdf · Exam MFE / 3F Actuarial Models Financial Economics Segment Solutions

Exam MFE/3F Solutions Chapter 2 – Comparing Options

© ActuarialBrew.com 2014 Page 2.17

The 1.5-year option

After 1.5 years, the stock price is $52.50. Therefore, the 1.5-year option is exercised against the arbitrageur. The arbitrageur borrows a share of stock and sells it for the strike price of $52. As a result, at the end of 2 years the arbitrageur owes the share of stock and has the accumulated value of the $52. This position results in the following cash flow at the end of 2 years:

¥- + =0.06 0.552.50 52 1.0836e

The 2-year option

The stock price of $52.50 at the end of 2 years is less than the strike price of the 2-year option, which is $53. Therefore, the 2-year call option expires worthless, and the resulting cash flow is zero.

The net cash flow

The net cash flow at the end of 2 years is the sum of the accumulated value of the $0.20 that was obtained by establishing the position, the $1.0836 resulting from the 1.5-year option, and the $0.00 resulting from the 2-year option:

¥ + + =0.06 20.20 1.0836 0.00 1.3091e

Solution 2.18

D Arbitrage

Let X be the number of puts with a strike price of $60 that are sold for Jill’s portfolio. The fact that the net cost of establishing the portfolio is zero allows us to solve for X:

- + + - ¥ =- + ¥ + - =

=

(45) 3 (55) 1 (60) 04 3 9 1 12 0

2

P P P XX

X

An arbitrage strategy does not allow the cash flow at expiration to be negative. But suppose that only the $60-strike put option is in-the-money at expiration. Since Jill is short the $60-strike put option, this results in a negative cash flow. In particular, if the stock price at expiration is between $55 and $59, then the payoff from Jill’s strategy will be negative. For example, if the stock price at expiration is $57, the payoff from Jill’s strategy will be:

[45-strike payoff ] 3[55-strike payoff ] 2[60-strike payoff ] [Proceeds from loan]

0 0 2(60 57) 1 5

Since Jill’s strategy can result in a negative payoff at expiration, Jill’s strategy is not arbitrage.

Page 40: ActuarialBrew.com Exam MFE / 3F Actuarial Models …actuarialbrew.com/data/documents/MFE-Solutions-2014-1st-ed.pdf · Exam MFE / 3F Actuarial Models Financial Economics Segment Solutions

Exam MFE/3F Solutions Chapter 2 – Comparing Options

© ActuarialBrew.com 2014 Page 2.18

Let Y be the number of calls with a strike price of $60 that are purchased for Sabrina’s portfolio. The fact that the net cost of establishing the portfolio is zero allows us to solve for Y:

- + + ¥ =

- ¥ + + ==

(45) 3 (55) 1 (60) 0

12 3 7 1 4 02

C C C Y

YY

The table below shows that regardless of the stock price at time T, Sabrina’s payoff is positive. Therefore, Sabrina is correct.

Sabrina’s Portfolio Time T

Transaction Time 0 < 45TS £ £45 55TS £ £55 60TS <60 TS

Buy 1 of (45)C –12.00 0.00 - 45TS - 45TS - 45TS

Sell 3 of (55)C 3(7.00) 0.00 0.00 - -3( 55)TS - -3( 55)TS

Buy 2 of (60)C –2(4.00) 0.00 0.00 0.00 -2( 60)TS

Lend $1 –1.00 1 1 1 1

Total 0.00 1 + -1 45TS + -1 120 2 TS 1

Let Z be the number of calls purchased and puts sold with a strike price of $55 for Kelly’s portfolio.

Since the net cost of establishing the portfolio is zero, we can solve for Z:

[ ] [ ] [ ][ ] [ ] [ ]

- + - + - - =

- + - + - - =- + + + ==

2 (60) (60) 1 (45) (45) 2 (55) (55) 0

2 4 12 12 4 2 7 9 0

16 8 2 2 03

C P C P Z C P

Z

ZZ

In evaluating Kelly’s portfolio, we can make use of the fact that purchasing a call option and selling a put option is equivalent to purchasing a prepaid forward on the stock and borrowing the present value of the strike price. We can see this by writing put-call parity as:

-- = -0,( , ) ( , ) ( )P rTEur Eur TC K T P K T F S Ke

Therefore, purchasing a call option and selling a put option results in a payoff of:

-TS K

Since Kelly purchases offsetting amounts of puts and calls for any given strike price, we can use this result to evaluate her payoffs.

Page 41: ActuarialBrew.com Exam MFE / 3F Actuarial Models …actuarialbrew.com/data/documents/MFE-Solutions-2014-1st-ed.pdf · Exam MFE / 3F Actuarial Models Financial Economics Segment Solutions

Exam MFE/3F Solutions Chapter 2 – Comparing Options

© ActuarialBrew.com 2014 Page 2.19

Kelly’s Portfolio

Transaction Time 0 Time T

Buy 2 of (60)C & sell 2 of P(60) -2(12.00 4.00) -2( 60)TS

Buy 1 of (45)C & sell 1 of P(45) -4.00 12.00 - 45TS

Sell 3 of (55)C & buy 3 of P(55) -3(7.00 9.00) - -3( 55)TS

Lend $2 –2.00 2

Total 0.00 2

Kelly’s portfolio is certain to have a positive payoff at time T, so Kelly is correct.

Solution 2.19

D Bounds on Option Prices

From Propositions 1 and 2, we see that Statement I is true:

-

-

≥ <fi £ -

- £ - <

fi - £ -

2 1 1 2

2 1 2 1 1 2

Proposition 1: ( ) ( ) for

0 (85) (80)

Proposition 2: ( ) ( ) ( ) for

(85) (80) (85 80)

rTEur Eur

rTEur Eur

P K P K K K

P P

P K P K K K e K K

P P e

We make use of put-call parity for Statements II and III:

-+ = +( ) ( )rTC K K S P K

When the strike price for a call is increased, its price goes down, so the first inequality in Statement II is true:

- -- + = fi - + ≥(75) (75) 75 (75) (80) 75rT rTP C S e P C S e

When the strike price for a put is decreased, its price goes down, so the second inequality in Statement II is true:

- -- + = fi - + £(80) (80) 80 (75) (80) 80rT rTP C S e P C S e

As we saw in Statement II (directly above), the first inequality in Statement III is false:

- -- + = fi - + £(80) (80) 80 (75) (80) 80rT rTP C S e P C S e

The second inequalty in Statement III is true (but Statement III is still false):

- -- + = fi - + £ fi - + £(80) (80) 80 (75) (80) 80 (75) (80) 85rT rTP C S e P C S e P C S

Page 42: ActuarialBrew.com Exam MFE / 3F Actuarial Models …actuarialbrew.com/data/documents/MFE-Solutions-2014-1st-ed.pdf · Exam MFE / 3F Actuarial Models Financial Economics Segment Solutions

Exam MFE/3F Solutions Chapter 2 – Comparing Options

© ActuarialBrew.com 2014 Page 2.20

Solution 2.20

D Early Exercise of an American Call

If it is optimal to exercise an American call prior to maturity, then the early exercise takes place just before a dividend payment. Therefore the call is not exercised at time 0, and Choice A is not the correct answer.

The call is not exercised early if the present value of the interest on the strike exceeds the present value of the dividends:

- -- > fi( ), ( ) Don't exercise earlyr T t

t TK Ke PV div

Let’s consider time 0.25:

- - - -- > +

>

0.09(1 0.25) 0.09(0.75 0.25)86 86 2 1.9

5.6134 3.8164

e e

Since the present value of the interest on the strike exceeds the present value of the dividends, the call option is not exercised at time 0.25, so Choice B is not the correct answer.

Let’s consider time 0.75:

- -- >

>

0.09(1 0.75)86 86 1.91.9134 1.9

e

Since the present value of the interest on the strike exceeds the present value of the dividends, the call option is not exercised at time 0.75, so Choice C is not the correct answer.

Since the call was exercised, it must have been exercised at maturity, at the end of 12 months. Therefore, the correct answer is Choice D.

Solution 2.21

A Application of Option Pricing Concepts

We are told that the price of a European put option with a strike price of $54.08 has a value of $8.96. The payoff of the put option at time 2 is:

0,54.08 (2)Max S

Once we express the payoff of the single premium deferred annuity in terms of the expression above, we will be able to obtain the price of the annuity.

Page 43: ActuarialBrew.com Exam MFE / 3F Actuarial Models …actuarialbrew.com/data/documents/MFE-Solutions-2014-1st-ed.pdf · Exam MFE / 3F Actuarial Models Financial Economics Segment Solutions

Exam MFE/3F Solutions Chapter 2 – Comparing Options

© ActuarialBrew.com 2014 Page 2.21

The payoff at time 2 is:

2

2

(2)Time 2 Payoff (1 %) , 1.04

501

(1 %) (2), 50 1.04501

(1 %) (2), 54.08501

(1 %) 0, 54.08 (2) (2)50

SP y Max

P y Max S

P y Max S

P y Max S S

The current value of a payoff of 0,54.08 (2)Max S at time 2 is $8.96. The current value

of (2)S is its prepaid forward price:

d- - ¥= = ¥ =2 2 0.070,2( ) (0) 50 43.4679PF S e S e

Therefore, the current value of the payoff is:

{ }= - ¥ +1Current value of payoff (1 %) 8.96 43.4679

50P y

For the company to break even on the contract, the current value of the payoff must be equal to the single premium of P:

1(1 %) 8.96 43.4679

50(1 %) 1.04856 1

% 4.631%

P y P

yy

Page 44: ActuarialBrew.com Exam MFE / 3F Actuarial Models …actuarialbrew.com/data/documents/MFE-Solutions-2014-1st-ed.pdf · Exam MFE / 3F Actuarial Models Financial Economics Segment Solutions

Exam MFE/3F Solutions Chapter 2 – Comparing Options

© ActuarialBrew.com 2014 Page 2.22

This page is intentionally blank.

Page 45: ActuarialBrew.com Exam MFE / 3F Actuarial Models …actuarialbrew.com/data/documents/MFE-Solutions-2014-1st-ed.pdf · Exam MFE / 3F Actuarial Models Financial Economics Segment Solutions

Exam MFE/3F Solutions Chapter 3 – Binomial Trees: Part I

© ActuarialBrew.com 2014 Page 3.01

Chapter 3 – Solutions

Solution 3.01

C One-Period Binomial Tree

If the stock price moves up, then the call option pays $25. If the stock price moves down, then the call pays $0:

75 25 = Max[75 – 50, 0] 51 V

40 0 = Max[40 – 50, 0] In this case, 75 / 51u and 40 / 51d .

The risk-neutral probability of an upward movement is:

( ) 0.07(1) 0.07(1)40 / 51 51 40

* 0.4199475 / 51 40 / 51 75 40

r he d e ep

u d

The value of the call option is:

[ ]0.07(1)( *) (1 *) 0.41994(25) 0 9.79- -= + - = + =È ˘Î ˚rh

u dV e p V p V e

Solution 3.02

B One-Period Binomial Tree

If the stock price moves up, then the put option pays $0. If the stock price moves down, then the put pays $10:

75 0 = Max[50 – 75, 0] 51 V

40 10 = Max[50 – 40, 0] The risk-neutral probability of an upward movement is:

( ) 0.07(1)51 40

* 0.4199475 40

d- - -= = =- -

r he d ep

u d

The value of the put option is:

[ ]0.07(1)( *) (1 *) 0.41994(0) (1 0.41994)10 5.41- -= + - = + - =È ˘Î ˚rh

u dV e p V p V e

Page 46: ActuarialBrew.com Exam MFE / 3F Actuarial Models …actuarialbrew.com/data/documents/MFE-Solutions-2014-1st-ed.pdf · Exam MFE / 3F Actuarial Models Financial Economics Segment Solutions

Exam MFE/3F Solutions Chapter 3 – Binomial Trees: Part I

© ActuarialBrew.com 2014 Page 3.02

Solution 3.03

D Delta

If the stock price moves up, then the call option pays $25. If the stock price moves down, then the call pays $0:

75 25 51 V

40 0 To replicate the option, the investor must purchase delta shares of the underlying stock:

0.03(1) 25 00.693

( ) 75 40d- -- -D = = =

- -h u dV V

e eS u d

Solution 3.04

A Replication

If the stock price moves up, then the call option pays $25. If the stock price moves down, then the call pays $0:

75 25 51 V

40 0 To replicate the option, the investor must lend B:

0.07(1) 75(0) 40(25)26.64

75 40- -- -= = = -

- -rh d uuV dV

B e eu d

Lending $26.64- is equivalent to borrowing $26.64.

Solution 3.05

B One-Period Binomial Tree

We can use the binomial option pricing model to price any option that has defined values at the end of each period. We aren’t limited to pricing just calls and puts.

If the stock price moves up, then the option pays 275 . If the stock price moves down, then

the option pays 240 :

75 5,625 51 V

40 1,600 The risk-neutral probability of an upward movement is:

( ) (0.07 0.03)(1)51 40

* 0.3737575 40

d- -- -= = =- -

r he d ep

u d

The value of the option is:

Page 47: ActuarialBrew.com Exam MFE / 3F Actuarial Models …actuarialbrew.com/data/documents/MFE-Solutions-2014-1st-ed.pdf · Exam MFE / 3F Actuarial Models Financial Economics Segment Solutions

Exam MFE/3F Solutions Chapter 3 – Binomial Trees: Part I

© ActuarialBrew.com 2014 Page 3.03

[ ]0.07(1)( *) (1 *) 0.37375(5,625) (1 0.37375)(1,600)

2,894.48

- -= + - = + -È ˘Î ˚=

rhu dV e p V p V e

Solution 3.06

E Arbitrage

The first step is to find the values of u and d:

( ) (0.08 0.05)(1) 0.27 1

( ) (0.08 0.05)(1) 0.27 1

1.34986

0.78663

d s

d s

- + - +

- - - -

= = =

= = =

r h h

r h h

u e e

d e e

The possible stock prices are therefore:

65 65(1.34986) 87.740865 65(0.78663) 51.1308

= == =

ud

If the stock price moves up, then the option pays $0. If the stock price moves down, then the option pays 11.8692:

87.7408 0.0000 65 V 51.1308 11.8692

To replicate the option, the investor must purchase delta shares of the underlying stock:

0.05(1) 0.000 11.86920.3084

( ) 87.7408 51.1308d- -- -D = = = -

- -h u dV V

e eS u d

To replicate the option, the investor must lend B:

0.08(1) 1.34986(11.8692) 0.78663(0.000)26.259

1.34986 0.78663- -- -= = =

- -rh d uuV dV

B e eu d

The purchase of the option can therefore be replicated by selling 0.3084 shares of stock and lending $26.259.

Therefore, the price of the put option should be:

0 65( 0.3084) 26.259 6.213= D + = - + =V S B

But the market price of the option is $6.00. Therefore an arbitrageur can earn an arbitrage profit by purchasing the option for $6.00 and synthetically selling it for $6.213.

The synthetic sale of the option requires that the arbitrageur buy 0.3084 shares of stock and borrow $26.259.

Page 48: ActuarialBrew.com Exam MFE / 3F Actuarial Models …actuarialbrew.com/data/documents/MFE-Solutions-2014-1st-ed.pdf · Exam MFE / 3F Actuarial Models Financial Economics Segment Solutions

Exam MFE/3F Solutions Chapter 3 – Binomial Trees: Part I

© ActuarialBrew.com 2014 Page 3.04

Solution 3.07

C Delta

Delta is the slope of the line that passes through , dSd C and , uSu C :

( )

h u d u dV V C Ce

S u d Su Sd

Although we are not given the value of Su, we can still find the slope of the line because we have two points on the line:

(0, –26.58) and (40.53, 0)

The “rise over run” for the two points is:

0 ( 26.58) 26.58

0.65640.53 0.00 40.53- -D = = =

-

Solution 3.08

D Risk-Neutral Pricing

The values of u and d are:

( ) (0.08 0.00)(1) 0.30 1

( ) (0.08 0.00)(1) 0.30 1

1.46228

0.80252

d s

d s

- + - +

- - - -

= = =

= = =

r h h

r h h

u e e

d e e

The possible stock prices at the end of one year are:

50 50 1.46228 73.114250 50 0.80252 40.1259

= ¥ == ¥ =

ud

The payoffs for the call option in 1 year are:

Up state: [0, 73.1142 48] 25.1142

Down state: [0, 40.1259 48] 0.0000

Max

Max

The stock price tree and the call price tree are:

Stock Call 73.1142 25.1142

50.0000 V 40.1259 0.0000

The risk-neutral probability of an upward movement is:

( ) (0.08 0.00)(1) 0.80252

* 0.425561.46228 0.80252

d- -- -= = =- -

r he d ep

u d

Page 49: ActuarialBrew.com Exam MFE / 3F Actuarial Models …actuarialbrew.com/data/documents/MFE-Solutions-2014-1st-ed.pdf · Exam MFE / 3F Actuarial Models Financial Economics Segment Solutions

Exam MFE/3F Solutions Chapter 3 – Binomial Trees: Part I

© ActuarialBrew.com 2014 Page 3.05

The value of the call option is:

[ ]0.08(1)( *) (1 *) 0.42556(25.1142) (1 0.42556)(0.0000)

9.8659

- -= + - = + -È ˘Î ˚=

rhu dV e p V p V e

Solution 3.09

D Replication

Delta is the slope of the line that passes through , dSd C and , uSu C :

( )

h u d u dV V C Ce

S u d Su Sd

Although we are not given the value of Sd, we can still find the slope of the line because we have two points on the line:

(0, –32.38) and (88.62, 26.62)

The “rise-over-run” for the two points is:

26.62 ( 32.38) 59.00

0.665888.62 0.00 88.62

The intercept at the vertical axis is the value of the replicating portfolio if the stock price is zero at the end of 1 year:

0.090 32.38

29.5931D ¥ + = -= -

BeB

The option can be replicated by purchasing 0.6658 shares of stock and borrowing $29.5931:

0 60(0.6658) 29.5931 10.35= D + = - =C S B

Solution 3.10

D Expected Return

The values of u and d are:

( ) (0.08 0.00)(1) 0.30 1

( ) (0.08 0.00)(1) 0.30 1

1.46228

0.80252

d s

d s

- + - +

- - - -

= = =

= = =

r h h

r h h

u e e

d e e

The stock price tree and the call price tree are:

Stock Call 73.1142 25.1142

50.0000 V 40.1259 0.0000

Page 50: ActuarialBrew.com Exam MFE / 3F Actuarial Models …actuarialbrew.com/data/documents/MFE-Solutions-2014-1st-ed.pdf · Exam MFE / 3F Actuarial Models Financial Economics Segment Solutions

Exam MFE/3F Solutions Chapter 3 – Binomial Trees: Part I

© ActuarialBrew.com 2014 Page 3.06

The risk-neutral probability of an upward movement is:

( ) (0.08 0.00)(1) 0.80252

* 0.425561.46228 0.80252

d- -- -= = =- -

r he d ep

u d

The value of the call option is:

[ ]0.08(1)( *) (1 *) 0.42556(25.1142) (1 0.42556)(0.0000)

9.8659

- -= + - = + -È ˘Î ˚=

rhu dV e p V p V e

Since we know the price of the option and the distribution of its payoffs, we can now found the expected return of the option:

[ ]( ) (1 )

9.8659 0.46 25.1142 (1 0.46) 0.0000

9.8659ln 0.15783

0.46 25.1142

g

g

g

-

-

= + -È ˘Î ˚

= ¥ + - ¥

Ê ˆ= - =Á ˜Ë ¯¥

hu dV e p V p V

e

Solution 3.11

E Risk-Neutral Probability

To find the price using the risk-free rate of return, we must use risk-neutral probabilities, not realistic probabilities. The 45% probability of an upward movement is a red herring in this question.

The values of u and d are:

( ) (0.05 0.02)(1) 0.30 1

( ) (0.05 0.02)(1) 0.30 1

1.39097

0.76338

d s

d s

- + - +

- - - -

= = =

= = =

r h h

r h h

u e e

d e e

If the stock price moves up, then the option pays $0. If the stock price moves down, then the option pays $14.5634:

97.3678 0.0000 70 V 53.4366 14.5634

The risk-neutral probability of an upward movement is:

( ) (0.05 0.02)(1) 0.76338

* 0.425561.39097 0.76338

d- -- -= = =- -

r he d ep

u d

The value of the put option is:

[ ]0.05(1)( *) (1 *) 0.42556(0.00) (1 0.42556)(14.5634)

7.96

- -= + - = + -È ˘Î ˚=

rhu dV e p V p V e

Page 51: ActuarialBrew.com Exam MFE / 3F Actuarial Models …actuarialbrew.com/data/documents/MFE-Solutions-2014-1st-ed.pdf · Exam MFE / 3F Actuarial Models Financial Economics Segment Solutions

Exam MFE/3F Solutions Chapter 3 – Binomial Trees: Part I

© ActuarialBrew.com 2014 Page 3.07

Solution 3.12

C Expected Return

The return on a put option is generally less than the risk-free rate of return. For a put option, D is negative and B is positive. This means that buying a put option is equivalent to selling stock and lending at the risk-free rate. The expected return is less than the risk-free rate since the stock’s expected return is greater than the risk-free rate.

The values of u and d are:

( ) (0.08 0.00)(1) 0.30 1

( ) (0.08 0.00)(1) 0.30 1

1.46228

0.80252

d s

d s

- + - +

- - - -

= = =

= = =

r h h

r h h

u e e

d e e

The stock price tree and the put price tree are:

Stock Put 73.1142 0.0000

50.0000 V 40.1259 7.8741

The risk-neutral probability of an upward movement is:

( ) (0.08 0.00)(1) 0.80252

* 0.425561.46228 0.80252

d- -- -= = =- -

r he d ep

u d

The value of the put option is:

[ ]0.08(1)( *) (1 *) 0.42556(0.0000) (1 0.42556)(7.8741)

4.1754

- -= + - = + -È ˘Î ˚=

rhu dV e p V p V e

Since we know the price of the option and the distribution of its payoffs, we can now found the expected return of the option:

[ ]( ) (1 )

4.1754 0.46 0.0000 (1 0.46) 7.8741

4.1754ln 0.01817

(1 0.46) 7.8741

g

g

g

-

-

= + -È ˘Î ˚

= ¥ + - ¥

Ê ˆ= - =Á ˜Ë ¯- ¥

hu dV e p V p V

e

Solution 3.13

E Arbitrage

Unless the following inequality is satisfied, arbitrage is available:

( )d-< <r hd e u

Page 52: ActuarialBrew.com Exam MFE / 3F Actuarial Models …actuarialbrew.com/data/documents/MFE-Solutions-2014-1st-ed.pdf · Exam MFE / 3F Actuarial Models Financial Economics Segment Solutions

Exam MFE/3F Solutions Chapter 3 – Binomial Trees: Part I

© ActuarialBrew.com 2014 Page 3.08

Let’s consider each of the possible choices:

d ( )d-r he u

A 0.872 1.013 1.176

B 0.805 0.995 1.230

C 0.996 1.002 1.008

D 0.783 1.000 1.278

E 0.920 1.105 1.100 The final row violates the inequality because the risk-free investment outperforms the risky asset in both scenarios. Therefore, the model described in Choice E permits arbitrage.

Solution 3.14

A Expected Return

The values of u and d are:

( ) (0.08 0.04)(1) 0.24 1

( ) (0.08 0.04)(1) 0.24 1

1.32313

0.81873

d s

d s

- + - +

- - - -

= = =

= = =

r h h

r h h

u e e

d e e

The stock price tree and the put price tree are below:

Stock Put 82.0340 0.0000

62.0000 V 50.7613 13.2387

The realistic probability of an upward movement is:

a d- -- -= = =

- -

( ) (0.12 0.04)(1) 0.818730.52450

1.32313 0.81873

he d ep

u d

The risk-neutral probability of an upward movement is:

( ) (0.08 0.04)(1) 0.81873

* 0.440291.32313 0.81873

d- -- -= = =- -

r he d ep

u d

The value of the put option can be calculated using the risk-neutral probability:

[ ]0.08(1)( *) (1 *) 0.44029(0.0000) (1 0.44029)(13.2387)

6.8402

- -= + - = + -È ˘Î ˚=

rhu dV e p V p V e

Page 53: ActuarialBrew.com Exam MFE / 3F Actuarial Models …actuarialbrew.com/data/documents/MFE-Solutions-2014-1st-ed.pdf · Exam MFE / 3F Actuarial Models Financial Economics Segment Solutions

Exam MFE/3F Solutions Chapter 3 – Binomial Trees: Part I

© ActuarialBrew.com 2014 Page 3.09

Since we know the price of the option and the distribution of its payoffs, we can now found the expected return of the option:

[ ]( ) (1 )

6.8402 0.52450 0.0000 (1 0.52450) 13.2387

6.8402ln 0.08305

(1 0.52450) 13.2387

g

g

g

-

-

= + -È ˘Î ˚

= ¥ + - ¥

Ê ˆ= - = -Á ˜Ë ¯- ¥

hu dV e p V p V

e

Solution 3.15

B One-Period Binomial Tree

This question can be answered using put-call parity:

00.08(0.5) 0.02(0.5)

( , ) ( , )

2.58 55 50 (55,0.5)

(55,0.5) 5.92

d- -

- -

+ = +

+ = +=

rT TEur Eur

Eur

Eur

C K T Ke S e P K T

e e P

P

Solution 3.16

B Realistic Probability

The values of u and d are:

( ) (0.07 0.02)(1) 0.27 1

( ) (0.07 0.02)(1) 0.27 1

1.37713

0.80252

d s

d s

- + - +

- - - -

= = =

= = =

r h h

r h h

u e e

d e e

The stock price tree and the call price tree are:

Stock Call 52.3309 12.3309

38.0000 V 30.4957 0.0000

The risk-neutral probability of an upward movement is:

( ) (0.07 0.02)(1) 0.80252

* 0.432911.37713 0.80252

d- -- -= = =- -

r he d ep

u d

The value of the call option is:

[ ]0.07(1)( *) (1 *) 0.43291(12.3309) (1 0.43291)(0.0000)

4.9772

- -= + - = + -È ˘Î ˚=

rhu dV e p V p V e

Page 54: ActuarialBrew.com Exam MFE / 3F Actuarial Models …actuarialbrew.com/data/documents/MFE-Solutions-2014-1st-ed.pdf · Exam MFE / 3F Actuarial Models Financial Economics Segment Solutions

Exam MFE/3F Solutions Chapter 3 – Binomial Trees: Part I

© ActuarialBrew.com 2014 Page 3.10

Since we know the price of the option, its possible payoffs, and its expected rate of return, we can find the realistic probability of an upward movement:

0.34836(1)

0.34836

( ) (1 )

4.9772 12.3309 (1 ) 0.0000

4.97720.57185

12.3309

hu dV e p V p V

e p p

ep

The realistic probability that the stock price moves down is:

1 0.57185 0.42815- =

Solution 3.17

C Expected Return

The values of u and d are:

( ) (0.07 0.02)(1) 0.27 1

( ) (0.07 0.02)(1) 0.27 1

1.37713

0.80252

d s

d s

- + - +

- - - -

= = =

= = =

r h h

r h h

u e e

d e e

The stock price tree and the call price tree are:

Stock Call 52.3309 12.3309

38.0000 V 30.4957 0.0000

The risk-neutral probability of an upward movement is:

( ) (0.07 0.02)(1) 0.80252

* 0.432911.37713 0.80252

d- -- -= = =- -

r he d ep

u d

The value of the call option is:

[ ]0.07(1)( *) (1 *) 0.43291(12.3309) (1 0.43291)(0.0000)

4.9772

- -= + - = + -È ˘Î ˚=

rhu dV e p V p V e

Since we know the price of the option, its possible payoffs, and its expected rate of return, we can find the realistic probability of an upward movement:

0.34836(1)

0.34836

( ) (1 )

4.9772 12.3309 (1 ) 0.0000

4.97720.57185

12.3309

hu dV e p V p V

e p p

ep

Page 55: ActuarialBrew.com Exam MFE / 3F Actuarial Models …actuarialbrew.com/data/documents/MFE-Solutions-2014-1st-ed.pdf · Exam MFE / 3F Actuarial Models Financial Economics Segment Solutions

Exam MFE/3F Solutions Chapter 3 – Binomial Trees: Part I

© ActuarialBrew.com 2014 Page 3.11

We can now use the realistic probability to find the expected return on the stock:

a d

a

a

-

- ¥

-=-

-=-

=

( )

( 0.02) 1 0.802520.57185

1.37713 0.802520.1432

he dp

u d

e

Solution 3.18

D Expected Return

The magnitude of g decreases as the stock price increases, so we only need to check the top nodes to answer this question.

The risk-neutral probability of an upward movement is constant throughout the tree for the standard binomial model. Therefore we can calculate it using any node. Below we use the first node:

(0.08 0.00)(1 / 3)( )

54.412663.000* 0.45681

76.9385 54.412663.000 63.000

r h ee dp

u d

Working from right to left, we create the tree of prices for the American call option:

Stock 114.7493 American Call 55.7493 93.9609 36.5134 76.9385 81.1533 21.4521 22.1533

63.0000 66.4512 11.8596 9.8535 54.4126 57.3934 4.3827 0.0000 46.9958 0.0000 40.5899 0.0000

Since the stock does not pay dividends, it is never optimal to exercise the call early.

The realistic probability is also constant throughout the tree, so as with the risk-neutral probability, we can use the values at the first node to calculate it:

(0.15 0.00)(1 / 3)( )

54.412663.000 0.52462

76.9385 54.412663.000 63.000

h ee dp

u d

Since g decreases as the stock price increases, we need only check the uppermost branch

of the tree of prices to find the lowest value of g .

Page 56: ActuarialBrew.com Exam MFE / 3F Actuarial Models …actuarialbrew.com/data/documents/MFE-Solutions-2014-1st-ed.pdf · Exam MFE / 3F Actuarial Models Financial Economics Segment Solutions

Exam MFE/3F Solutions Chapter 3 – Binomial Trees: Part I

© ActuarialBrew.com 2014 Page 3.12

When the stock price is $63.00, we have:

(1 / 3)

( ) (1 )

(0.52462)21.4521 (1 0.52462)(4.3827)11.8596

(0.52462)21.4521 (1 0.52462)(4.3827)ln 3 0.352

11.8596

g

g

g

+ -È ˘Î ˚=

+ -=

+ -Ê ˆ= ¥ =Á ˜Ë ¯

u dh p V p Ve

V

e

When the stock price is $76.9385, we have:

(1 / 3)

( ) (1 )

(0.52462)36.5134 (1 0.52462)(9.8535)21.4521

(0.52462)36.5134 (1 0.52462)(9.8535)ln 3 0.317

21.4521

g

g

g

+ -È ˘Î ˚=

+ -=

+ -Ê ˆ= ¥ =Á ˜Ë ¯

u dh p V p Ve

V

e

When the stock price is $93.9609, we have:

(1 / 3)

( ) (1 )

(0.52462)55.7493 (1 0.52462)(22.1533)36.5134

(0.52462)55.7493 (1 0.52462)(22.1533)ln 3 0.257

36.5134

g

g

g

+ -È ˘Î ˚=

+ -=

+ -Ê ˆ= ¥ =Á ˜Ë ¯

u dh p V p Ve

V

e

The lowest value of g occurs when the stock price is $93.9609.

Solution 3.19

D Expected Return

The risk-neutral probability of an upward movement is constant throughout the tree for the standard binomial model. Therefore we can calculate it using any node. Below we use the first node:

(0.08 0.00)(1 / 3)( )

54.412663.000* 0.45681

76.9385 54.412663.000 63.000

r h ee dp

u d

Page 57: ActuarialBrew.com Exam MFE / 3F Actuarial Models …actuarialbrew.com/data/documents/MFE-Solutions-2014-1st-ed.pdf · Exam MFE / 3F Actuarial Models Financial Economics Segment Solutions

Exam MFE/3F Solutions Chapter 3 – Binomial Trees: Part I

© ActuarialBrew.com 2014 Page 3.13

Working from right to left, we create the tree of prices for the European put option:

Stock 114.7493 European Put 0.0000 93.9609 0.0000 76.9385 81.1533 0.4494 0.0000

63.0000 66.4512 3.3235 0.8497 54.4126 57.3934 5.9058 1.6066 46.9958 10.4517 40.5899 18.4101

The realistic probability is also constant throughout the tree, so as with the risk-neutral probability, we can use the values at the first node to calculate it:

(0.15 0.00)(1 / 3)( )

54.412663.000 0.52462

76.9385 54.412663.000 63.000

h ee dp

u d

Since g decreases as the stock price increases, we need only check the uppermost branch

of the tree of prices to find the lowest value of g .

When the stock price is $63.00, we have:

(1 / 3)

( ) (1 )

(0.52462)0.4494 (1 0.52462)(5.9058)3.3235

(0.52462)0.4494 (1 0.52462)(5.9058)ln 3 0.264

3.3235

g

g

g

+ -È ˘Î ˚=

+ -=

+ -Ê ˆ= ¥ = -Á ˜Ë ¯

u dh p V p Ve

V

e

When the stock price is $76.9385, we have:

(1 / 3)

( ) (1 )

(0.52462)(0.0000) (1 0.52462)(0.8497)0.4494

(0.52462)(0.0000) (1 0.52462)(0.8497)ln 3 0.320

0.4494

g

g

g

+ -È ˘Î ˚=

+ -=

+ -Ê ˆ= ¥ = -Á ˜Ë ¯

u dh p V p Ve

V

e

When the stock price is $54.4126, we have:

(1 / 3)

( ) (1 )

(0.52462)(0.8497) (1 0.52462)(10.4517)5.9058

(0.52462)(0.8497) (1 0.52462)(10.4517)ln 3 0.261

5.9058

g

g

g

+ -È ˘Î ˚=

+ -=

+ -Ê ˆ= ¥ = -Á ˜Ë ¯

u dh p V p Ve

V

e

Page 58: ActuarialBrew.com Exam MFE / 3F Actuarial Models …actuarialbrew.com/data/documents/MFE-Solutions-2014-1st-ed.pdf · Exam MFE / 3F Actuarial Models Financial Economics Segment Solutions

Exam MFE/3F Solutions Chapter 3 – Binomial Trees: Part I

© ActuarialBrew.com 2014 Page 3.14

When the stock price is $66.4512 we have:

(1 / 3)

( ) (1 )

(0.52462)(0.0000) (1 0.52462)(1.6066)0.8497

(0.52462)(0.0000) (1 0.52462)(1.6066)ln 3 0.320

0.8497

g

g

g

+ -È ˘Î ˚=

+ -=

+ -Ê ˆ= ¥ = -Á ˜Ë ¯

u dh p V p Ve

V

e

When the stock price is $46.9958 we have:

(1 / 3)

( ) (1 )

(0.52462)(1.6066) (1 0.52462)(18.4101)10.4517

(0.52462)(1.6066) (1 0.52462)(18.4101)ln 3 0.257

10.4517

g

g

g

+ -È ˘Î ˚=

+ -=

+ -Ê ˆ= ¥ = -Á ˜Ë ¯

u dh p V p Ve

V

e

The tree of values for g is:

N/A –0.320

–0.264 –0.320 –0.261 –0.257

The lowest value is –32.0%.

It might seem surprising that g is lowest at the upper nodes. After all, the put option can be viewed as being more risky as it becomes more out-of-the money. So as the stock price increases, shouldn’t the expected return increase as well? The answer is no, because the put option is a hedging instrument. Therefore, its “risk” is inversely related to the risk of the stock price declining. That’s why put options earn less than the risk-free rate of return. Armed with this knowledge, we see that the lowest value of g is sure to be in one of the upper nodes. Therefore, we could have solved this problem by calculating the value of g at only the three uppermost nodes.

The general rule for both calls and puts is that the lowest value of g for each column in a tree is at the highest node.

Solution 3.20

C Cox-Ross-Rubinstein Binomial Tree

The values of u and d are:

0.30 1

0.30 1

1.34986

0.74082

s

s- -

= = =

= = =

h

h

u e e

d e e

Page 59: ActuarialBrew.com Exam MFE / 3F Actuarial Models …actuarialbrew.com/data/documents/MFE-Solutions-2014-1st-ed.pdf · Exam MFE / 3F Actuarial Models Financial Economics Segment Solutions

Exam MFE/3F Solutions Chapter 3 – Binomial Trees: Part I

© ActuarialBrew.com 2014 Page 3.15

The possible stock prices at the end of one year are:

38 38 1.34986 51.2946

38 38 0.74082 28.1511

= ¥ == ¥ =

ud

The payoffs for the call option in 1 year are:

Up state: [0, 51.2946 40] 11.2946

Down state: [0, 28.1511 40] 0.0000

Max

Max

The stock price tree and the call price tree are below:

Stock Call 51.2946 11.2946

38.0000 V 28.1511 0.0000

The risk-neutral probability of an upward movement is:

( ) (0.07 0.00)(1) 0.74082

* 0.544611.34986 0.74082

d- -- -= = =- -

r he d ep

u d

The value of the call option is:

[ ]0.07(1)( *) (1 *) 0.54461(11.2946) (1 0.54461)(0.0000)

5.735

- -= + - = + -È ˘Î ˚=

rhu dV e p V p V e

Solution 3.21

A Cox-Ross-Rubinstein Binomial Tree

The values of u and d are:

0.30 1

0.30 1

1.34986

0.74082

s

s- -

= = =

= = =

h

h

u e e

d e e

We can now solve for the expected return on the stock:

a d

a

a

-

- ¥

-=-

-=-

=

( )

( 0) 1 0.740820.661

1.34986 0.740820.1340

he dp

u d

e

Page 60: ActuarialBrew.com Exam MFE / 3F Actuarial Models …actuarialbrew.com/data/documents/MFE-Solutions-2014-1st-ed.pdf · Exam MFE / 3F Actuarial Models Financial Economics Segment Solutions

Exam MFE/3F Solutions Chapter 3 – Binomial Trees: Part I

© ActuarialBrew.com 2014 Page 3.16

Solution 3.22

B Cox-Ross-Rubinstein Binomial Tree

The values of u and d are:

0.30 1

0.30 1

1.34986

0.74082

s

s- -

= = =

= = =

h

h

u e e

d e e

The possible stock prices at the end of one year are:

38 38 1.34986 51.2946

38 38 0.74082 28.1511

= ¥ == ¥ =

u

d

The payoffs for the put option in 1 year are:

Up state: [0, 40 51.2946] 0.0000

Down state: [0, 40 28.1511] 11.8489

Max

Max

The stock price tree and the put price tree are below:

Stock Put 51.2946 0.0000

38.0000 V 28.1511 11.8489

The risk-neutral probability of an upward movement is:

( ) (0.07 0.00)(1) 0.74082

* 0.544611.34986 0.74082

d- -- -= = =- -

r he d ep

u d

The value of the put option is:

[ ]0.07(1)( *) (1 *) 0.54461(0.0000) (1 0.54461)(11.8489)

5.031

- -= + - = + -È ˘Î ˚=

rhu dV e p V p V e

Solution 3.23

A Cox-Ross-Rubinstein Binomial Tree

The values of u and d are:

0.30 0.25

0.30 0.25

1.16183

0.86071

s

s- -

= = =

= = =

h

h

u e e

d e e

The stock price tree and the call price tree are below:

Stock Call 116.1834 21.1834

100.0000 10.1239 86.0708 0.0000

Page 61: ActuarialBrew.com Exam MFE / 3F Actuarial Models …actuarialbrew.com/data/documents/MFE-Solutions-2014-1st-ed.pdf · Exam MFE / 3F Actuarial Models Financial Economics Segment Solutions

Exam MFE/3F Solutions Chapter 3 – Binomial Trees: Part I

© ActuarialBrew.com 2014 Page 3.17

The risk-neutral probability of an upward movement is:

( ) (0.08 0.05)(0.25) 0.86071

* 0.487571.16183 0.86071

d- -- -= = =- -

r he d ep

u d

The value of the call option is:

0.08(0.25)( *) (1 *) 0.48757(21.1834) (1 0.48757)(0.00)

10.124

rhu dV e p V p V e

Solution 3.24

B Cox-Ross-Rubinstein Binomial Tree

The values of u and d are:

0.30 0.25

0.30 0.25

1.16183

0.86071

s

s- -

= = =

= = =

h

h

u e e

d e e

The stock price tree and the call price tree are below:

Stock Call 116.1834 21.1834

100.0000 10.1239 86.0708 0.0000

The risk-neutral probability of an upward movement is:

( ) (0.08 0.05)(0.25) 0.86071

* 0.487571.16183 0.86071

d- -- -= = =- -

r he d ep

u d

The value of the call option is:

0.08(0.25)( *) (1 *) 0.48757(21.1834) (1 0.48757)(0.00)

10.1239

rhu dV e p V p V e

The realistic probability can now be determined:

(0.2153)(0.25)

(0.2153)(0.25)

( ) (1 )

10.1239 ( )(21.1834) (1 )(0.0000)

10.123921.1834

0.50434

hu dV e p V p V

e p p

ep

p

Page 62: ActuarialBrew.com Exam MFE / 3F Actuarial Models …actuarialbrew.com/data/documents/MFE-Solutions-2014-1st-ed.pdf · Exam MFE / 3F Actuarial Models Financial Economics Segment Solutions

Exam MFE/3F Solutions Chapter 3 – Binomial Trees: Part I

© ActuarialBrew.com 2014 Page 3.18

Now that we have the realistic probability, we can find the expected return for the stock:

(0.25) 0.05(0.25) 0.05(0.25)

0.05(0.25) 0.05(0.25)

( ) (1 )

100 (0.50434)(116.1834) (1 0.50434)(86.0708)

(0.50434)(116.1834) (1 0.50434)(86.0708) 1ln

100 0.25

0.100

a d d

a

a

-

-

È ˘= + -Î ˚È ˘= + -Î ˚

Ê ˆ+ -= ¥Á ˜Ë ¯

=

h h hS e p Sue p Sde

e e e

e e

Solution 3.25

D Jarrow and Rudd Binomial Tree

The values of u and d are:

2 2

2 2

( 0.5 ) (0.07 0.00 0.5 0.30 )(1) 0.3 1

( 0.5 ) (0.07 0.00 0.5 0.30 )(1) 0.3 1

1.38403

0.75957

d s s

d s s

- - + - - ¥ +

- - - - - ¥ -

= = =

= = =

r h h

r h h

u e e

d e e

The possible stock prices at the end of one year are:

38 38 1.38403 52.5932

38 38 0.75957 28.8637= ¥ == ¥ =

u

d

The payoffs for the call option in 1 year are:

Up state: [0, 52.5932 40] 12.5932

Down state: [0, 28.8637 40] 0.0000

Max

Max

The stock price tree and the call price tree are below:

Stock Call 52.5932 12.5932

38.0000 5.8842 28.8637 0.0000

The risk-neutral probability of an upward movement is:

( ) (0.07 0.00)(1) 0.75957

* 0.501131.38403 0.75957

d- -- -= = =- -

r he d ep

u d

The value of the call option is:

[ ]0.07(1)( *) (1 *) 0.50113(12.5932) (1 0.50113)(0.0000)

5.884

- -= + - = + -È ˘Î ˚=

rhu dV e p V p V e

Page 63: ActuarialBrew.com Exam MFE / 3F Actuarial Models …actuarialbrew.com/data/documents/MFE-Solutions-2014-1st-ed.pdf · Exam MFE / 3F Actuarial Models Financial Economics Segment Solutions

Exam MFE/3F Solutions Chapter 3 – Binomial Trees: Part I

© ActuarialBrew.com 2014 Page 3.19

Solution 3.26

C Jarrow and Rudd Binomial Tree

The values of u and d are:

2 2

2 2

( 0.5 ) (0.07 0.00 0.5 0.30 )(1) 0.3 1

( 0.5 ) (0.07 0.00 0.5 0.30 )(1) 0.3 1

1.38403

0.75957

d s s

d s s

- - + - - ¥ +

- - - - - ¥ -

= = =

= = =

r h h

r h h

u e e

d e e

The possible stock prices at the end of one year are:

38 38 1.38403 52.5932

38 38 0.75957 28.8637= ¥ == ¥ =

u

d

The stock price tree and the put price tree are below:

Stock Put 52.5932 0.0000

38.0000 5.1799 28.8637 11.1363

The risk-neutral probability of an upward movement is:

( ) (0.07 0.00)(1) 0.75957

* 0.501131.38403 0.75957

d- -- -= = =- -

r he d ep

u d

The value of the put option is:

[ ]0.07(1)( *) (1 *) 0.50113(0.0000) (1 0.50113)(11.1363)

5.180

- -= + - = + -È ˘Î ˚=

rhu dV e p V p V e

Solution 3.27

C Jarrow and Rudd Binomial Tree

The values of u and d are:

2 2

2 2

( 0.5 ) (0.08 0.05 0.5 0.30 )(0.25) 0.3 0.25

( 0.5 ) (0.08 0.05 0.5 0.30 )(0.25) 0.3 0.25

1.15749

0.85749

d s s

d s s

- - + - - ¥ +

- - - - - ¥ -

= = =

= = =

r h h

r h h

u e e

d e e

The stock price tree and the call price tree are below:

Stock Call 115.7486 20.7486

100.0000 10.1717 85.7486 0.0000

The risk-neutral probability of an upward movement is:

( ) (0.08 0.05)(0.25) 0.85749

* 0.500141.15749 0.85749

d- -- -= = =- -

r he d ep

u d

Page 64: ActuarialBrew.com Exam MFE / 3F Actuarial Models …actuarialbrew.com/data/documents/MFE-Solutions-2014-1st-ed.pdf · Exam MFE / 3F Actuarial Models Financial Economics Segment Solutions

Exam MFE/3F Solutions Chapter 3 – Binomial Trees: Part I

© ActuarialBrew.com 2014 Page 3.20

The value of the call option is:

[ ] [ ]- -= + - = + -

=

0.08(0.25)( *) (1 *) 0.50014(20.7486) (1 0.50014)(0.0000)

10.172

rhu dV e p V p V e

Solution 3.28

D Multiple-Period Binomial Tree

It isn’t very reasonable to expect us to work through an 8-period binomial tree during the exam, so there must be a shortcut to the answer. As it turns out, the option is in-the-money at only one final node. That makes it fairly easy to find the correct answer.

The value of h is 1/12 since the intervals are monthly periods. The values of u and d are:

( ) (0.07 0.02)(1 /12) 0.35 1 /12

( ) (0.07 0.02)(1 /12) 0.35 1 /12

1.11094

0.90767

r h h

r h h

u e e

d e e

The risk-neutral probability of an upward movement is:

( ) (0.07 0.02)(1 /12) 0.90767

* 0.474761.11094 0.90767

d- -- -= = =- -

r he d ep

u d

If the stock price increases each month, then at the end of 8 months, the price is:

8 8130 130(1.11094) 301.6170= =u

If the stock price increases for 7 of the months and decreases for 1 of the months, then the price is:

7 7130 130(1.11094) (0.90767) 246.4319= =u d

Since the price of $246.4319 is out-of-the-money, all other lower prices are also out-of-the-money. This means that the only price at which the option expires in-the-money is the highest possible price, which is $301.6170. Consequently, the value of the option is:

( )0 0

0

0.07(8 /12) 8

( , ,0) ( *) (1 *) ( , , )

[0 0 0 0 0 0 0 (0.47476) (301.6170 247)] 0.1345

- - -

=-

È ˘Ê ˆ= -Í ˙Á ˜Ë ¯Î ˚

= + + + + + + + -=

Ân

r hn j n j j n j

j

nV S K e p p V S u d K hn

j

e

Solution 3.29

D Jarrow and Rudd Binomial Tree

The values of u and d are:

2 2

2 2

( 0.5 ) (0.08 0.05 0.5 0.30 )(0.25) 0.3 0.25

( 0.5 ) (0.08 0.05 0.5 0.30 )(0.25) 0.3 0.25

1.15749

0.85749

d s s

d s s

- - + - - ¥ +

- - - - - ¥ -

= = =

= = =

r h h

r h h

u e e

d e e

Page 65: ActuarialBrew.com Exam MFE / 3F Actuarial Models …actuarialbrew.com/data/documents/MFE-Solutions-2014-1st-ed.pdf · Exam MFE / 3F Actuarial Models Financial Economics Segment Solutions

Exam MFE/3F Solutions Chapter 3 – Binomial Trees: Part I

© ActuarialBrew.com 2014 Page 3.21

The stock price tree and the call price tree are below:

Stock Call 115.7486 20.7486

100.0000 10.1717 85.7486 0.0000

The risk-neutral probability of an upward movement is:

( ) (0.08 0.05)(0.25) 0.85749

* 0.500141.15749 0.85749

d- -- -= = =- -

r he d ep

u d

The value of the call option is:

[ ]0.07(1)( *) (1 *) 0.50014(20.7486) (1 0.50014)(0.0000)

10.1717

- -= + - = + -È ˘Î ˚=

rhu dV e p V p V e

The realistic probability can now be determined:

(0.5281)(0.25)

(0.5281)(0.25)

( ) (1 )

10.1717 ( )(20.7486) (1 )(0.0000)

10.171720.7486

0.55943

hu dV e p V p V

e p p

ep

p

Now that we have the realistic probability, we can find the expected return for the stock:

( )

( 0.05) 0.25 0.857490.55943

1.15749 0.857490.150

he dp

u d

e

Solution 3.30

C Multiple-Period Binomial Tree

If we work through the entire binomial tree, this is a very time-consuming problem. Even using the direct method takes a lot of time since the put option provides a positive payoff at multiple final nodes. But once we notice that the value of the corresponding call option can be calculated fairly quickly, we can use put-call parity to find the value of the put option.

The value of h is 1/12 since the intervals are monthly periods. The values of u and d are:

( ) (0.07 0.02)(1 /12) 0.35 1 /12

( ) (0.07 0.02)(1 /12) 0.35 1 /12

1.11094

0.90767

r h h

r h h

u e e

d e e

Page 66: ActuarialBrew.com Exam MFE / 3F Actuarial Models …actuarialbrew.com/data/documents/MFE-Solutions-2014-1st-ed.pdf · Exam MFE / 3F Actuarial Models Financial Economics Segment Solutions

Exam MFE/3F Solutions Chapter 3 – Binomial Trees: Part I

© ActuarialBrew.com 2014 Page 3.22

The risk-neutral probability of an upward movement is:

( ) (0.07 0.02)(1 /12) 0.90767

* 0.474761.11094 0.90767

d- -- -= = =- -

r he d ep

u d

If the stock price increases each month, then at the end of 8 months, the price is:

8 8130 130(1.11094) 301.6170= =u

If the stock price increases for 7 of the months and decreases for 1 of the months, then the price is:

7 7130 130(1.11094) (0.90767) 246.4319= =u d

Consider the corresponding call option with a strike price of $247. Since the price of $246.4319 is out-of-the-money, all other lower prices are also out-of-the-money. This means that the only price at which the call option expires in-the-money is the highest possible price, which is $301.6170.

Calculating the price of the put option is daunting, but the corresponding call option price can be found fairly easily. The value of the call option is:

( )0 0

0

0.07(8 /12) 8

( , ,0) ( *) (1 *) ( , , )

[0 0 0 0 0 0 0 (0.47476) (301.6170 247)] 0.1345

- - -

=-

È ˘Ê ˆ= -Í ˙Á ˜Ë ¯Î ˚

= + + + + + + + -=

Ân

r hn j n j j n j

j

nV S K e p p V S u d K hn

j

e

Now we can use put-call parity to find the value of the corresponding put option:

00.07(8 /12) 0.02(8 /12)

( , ) ( , )

0.1345 247 130 ( , )

( , ) 107.59

d- -

- -

+ = +

+ = +=

rT TEur Eur

Eur

Eur

C K T Ke S e P K T

e e P K T

P K T

Solution 3.31

A Jarrow and Rudd Binomial Tree

The values of u and d are:

2 2

2 2

( 0.5 ) (0.12 0.045 0.5 0.27 )(0.5) 0.27 0.5

( 0.5 ) (0.12 0.045 0.5 0.27 )(0.5) 0.27 0.5

1.23392

0.84228

d s s

d s s

- - + - - ¥ +

- - - - - ¥ -

= = =

= = =

r h h

r h h

u e e

d e e

The possible stock prices at the end of one year are:

53 53 1.23392 65.397653 53 0.84228 44.6408

ud= ¥ == ¥ =

Page 67: ActuarialBrew.com Exam MFE / 3F Actuarial Models …actuarialbrew.com/data/documents/MFE-Solutions-2014-1st-ed.pdf · Exam MFE / 3F Actuarial Models Financial Economics Segment Solutions

Exam MFE/3F Solutions Chapter 3 – Binomial Trees: Part I

© ActuarialBrew.com 2014 Page 3.23

The stock price tree and the call price tree are below:

Stock Call 65.3976 5.3976

53.0000 2.5431 44.6408 0.0000

The value of delta is:

0.045(0.5) 5.3976 0.00000.254

( ) 65.3976 44.6408d- -- -D = = =

- -h u dV V

e eS u d

Solution 3.32

B Put-Call Parity

This problem is fairly easy if we use put-call parity.

The value of h is 1/12 since the intervals are monthly periods. The values of u and d are:

( ) (0.07 0.02)(1 /12) 0.35 1 /12

( ) (0.07 0.02)(1 /12) 0.35 1 /12

1.11094

0.90767

r h h

r h h

u e e

d e e

The risk-neutral probability of an upward movement is:

( ) (0.07 0.02)(1 /12) 0.90767

* 0.474761.11094 0.90767

d- -- -= = =- -

r he d ep

u d

If the stock price increases each month, then at the end of 4 months, the price is:

4 4130 130(1.11094) 198.0157u

Since the other possible prices at the end of 4 months will also be below $200, all of the nodes are in-the-money.

But none of the nodes are in-the-money for the corresponding European call option. This means that the value of the corresponding European call option is zero.

We can use put-call parity to find the value of the corresponding put option:

00.07(4 /12) 0.02(4 /12)

( , ) ( , )

0 200 130 ( , )

( , ) 66.25

d- -

- -

+ = +

+ = +=

rT TEur Eur

Eur

Eur

C K T Ke S e P K T

e e P K T

P K T

Page 68: ActuarialBrew.com Exam MFE / 3F Actuarial Models …actuarialbrew.com/data/documents/MFE-Solutions-2014-1st-ed.pdf · Exam MFE / 3F Actuarial Models Financial Economics Segment Solutions

Exam MFE/3F Solutions Chapter 3 – Binomial Trees: Part I

© ActuarialBrew.com 2014 Page 3.24

Solution 3.33

A Jarrow and Rudd Binomial Tree

The values of u and d are:

2 2

2 2

( 0.5 ) (0.05 0.01 0.5 0.35 )(0.25) 0.35 0.25

( 0.5 ) (0.05 0.01 0.5 0.35 )(0.25) 0.35 0.25

1.18493

0.83501

d s s

d s s

- - + - - ¥ +

- - - - - ¥ -

= = =

= = =

r h h

r h h

u e e

d e e

The stock price tree, the put payoffs and the risk-neutral probability of each payoff are below:

Put Payoff

# of Paths

Probability

Stock 98.5706 0.0000 1 0.06261 83.1865 70.2035 69.4615 0.0000 4 0.25022 59.2467 58.6206

50.0000 49.4716 48.9487 0.0000 6 0.37500 41.7505 41.3092 34.8620 34.4936 0.0000 4 0.24978 29.1101 24.3072 9.6928 1 0.06239

Although we show the entire table above, there is no need to calculate the stock prices higher than $34.4936 to answer this question. At those prices, the option is clearly out-of-the-money.

Only one of the final stock prices is below the strike price of $34. The payoff if the lowest stock price is reached is:

34 24.3072 9.6928- =

The risk-neutral probability of an upward movement is:

( ) (0.05 0.01)(0.25) 0.83501

* 0.500221.18493 0.83501

d- -- -= = =- -

r he d ep

u d

The probability of 4 downward movements is:

4 4(1 *) (1 0.50022) 0.06239p

Since the option is a European option, we can find its value directly as the present value of its expected value:

( ) ( )

0 00

0.05

( , ,0) ( *) (1 *) ( , , )

0.06239 9.6928 0.5752

- - -

=-

È ˘Ê ˆ= -Í ˙Á ˜Ë ¯Î ˚

= ¥ ¥ =

Ân

r hn n i i n i i

i

nV S K e p p V S u d K hn

i

e

Page 69: ActuarialBrew.com Exam MFE / 3F Actuarial Models …actuarialbrew.com/data/documents/MFE-Solutions-2014-1st-ed.pdf · Exam MFE / 3F Actuarial Models Financial Economics Segment Solutions

Exam MFE/3F Solutions Chapter 3 – Binomial Trees: Part I

© ActuarialBrew.com 2014 Page 3.25

Solution 3.34

C Replication

The values of u and d are:

( ) (0.07 0.02)(1 /12) 0.35 1 /12

( ) (0.07 0.02)(1 /12) 0.35 1 /12

1.11094

0.90767

r h h

r h h

u e e

d e e

The risk-neutral probability of an upward movement is:

( ) (0.07 0.02)(1 /12) 0.90767

* 0.474761.11094 0.90767

d- -- -= = =- -

r he d ep

u d

If the stock price increases during each of the first 3 months, then the price is:

3 3130 130(1.11094) 178.2422= =u

The relevant portions of the stock price tree and the option price tree are shown below:

Stock European Put 198.0157 1.9843

178.2422 — 161.786 38.2141

The amount that must be lent at the risk-free rate of return is:

0.07(1 /12) 1.11094(38.2141) 0.90767(1.9843)198.84

1.11094 0.90767- -- -= = =

- -rh d uuV dV

B e eu d

Solution 3.35

D J-R and CRR Binomial Trees

In the J-R model, the values of u and d are:

d s s

d s s

- - + - - ¥ + +

- - - - - ¥ - -

= = =

= = =

2 2

2 2

( 0.5 ) (0.07 0.025 0.5 0.30 )(0.25) 0.30 0.25 0 0.30 0.25

( 0.5 ) (0.07 0.025 0.5 0.30 )(0.25) 0.30 0.25 0 0.30 0.25

r h h

r h h

u e e e

d e e e

In the CRR model, the values of u and d are:

s

s- -

= =

= =

0.30 0.25

0.30 0.25

h

h

u e e

d e e

Page 70: ActuarialBrew.com Exam MFE / 3F Actuarial Models …actuarialbrew.com/data/documents/MFE-Solutions-2014-1st-ed.pdf · Exam MFE / 3F Actuarial Models Financial Economics Segment Solutions

Exam MFE/3F Solutions Chapter 3 – Binomial Trees: Part I

© ActuarialBrew.com 2014 Page 3.26

Since this J-R model’s up and down factors are the same as this CRR model’s up and down factors, Ann and Betsy are using the same binomial model. Therefore, the prices they produce satisfy put-call parity:

d- -

- ¥ - ¥

+ = +

+ = +- =

00.07 1 0.025 150 502.1458

rT TEur EurC Ke S e P

A e e BA B

Solution 3.36

B Alternative Binomial Trees

If the option has a positive payoff in both the up and down states, then:

0.11 0.25 (38 ) (38 )0.9729 0.9729

( )h u dV V Su Sd Sd Su

e eS u d Su Sd Su Sd

d- - ¥- - - - -D = = = = -- - -

But D = -0.262 , so it must be the case that = 0uV .

Since we are given the values of D and B, we can solve for u and d:

d- - ¥

- - ¥

- - - -- = D = = =- - -- - - ¥ -= = = =- - -

0.11 0.25

0.15 0.25

0 (38 40 ) 40 380.262 0.9729

( ) 40 40 40( )

(38 40 ) 0 (38 40 )11.76 0.9632

h u d

rh d u

V V d de e

S u d u d u duV dV u d d u d

B e eu d u d u d

Dividing the first equation by the second allows us to solve for u:

40 380.9729

0.262 40( )(38 40 )11.76 0.9632

10.02228 0.02525

1.1334

du d

u du d

uu

Now we can find the value of d:

-- =--- =

-- + = -=

40 380.262 0.9729

40( )40 38

0.262 0.972940(1.1334 )

12.2094 10.7722 40 380.8824

du d

dd

d dd

Page 71: ActuarialBrew.com Exam MFE / 3F Actuarial Models …actuarialbrew.com/data/documents/MFE-Solutions-2014-1st-ed.pdf · Exam MFE / 3F Actuarial Models Financial Economics Segment Solutions

Exam MFE/3F Solutions Chapter 3 – Binomial Trees: Part I

© ActuarialBrew.com 2014 Page 3.27

Both the Cox-Ross-Rubinestein model and the Jarrow-Rudd model have the same ratio of u to d:

s

s

s

=

=

=

2

2 0.251.13340.8824

0.2503

hue

d

e

Solution 3.37

B Arbitrage

The Cox-Ross-Rubinstein model is:

s

s-

=

=

h

h

u e

d e

The Jarrow-Rudd model is:

2

2

( 0.5 )

( 0.5 )

d s s

d s s

- - +

- - -

=

=

r h h

r h h

u e

d e

From Chapter 10, we know that to avoid arbitrage, a model must satisfy the following:

( )d-< <r hd e u

The values of d, ( )d-r he , and u are calculated for each model below:

Model r d s h d ( )d-r he u Type

A 0.25 0.01 0.30 0.25 0.8607 1.0618 1.1618 CRR B 0.21 0.00 0.20 1.00 0.8187 1.2337 1.2214 CRR C 0.14 0.03 0.10 0.50 0.9317 1.0565 1.0733 CRR D 0.14 0.05 0.50 0.50 0.6900 1.0460 1.3994 J-R E 0.07 0.00 0.10 1.00 0.9656 1.0725 1.1794 J-R

Model B violates the restriction because it has the risk-free asset earning more than the risky asset after an upward move:

1.2337 1.2214>

Page 72: ActuarialBrew.com Exam MFE / 3F Actuarial Models …actuarialbrew.com/data/documents/MFE-Solutions-2014-1st-ed.pdf · Exam MFE / 3F Actuarial Models Financial Economics Segment Solutions

Exam MFE/3F Solutions Chapter 3 – Binomial Trees: Part I

© ActuarialBrew.com 2014 Page 3.28

Solution 3.38

B Realistic Probability

We can solve for p, the true probability of the stock price going up, using the following formula:

a d- - ¥- -= = =

- -

( ) (0.10 0) 1 0.7630.47923

1.477 0.763

he d ep

u d

Solution 3.39

D Risk-Neutral Probability

The values of u and d are:

( ) (0.085 0.022)(0.25) 0.28 0.25

( ) (0.085 0.022)(0.25) 0.28 0.25

1.16853

0.88316

r h h

r h h

u e e

d e e

The risk-neutral probability of an upward movement is:

( ) (0.085 0.022)(0.25) 0.88316

* 0.465061.16853 0.88316

r he d ep

u d

d- -- -= = =- -

The risk-neutral probability of a downward movement is:

(1 *) 1 0.46506 0.53494p- = - =

Solution 3.40

D Replication

The call option pays 11 in the up-state and 0 in the down-state:

[38 27,0] 11

[21 27,0] 0u

d

V Max

V Max

The dividend rate is zero, so the number of shares needed to replicate the option is:

0 1 11 0

0.6471( ) 38 21

h hu d u dV V V Ve e e

S u d Su Sd

Solution 3.41

B One-Period Binomial Tree

A straddle consists of a long call and a long put on the same stock, where both options have the same strike price and the same expiration date. The question describes the payoff of a straddle, so it is not necessary to know the definition of a straddle to answer this question.

Page 73: ActuarialBrew.com Exam MFE / 3F Actuarial Models …actuarialbrew.com/data/documents/MFE-Solutions-2014-1st-ed.pdf · Exam MFE / 3F Actuarial Models Financial Economics Segment Solutions

Exam MFE/3F Solutions Chapter 3 – Binomial Trees: Part I

© ActuarialBrew.com 2014 Page 3.29

If the stock price moves up, then the straddle pays $25. If the stock price moves down, then the straddle pays $2:

95 25 = -70 95

75 V 68 2 = -70 68

In this case, 95 /75u and 68 /75d .

The risk-neutral probability of an upward movement is:

( ) (0.07 0.00)1 68 /75* 0.46067

95 /75 68 /75

r he d ep

u d

The value of the straddle is:

[ ]- -= + - = + -È ˘Î ˚=

0.07(1)( *) (1 *) 0.46067(25) (1 0.46067)(2)

11.74

rhu dV e p V p V e

Solution 3.42

C Replication

The end-of-year payoffs of the call and put options in each scenario are shown in the table below. The rightmost column is the payoff resulting from buying the put option and selling the call option.

End of Year Price of Stock A

End of Year Price of Stock B

( )C A Payoff

( )P B Payoff

-( ) ( )P B C A Payoff

Scenario 1 $80 $0 45 35 –10

Scenario 2 $30 $0 0 35 35

Scenario 3 $0 $400 0 0 0

We need to determine the cost of replicating the payoffs in the rightmost column above. We can replicate those payoffs by determining the proper amount of Stock A, Stock B, and the risk-free asset to purchase.

Let’s define the following variables:

===

Number of shares of Stock A to purchaseNumber of shares of Stock B to purchase Amount to lend at the risk-free rate

ABC

Page 74: ActuarialBrew.com Exam MFE / 3F Actuarial Models …actuarialbrew.com/data/documents/MFE-Solutions-2014-1st-ed.pdf · Exam MFE / 3F Actuarial Models Financial Economics Segment Solutions

Exam MFE/3F Solutions Chapter 3 – Binomial Trees: Part I

© ActuarialBrew.com 2014 Page 3.30

We have 3 equations and 3 unknown variables:

+ + = -

+ + =

+ =

0.08

0.08

0.08

Scenario 1: 80 0 10

Scenario 2: 30 0 35

Scenario 3: 400 0

A B Ce

A B Ce

B Ce

Subtracting the second equation from the first equation allows us to find A:

= -= -

50 450.9

AA

We can put this value of A into the first equation to find the value of C:

- + = -

==

0.08

0.08

80( 0.9) 10

6257.2332

Ce

CeC

We can use this value of C in the third equation to find the value of B:

+ == -

0.08400 57.2332 00.155

B eB

The cost now of replicating the payoffs resulting from buying the put and selling the call is equal to the cost of establishing a position consisting of A shares of Stock A, B shares of Stock B, and C lent at the risk-free rate:

+ + = ¥ - + ¥ - + =50 50 50 ( 0.9) 50 ( 0.155) 57.2332 4.48A B C

Solution 3.43

D Replication

The end-of-year payoffs of the call and put options in each scenario are shown in the table below. The rightmost column is the payoff resulting from buying the call option and selling the put option.

End of Year Price of Stock A

End of Year Price of Stock B

(45)AP

Payoff

(40)BC

Payoff

-(40) (45)B AC P

Payoff

Scenario 1 $30 $0 15 0 –15

Scenario 2 $40 $50 5 10 5

Scenario 3 $50 $40 0 0 0

We need to determine the cost of replicating the payoffs in the rightmost column above. We can replicate those payoffs by determining the proper amount of Stock A, Stock B, and the risk-free asset to purchase.

Page 75: ActuarialBrew.com Exam MFE / 3F Actuarial Models …actuarialbrew.com/data/documents/MFE-Solutions-2014-1st-ed.pdf · Exam MFE / 3F Actuarial Models Financial Economics Segment Solutions

Exam MFE/3F Solutions Chapter 3 – Binomial Trees: Part I

© ActuarialBrew.com 2014 Page 3.31

Let’s define the following variables:

===

Number of shares of Stock A to purchase

Number of shares of Stock B to purchase Amount to lend at the risk-free rate

A

BC

We have 3 equations and 3 unknown variables:

+ + = -

+ + =

+ + =

0.05

0.05

0.05

Scenario 1: 30 0 15

Scenario 2: 40 50 5

Scenario 3: 50 40 0

A B Ce

A B Ce

A B Ce

The two equations below are obtained by subtracting the first equation above from the second and third equations above:

+ =+ =

10 50 2020 40 15

A BA B

We can solve for B by subtracting twice the first equation above from the second equation:

- = -

=

60 255

12

B

B

We can put this value of B into one of the two equations found earlier in order to find the value of A:

+ =

+ ¥ =

= -

10 50 205

10 50 2012

112

A B

A

A

We can now find the value of C using one of the 3 original equations:

+ + = --¥ + + = -

= -

0.05

0.05

30 0 151

30 0 151211.89

A B Ce

Ce

C

Since C is negative, the payoffs resulting from buying the call and selling the put are replicated by borrowing $11.89, which is equivalent to lending –$11.89.

The cost now of replicating the payoffs resulting from buying the call and selling the put is equal to the cost of establishing a position consisting of A shares of Stock A, B shares of Stock B, and C lent at the risk-free rate. Since the price of Stock A is $40 and the price of Stock B is $30, we have:

-+ + = ¥ + ¥ - = -1 5

40 30 40 30 11.89 2.7212 12

A B C

Page 76: ActuarialBrew.com Exam MFE / 3F Actuarial Models …actuarialbrew.com/data/documents/MFE-Solutions-2014-1st-ed.pdf · Exam MFE / 3F Actuarial Models Financial Economics Segment Solutions

Exam MFE/3F Solutions Chapter 3 – Binomial Trees: Part I

© ActuarialBrew.com 2014 Page 3.32

Solution 3.44

E Replication

The risk-free asset can be replicated with the three stocks. Let’s find the cost of replicating an asset that is certain to pay a fixed amount at time 1. We can choose any fixed amount, and in this solution we use $100. We begin by defining the following variables:

===

Number of shares of Stock A to purchaseNumber of shares of Stock B to purchaseNumber of shares of Stock C to purchase

ABC

We have 3 equations and 3 unknown variables. The $100 on the right side of the equations below is the fixed amount certain to be paid by our risk-free asset:

+ + =

+ + =+ + =

Scenario 1: 200 0 0 100

Scenario 2: 50 0 100 100Scenario 3: 0 300 50 100

A B C

A B CA B C

The solution to these equations is:

==

=

0.50.755

24

AC

B

The cost now of replicating an asset that is certain to pay $100 at time 1 is equal to the cost of establishing a position consisting of A shares of Stock A, B shares of Stock B, and C shares of Stock C. Since the price of Stock A is $100, the price of Stock B is $50, and the price of Stock C is $40, the cost of the risk-free asset is:

+ + = ¥ + ¥ + ¥ =5100 50 40 100 0.5 50 40 0.75 90.4167

24A B C

Since t is one year, we can now solve for r:

==

90.4167 1000.1007

rter

Page 77: ActuarialBrew.com Exam MFE / 3F Actuarial Models …actuarialbrew.com/data/documents/MFE-Solutions-2014-1st-ed.pdf · Exam MFE / 3F Actuarial Models Financial Economics Segment Solutions

Exam MFE/3F Solutions Chapter 3 – Binomial Trees: Part I

© ActuarialBrew.com 2014 Page 3.33

Solution 3.45

E Replication

The end-of-year payoffs of the call and put options in each scenario are shown in the table below. The rightmost column is the payoff resulting from buying the put option and selling the call option.

Scenario

End of Year Price of Stock

A

End of Year Price of Stock B

(100)AP

Payoff

(200)BC

Payoff

-(100) (200)A BP C

Payoff

1 $200 $0 0 0 0

2 $50 $0 50 0 50

3 $25 $300 75 100 –25

We need to determine the cost of replicating the payoffs in the rightmost column above. We can replicate those payoffs by determining the proper amount of Stock A, Stock B, and the risk-free asset to purchase.

Let’s define the following variables:

===

Number of shares of Stock A to purchase

Number of shares of Stock B to purchase Amount to lend at the risk-free rate

A

BC

Since Stock A pays a $10 dividend at time 0.5, each share of Stock A that is purchased provides its holder with the final price of Stock A at time 1 plus the accumulated value of $10. Since Stock B pays continuously compounded dividends of 5%, each share of stock B

purchased at time 0 grows to 0.05e shares of Stock B at time 1.

We have 3 equations and 3 unknown variables:

È ˘+ + + =Î ˚È ˘+ + + =Î ˚È ˘+ + + = -Î ˚

0.08(0.5) 0.08

0.08(0.5) 0.08

0.08(0.5) 0.05 0.08

Scenario 1: 200 10 0 0

Scenario 2: 50 10 0 50

Scenario 3: 25 10 300 25

e A B Ce

e A B Ce

e A e B Ce

Subtracting the first equation from the second equation allows us to solve for A:

È ˘ È ˘+ - + =Î ˚ Î ˚

= = -+ - -

0.08(0.5) 0.08(0.5)

0.04 0.04

50 10 200 10 50

50 1350 10 200 10

e A e A

Ae e

Page 78: ActuarialBrew.com Exam MFE / 3F Actuarial Models …actuarialbrew.com/data/documents/MFE-Solutions-2014-1st-ed.pdf · Exam MFE / 3F Actuarial Models Financial Economics Segment Solutions

Exam MFE/3F Solutions Chapter 3 – Binomial Trees: Part I

© ActuarialBrew.com 2014 Page 3.34

The equation associated with Scenario 1 can now be used to find C:

0.08(0.5) 0.08

0.08(0.5) 0.08 0.08(0.5) 0.08

200 10 0 0

1200 10 200 10 64.7437

3

e A B Ce

C e Ae e e

We use the equation associated with Scenario 3 to find B:

È ˘+ + + = -Î ˚

È ˘- + + + = -Î ˚= -

0.08(0.5) 0.05 0.08

0.08(0.5) 0.05 0.08

25 10 300 25

125 10 300 64.7437 25

30.2642

e A e B Ce

e e B e

B

The cost now of replicating the payoffs resulting from buying the put and selling the call is equal to the cost of establishing a position consisting of A shares of Stock A, B shares of Stock B, and C lent at the risk-free rate. Since the price of Stock A is $100 and the price of Stock B is $75, we have:

-+ + = ¥ + ¥ - + =1

100 75 100 75 ( 0.2642) 64.7437 11.59313

A B C

Solution 3.46

A Delta

The delta of the put option is –0.3275. The delta of the corresponding call option satisfies the following formula:

d-

-

D - D =

D - - =D =

0.07(0.75)( 0.3275)

0.6214

TCall Put

Call

Call

e

e

Solution 3.47

A Replication

Since we are given the value of the put option and the delta of the put option, we can find the amount that must be lent to replicate the put option:

= D += - +=

0

3.89 52( 0.3275)

20.92

Put Put Put

Put

Put

V S B

B

B

Page 79: ActuarialBrew.com Exam MFE / 3F Actuarial Models …actuarialbrew.com/data/documents/MFE-Solutions-2014-1st-ed.pdf · Exam MFE / 3F Actuarial Models Financial Economics Segment Solutions

Exam MFE/3F Solutions Chapter 3 – Binomial Trees: Part I

© ActuarialBrew.com 2014 Page 3.35

The put option is replicated by lending $20.92, so = 20.92X . We can now solve for CallB :

-

-

- =

- == -

0.10(0.75)20.92 50

25.47

rTPut Call

Call

Call

B B Ke

B e

B

Therefore $25.47 must be borrowed to replicate the call option.

Solution 3.48

B Arbitrage in the Binomial Model

If the stock price moves up, then the option pays Max[0, 110 – 100] = $10. If the stock price moves down, then the option pays Max[0, 80 – 100] = $0:

110 10 100 V

80 0

Let’s use the replication method to find the value of the option:

0.10(1)

0.04(1)

10 00.3016

( ) 110 80110(0) 80(10)

25.6211110 80

h u d

rh rhd u d u

V Ve e

S u duV dV SuV SdV

B e e eu d Su Sd

If no arbitrage is available, then the price of the option is:

= D + = - =0 100(0.3016) 25.6211 4.5402V S B

The actual option price of $4.00 is less than $4.5402, so the option’s price is too low. Arbitrage is obtained by buying the option (buy low) and also replicating the sale of the option. The sale of the option is replicated by selling D shares and borrowing B. In this case, B is negative, so borrowing B is equivalent to lending 25.6211.

To summarize, arbitrage is obtained by:

buying the option for $4.00

selling 0.3016 shares of stock

lending $25.6211 at the risk-free rate.

These actions are described in Choice B.

Page 80: ActuarialBrew.com Exam MFE / 3F Actuarial Models …actuarialbrew.com/data/documents/MFE-Solutions-2014-1st-ed.pdf · Exam MFE / 3F Actuarial Models Financial Economics Segment Solutions

Exam MFE/3F Solutions Chapter 3 – Binomial Trees: Part I

© ActuarialBrew.com 2014 Page 3.36

This page is intentionally blank.

Page 81: ActuarialBrew.com Exam MFE / 3F Actuarial Models …actuarialbrew.com/data/documents/MFE-Solutions-2014-1st-ed.pdf · Exam MFE / 3F Actuarial Models Financial Economics Segment Solutions

Exam MFE/3F Solutions Chapter 4 – Binomial Trees: Part II

© ActuarialBrew.com 2014 Page 4.01

Chapter 4 – Solutions

Solution 4.01

C State Prices

The question tells us that the stock price might decrease to $X, which implies that:

< 100X

The stock price tree and the call option tree are:

Stock Call 150.0000 50.0000

100.0000 23.7200 X 0.0000

We can use the price of the call option to obtain the state price associated with an upward movement:

=

=23.72 50

0.4744u

u

Q

Q

The risk-free rate of return can now be used to obtain the state price associated with a downward movement:

-

- ¥

= +

= +=

0.10 1 0.4744

0.4304

rhu d

d

d

e Q Q

e Q

Q

We can use the stock’s price and the state prices to obtain the value of X:

= += ¥ + ¥

=

100 150

100 150 0.4744 0.430467.0016

u dQ XQ

XX

The strike price of the put option is:

= = ¥ =1.6 1.6 67.0016 107.2026K X

The put option pays off only if the stock’s price falls to 67.0016, so the current value of the put option is:

+ - = + ¥ =0 (107.2026 67.0016) 0 40.2010 0.4304 17.3040u dQ Q

Alternate Solution

We don’t have to use state prices to answer this question.

Page 82: ActuarialBrew.com Exam MFE / 3F Actuarial Models …actuarialbrew.com/data/documents/MFE-Solutions-2014-1st-ed.pdf · Exam MFE / 3F Actuarial Models Financial Economics Segment Solutions

Exam MFE/3F Solutions Chapter 4 – Binomial Trees: Part II

© ActuarialBrew.com 2014 Page 4.02

Since the call option has a payoff of 50 if the stock price increases and 0 if the stock price falls, we can use the call price to obtain the risk-neutral probability of an upward movement:

[ ]-= + -=

0.1023.72 50 * 0(1 *)

* 0.5243

e p p

p

We can now use the current stock price to solve for X:

-

-

= + -

= ¥ + -=

0.10

0.10

100 [150 * (1 *)]

100 [150 0.5243 (1 0.5243)]67.0016

e p X p

e XX

The strike price of the put option is:

= = ¥ =1.6 1.6 67.0016 107.2026K X

The put option pays off only if the stock price falls to 67.0016, so the current value of the put option is:

[ ]- ¥ + - - =0.10 0 0.5243 (107.2026 67.0016)(1 0.5243) 17.3040e

Solution 4.02

B Two-Period Binomial Tree

The values of u and d are:

( ) (0.08 0.03)(1) 0.23 1

( ) (0.08 0.03)(1) 0.23 1

1.32313

0.83527

d s

d s

- + - +

- - - -

= = =

= = =

r h h

r h h

u e e

d e e

The risk-neutral probability of an upward movement is:

0.23 1

1 1* 0.44275

1 1hp

e es= = =

+ +

The stock price tree and its corresponding tree of option prices are:

Stock American Call 126.0484 52.0484 95.2653 24.1392

72.0000 79.5723 11.0375 5.5723 60.1395 2.2775 50.2327 0.0000

Page 83: ActuarialBrew.com Exam MFE / 3F Actuarial Models …actuarialbrew.com/data/documents/MFE-Solutions-2014-1st-ed.pdf · Exam MFE / 3F Actuarial Models Financial Economics Segment Solutions

Exam MFE/3F Solutions Chapter 4 – Binomial Trees: Part II

© ActuarialBrew.com 2014 Page 4.03

The tree of prices for the call option is found by working from right to left. The rightmost column is found as follows:

[ ][ ][ ]

0, 126.0484 74 52.0484

0, 79.5723 74 5.5723

0, 50.232 74 0.0000

- =

- =

- =

Max

Max

Max

The prices after 1 year are found using the risk-neutral probabilities:

[ ][ ]

0.08(1)

0.08(1)

(0.44275)(52.0484) (1 0.44275)(5.5723) 24.1392

(0.44275)(5.5723) (1 0.44275)(0.0000) 2.2775

-

-

+ - =

+ - =

e

e

There is only one opportunity to call the option early, and this occurs after 1 year if the stock price has risen. Early exercise is not optimal since:

95.2653 74 21.2653 and 21.2653 24.1392- = <

The current price of the option is:

[ ]0.08(1) (0.44275)(24.1392) (1 0.44275)(2.2775) 11.0375- + - =e

Solution 4.03

D Two-Period Binomial Tree

The values of u and d are:

( ) (0.08 0.03)(1) 0.23 1

( ) (0.08 0.03)(1) 0.23 1

1.32313

0.83527

d s

d s

- + - +

- - - -

= = =

= = =

r h h

r h h

u e e

d e e

The risk-neutral probability of an upward movement is:

0.23 1

1 1* 0.44275

1 1hp

e es= = =

+ +

The stock price tree and its corresponding tree of option prices are:

Stock American Call 126.0484 52.0484 95.2653 24.1392

72.0000 79.5723 11.0375 5.5723 60.1395 2.2775 50.2327 0.0000

We are told that after 1 year, the stock price has increased. Therefore, the new stock price is $95.2653. At this point the value of delta is:

0.03(1) 52.0484 5.57230.9704

( ) 126.0484 79.5723d- -- -D = = =

- -h u dV V

e eS u d

Page 84: ActuarialBrew.com Exam MFE / 3F Actuarial Models …actuarialbrew.com/data/documents/MFE-Solutions-2014-1st-ed.pdf · Exam MFE / 3F Actuarial Models Financial Economics Segment Solutions

Exam MFE/3F Solutions Chapter 4 – Binomial Trees: Part II

© ActuarialBrew.com 2014 Page 4.04

Therefore, the investor must hold 0.9704 shares at the end of 1 year in order to replicate the call option.

Solution 4.04

B Expected Return

The realistic probability of an up jump is constant throughout the tree. The expected return on the stock is also constant throughout the tree. The return on the option, however, can change from one node to the next.

The values of u and d are:

( ) (0.06 0.05)(0.5) 0.30 0.5

( ) (0.06 0.05)(0.5) 0.30 0.5

1.24251

0.81291

d s

d s

- + - +

- - - -

= = =

= = =

r h h

r h h

u e e

d e e

The risk-neutral probability of an upward movement is:

( ) (0.06 0.05)(0.5) 0.81291

* 0.447161.24251 0.81291

d- -- -= = =- -

r he d ep

u d

The stock price tree and the call price tree are shown below:

Stock European Call 77.1913 30.1913 62.1254 14.9806

50.0000 50.5025 7.3162 3.5025 40.6456 1.5199 33.0413 0.0000

Although the entire tree is shown above, it is not necessary to calculate the values along the lowest path in order to answer this problem.

The realistic probability of an up jump is:

a d- -- -= = =

- -

( ) (0.10 0.05)(0.5) 0.812910.49442

1.24251 0.81291

he d ep

u d

If the stock price increases during the first 6 months, then the expected rate of return during the second 6 months must satisfy the following equation:

[ ]0.5

( ) (1 )

14.9806 0.49442 30.1913 (1 0.49442) 3.5025

0.49442 30.1913 (1 0.49442) 3.5025 1ln 0.21708

14.9806 0.5

g

g

g

-

-

= + -È ˘Î ˚

= ¥ + - ¥

¥ + - ¥Ê ˆ= ¥ =Á ˜Ë ¯

hu dV e p V p V

e

Page 85: ActuarialBrew.com Exam MFE / 3F Actuarial Models …actuarialbrew.com/data/documents/MFE-Solutions-2014-1st-ed.pdf · Exam MFE / 3F Actuarial Models Financial Economics Segment Solutions

Exam MFE/3F Solutions Chapter 4 – Binomial Trees: Part II

© ActuarialBrew.com 2014 Page 4.05

Solution 4.05

B Two-Period Binomial Tree

The values of u and d are:

( ) (0.08 0.03)(1) 0.23 1

( ) (0.08 0.03)(1) 0.23 1

1.32313

0.83527

d s

d s

- + - +

- - - -

= = =

= = =

r h h

r h h

u e e

d e e

The risk-neutral probability of an upward movement is:

0.23 1

1 1* 0.44275

1 1hp

e es= = =

+ +

The stock price tree and its corresponding tree of option prices are:

Stock European Put 126.0484 0.0000 95.2653 0.0000

72.0000 79.5723 6.2891 0.0000 60.1395 12.2260 50.2327 23.7673

The put option can be exercised only if the stock price falls to $50.2327. At that point, the put option has a payoff of $23.7673. Although the entire table of put prices is filled in above, we can directly find the value of the put option as follows:

( )0 0

0

0.08(1)(2) 2

( , ,0) ( *) (1 *) ( , , )

(1 0.44275) 23.7673 0 0 6.289

- - -

=

-

È ˘Ê ˆ= -Í ˙Á ˜Ë ¯Î ˚

È ˘= - + + =Î ˚

Ân

r hn j n j j n j

j

nV S K e p p V S u d K hn

j

e

Solution 4.06

C Expected Return

The values of u and d are:

( ) (0.06 0.05)(0.5) 0.30 0.5

( ) (0.06 0.05)(0.5) 0.30 0.5

1.24251

0.81291

d s

d s

- + - +

- - - -

= = =

= = =

r h h

r h h

u e e

d e e

The risk-neutral probability of an upward movement is:

( ) (0.06 0.05)(0.5) 0.81291

* 0.447161.24251 0.81291

d- -- -= = =- -

r he d ep

u d

Page 86: ActuarialBrew.com Exam MFE / 3F Actuarial Models …actuarialbrew.com/data/documents/MFE-Solutions-2014-1st-ed.pdf · Exam MFE / 3F Actuarial Models Financial Economics Segment Solutions

Exam MFE/3F Solutions Chapter 4 – Binomial Trees: Part II

© ActuarialBrew.com 2014 Page 4.06

The stock price tree and the call price tree are shown below:

Stock American Put 77.1913 0.0000 62.1254 0.0000

50.0000 50.5025 4.0177 0.0000 40.6456 7.4888 33.0413 13.9587

Although the entire tree is shown above, it is not necessary to calculate the values along the uppermost path in order to answer this problem.

Early exercise of the American put option is never optimal.

The realistic probability of an up-move is:

a d- -- -= = =

- -

( ) (0.10 0.05)(0.5) 0.812910.49442

1.24251 0.81291

he d ep

u d

If the stock price decreases during the first 6 months, then the expected rate of return during the second 6 months must satisfy the following equation:

[ ]0.5

( ) (1 )

7.4888 0.49442 0.0000 (1 0.49442) 13.9587

(1 0.49442) 13.9587 1ln 0.1187

7.4888 0.5

g

g

g

-

-

= + -È ˘Î ˚

= ¥ + - ¥

- ¥Ê ˆ= ¥ = -Á ˜Ë ¯

hu dV e p V p V

e

Solution 4.07

D Two-Period Binomial Tree

The values of u and d are:

( ) (0.08 0.03)(1) 0.23 1

( ) (0.08 0.03)(1) 0.23 1

1.32313

0.83527

d s

d s

- + - +

- - - -

= = =

= = =

r h h

r h h

u e e

d e e

The risk-neutral probability of an upward movement is:

0.23 1

1 1* 0.44275

1 1hp

e es= = =

+ +

The stock price tree and its corresponding tree of option prices are:

Stock American Put 126.0484 0.0000 95.2653 0.0000

72.0000 79.5723 7.1299 0.0000 60.1395 13.8605 50.2327 23.7673

Page 87: ActuarialBrew.com Exam MFE / 3F Actuarial Models …actuarialbrew.com/data/documents/MFE-Solutions-2014-1st-ed.pdf · Exam MFE / 3F Actuarial Models Financial Economics Segment Solutions

Exam MFE/3F Solutions Chapter 4 – Binomial Trees: Part II

© ActuarialBrew.com 2014 Page 4.07

If the stock price initially moves down, then the resulting put price is $13.8605. This price is in bold type above to indicate that it is optimal to exercise early at this node:

74 60.1395 13.8605- =

The exercise value of 13.8605 is greater than the value of holding the option, which is:

[ ]0.08(1) (0.44275)0.0000 (1 0.44275)(23.7673) 12.2260- + - =e

The current value of the American option is:

[ ]0.08(1) (0.44275)0.0000 (1 0.44275)(13.8605) 7.1299- + - =e

Solution 4.08

E Expected Return

The values of u and d are:

( ) (0.11 0.10)(1.0) 0.40 1

( ) (0.11 0.10)(1.0) 0.40 1

1.50682

0.67706

d s

d s

- + - +

- - - -

= = =

= = =

r h h

r h h

u e e

d e e

The risk-neutral probability of an upward movement is:

( ) (0.11 0.10)(1.0) 0.67706

* 0.401311.50682 0.67706

d- -- -= = =- -

r he d ep

u d

The stock price tree and the call price tree are shown below:

Stock American Call 113.5250 73.5250 75.3409 35.3409

50.0000 51.0101 14.8283 11.0101 33.8528 3.9582 22.9203 0.0000

If the stock moves up during the first year, then it is optimal to exercise the American option early. This is indicated above by the bold type for the option price of $35.3409.

The realistic probability of an up-move is:

a d- -- -= = =

- -

( ) (0.24 0.10)(1.0) 0.677060.57031

1.50682 0.67706

he d ep

u d

The expected return on the call option can be calculated using the realistic probability:

[ ](1)

( ) (1 )

14.8283 0.57031 35.3409 (1 0.57031) 3.9582

0.57031 35.3409 (1 0.57031) 3.9582ln 0.3879

14.8283

g

g

g

-

-

= + -È ˘Î ˚

= ¥ + - ¥

¥ + - ¥Ê ˆ= =Á ˜Ë ¯

hu dV e p V p V

e

Page 88: ActuarialBrew.com Exam MFE / 3F Actuarial Models …actuarialbrew.com/data/documents/MFE-Solutions-2014-1st-ed.pdf · Exam MFE / 3F Actuarial Models Financial Economics Segment Solutions

Exam MFE/3F Solutions Chapter 4 – Binomial Trees: Part II

© ActuarialBrew.com 2014 Page 4.08

Solution 4.09

B Three-Period Binomial Tree

The values of h, u and d are:

( ) (0.10 0.08)(0.25) 0.32 0.25

( ) (0.10 0.08)(0.25) 0.32 0.25

9 /12 1/ 3 1 / 4

1.17939

0.85642

r h h

r h h

h

u e e

d e e

The risk-neutral probability of an upward movement is:

( ) (0.10 0.08)(0.25) 0.85642

* 0.460091.17939 0.85642

d- -- -= = =- -

r he d ep

u d

The stock price tree and its corresponding tree of option prices are:

Stock 180.4548 European Call 80.4548 153.0065 52.4458 129.7332 131.0371 30.8676 31.0371

110.0000 111.1055 17.1419 13.9271 94.2057 95.1525 6.2495 0.0000 80.6792 0.0000 69.0949 0.0000

Although we have shown all of the prices in the tree above, we can also solve more directly for the price of the call option (n is 3) as follows:

( )0 0

0

0.10(0.75) 2 3

( , ,0) ( *) (1 *) ( , , )

0 0 3(0.46009) (1 0.46009)(31.0371) (0.46009) (80.4548)

17.14

nr hn j n j j n j

j

nV S K e p p V S u d K hn

j

e

Solution 4.10

A Three-Period Binomial Tree

The values of u and d are:

( ) (0.10 0.08)(0.25) 0.32 0.25

( ) (0.10 0.08)(0.25) 0.32 0.25

1.17939

0.85642

d s

d s

- + - +

- - - -

= = =

= = =

r h h

r h h

u e e

d e e

The risk-neutral probability of an upward movement is:

( ) (0.10 0.08)(0.25) 0.85642

* 0.460091.17939 0.85642

d- -- -= = =- -

r he d ep

u d

Page 89: ActuarialBrew.com Exam MFE / 3F Actuarial Models …actuarialbrew.com/data/documents/MFE-Solutions-2014-1st-ed.pdf · Exam MFE / 3F Actuarial Models Financial Economics Segment Solutions

Exam MFE/3F Solutions Chapter 4 – Binomial Trees: Part II

© ActuarialBrew.com 2014 Page 4.09

The stock price tree and its corresponding tree of option prices are:

Stock 180.4548 American Call 80.4548 153.0065 53.0065 129.7332 131.0371 31.1192 31.0371

110.0000 111.1055 17.2548 13.9271 94.2057 95.1525 6.2495 0.0000 80.6792 0.0000 69.0949 0.0000

At each node, the value of holding the option must be compared with the value obtained by exercising it. If the stock price reaches $153.0065, then it is optimal to exercise the option early because:

0.10(0.25)153.0065 100 (0.46009) 80.4548 (1 0.46009) 31.0371

53.0065 52.4458

e

Working from right to left, we calculate the current value of the option to be $17.25.

Solution 4.11

A Three-Period Binomial Tree

The values of u and d are:

( ) (0.10 0.08)(0.25) 0.32 0.25

( ) (0.10 0.08)(0.25) 0.32 0.25

1.17939

0.85642

d s

d s

- + - +

- - - -

= = =

= = =

r h h

r h h

u e e

d e e

The risk-neutral probability of an upward movement is:

( ) (0.10 0.08)(0.25) 0.85642

* 0.460091.17939 0.85642

d- -- -= = =- -

r he d ep

u d

The stock price tree and its corresponding tree of option prices are:

Stock 180.4548 European Put 0.0000 153.0065 0.0000 129.7332 131.0371 1.3442 0.0000

110.0000 111.1055 6.3222 2.5526 94.2057 95.1525 10.8606 4.8475 80.6792 18.4494 69.0949 30.9051

Page 90: ActuarialBrew.com Exam MFE / 3F Actuarial Models …actuarialbrew.com/data/documents/MFE-Solutions-2014-1st-ed.pdf · Exam MFE / 3F Actuarial Models Financial Economics Segment Solutions

Exam MFE/3F Solutions Chapter 4 – Binomial Trees: Part II

© ActuarialBrew.com 2014 Page 4.10

Although we have shown all of the prices in the trees above, we can also solve more directly for the price of the European put option as follows:

( )0 0

0

0.10(0.75) 3 2

( , ,0) ( *) (1 *) ( , , )

(1 0.46009) (30.9051) 3(0.46009)(1 0.46009) (4.8475) 0 0

6.3222

nr hn j n j j n j

j

nV S K e p p V S u d K hn

j

e

Solution 4.12

C Three-Period Binomial Tree

The values of u and d are:

( ) (0.10 0.08)(0.25) 0.32 0.25

( ) (0.10 0.08)(0.25) 0.32 0.25

1.17939

0.85642

d s

d s

- + - +

- - - -

= = =

= = =

r h h

r h h

u e e

d e e

The risk-neutral probability of an upward movement is:

( ) (0.10 0.08)(0.25) 0.85642

* 0.460091.17939 0.85642

d- -- -= = =- -

r he d ep

u d

The stock price tree and its corresponding tree of option prices are:

Stock 180.4548 American Put 0.0000 153.0065 0.0000 129.7332 131.0371 1.3442 0.0000

110.0000 111.1055 6.5638 2.5526 94.2057 95.1525 11.3195 4.8475 80.6792 19.3208 69.0949 30.9051

At each node, the value of holding the option must be compared with the value obtained by exercising it. If the stock price reaches $80.6792, then it is optimal to exercise the option early because:

0.10(0.25)100 80.6792 (0.46009) 4.8475 (1 0.46009) 30.9051

19.3208 18.4494

e

Working from right to left, we calculate the current value of the option to be $6.5638.

Page 91: ActuarialBrew.com Exam MFE / 3F Actuarial Models …actuarialbrew.com/data/documents/MFE-Solutions-2014-1st-ed.pdf · Exam MFE / 3F Actuarial Models Financial Economics Segment Solutions

Exam MFE/3F Solutions Chapter 4 – Binomial Trees: Part II

© ActuarialBrew.com 2014 Page 4.11

Solution 4.13

C Three-Period Binomial Tree

The values of u and d are:

( ) (0.10 0.08)(0.25) 0.32 0.25

( ) (0.10 0.08)(0.25) 0.32 0.25

1.17939

0.85642

d s

d s

- + - +

- - - -

= = =

= = =

r h h

r h h

u e e

d e e

The risk-neutral probability of an upward movement is:

( ) (0.10 0.08)(0.25) 0.85642

* 0.460091.17939 0.85642

d- -- -= = =- -

r he d ep

u d

The stock price tree and its corresponding tree of option prices are:

Stock — American Put — — — 129.7332 131.0371 — 0.0000

110.0000 111.1055 — — 94.2057 95.1525 — 4.8475 — — — —

We have left much of the tree blank because there is no need to fill in all of the cells to answer this question. The amount that must be lent at the risk-free rate of return is:

0.10(0.25) 1.17939(4.8475) 0.85642(0.0000)17.26

1.17939 0.85642- -- -= = =

- -rh d uuV dV

B e eu d

Solution 4.14

C Three-Period Binomial Tree

The values of u and d are:

( ) (0.04 0.03)(1 / 3) 0.30 1 / 3

( ) (0.04 0.03)(1 / 3) 0.30 1 / 3

1.19308

0.84377

d s

d s

- + - +

- - - -

= = =

= = =

r h h

r h h

u e e

d e e

The risk-neutral probability of an upward movement is:

( ) (0.04 0.03)(1 / 3) 0.84377

* 0.456811.19308 0.84377

d- -- -= = =- -

r he d ep

u d

Page 92: ActuarialBrew.com Exam MFE / 3F Actuarial Models …actuarialbrew.com/data/documents/MFE-Solutions-2014-1st-ed.pdf · Exam MFE / 3F Actuarial Models Financial Economics Segment Solutions

Exam MFE/3F Solutions Chapter 4 – Binomial Trees: Part II

© ActuarialBrew.com 2014 Page 4.12

The stock price tree and its corresponding tree of option prices are below:

Stock 152.8451 American Put 0.0000 128.1096 0.0000 107.3772 108.0955 2.7444 0.0000

90.0000 90.6020 8.7728 5.1201 75.9396 76.4475 14.0593 9.5525 64.0758 21.9242 54.0654 31.9346

At each node, the value of holding the option must be compared with the value obtained by exercising it. If the stock price reaches $64.0758, then it is optimal to exercise the option early because:

0.04(1 / 3)86 64.0758 (0.45681)9.5525 (1 0.45681)31.9346

21.9242 21.4227

e

When the stock price is $75.9396, the value of the put option is:

0.04(1 / 3) (0.45681)5.1201 (1 0.45681)21.9242 14.0593e

Solution 4.15

B Three-Period Binomial Tree

The values of u and d are:

( ) (0.04 0.03)(1 / 3) 0.30 1 / 3

( ) (0.04 0.03)(1 / 3) 0.30 1 / 3

1.19308

0.84377

d s

d s

- + - +

- - - -

= = =

= = =

r h h

r h h

u e e

d e e

The risk-neutral probability of an upward movement is:

( ) (0.04 0.03)(1 / 3) 0.84377

* 0.456811.19308 0.84377

d- -- -= = =- -

r he d ep

u d

The stock price tree and its corresponding tree of option prices are below:

Stock 152.8451 American Put 0.0000 128.1096 0.0000 107.3772 108.0955 2.7444 0.0000

90.0000 90.6020 8.7728 5.1201 75.9396 76.4475 14.0593 9.5525 64.0758 21.9242 54.0654 31.9346

We are told that during the first period, the stock price decreases. Therefore, the new stock price is $75.9396. At this point the value of delta is:

0.03(1 / 3) 5.1201 21.92420.627

( ) 90.6020 64.0758d- -- -D = = = -

- -h u dV V

e eS u d

Page 93: ActuarialBrew.com Exam MFE / 3F Actuarial Models …actuarialbrew.com/data/documents/MFE-Solutions-2014-1st-ed.pdf · Exam MFE / 3F Actuarial Models Financial Economics Segment Solutions

Exam MFE/3F Solutions Chapter 4 – Binomial Trees: Part II

© ActuarialBrew.com 2014 Page 4.13

Solution 4.16

B Four-Period Binomial Tree

The values of u and d are:

( ) (0.07)(0.25) 0.30 0.25

( ) (0.07)(0.25) 0.30 0.25

1.18235

0.87590

d s

d s

- + +

- - -

= = =

= = =

r h h

r h h

u e e

d e e

The risk-neutral probability of an upward movement is:

( ) (0.07)(0.25) 0.87590

* 0.462571.18235 0.87590

d- - -= = =- -

r he d ep

u d

Since the stock does not pay dividends, the price of the American call option is equal to the price of an otherwise equivalent European call option. Therefore, we do not need to calculate the possible values of the stock at times 0.25, 0.50, and 0.75. Furthermore, since the option expires out of the money if the final price is less than $131, it makes sense to begin by calculating the highest possible final prices:

4 4

3 3

90 90(1.18235) 175.8814

90 90(1.18235) (0.87590) 130.2961

= = =

= = =uuuu

uuud

S u

S u d

The portions of the stock price tree that we need are below:

Stock 175.8814 — — 130.2961 — —

90.0000 — — — — — — — —

We did not complete the stock price tree above because the option is in-the-money at the end of the year only if the final stock price is $175.8814. Since it is not in-the-money if the stock price is $130.2961, it clearly will not be in-the-money at any lower stock price. Since the end of the year payoff is zero for all of the nodes except the uppermost one, it is relatively quick to find the value of the option:

( )

0 00

0.07(1) 4

( , ,0) ( *) (1 *) ( , , )

(0.46257) (175.8814 131) 1.9159

nr hn j n j j n j

j

nV S K e p p V S u d K hn

j

e

Page 94: ActuarialBrew.com Exam MFE / 3F Actuarial Models …actuarialbrew.com/data/documents/MFE-Solutions-2014-1st-ed.pdf · Exam MFE / 3F Actuarial Models Financial Economics Segment Solutions

Exam MFE/3F Solutions Chapter 4 – Binomial Trees: Part II

© ActuarialBrew.com 2014 Page 4.14

Solution 4.17

E Three-Period Binomial Tree

The values of h, u and d are provided in the question:

1 / 4 1 / 3 1/12

1.200.90

h

ud

The risk-neutral probability of an upward movement is:

( ) (0.03 0.05)(1 /12) 0.90

* 0.327781.20 0.90

d- -- -= = =- -

r he d ep

u d

The tree of stock prices and the tree of option prices are below:

Stock 69.1200 European Call 36.1200 57.6000 24.4429 48.0000 51.8400 14.7663 18.8400

40.0000 43.2000 7.9074 10.1028 36.0000 38.8800 4.5924 5.8800 32.4000 1.9225 29.1600 0.0000

Although we have shown all of the prices in the trees above, we can also solve more directly for the price of the European option as follows:

( )0 0

0

0.03(3 /12) 2

2 3

( , ,0) ( *) (1 *) ( , , )

[0 3(0.32778)(1 0.32778) (5.88)

3(0.32778) (1 0.32778)(18.84) (0.32778) (36.12)] 7.9074

nr hn j n j j n j

j

nV S K e p p V S u d K hn

j

e

Solution 4.18

D Three-Period Binomial Tree

The values of h, u and d are provided in the question:

1 / 4 1 / 3 1/121.200.90

hud

The risk-neutral probability of an upward movement is:

( ) (0.03 0.05)(1 /12) 0.90

* 0.327781.20 0.90

d- -- -= = =- -

r he d ep

u d

Page 95: ActuarialBrew.com Exam MFE / 3F Actuarial Models …actuarialbrew.com/data/documents/MFE-Solutions-2014-1st-ed.pdf · Exam MFE / 3F Actuarial Models Financial Economics Segment Solutions

Exam MFE/3F Solutions Chapter 4 – Binomial Trees: Part II

© ActuarialBrew.com 2014 Page 4.15

The tree of stock prices and the tree of option prices are below:

Stock 69.1200 American Call 36.1200 57.6000 24.6000 48.0000 51.8400 15.0000 18.8400

40.0000 43.2000 8.0052 10.2000 36.0000 38.8800 4.6242 5.8800 32.4000 1.9225 29.1600 0.0000

Early exercise is optimal at the three nodes shown in bold type. After two up jumps, the option exercise value of 24.60 exceeds the option expected present value of 24.4429, which we calculated in the prior solution. After one up jump and one down jump, the option exercise value of 10.20 exceeds the option expected present value of 10.1028, which we also calculated in the prior solution. After one up jump, the option exercise value of 15.00 exceeds the option expected present value of:

0.03(1 /12) (0.32778)24.6000 (1 0.32778)10.2000 14.8828e

The value of the American call option is:

0.03(1 /12) (0.32778)15.0000 (1 0.32778)4.6242 8.0052e

Solution 4.19

C Option on a Stock Index

The value of h is 1 since the intervals are annual periods. The values of u and d are:

( ) (0.05 0.04)(1) 0.34 1

( ) (0.05 0.04)(1) 0.34 1

1.41907

0.71892

d s

d s

- + - +

- - - -

= = =

= = =

r h h

r h h

u e e

d e e

The risk-neutral probability of an upward movement is:

( ) (0.05 0.04)(1) 0.71892

* 0.415811.41907 0.71892

d- -- -= = =- -

r he d ep

u d

The tree of stock index prices and the tree of option prices are below:

Stock Index 285.7651 European Call 190.7651 201.3753 103.1124 141.9068 144.7735 51.7241 49.7735

100.0000 102.0201 24.7855 19.6869 71.8924 73.3447 7.7868 0.0000 51.6851 0.0000 37.1577 0.0000

Page 96: ActuarialBrew.com Exam MFE / 3F Actuarial Models …actuarialbrew.com/data/documents/MFE-Solutions-2014-1st-ed.pdf · Exam MFE / 3F Actuarial Models Financial Economics Segment Solutions

Exam MFE/3F Solutions Chapter 4 – Binomial Trees: Part II

© ActuarialBrew.com 2014 Page 4.16

Although we have shown all of the prices in the trees above, we can also solve more directly for the price of the European option as follows:

( )0 0

0

0.05(3) 2 3

( , ,0) ( *) (1 *) ( , , )

[0 0 3(0.41581) (1 0.41581)(49.7735) (0.41581) (190.7651)] 24.7855

nr hn j n j j n j

j

nV S K e p p V S u d K hn

j

e

Solution 4.20

A Option on a Stock Index

The value of h is 1 since the intervals are annual periods. The values of u and d are:

( ) (0.05 0.04)(1) 0.34 1

( ) (0.05 0.04)(1) 0.34 1

1.41907

0.71892

d s

d s

- + - +

- - - -

= = =

= = =

r h h

r h h

u e e

d e e

The risk-neutral probability of an upward movement is:

( ) (0.05 0.04)(1) 0.71892

* 0.415811.41907 0.71892

d- -- -= = =- -

r he d ep

u d

The tree of stock index prices and the tree of option prices are below:

Stock Index 285.7651 European Put 0.0000 201.3753 0.0000 141.9068 144.7735 6.6872 0.0000

100.0000 102.0201 17.8608 12.0338 71.8924 73.3447 27.3813 21.6553 51.6851 40.7083 37.1577 57.8423

Although we have shown all of the prices in the trees above, we can also solve more directly for the price of the European option as follows:

( )0 0

0

0.05(3) 3 2

( , ,0) ( *) (1 *) ( , , )

[(1 0.41581) (57.8423) 3(0.41581)(1 0.41581) (21.6553) 0 0] 17.8608

- - -

=-

È ˘Ê ˆ= -Í ˙Á ˜Ë ¯Î ˚

= - + - + +=

Ân

r hn j n j j n j

j

nV S K e p p V S u d K hn

j

e

Solution 4.21

C Utility Values and State Prices

The payoffs of the call option are:

= - == - =

(0,200 130) 70

(0,50 130) 0u

d

V Max

V Max

Page 97: ActuarialBrew.com Exam MFE / 3F Actuarial Models …actuarialbrew.com/data/documents/MFE-Solutions-2014-1st-ed.pdf · Exam MFE / 3F Actuarial Models Financial Economics Segment Solutions

Exam MFE/3F Solutions Chapter 4 – Binomial Trees: Part II

© ActuarialBrew.com 2014 Page 4.17

The price of the call option is:

= + = ¥ + - ¥ = ¥ + ¥ =70 (1 ) 0 0.55(0.9) 70 0.45(1.03) 0 34.65u u d d u dV Q V Q V pU p U

Solution 4.22

A Option on a Stock Index

The values of u and d are:

( ) (0.05 0.04)(1) 0.34 1

( ) (0.05 0.04)(1) 0.34 1

1.41907

0.71892

d s

d s

- + - +

- - - -

= = =

= = =

r h h

r h h

u e e

d e e

The risk-neutral probability of an upward movement is:

( ) (0.05 0.04)(1) 0.71892

* 0.415811.41907 0.71892

d- -- -= = =- -

r he d ep

u d

The tree of stock index prices and the tree of option prices are below:

Stock Index 285.7651 American Put 0.0000 201.3753 0.0000 141.9068 144.7735 6.6872 0.0000

100.0000 102.0201 18.6657 12.0338 71.8924 73.3447 28.8298 21.6553 51.6851 43.3149 37.1577 57.8423

If the stock price reaches $51.6851 after 2 years, then early exercise is optimal since the exercise value of the option exceeds the expected present value of the option at that node.

We have:

0.05 (0.41581)(12.0338) (1 0.41581)(43.3149) 28.8298e

The value of the American put option is:

0.05 (0.41581)(6.6872) (1 0.41581)(28.8298) 18.6657e

Solution 4.23

E Utility Values and State Prices

The payoffs of the call option are:

= - == - =

(0,200 130) 70

(0,50 130) 0u

d

V Max

V Max

The price of the call option is:

= + = ¥ + - ¥ = ¥ + ¥ =70 (1 ) 0 0.55(0.9) 70 0.45(1.03) 0 34.65u u d d u dV Q V Q V pU p U

Page 98: ActuarialBrew.com Exam MFE / 3F Actuarial Models …actuarialbrew.com/data/documents/MFE-Solutions-2014-1st-ed.pdf · Exam MFE / 3F Actuarial Models Financial Economics Segment Solutions

Exam MFE/3F Solutions Chapter 4 – Binomial Trees: Part II

© ActuarialBrew.com 2014 Page 4.18

The expected return on the call option can now be calculated:

g

g

g

+ -+ =

¥ + ¥+ =

=

0.5

0.5

(1 )(1 )

0.55 70 0.45 0(1 )

34.650.2346

u dCall

Call

Call

pV p VV

The payoffs of the put option are:

= - == - =

(0,100 200) 0

(0,100 50) 50u

d

V Max

V Max

The price of the put option is:

= + = ¥ + - ¥ = ¥ + ¥=

0 (1 ) 50 0.55(0.9) 0 0.45(1.03) 50

23.175u u d d u dV Q V Q V pU p U

The expected return on the put option can now be calculated:

g

g

g

+ -+ =

¥ + ¥+ =

= -

0.5

0.5

(1 )(1 )

0.55 0 0.45 50(1 )

23.1750.0574

u dPut

Put

Put

pV p VV

The difference in the expected returns is:

g g- = - - =0.2346 ( 0.0574) 0.2920Call Put

Solution 4.24

E Options on Currencies

The value of h is 1/4 since the intervals are quarterly periods. The values of u and d are:

( ) (0.05 0.09)0.25 0.15 0.25

( ) (0.05 0.09)0.25 0.15 0.25

1.06716

0.91851

s

s

- + - +

- - - -

= = =

= = =

f

f

r r h h

r r h h

u e e

d e e

The risk-neutral probability of an upward movement is:

( ) (0.05 0.09)(0.25) 0.91851

* 0.481261.06716 0.91851

d- -- -= = =- -

r he d ep

u d

Page 99: ActuarialBrew.com Exam MFE / 3F Actuarial Models …actuarialbrew.com/data/documents/MFE-Solutions-2014-1st-ed.pdf · Exam MFE / 3F Actuarial Models Financial Economics Segment Solutions

Exam MFE/3F Solutions Chapter 4 – Binomial Trees: Part II

© ActuarialBrew.com 2014 Page 4.19

The tree of euro prices and the tree of option prices are below:

Euros 1.4584 European Call 0.3584 1.3666 0.2499 1.2806 1.2552 0.1565 0.1552

1.2000 1.1762 0.0924 0.0738 1.1022 1.0804 0.0351 0.0000 1.0124 0.0000 0.9299 0.0000

Although we have shown all of the prices in the trees above, we can also solve more directly for the price of the European option as follows:

( )0 0

0

0.05(0.75) 2 3

( , ,0) ( *) (1 *) ( , , )

[0 0 3(0.48126) (1 0.48126)(0.1552) (0.48126) (0.3584)] 0.0924

- - -

=-

È ˘Ê ˆ= -Í ˙Á ˜Ë ¯Î ˚

= + + - +=

Ân

r hn j n j j n j

j

nV S K e p p V S u d K hn

j

e

Solution 4.25

E Utility Values and State Prices

The price of the stock is:

d d d d

¥ ¥

= + = + -

= ¥ ¥ + ¥ ¥ =0.06 1 0.06 1

(1 )

0.55 0.87 200 0.45 0.98 50 125.0313

h h h hu u d d u u d dS Q S e Q S e pU S e p U S e

e e

The expected return can now be calculated:

d da

a

a

¥ ¥

+ -+ =

¥ + ¥+ =

=

0.06 1 0.06 11

(1 )(1 )

0.55 200 0.45 50(1 )

125.03130.1253

h hh u dpS e p S e

S

e e

Solution 4.26

E Options on Currencies

The value of h is 1/4 since the intervals are quarterly periods. The values of u and d are:

( ) (0.05 0.09)0.25 0.15 0.25

( ) (0.05 0.09)0.25 0.15 0.25

1.06716

0.91851

s

s

- + - +

- - - -

= = =

= = =

f

f

r r h h

r r h h

u e e

d e e

The risk-neutral probability of an upward movement is:

( ) (0.05 0.09)(0.25) 0.91851

* 0.481261.06716 0.91851

d- -- -= = =- -

r he d ep

u d

Page 100: ActuarialBrew.com Exam MFE / 3F Actuarial Models …actuarialbrew.com/data/documents/MFE-Solutions-2014-1st-ed.pdf · Exam MFE / 3F Actuarial Models Financial Economics Segment Solutions

Exam MFE/3F Solutions Chapter 4 – Binomial Trees: Part II

© ActuarialBrew.com 2014 Page 4.20

The tree of euro prices and the tree of option prices are below:

Euros 1.4584 American Call 0.3584 1.3666 0.2666 1.2806 1.2552 0.1806 0.1552

1.2000 1.1762 0.1044 0.0762 1.1022 1.0804 0.0362 0.0000 1.0124 0.0000 0.9299 0.0000

The three nodes that are in bold type in the American call option tree above indicate that early exercise is optimal at those nodes since the option exercise value exceeds the option expected present value at those nodes.

The value of the American call option is:

[ ]0.05(0.25) (0.48126)(0.1806) (1 0.48126)(0.0362) 0.1044- + - =e

Solution 4.27

D Utility Values and State Prices

Although we called the two states “up” and “down” in the Review Notes, any consistent labeling system is valid. In the formulas, we can replace references to the up state with references to Scenario 1, and we can replace references to the down state with references to Scenario 2.

The price of the stock is:

a d d- - ¥ ¥ ¥È ˘ È ˘= + - = ¥ + ¥Î ˚ Î ˚

=

0.07 1 0 1 0 11 2(1 ) 0.3 50 0.7 100

79.2535

h h hS e pS e p S e e e e

We can use Stock X and the risk-free asset to obtain two equations and two unknowns:

= +

= +1 2

1 2

79.2535 50 100

0.92

Q Q

Q Q

Multiplying the bottom equation by 50 and subtracting it from the top equation, we have:

- ¥ = - + -=

=

1 1 2 2

2

2

79.2535 50 0.92 50 50 100 50

33.2535 50

0.6651

Q Q Q Q

Q

Q

Solution 4.28

D Options on Currencies

The dollar call option is an option to buy a dollar. Since it is yen-denominated, the strike price is denominated in yen. Since it is at-the-money, the strike price is 118 yen.

Since the call is yen-denominated, we treat the yen as the base currency.

Page 101: ActuarialBrew.com Exam MFE / 3F Actuarial Models …actuarialbrew.com/data/documents/MFE-Solutions-2014-1st-ed.pdf · Exam MFE / 3F Actuarial Models Financial Economics Segment Solutions

Exam MFE/3F Solutions Chapter 4 – Binomial Trees: Part II

© ActuarialBrew.com 2014 Page 4.21

Unless told otherwise, we assume that the price of a yen-denominated option is denominated in yen. Therefore, the answer choices are in yen, not dollars.

The value of h is 1/3 since the intervals are 1/3 of a year. The values of u and d are:

¥

¥

$

$

( ) (0.01 0.06)(1 / 3) 0.11 1 / 3

( ) (0.01 0.06)(1 / 3) 0.11 1 / 3

1.04796

0.92295

s

s

- + - +

- - - -

= = =

= = =

r r h h

r r h h

u e e

d e e

The risk-neutral probability of an upward movement is:

¥ $( ) (0.01 0.06)(1 / 3) 0.92295

* 0.484131.04796 0.92295

- -- -= = =- -

r r he d ep

u d

The tree of prices for a dollar and the tree of option prices are below. Both trees are expressed in yen:

Dollar 135.8037 American Call 17.8037 129.5891 11.5891 123.6588 119.6048 5.9901 1.6048

118.0000 114.1315 3.0824 0.7744 108.9086 105.3382 0.3736 0.0000 100.5177 0.0000 92.7733 0.0000

The node that is in bold type in the American call option tree above indicates that early exercise is optimal since the option exercise value exceeds the option expected present value at that node.

The value of the American call option is:

[ ]0.01(1 / 3) (0.48413)(5.9901) (1 0.48413)(0.3736) 3.0824- + - =e

Solution 4.29

A Utility Values and State Prices

Although the Derivatives Markets textbook presented state prices with 2 states, it is not difficult to extend the analysis to cover 3 states.

We can find the utility value for the third state:

d d d

d d d= + +

= + += ¥ ¥ + ¥ ¥ + ¥ ¥=

1 1 2 2 3 3

1 1 1 2 2 2 3 3 3

2

2

100 0.46 0.8554 200 0.06 120 0.48 0.9819 35

0.6677

h h h

h h h

S Q S e Q S e Q S e

S p U S e p U S e p U S e

U

U

Page 102: ActuarialBrew.com Exam MFE / 3F Actuarial Models …actuarialbrew.com/data/documents/MFE-Solutions-2014-1st-ed.pdf · Exam MFE / 3F Actuarial Models Financial Economics Segment Solutions

Exam MFE/3F Solutions Chapter 4 – Binomial Trees: Part II

© ActuarialBrew.com 2014 Page 4.22

The values of the call option on Stock B at time T are:

= - == - == - =

1

2

3

(0,70 60) 10

(0,90 60) 30

(0,40 60) 0

V Max

V Max

V Max

The value of the option is:

= + += + += ¥ ¥ + ¥ ¥ + ¥ ¥ =

1 1 2 2 3 3

1 1 1 2 2 2 3 3 3

0.46 0.8554 10 0.06 0.6677 30 0.48 0.9819 0 5.137

V Q V Q V Q V

V p U V p U V p U V

V

Solution 4.30

E Options on Currencies

The dollar put option is an option to sell a dollar. Since it is yen-denominated, the strike price is denominated in yen. Since it is at-the-money, the strike price is 118 yen.

Since the put is yen-denominated, we treat the yen as the base currency.

The value of h is 1/3 since the intervals are 1/3 of a year. The values of u and d are:

¥

¥

$

$

( ) (0.01 0.06)(1 / 3) 0.11 1 / 3

( ) (0.01 0.06)(1 / 3) 0.11 1 / 3

1.04796

0.92295

s

s

- + - +

- - - -

= = =

= = =

r r h h

r r h h

u e e

d e e

The risk-neutral probability of an upward movement is:

¥ $( ) (0.01 0.06)(1 / 3) 0.92295

* 0.484131.04796 0.92295

- -- -= = =- -

r r he d ep

u d

The tree of prices for a dollar and the tree of option prices are below. Both trees are expressed in yen:

Dollar 135.8037 American Put 0.0000 129.5891 0.0000 123.6588 119.6048 3.3472 0.0000

118.0000 114.1315 8.2741 6.5101 108.9086 105.3382 12.9513 12.6618 100.5177 19.0800 92.7733 25.2267

It is not rational to exercise this American put option early.

The value of the American put option is:

[ ]0.01(1 / 3) (0.48413)(3.3472) (1 0.48413)(12.9513) 8.2741- + - =e

Page 103: ActuarialBrew.com Exam MFE / 3F Actuarial Models …actuarialbrew.com/data/documents/MFE-Solutions-2014-1st-ed.pdf · Exam MFE / 3F Actuarial Models Financial Economics Segment Solutions

Exam MFE/3F Solutions Chapter 4 – Binomial Trees: Part II

© ActuarialBrew.com 2014 Page 4.23

Solution 4.31

D Utility Values and State Prices

This stock has a higher value in the low state than it does in the high state. Although this situation does not affect our solution to the question, it suggests that the stock could be used as a hedge for whatever asset (which might, for example, be the entire market) is used to define high state and low state.

The price of the risk-free asset is:

( ) ( )- = + = + - = ¥ + ¥

=

110 110( ) 110 (1 ) 110 0.4 0.85 0.6 0.95

100.10

rhu d u de Q Q pU p U

The price of the stock is:

d d d d

¥ ¥

= + = + -

= ¥ ¥ + ¥ ¥ =0.05 1 0.05 1

(1 )

0.40 0.85 75 0.60 0.95 125 101.71

h h h hu u d d u u d dS Q S e Q S e pU S e p U S e

e e

The stock price exceeds the price of the risk-free bond by:

- =101.71 100.10 1.61

Solution 4.32

A Options on Currencies

The pound call is an option to buy a pound. Since it is franc-denominated, the strike price is denominated in francs.

Since the call is franc-denominated, we treat the franc as the base currency. For this problem, the subscript f denotes franc.

The value of h is 1 since the intervals are annual periods. The values of u and d are:

£

£

( ) (0.05 0.07)(1) 0.12 1

( ) (0.05 0.07)(1) 0.12 1

1.10517

0.86936

s

s

- + - +

- - - -

= = =

= = =

f

f

r r h h

r r h h

u e e

d e e

The risk-neutral probability of an upward movement is:

£( ) (0.05 0.07)(1) 0.86936

* 0.470041.10517 0.86936

- -- -= = =- -

fr r he d ep

u d

The tree of prices for a pound and the tree of option prices are below. Both trees are expressed in francs:

Pound American Call 2.9069 0.5469 2.6303 0.2703

2.3800 2.2867 0.1209 0.0000 2.0691 0.0000 1.7988 0.0000

Page 104: ActuarialBrew.com Exam MFE / 3F Actuarial Models …actuarialbrew.com/data/documents/MFE-Solutions-2014-1st-ed.pdf · Exam MFE / 3F Actuarial Models Financial Economics Segment Solutions

Exam MFE/3F Solutions Chapter 4 – Binomial Trees: Part II

© ActuarialBrew.com 2014 Page 4.24

The node that is in bold type in the American call option tree above indicates that early exercise is optimal at that node since the option exercise value exceeds the option expected present value at that node.

The value of the American call option is:

[ ]0.05(1) (0.47004)(0.2703) (1 0.47004)(0.000) 0.1209- + - =e

Solution 4.33

A Utility Values and State Prices

Since the probability of the high state is 0.65, the probability of the low state is:

- = - =1 1 0.65 0.35p

The stock’s cash flow in the low state can now be determined:

d d

d d

¥ ¥

= +

= + -

= ¥ ¥ + ¥ ¥=

0 2 0 2

(1 )

36.53 0.65 0.90 50 0.35 1.04

20

h hu u d d

h hu u d d

L

L

S Q S e Q S e

S pU S e p U S e

e C e

C

The payoffs of the derivative are:

= =

= =

3

3

ln(50 ) 11.7361

ln(20 ) 8.9872

u

d

V

V

The value of the derivative is:

= + = ¥ + - ¥= ¥ + ¥ =

(1 )

0.65(0.9) 11.7361 0.35(1.04) 8.9872 10.1369u u d d u u d dV Q V Q V pU V p U V

Solution 4.34

D Options on Futures Contracts

Since there is only one period, it doesn’t make any difference whether the call option is a European option or an American option.

The lease rate was thrown in as a red herring. It could be used to calculate the current price of gold, but we don’t need to know the current price of gold. But if you are curious, it is:

d- - - -= = =(r )T (0.07 0.04)(1)0 0,TS F e 600e 582.27

The values of Fu and Fd are:

0.12 1

0.12 1

1.12750

0.88692

s

s- -

= = =

= = =

hF

hF

u e e

d e e

Page 105: ActuarialBrew.com Exam MFE / 3F Actuarial Models …actuarialbrew.com/data/documents/MFE-Solutions-2014-1st-ed.pdf · Exam MFE / 3F Actuarial Models Financial Economics Segment Solutions

Exam MFE/3F Solutions Chapter 4 – Binomial Trees: Part II

© ActuarialBrew.com 2014 Page 4.25

The risk-neutral probability of an upward movement is:

1 1 0.88692

* 0.470041.12750 0.88692

- -= = =- -

F

F F

dp

u d

The futures price tree and the call option tree are below:

0,1F 1,1F Call Option

676.4981 56.4981 600.0000 24.7608

532.1523 0.0000 If the futures price rises to $676.4981, then the call option has a payoff of:

676.4981 620 56.4981- =

The price of the call option is:

0.07(1) (0.47004)(56.4981) (1 0.47004)(0.0000) 24.7608e

Solution 4.35

C Utility Values and State Prices

Since the probability of the high state is 0.65, the probability of the low state is:

- = - =1 1 0.65 0.35p

The stock’s cash flow in the low state can now be determined:

d d

d d

¥ ¥

= +

= + -

= ¥ ¥ + ¥ ¥=

0 2 0 2

(1 )

36.53 0.65 0.90 50 0.35 1.04

20

h hu u d d

h hu u d d

L

L

S Q S e Q S e

S pU S e p U S e

e C e

C

The risk-free rate can be found with the probabilities and utility values:

-

-

- ¥

= +

= + -

= ¥ +=

2

(1 )

0.65 0.90 0.35 *1.04

0.0262

rhu d

rhu d

r

e Q Q

e pU p U

e

r

The payoffs of the derivative at time 3 are:

= =

= =

3

3

(3) ln(50 ) 11.7361

(3) ln(20 ) 8.9872

u

d

V

V

Page 106: ActuarialBrew.com Exam MFE / 3F Actuarial Models …actuarialbrew.com/data/documents/MFE-Solutions-2014-1st-ed.pdf · Exam MFE / 3F Actuarial Models Financial Economics Segment Solutions

Exam MFE/3F Solutions Chapter 4 – Binomial Trees: Part II

© ActuarialBrew.com 2014 Page 4.26

Although the derivative does not pay until time 3, its payoffs are known with certainty at time 2. Therefore, we can discount the payoffs back to time 2 at the risk-free interest rate.

- ¥ -

- ¥ -

= = =

= = =

1 0.0262

1 0.0262

(2) (3) 11.7361 11.4329

(2) (3) 8.9872 8.7550

ru u

rd d

V V e e

V V e e

The value of the derivative is:

(0) (2) (2) (2) (1 ) (2)

0.65 0.9 11.4329 0.35 1.04 8.7550 9.8751u u d d u u d dV Q V Q V pU V p U V

Solution 4.36

A Options on Futures Contracts

The value of h is 1/3 since the intervals are 1/3 of a year. The values of Fu and Fd are:

0.30 1 / 3

0.30 1 / 3

1.18911

0.84097

s

s- -

= = =

= = =

hF

hF

u e e

d e e

The risk-neutral probability of an upward movement is:

1 1 0.84097

* 0.456811.18911 0.84097

- -= = =- -

F

F F

dp

u d

The futures price tree and the call option tree are below:

0,1F 13

,1F 23

,1F 1,1F American Call

1,681.3806 681.3806 1,413.9825 413.9825 1,189.1099 1,189.1099 229.5336 189.1099

1,000.0000 1,000.0000 122.4206 84.3943 840.9651 840.9651 37.6628 0.0000 707.2224 0.0000 594.7493 0.0000

If the futures price reaches $1,413.9825, then it is optimal to exercise the call option early since the option exercise value exceeds the expected present value of the option at that node.

The value of the American option is:

0.07(1 / 3) (0.45681)(229.5336) (1 0.45681)(37.6628) 122.4206e

Page 107: ActuarialBrew.com Exam MFE / 3F Actuarial Models …actuarialbrew.com/data/documents/MFE-Solutions-2014-1st-ed.pdf · Exam MFE / 3F Actuarial Models Financial Economics Segment Solutions

Exam MFE/3F Solutions Chapter 4 – Binomial Trees: Part II

© ActuarialBrew.com 2014 Page 4.27

Solution 4.37

B Utility Values and State Prices

The expected return on the call options can be found with the formula below:

g+ - + -

+ = =+ -

(1 ) (1 )1

(1 )u d u d

Callu u d d

pV p V pV p VV pU V p U V

The expected returns for the first 2 choices are:

g

g

+ - ¥ + ¥= - = - =+ - ¥ ¥ + ¥ ¥+ - ¥ + ¥= - = - =+ - ¥ ¥ + ¥ ¥

(1 ) 0.6 60 0.4 20A: 1 1 5.97%

(1 ) 0.6 0.92 60 0.4 1.05 20

(1 ) 0.6 35 0.4 0B: 1 1 8.70%

(1 ) 0.6 0.92 35 0.4 1.05 0

u dCall

u u d d

u dCall

u u d d

pV p VpU V p U V

pV p VpU V p U V

For Choice C, the strike price is greater than either of the possible stock prices, so the value of the call option is zero. Consequently, the expected return is undefined for the option described in Choice C.

The expected returns for the fourth and fifth choices are:

g

g

+ - ¥ + ¥= - = - = -+ - ¥ ¥ + ¥ ¥+ - ¥ + ¥= - = - = -+ - ¥ ¥ + ¥ ¥

(1 ) 0.6 0 0.4 15D: 1 1 4.76%

(1 ) 0.6 0.92 0 0.4 1.05 15

(1 ) 0.6 20 0.4 60E: 1 1 0.66%

(1 ) 0.6 0.92 20 0.4 1.05 60

u dPut

u u d d

u dPut

u u d d

pV p VpU V p U V

pV p VpU V p U V

The option described in Choice B has the highest expected return.

Solution 4.38

A Options on Futures Contracts

The values of Fu and Fd are:

0.30 1 / 3

0.30 1 / 3

1.18911

0.84097

s

s- -

= = =

= = =

hF

hF

u e e

d e e

The risk-neutral probability of an upward movement is:

1 1 0.84097

* 0.456811.18911 0.84097

- -= = =- -

F

F F

dp

u d

Page 108: ActuarialBrew.com Exam MFE / 3F Actuarial Models …actuarialbrew.com/data/documents/MFE-Solutions-2014-1st-ed.pdf · Exam MFE / 3F Actuarial Models Financial Economics Segment Solutions

Exam MFE/3F Solutions Chapter 4 – Binomial Trees: Part II

© ActuarialBrew.com 2014 Page 4.28

The futures price tree and the call option tree are below:

0,1F 13

,1F 23

,1F 1,1F American Call

1,681.3806 681.3806 1,413.9825 413.9825 1,189.1099 1,189.1099 229.5336 189.1099

1,000.0000 1,000.0000 122.4206 84.3943 840.9651 840.9651 37.6628 0.0000 707.2224 0.0000 594.7493 0.0000

If the futures price reaches $1,413.9825, then it is optimal to exercise the call option early since the option exercise value exceeds the expected present value of the option at that node.

The number of futures contracts that the investor must be long is:

229.5336 37.6628

0.5511( ) 1,189.1099 840.9651

- -D = = =- -

u d

F F

V VF u d

Solution 4.39

E Two-Period Binomial Model

Since the stock does not pay dividends, the price of the American call option is equal to the price of an otherwise equivalent European call option.

The stock price tree and the associated option payoffs at the end of 2 years are:

Call Payoff 75.6877 27.6877 58.3605

45.0000 50.7386 2.7386 39.1230 34.0135 0.0000

The risk-neutral probability of an upward movement is:

( ) (0.06 0.00)1 0.8694

* 0.450141.2969 0.8694

r he d ep

u d

d- -- -= = =- -

The value of the call option is:

( ) ( )0 0

0

0.06(2) 2

( , ,0) ( *) (1 *) ( , , )

(0.45014) (27.6877) 2(0.45014)(1 0.45014)(2.7386) 0

6.1783

nr hn n i i n i i

i

nV S K e p p V S u d K hn

i

e

- - -

=-

È ˘Ê ˆ= -Í ˙Á ˜Ë ¯Î ˚

È ˘= + - +Î ˚=

Â

Page 109: ActuarialBrew.com Exam MFE / 3F Actuarial Models …actuarialbrew.com/data/documents/MFE-Solutions-2014-1st-ed.pdf · Exam MFE / 3F Actuarial Models Financial Economics Segment Solutions

Exam MFE/3F Solutions Chapter 4 – Binomial Trees: Part II

© ActuarialBrew.com 2014 Page 4.29

Solution 4.40

E Three-Period Binomial Model for Currency

When the type of binomial model is not specified in the question, we should assume that that we are to use the standard binomial model from the Derivatives Markets textbook. Question 5 of the SOA’s sample exam questions is an example of a question where we are expected to assume that the standard binomial model applies.

The values of u and d are:

( ) (0.10 0.04)0.25 0.32 0.25

( ) (0.10 0.04)0.25 0.32 0.25

1.19125

0.86502

f

f

r r h h

r r h h

u e e

d e e

s

s

- + - +

- - - -

= = =

= = =

The risk-neutral probability of an upward movement is:

( ) (0.10 0.04)(0.25) 0.86502

* 0.460091.19125 0.86502

r he d ep

u d

d- -- -= = =- -

The tree of pound prices and the tree of option prices are below:

Pound 3.2119 American Put 0.0000 2.6962 0.0000 2.2634 2.3323 0.0988 0.0000

1.9000 1.9579 0.2629 0.1877 1.6435 1.6936 0.4151 0.3564 1.4217 0.6283 1.2298 0.8202

If the exchange rate falls to 1.4217 at the end of 6 months, then early exercise is optimal. The value of the American put option at time 0 is 0.2629.

Solution 4.41

D Greeks in the Jarrow-Rudd Binomial Model

How do we know that gamma in the question refers to G and not g ? Because we would need to know the realistic probability of an upward movement in order to determine the expected return on the option, g , but there is no way of knowing the realistic probability of an upward movement.

The values of u and d are:

2 2

2 2

( 0.5 ) (0.11 0.04 0.5 0.32 )(1) 0.32 1

( 0.5 ) (0.11 0.04 0.5 0.32 )(1) 0.32 1

1.40326

0.73993

r h h

r h h

u e e

d e e

The risk-neutral probability of an upward movement is:

d- -- -= = =- -

( ) (0.11 0.04)(1) 0.73993* 0.50137

1.40326 0.73993

r he d ep

u d

Page 110: ActuarialBrew.com Exam MFE / 3F Actuarial Models …actuarialbrew.com/data/documents/MFE-Solutions-2014-1st-ed.pdf · Exam MFE / 3F Actuarial Models Financial Economics Segment Solutions

Exam MFE/3F Solutions Chapter 4 – Binomial Trees: Part II

© ActuarialBrew.com 2014 Page 4.30

The stock price tree and its corresponding tree of option prices are:

Stock American Put 78.7658 0.0000 56.1305 0.2088

40.0000 41.5326 5.6339 0.4674 29.5972 12.4028 21.8998 20.1002 If the stock price initially moves down, then the resulting put price is $12.4028. This price is in bold type above to indicate that it is optimal to exercise early at this node:

- =42 29.5972 12.4028

This exercise value of 12.4028 is greater than the value of holding the option, which is:

[ ]- + - =0.11(1) (0.50137)(0.4674) (1 0.50137)(20.1002) 9.1884e

There is no need to calculate the current value of the American put to answer this question, but it is provided in the tree above for completeness.

We need to calculate the two possible values of delta at time 1:

d

d

- - ¥

- - ¥

- -D = = = ---

- -D = = = ---

0.04 12

0.04 12

0.0000 0.4674( , ) 0.0121

78.7658 41.5326

0.4674 20.1002( , ) 0.9608

41.5326 21.8998

h uu ud

h ud dd

V VSu h e e

Su SudV V

Sd h e eSud Sd

We can now calculate gamma:

D - D - - -G ª G = = =

- -( , ) ( , ) 0.0121 ( 0.9608)

( ,0) ( , ) 0.035856.1305 29.5972h

Su h Sd hS S h

Su Sd

Solution 4.42

D State Prices

Since the strike price is $10, the put option pays Max[0, 10 – 15] = $0 if the up state occurs. If the down state occurs, the put option pays Max[0, 10 – 7] = $3. We can use the stock price and the option price to solve for the original state prices:

10 15 7 0.40381.69 0 3 0.5633

u d u

u d d

Q Q Q

Q Q Q

The discount factor for one year is:

- = + = + =0.4038 0.5633 0.9671ru de Q Q

Page 111: ActuarialBrew.com Exam MFE / 3F Actuarial Models …actuarialbrew.com/data/documents/MFE-Solutions-2014-1st-ed.pdf · Exam MFE / 3F Actuarial Models Financial Economics Segment Solutions

Exam MFE/3F Solutions Chapter 4 – Binomial Trees: Part II

© ActuarialBrew.com 2014 Page 4.31

After the correction is made, the new state prices still result in the same risk-free rate of return:

- = + =ˆ ˆ 0.9671ru de Q Q

We can use the stock price with the corrected value after a down-move and the expression for the risk-free discount factor to obtain another system of 2 equations and 2 unknowns:

¸= + Ô fi =˝

= + Ô

ˆ ˆ10 15 5 ˆ 0.4507ˆ ˆ0.9671

u dd

u d

Q QQ

Q Q

The new state price can now be used to find the value of the option. Since the payoff of the put option is Max[0, 10 – 5] = $5 when we use the correct lower stock price of $5, we have the following price for the put option:

ˆ ˆ0 5 5 0.4507 2.2533u dQ Q

Alternate Solution

We don’t have to use state prices to answer this question.

We can use the stock price and the option price to solve for the risk-neutral probability of an upward movement and the discount factor:

[ ][ ]

- -

-

¸= + - Ô =fi˝== + - Ô

10 15 * 7(1 *) 0.9671* 0.41751.69 0 * 3(1 *)

r r

r

e p p epe p p

After the correction is made, we still have the same discount factor of 0.9671, but we have a new risk-neutral probability of an upward movement. We can use the stock price with the corrected value after a down-move to solve for the new, corrected risk-neutral probability of an up move:

[ ]= + -

= +

=

ˆ ˆ10 0.9671 15 * 5(1 *)

10ˆ10 * 5

0.9671ˆ* 0.5340

p p

p

p

The payoff of the put option is $5 when we use the correct lower stock price of $5. We have the following price for the put option:

-- = - =ˆ5(1 *) 5(1 0.5340)(0.9671) 2.2533rp e

Solution 4.43

E Utility Values and State Prices

The payoffs of the call option are:

= - = -= - =

(0,100 ) 100

(0,60 ) 0u C C

d C

V Max K K

V Max K

Page 112: ActuarialBrew.com Exam MFE / 3F Actuarial Models …actuarialbrew.com/data/documents/MFE-Solutions-2014-1st-ed.pdf · Exam MFE / 3F Actuarial Models Financial Economics Segment Solutions

Exam MFE/3F Solutions Chapter 4 – Binomial Trees: Part II

© ActuarialBrew.com 2014 Page 4.32

The price of the call option is:

= + = ¥ - + - ¥ = -(100 ) (1 ) 0 0.6(0.92)(100 )u u d d u C d CV Q V Q V pU K p U K

The expected return on the call option can now be calculated:

g

g

g

g

+ -+ =

-+ =

-

+ =

=

(1 )1

0.6(100 )1

0.6(0.92)(100 )

11

0.920.0870

u dCall

CCall

C

Call

Call

pV p VV

KK

The payoffs of the put option are:

= - == - = -

(0, 100) 0

(0, 60) 60u P

d P P

V Max K

V Max K K

The price of the put option is:

0 (1 ) ( 60) 0.4(1.05)( 60)u u d d u d P PV Q V Q V pU p U K K

The expected return on the put option can now be calculated:

(1 )1

0.4( 60)1

0.4(1.05)( 60)

11

1.050.0476

u dPut

PPut

P

Put

Put

pV p VVK

K

The difference in the expected returns is:

g g- = - - =0.0870 ( 0.0476) 0.1346Call Put

Solution 4.44

B Utility Values and State Prices

Notice that the expected return on the call option is expressed as an annual effective rate, not a continuously compounded rate.

Since the call option consists of 20 of the up-state Arrow-Debreu securities, the expected return of each up-state Arrow-Debreu security is equal to the expected return of the call option:

g g= = 0.23u Call

Page 113: ActuarialBrew.com Exam MFE / 3F Actuarial Models …actuarialbrew.com/data/documents/MFE-Solutions-2014-1st-ed.pdf · Exam MFE / 3F Actuarial Models Financial Economics Segment Solutions

Exam MFE/3F Solutions Chapter 4 – Binomial Trees: Part II

© ActuarialBrew.com 2014 Page 4.33

The utility value is the present value of $1, calculated using the expected return of the corresponding Arrow-Debreu security. Therefore, the up-state utility value is:

g= =

+1 1

1 1.23uu

U

The call option pays off $20 in the up state and $0 in the down state. We can use the call option to obtain the state price for the up state:

= +

= =

7.37 20 0

7.370.3685

20

u d

u

Q Q

Q

We can use the state price of the up-state to obtain the realistic probability that the up state occurs:

= ¥

= ¥

=

10.3685

1.230.4533

u uQ p U

p

p

We can now find the down-state utility value:

= += + -

= ¥ ¥ + -

=

(1 )

176 0.4533 100 (1 0.4533)70

1.231.0229

u u d d

u u d d

d

d

S Q S Q S

S U pS U p S

U

U

The sum of the up-state and the down-state utility values is:

+ = + =1

1.0229 1.83591.23u dU U

Solution 4.45

E Greeks in the Binomial Model

The values of u and d are:

d s

d s

- + - +

- - - -

= = =

= = =

( ) (0.11 0.04)(1) 0.32 1

( ) (0.11 0.04)(1) 0.32 1

1.47698

0.77880

r h h

r h h

u e e

d e e

The risk-neutral probability of an upward movement is:

d- -- -= = =- -

( ) (0.11 0.04)(1) 0.77880* 0.42068

1.47698 0.77880

r he d ep

u d

Page 114: ActuarialBrew.com Exam MFE / 3F Actuarial Models …actuarialbrew.com/data/documents/MFE-Solutions-2014-1st-ed.pdf · Exam MFE / 3F Actuarial Models Financial Economics Segment Solutions

Exam MFE/3F Solutions Chapter 4 – Binomial Trees: Part II

© ActuarialBrew.com 2014 Page 4.34

The stock price tree and its corresponding tree of option prices are:

Stock American Put 87.2589 0.0000 59.0792 0.0000

40.0000 46.0110 5.6299 0.0000 31.1520 10.8480 24.2612 17.7388

If the stock price initially moves down, then the resulting put price is $10.8480. This price is in bold type above to indicate that it is optimal to exercise early at this node:

- =42 31.1520 10.8480

The exercise value of 10.8480 is greater than the value of holding the option, which is:

[ ]- + - =0.11(1) (0.42068)0.0000 (1 0.42068)(17.7388) 9.2060e

The current value of the American option is:

[ ]- + - =0.11(1) (0.42068)0.0000 (1 0.42068)(10.8480) 5.6299e

We need to calculate 3 values of delta:

d

d

d

- - ¥

- - ¥

- - ¥

- -D = = = -- -

- -D = = =--

- -D = = = ---

0.04 1

0.04 12

0.04 12

0 10.8480( ,0) 0.3732

59.0792 31.15200.0000 0.0000

( , ) 0.000087.2589 46.0110

0.0000 17.7388( , ) 0.

46.0110 24.2612

h u d

h uu ud

h ud dd

V VS e e

Su SdV V

Su h e eSu SudV V

Sd h e eSud Sd

7836

We can now calculate gamma:

D - D - -G ª G = = =

- -( , ) ( , ) 0 ( 0.7836)

( ,0) ( , ) 0.028159.0792 31.1520h

Su h Sd hS S h

Su Sd

We can now estimate theta:

q

-- - - D - G=

-- - - - -=

¥= -

2

2

( )( ) ( ,0) ( ,0)

2( ,0)2

(46.0110 40.0000)0 5.6299 (46.0110 40.0000)( 0.3732) (0.0281)

22 1

1.9467

udSud S

V V Sud S S SS

h

Page 115: ActuarialBrew.com Exam MFE / 3F Actuarial Models …actuarialbrew.com/data/documents/MFE-Solutions-2014-1st-ed.pdf · Exam MFE / 3F Actuarial Models Financial Economics Segment Solutions

Exam MFE/3F Solutions Chapter 4 – Binomial Trees: Part II

© ActuarialBrew.com 2014 Page 4.35

Solution 4.46

B Greeks in the Binomial Model

The question does not tell us when the call option expires, so we cannot assume that it expires in 6 months.

The risk-neutral probability of an upward movement is:

d- -- -= = =- -

( ) (0.10 0.03)0.25 61.60 /70* 0.4637

82.38 /70 61.60 /70

r he d ep

u d

The time 0 value of the option is:

[ ]- - ¥= + - = ¥ + - ¥

=

0.10 0.250 ( *) (1 *) [0.4637 14.91 (1 0.4637) 2.72]

8.1658

rhu dC e p C p C e

The delta-gamma-theta approximation is:

2

0

2

( )( ) ( ,0) ( ,0) 2 ( ,0)

2

(72.49 70)8.1658 (72.49 70)(0.5822) (0.0233) 2(0.25)( 7.3361)

26.0197

udSud S

C C Sud S S S h S

Solution 4.47

B Greeks in the Binomial Model

The risk-neutral probability of an upward movement is:

d- -- -= = =- -

( ) (0.10 0.03)0.25 61.60 /70* 0.4637

82.38 /70 61.60 /70

r he d ep

u d

The time 0 value of the option is:

[ ]- - ¥= + - = ¥ + - ¥

=

0.10 0.250 ( *) (1 *) [0.4637 14.91 (1 0.4637) 2.72]

8.1658

rhu dC e p C p C e

The delta-gamma-theta approximation is:

2

0

2

( )( ) ( ,0) ( ,0) ( ,0)

2

(82.38 70)8.1658 (82.38 70)(0.5822) (0.0233) (0.25)( 7.3361)

215.32

uSu S

C C Su S S S h S

The difference between the estimate and the actual value of $14.91 (which can be read from the tree of option prices) is:

- =15.32 14.91 0.41

Page 116: ActuarialBrew.com Exam MFE / 3F Actuarial Models …actuarialbrew.com/data/documents/MFE-Solutions-2014-1st-ed.pdf · Exam MFE / 3F Actuarial Models Financial Economics Segment Solutions

Exam MFE/3F Solutions Chapter 4 – Binomial Trees: Part II

© ActuarialBrew.com 2014 Page 4.36

Solution 4.48

E Three-Period Binomial Tree

The risk-neutral probability of an upward movement is:

d - ¥- --= = =- -

(0.10 0.065) 1 210( )300

375 210300 300

* 0.6102r h ee d

pu d

The tree of prices for the American call option is shown below:

American Call 285.9375 168.7500 98.6519 28.1250

57.4945 15.5292 8.5744 0.0000 0.0000 0.0000

Early exercise is optimal if the stock price increases to $468.75 at the end of 2 years. This is indicated by the bolding of that node in the tree above.

The price of the option is:

0.10 1 0.6102 98.6519 (1 0.6102) 8.5744 57.4945e

Solution 4.49

A Greeks in the Binomial Model

We need to calculate the two possible values of delta at the end of 1 year:

0.065 12

0.065 12

168.75 15.5292( , ) 0.6961

468.75 262.50

15.5292 0.0000( , ) 0.1260

262.50 147.00

h uu ud

h ud dd

V VSu h e e

Su SudV V

Sd h e eSud Sd

We can now calculate gamma:

( , ) ( , ) 0.6961 0.1260

( ,0) ( , ) 0.003455375 210h

Su h Sd hS S h

Su Sd

Solution 4.50

A Three-Period Binomial Tree

The risk-neutral probability of an upward movement is:

d - ¥- --= = =- -

(0.10 0.065) 1 210( )300

375 210300 300

* 0.6102r h ee d

pu d

Page 117: ActuarialBrew.com Exam MFE / 3F Actuarial Models …actuarialbrew.com/data/documents/MFE-Solutions-2014-1st-ed.pdf · Exam MFE / 3F Actuarial Models Financial Economics Segment Solutions

Exam MFE/3F Solutions Chapter 4 – Binomial Trees: Part II

© ActuarialBrew.com 2014 Page 4.37

The tree of prices for the American put option is shown below:

American put 0.0000 0.0000 13.8384 0.0000

37.6195 39.2367 85.0000 111.2500 148.0000 192.1000

It is not necessary to calculate all of the prices in the tree above to answer this question, but we included the full tree for the sake of completeness.

Early exercise is optimal if the stock price falls to $210 at the end of 1 year or falls to $147 at the end of 2 years. This is indicated by the bolding of those two nodes in the tree above.

The only year-2 node at which early exercise occurs is the bottom node. But to reach that node, the stock must first pass through the year-1 bottom node. Since it will be exercised at the end of year 1 if the stock price reaches $210, there is no possibility of the stock being exercised at the end of year 2. Therefore, =2 0p .

Solution 4.51

A Options on Futures Contracts

We are given that the ratio of the factors applicable to the futures price is:

= 43

F

F

ud

The formula for the risk-neutral probability of an up move can be used to find Fu and

Fd :

-=-

-=

-

-=

-

=

= ¥ =

1

1

43

1*

*

113 1

0.9

41.2

3

F

F F

F F

F F

F

F

F Fd

d du dd d

d

F

F F

dp

u d

p

d

u d

Page 118: ActuarialBrew.com Exam MFE / 3F Actuarial Models …actuarialbrew.com/data/documents/MFE-Solutions-2014-1st-ed.pdf · Exam MFE / 3F Actuarial Models Financial Economics Segment Solutions

Exam MFE/3F Solutions Chapter 4 – Binomial Trees: Part II

© ActuarialBrew.com 2014 Page 4.38

The tree of futures prices is therefore:

Futures Prices 115.2000 96.0000

80.0000 86.4000 72.0000 64.8000

The tree of prices for the European put option is:

European Put 0.0000 0.0000

8.5399 0.0000 13.1342 20.2000

The price of the European put is:

0.05 0.5 (1 / 3)0.000 (2 / 3)13.1342 8.5399e

The tree of prices for the American put option is:

American Put 0.0000 0.0000

8.5399 0.0000 13.1342 20.2000

The price of the American put is:

0.05 0.5 (1 / 3)0.0000 (2 / 3)13.1342 8.5399e

Early exercise of the American option is never optimal, so the prices of the European option and the American option are the same.

The price of the American put option exceeds the price of the European put option by:

8.5399 8.5399 0.0000

Solution 4.52

C Options on Futures Contracts

We are given that the ratio of the factors applicable to the futures price is:

= 0.6F

F

du

Page 119: ActuarialBrew.com Exam MFE / 3F Actuarial Models …actuarialbrew.com/data/documents/MFE-Solutions-2014-1st-ed.pdf · Exam MFE / 3F Actuarial Models Financial Economics Segment Solutions

Exam MFE/3F Solutions Chapter 4 – Binomial Trees: Part II

© ActuarialBrew.com 2014 Page 4.39

The formula for the risk-neutral probability of an up move can be used to find Fu and

Fd :

-=

-

-=

-

-=

-== ¥ =

1

1

1*

*

0.616 1 0.6

1.5

0.6 0.9

F

F F

F F

F F

F

F

F Fd

u uu du u

u

F

F F

dp

u d

p

u

d u

The tree of futures prices is therefore:

Futures Prices 157.5000 105.0000

70.0000 94.5000 63.0000 56.7000

The tree of prices for the European call option is:

European Call 92.5000 38.8178

10.1370 29.5000 4.7714 0.0000

The price of the European call is:

[ ]- ¥ + =0.06 0.5 (1 / 6)(38.8178) (5 / 6)(4.7714) 10.1370e

The tree of prices for the American call option is:

American Call 92.5000 40.0000

10.3283 29.5000 4.7714 0.0000

The price of the American call is:

[ ]- ¥ + =0.06 0.5 (1 / 6)(40.0000) (5 / 6)(4.7714) 10.3283e

If the futures price moves up at the end of 6 months, then early exercise of the American option is optimal, and this is indicated by the bolding of that node above.

Page 120: ActuarialBrew.com Exam MFE / 3F Actuarial Models …actuarialbrew.com/data/documents/MFE-Solutions-2014-1st-ed.pdf · Exam MFE / 3F Actuarial Models Financial Economics Segment Solutions

Exam MFE/3F Solutions Chapter 4 – Binomial Trees: Part II

© ActuarialBrew.com 2014 Page 4.40

The price of the American call option exceeds the price of the European call option by:

- =10.3283 10.1370 0.1912

Solution 4.53

C American Put Option

The values of u and d are:

( ) (0.06 0.00)0.5 0.35 0.5

( ) (0.06 0.00)0.5 0.35 0.5

1.3198

0.8045

r h h

r h h

u e e

d e e

The risk-free probability of an upward movement is:

( ) (0.06 0.00)0.5 0.8045

* 0.43841.3198 0.8045

r he d ep

u d

The stock price tree is:

98.99

75

60.34

Let’s use trial and error. We begin with the middle strike price of $93. The value of immediate exercise is:

- =93 75 18

The value of holding on to the option is:

- ¥- ¥ ¥ - =0.06 0.5(93 60.34) (1 0.4384) 17.80e

Since <17.80 18.00 , the investor will exercise immediately if the strike price is $93.

If the strike price is higher than $93, then the put option will be even more in-the-money, so the option is sure to be exercised at strike prices that are greater than or equal to $93.

Next, let’s try a strike price of $92. The value of immediate exercise is:

- =92 75 17

The value of holding on to the option is:

- ¥- ¥ ¥ - =0.06 0.5(92 60.34) (1 0.4384) 17.25e

Since >17.25 17.00 , the investor will not exercise immediately if the strike price is $92.

If the strike price is less than $92, then the option is even less in-the-money, meaning that it is even less likely to be exercised early.

Therefore, the smallest integer-valued strike price for which the investor will exercise the put option at the beginning of the period is $93.

Page 121: ActuarialBrew.com Exam MFE / 3F Actuarial Models …actuarialbrew.com/data/documents/MFE-Solutions-2014-1st-ed.pdf · Exam MFE / 3F Actuarial Models Financial Economics Segment Solutions

Exam MFE/3F Solutions Chapter 4 – Binomial Trees: Part II

© ActuarialBrew.com 2014 Page 4.41

Alternate Solution

The strike price K, for which an investor will exercise the put option at the beginning of the period must be at least $75, since otherwise the payoff to immediate exercise would be zero. Since we are seeking the lowest strike price that results in immediate exercise, let’s begin by determining whether there is a strike price that is greater than $75 but less than $98.99 that results in immediate exercise.

If there is such a strike price, then the value of exercising now must exceed the value of holding the option:

0.06(0.5)

0.06(0.5)

75 ( 60.34)(1 *)

75 ( 60.34)(1 0.4384)75 0.5450 32.88

0.4550 42.1292.56

K K p e

K K eK K

KK

The smallest integer that satisfies this inequality is $93.

Solution 4.54

A American Call Option

The stock does not pay dividends. Therefore, the general rule is that an American call option on the stock will not be exercised early.

But we must select one of the answer choices, so let’s consider the unusual situation of a strike price of zero. If = 0K , then the exercise value is equal to the $75. Furthermore, the value of holding the option is $75 as well, since the option delivers the same future cash flows as ownership of one share of stock.

Therefore, the owner of the option is indifferent to exercising the option when the strike price is 0. That is the only possible strike price at which the investor will exercise the call option, so Choice A is the best answer to this question.

Referring back to the solution to Question 60, some students might be tempted to solve the inequality below to obtain an answer:

0.06(0.5 )75 K (98.99 K ) 0.4384e75 K (98.99 K ) 0.42550.5745K 32.8832

K 57.2363

Page 122: ActuarialBrew.com Exam MFE / 3F Actuarial Models …actuarialbrew.com/data/documents/MFE-Solutions-2014-1st-ed.pdf · Exam MFE / 3F Actuarial Models Financial Economics Segment Solutions

Exam MFE/3F Solutions Chapter 4 – Binomial Trees: Part II

© ActuarialBrew.com 2014 Page 4.42

Does the solution above suggest that the correct answer is $57? No, because the inequality above is only valid if K 60.34≥ , which is obviously contradicted by a solution of K 57.235= . If the strike price is less than $60.34, then both 6-month nodes produce a payoff, and the inequality becomes:

0.06(0.5 ) 0.06(0.5 )75 K (98.99 K) 0.4384e (60.34 K) (1 0.4384)e75 K (98.99 K) 0.4255 (60.34 K) 0.545075 K 42.1168 0.4255K 32.8832 0.5450KK 0

- -- ≥ - ¥ + - ¥ -- ≥ - ¥ + - ¥- ≥ - + -£

Solution 4.55

E American Put Option

The values of u and d are:

( ) (0.06 0.08)0.5 0.35 0.5

( ) (0.06 0.08)0.5 0.35 0.5

1.2681

0.7730

r h h

r h h

u e e

d e e

The risk-free probability of an upward movement is:

( ) (0.06 0.08)0.5 0.7730

* 0.43841.2681 0.7730

r he d ep

u d

The stock price tree is:

95.10

75

57.97

Let’s use trial and error. We begin with the middle strike price of $96. The value of immediate exercise is:

- =93 75 21

The value of holding on to the option is:

- ¥ - ¥- ¥ ¥ + - ¥ ¥ - =0.06 0.5 0.06 0.5(96 95.10) 0.4384 (96 57.97) (1 0.4384) 21.10e e

Since <21.00 21.10 , the investor will not exercise immediately if the strike price is $96.

If the strike price is less than $96, then the option is even less in-the-money, meaning that it is even less likely to be exercised early. Therefore, for our next guess, we’ll try a higher strike price.

Next, let’s try a strike price of $99. The value of immediate exercise is:

- =99 75 24

The value of holding on to the option is:

- ¥ - ¥- ¥ ¥ + - ¥ ¥ - =0.06 0.5 0.06 0.5(99 95.10) 0.4384 (99 57.97) (1 0.4384) 24.01e e

Page 123: ActuarialBrew.com Exam MFE / 3F Actuarial Models …actuarialbrew.com/data/documents/MFE-Solutions-2014-1st-ed.pdf · Exam MFE / 3F Actuarial Models Financial Economics Segment Solutions

Exam MFE/3F Solutions Chapter 4 – Binomial Trees: Part II

© ActuarialBrew.com 2014 Page 4.43

Since <24.00 24.01 , the investor will not exercise immediately if the strike price is $99.

By the process of elimination, we know that the correct answer must be Choice E, which is $100. Let’s verify this. The value of immediate exercise is:

- =100 75 25

The value of holding on to the option is:

- ¥ - ¥- ¥ ¥ + - ¥ ¥ - =0.06 0.5 0.06 0.5(100 95.10) 0.4384 (100 57.97) (1 0.4384) 24.99e e

Since >25.00 24.99 , the option is exercised immediately if the strike price is $100. Therefore, the smallest integer-valued strike price for which the investor will exercise the put option at the beginning of the period is $100.

Alternate Solution

The strike price K, for which an investor will exercise the put option at the beginning of the period must be at least $75, since otherwise the payoff to immediate exercise would be zero. Since we are seeking the lowest strike price that results in immediate exercise, let’s begin by determining whether there is a strike price that is greater than $75 but less than $95.10 that results in immediate exercise.

If there is strike price that is less than $95.10, then the value of exercising now must exceed the value of holding the option:

0.06(0.5)

0.06(0.5)

75 ( 57.97)(1 *)

75 ( 57.97)(1 0.4384)75 0.5450 31.59

0.4550 43.4195.39

K K p e

K K eK K

KK

The inequality above suggests that the strike price must be greater than 95.39, but the original inequality was predicated on the assumption that < 95.10K . Clearly, the strike price cannot simultaneously be greater than 95.39 and less than 95.10. Therefore, the assumption that the strike price is less than 95.10 is incorrect.

We now consider the possibility that the strike price is greater than $95.10. Once again, the value of exercising now exceeds the value of holding the option:

0.06(0.5) 0.06(0.5)

0.06(0.5) 0.03

75 ( 95.10)( *) ( 57.97)(1 *)

75 ( 95.10)(0.4384) ( 57.97)(1 0.4384)75 0.9704 72.06

0.0296 2.9499.50

K K p e K p e

K K e K eK K

KK

The smallest integer that satisfies this inequality is $100.

Page 124: ActuarialBrew.com Exam MFE / 3F Actuarial Models …actuarialbrew.com/data/documents/MFE-Solutions-2014-1st-ed.pdf · Exam MFE / 3F Actuarial Models Financial Economics Segment Solutions

Exam MFE/3F Solutions Chapter 4 – Binomial Trees: Part II

© ActuarialBrew.com 2014 Page 4.44

Solution 4.56

C Theta in the Binomial Model

The formula for theta is:

q

-- - - D - G=

2( )( ) ( ,0) ( ,0)

2( ,0)2

udSud S

V V Sud S S SS

h

Since we have 60S Sud= = , this simplifies to:

q-

=( ,0)2

udV VS

h

The risk-neutral probability of an upward move is:

-= =

-

0.0860 48* 0.6295

75 48e

p

Since the put option is an American option, we solve for its value from right to left. The completed tree is shown below.

0.0000 0.0000 1.4035 0.0000

4.9195 4.1039 12.0000 12.0000 21.6000 29.2800

The bolded entries in the table above indicate where early exercise is optimal.

The value of theta can now be found:

q- -= = = -

¥4.1039 4.9195

( ,0) 0.40782 2 1

udV VS

h

Page 125: ActuarialBrew.com Exam MFE / 3F Actuarial Models …actuarialbrew.com/data/documents/MFE-Solutions-2014-1st-ed.pdf · Exam MFE / 3F Actuarial Models Financial Economics Segment Solutions

Exam MFE/3F Solutions Chapter 5 – Lognormally Distributed Prices

© ActuarialBrew.com 2014 Page 5.01

Chapter 5 – Solutions

Solution 5.01

C The Lognormal Distribution and its Parameters

The page references below refer to the Second Edition of Derivatives Markets.

Choice A is false. The observed mean is determined by the difference between the beginning and ending stock price, so more frequent observations do not change the observed mean. (See page 607.)

Choice B is false. Observations taken over a longer period of time are more likely to reflect the true mean and therefore they increase the precision of the estimate. (See page 607.)

Choice C is true. More frequent observations allow us to better observe the extent to which the returns vary. (See page 607.)

Choice D is false. The median price is less than the expected price:

a d s a d- - -= =2( 0.5 ) ( )

0 0Median of [ ]T TT TS S e E S S e

By definition, the return is less than or equal to the median return 50% of the time. Since the median return is less than the expected return, this implies that the return is less than or equal to the mean return more than 50% of the time. (See page 597.)

Choice E is false. The product of independent lognormal random variables is lognormal. Suppose that 1y and 2y are lognormal, and 1x and 2x are normal and independent. Suppose further that:

= =1 21 2

x xy e y e

The product of the two independent lognormal random variables is lognormal, because the sum of the two independent normal random variables is normal:

+¥ = =1 2 1 21 2

x x x xy y e e e

(See page 593.)

Solution 5.02

C Converting to a Standard Normal Random Variable

We use the following relationship to convert the draws into their standard normal equivalents:

m

s-= x

z

The 3 standard normal equivalents are:

- - - -= = - = = = =4 3 5 3 3 3

2.21 0.63 0.0010 10 10

z z z

Page 126: ActuarialBrew.com Exam MFE / 3F Actuarial Models …actuarialbrew.com/data/documents/MFE-Solutions-2014-1st-ed.pdf · Exam MFE / 3F Actuarial Models Financial Economics Segment Solutions

Exam MFE/3F Solutions Chapter 5 – Lognormally Distributed Prices

© ActuarialBrew.com 2014 Page 5.02

Solution 5.03

A Sums of Normal Random Variables

There is no need to calculate the expected value to answer this question, because all of the answer choices have the same expected value. For the sake of thoroughness, we calculate the expected value below.

The expected value of the linear combination is:

1 2 33 2 ( 1) 3 4 15E x x x

The variance of the linear combination is:

[ ][ ]

s s s s s s

s s s r s s r s s r s s

+ + = + + + + +

- + = + + - - ¥ + ¥

= + + ¥ - ¥ ¥ ¥ - ¥ ¥ ¥ ¥ + ¥ ¥ - ¥ ¥=

2 2 2 2 2 21 2 3 1 2 3 1,2 2,3 1,3

2 2 21 2 3 1 2 3 1,2 1 2 2,3 2 3 1,3 1 3

2 2 2

3 9 2 2 3 2 3

4 3 9 6 2 0.3 4 3 2 3 0.4 3 6 2 3 ( 0.5) 4 634.04

Var x bx cx b c b bc c

Var x x x

a a a a

Solution 5.04

D Median Stock Price

The formulas for the expected value and the median are:

a d

a d s

- -

- - -

=

=2

( )( )

( 0.5 )( )

[ ]

Median

T tT t

T tt

E S S e

S e

We can substitute the expected value into the formula for the median:

a d s a d s s- - - - - - - - -= = =2 2 2( 0.5 )( ) ( )( ) 0.5 ( ) 0.5 ( )Median [ ]T t T t T t T t

t t TS e S e e E S e

The median price of the stock in 3 years is:

s- - - -= = =2 20.5 ( ) 0.5(0.3) (3 0)Median [ ] 135 117.95T t

TE S e e

Solution 5.05

A Expected Value

We can use the expected value of the stock price to determine a d-( ) :

a d

a d

a d

- -

- -

=

=- =

( )( )

( )(4 0)

[ ]

1.27125 1( ) 0.0600

T tT t tE S S S e

e

Since the Black-Scholes framework holds, we know that:

a d s sÊ ˆ È ˘- - - - >Á ˜ Î ˚Ë ¯

2 2ln ( 0.5 )( ), ( )T

t

SN T t T t T t

S

Page 127: ActuarialBrew.com Exam MFE / 3F Actuarial Models …actuarialbrew.com/data/documents/MFE-Solutions-2014-1st-ed.pdf · Exam MFE / 3F Actuarial Models Financial Economics Segment Solutions

Exam MFE/3F Solutions Chapter 5 – Lognormally Distributed Prices

© ActuarialBrew.com 2014 Page 5.03

The variance of the natural log of the stock price can be used to determine s :

ss

È ˘ =Î ˚

- ==

2

ln (4) (0) 0.49

(4 0) 0.490.35

Var S S

The expected value of the natural log of the time-8 stock price is:

a d sÈ ˘È ˘ = = ¥ - - -Í ˙Î ˚ Î ˚

= - ¥ = -

2

2

ln (8)ln (8) (0) (0) 1 ( 0.5 )(8 0)

(0)

(0.0600 0.5 0.35 )8 0.01

SE S S S E

S

Solution 5.06

B Stock Price Probabilities

The probability that the future stock price exceeds a specified amount can be found with the following formula:

( )a d s

s

Ê ˆ + - - -Á ˜Ë ¯> = =

-

2

2 2

ln ( 0.5 )( )ˆ ˆProb ( ) where:

t

T

ST t

KS K N d d

T t

We begin by finding:

2 2

2

50ln ( 0.5 )( ) ln (0.10 0 0.5(0.25) )(2 0)

65ˆ 0.25 2 0

0.35317

tST t

Kd

T t

a d s

s

Ê ˆ Ê ˆ+ - - - + - - -Á ˜ Á ˜Ë ¯ Ë ¯= =

- -= -

We can now find the probability that tS is greater than $65:

( )Prob 65 ( 0.35317) 0.36198TS N> = - =

Solution 5.07

E Conditional Expectation

The conditional expected value is:

a d- -

È ˘> =Î ˚( )( )

0 1

2

ˆ( )ˆ( )

T t

T TS e N d

E S S KN d

Page 128: ActuarialBrew.com Exam MFE / 3F Actuarial Models …actuarialbrew.com/data/documents/MFE-Solutions-2014-1st-ed.pdf · Exam MFE / 3F Actuarial Models Financial Economics Segment Solutions

Exam MFE/3F Solutions Chapter 5 – Lognormally Distributed Prices

© ActuarialBrew.com 2014 Page 5.04

The values of 1d and 2d are:

2 2

1

2 2

2

50ln ( 0.5 )( ) ln (0.10 0 0.5(0.25) )(2 0)

65ˆ0.25 2 0

0.00038

50ln ( 0.5 )( ) ln (0.10 0 0.5(0.25) )(2 0)

65ˆ0.25 2 0

0.35317

t

t

ST t

Kd

T t

ST t

Kd

T t

a d s

s

a d s

s

Ê ˆ Ê ˆ+ - + - + - + -Á ˜ Á ˜Ë ¯ Ë ¯= =

- -=

Ê ˆ Ê ˆ+ - - - + - - -Á ˜ Á ˜Ë ¯ Ë ¯= =

- -= -

The values of 1( )N d and 2ˆ( )N d are:

1

2

ˆ( ) (0.00038) 0.50015ˆ( ) ( 0.35317) 0.36198

N d N

N d N

= =

= - =

The conditional expectation is:

(0.10 0)(2 0)

2 250 0.50015

65 84.38100.36198

eE S S

- - ¥È ˘> = =Î ˚

Solution 5.08

E Effect of Increasing Time Until Maturity

If we can find an exception to a statement, then we know that the statement is not true.

Let’s begin by considering the probability that the time t stock price is less than K:

( )a d s

s

Ê ˆÊ ˆ + - -Á ˜Á ˜Ë ¯Á ˜< = - = -Á ˜Á ˜Ë ¯

20

2

ln ( 0.5 )ˆProb ( )t

St

KS K N d N

t

If we can find parameters for a d s0 , , , , and S K such that an increase in t leads to a

decrease in ( )<Prob tS K , then Choice A must be false. To simplify the expression, let’s

choose =0S K , so that we have:

( ) ( ) a d s a d sss

Ê ˆ Ê ˆ+ - - - + +< = - =Á ˜ Á ˜Á ˜ Ë ¯Ë ¯

2 2ln 1 ( 0.5 ) 0.5Prob t

tS K N N t

t

From the above expression we can see that the probability decreases as t increases when the numerator of the expression at right above is negative. We can make the numerator negative by setting the parameters so that:

a d s> + 20.5

Page 129: ActuarialBrew.com Exam MFE / 3F Actuarial Models …actuarialbrew.com/data/documents/MFE-Solutions-2014-1st-ed.pdf · Exam MFE / 3F Actuarial Models Financial Economics Segment Solutions

Exam MFE/3F Solutions Chapter 5 – Lognormally Distributed Prices

© ActuarialBrew.com 2014 Page 5.05

Therefore, Choice A is false.

Furthermore, we can see that the probability increases as t increases when the numerator of the expression is positive. We can make the numerator positive by setting the parameters so that:

a d s< + 20.5

Therefore, Choice B is false.

For Choice C, the conditional expectation is:

a d

a d

a d s

s

a d s

s

-

-

Ê ˆÊ ˆ + - +Á ˜Á ˜Ë ¯Á ˜-Á ˜Á ˜Ë ¯-È ˘< = =Î ˚ Ê ˆÊ ˆ- + - -Á ˜Á ˜Ë ¯Á ˜-

Á ˜Á ˜Ë ¯

20

( )0

( )0 1

202

ln ( 0.5 )

ˆ( )ˆ( )

ln ( 0.5 )

t

t

t t

St

KS e N

tS e N d

E S S KSN d

tK

Nt

To simplify the expression, let’s choose =0S K and a d= , so that we have:

( )

( )( )( )

( )( ) ( )

sss

sss

s

s s

Ê ˆ+-Á ˜Á ˜ -Ë ¯

È ˘< = =Î ˚ Ê ˆ--Á ˜Á ˜Ë ¯

-= = -

2

0

02

0 0 0

ln 1 (0.5 )

0.5

0.5ln 1 0.5 )

1 0.5 1

0.5 0.5

t t

tS N

N ttE S S K S

N ttN

t

N tS S S

N t N t

From the expression above, we can see that the conditional expectation decreases when t increases. Therefore, Choice C is false.

In fact, t tE S S KÈ ˘<Î ˚ always monotonically decreases as t increases, regardless of our

choices for 0S , K , , , and a d s .

For Choice D, the partial expectation is:

a d a da d s

s- -

Ê ˆÊ ˆ + - +Á ˜Á ˜Ë ¯Á ˜È ˘< = - = -Î ˚ Á ˜Á ˜Ë ¯

20

( ) ( )0 1 0

ln ( 0.5 )ˆ( )t t

t t

St

KPE S S K S e N d S e N

t

Page 130: ActuarialBrew.com Exam MFE / 3F Actuarial Models …actuarialbrew.com/data/documents/MFE-Solutions-2014-1st-ed.pdf · Exam MFE / 3F Actuarial Models Financial Economics Segment Solutions

Exam MFE/3F Solutions Chapter 5 – Lognormally Distributed Prices

© ActuarialBrew.com 2014 Page 5.06

To simplify the expression, let’s choose =0S K and a d= , so that we have:

( ) ( )s

ss

Ê ˆ+È ˘< = - = -Á ˜Î ˚ Á ˜Ë ¯

2

0 0ln 1 0.5

0.5t tt

PE S S K S N S N tt

From the expression above, we can see that the partial expectation decreases when t increases. Therefore, Choice D is false.

Since Choices A, B, C, and D are false, Choice E is the correct answer.

Solution 5.09

D Confidence Intervals

For a 90% lognormal confidence interval, we set = 10%p in the expression below:

( )a d s s- - - + -= > =2( 0.5 )( ) where:

2

UU T t z T t UtT

pS S e P z z

First, we use the standard normal calculator to determine Uz :

( )( )

( )( )

0.102

0.05

1 0.05

0.95

1.64485

U

U

U

U

U

P z z

P z z

P z z

P z z

z

> =

> =

- < =

< =

=

The upper bound is:

2(0.16 0 0.5(0.30) )(0.5 0) 0.3(1.64485) 0.5 075 112.608U

TS e - - - + -= =

Solution 5.10

C Probability that Stock Price is Greater Than K

Let F be the fund amount. The time-5 cash flow to the company that sold the put option is:

[ ]- -55(1.05) 0,50F Max S

For this cash flow to be negative, the put option must be in-the-money, meaning that 5S

must be less than 50. Furthermore, the put option’s nonzero payoff must be greater than 5(1.05)F . That is, the following must be negative:

- -55(1.05) (50 )F S

Page 131: ActuarialBrew.com Exam MFE / 3F Actuarial Models …actuarialbrew.com/data/documents/MFE-Solutions-2014-1st-ed.pdf · Exam MFE / 3F Actuarial Models Financial Economics Segment Solutions

Exam MFE/3F Solutions Chapter 5 – Lognormally Distributed Prices

© ActuarialBrew.com 2014 Page 5.07

The company wants the probability of a nonnegative cash flow to be 98%, so:

È ˘- - > =Î ˚È ˘- + > =Î ˚È ˘> - =Î ˚

55

55

55

(1.05) (50 ) 0 0.98

(1.05) 50 0 0.98

50 (1.05) 0.98

P F S

P F S

P S F

We make use of the following formula for the probability of the price of a stock exceeding a threshold:

( )> = 2ˆProb ( )TS K N d with = - 550 (1.05)K F

We can use the normal distribution table to find 2d :

2 2ˆ ˆ( ) 0.98000 2.05375N d d= fi =

We have:

2

2

25

5

0.7043315

ln ( 0.5 )( )ˆ

50ln (0.14 0.02 0.5 0.25 )(5 0)

50 (1.05)2.05375

0.25 5 0

500.704331 ln

50 (1.05)

50

50 (1.05)19.8060

tST t

Kd

T t

F

F

eF

F

a d s

s

Ê ˆ + - - -Á ˜Ë ¯=

-Ê ˆ

+ - - ¥ -Á ˜Ë ¯-=

-Ê ˆ

= Á ˜Ë ¯-

=-

=

Solution 5.11

C Expected Value of Future Stock Price

From the distribution provided, we have:

2 2

0.09 0.30

0.5 0.035 0.035 0.5 0.035 0.03 0.5(0.09) 0.11

The expected value of the stock price at time 4 is:

a d- - ¥= = =( ) (0.11 0.03) 40[ ] 75 103.28t

tE S S e e

Page 132: ActuarialBrew.com Exam MFE / 3F Actuarial Models …actuarialbrew.com/data/documents/MFE-Solutions-2014-1st-ed.pdf · Exam MFE / 3F Actuarial Models Financial Economics Segment Solutions

Exam MFE/3F Solutions Chapter 5 – Lognormally Distributed Prices

© ActuarialBrew.com 2014 Page 5.08

Solution 5.12

A Median Value of Future Stock Price

From the distribution provided, we have:

a d s- - =20.5 0.035

The median value of the stock price at time 4 is:

a d s- - ¥= = =2( 0.5 ) 0.035 4

0Median 75 86.27tS e e

Solution 5.13

B Probability of Future Stock Price

From the distribution provided, we have:

2

0.09 0.30

0.5 0.035

The probability that the stock price at the end of 4 years is less than $75 is:

[ ]< = - 2ˆProb (4) 75 ( )S N d

The value of 2d is:

20

2

75ln ( 0.5 ) ln 0.035 4

75ˆ 0.233330.30 4

St

Kd

t

a d s

s

Ê ˆ Ê ˆ+ - - + ¥Á ˜ Á ˜Ë ¯ Ë ¯= = =

The probability is:

2ˆ( ) ( 0.23333) 0.40775N d N- = - =

Solution 5.14

B Median Value of Future Stock Price

From the distribution provided, we have:

a d s- - =20.5 0.035

The median value of the stock price at time 4 is:

a d s- - ¥= = =2( 0.5 ) 0.035 4

0Median 75 86.27tS e e

Since the investor reinvests the dividends, the investor has the following quantity of shares at the end of 4 years:

d ¥= =0.03 4 1.1275te e

Page 133: ActuarialBrew.com Exam MFE / 3F Actuarial Models …actuarialbrew.com/data/documents/MFE-Solutions-2014-1st-ed.pdf · Exam MFE / 3F Actuarial Models Financial Economics Segment Solutions

Exam MFE/3F Solutions Chapter 5 – Lognormally Distributed Prices

© ActuarialBrew.com 2014 Page 5.09

The median value of the investor’s position at the end of 4 years is the quantity of shares owned times the median stock price:

¥ =1.1275 86.27 97.27

Solution 5.15

E Probability that Stock Price is Less Than K

The probability that the stock price at the end of T years is lower than K is:

( )< = - 2ˆProb ( )TS K N d

Since for = 0K S this probability is greater than 50%, the value of 2d must be negative:

- > fi - > fi <2 2 2ˆ ˆ ˆ( ) 50% 0 0N d d d

Setting K equal to the initial price, we have the following value of 2d :

a d sa d s

ss

Ê ˆ+ - -Á ˜Ë ¯ - -= =

202

02

ln ( 0.5 )( )( 0.5 )ˆ

ST

Sd T

T

Since T is positive, for 2d to be negative, the following expression must be negative:

a d s

s- - 2( 0.5 )

Let’s evaluate the expression for each of the choices:

Choice A: - - ¥ =2(0.10 0.00 0.5 0.40 ) / 0.40 0.05

Choice B: - - ¥ =2(0.11 0.02 0.5 0.40 ) / 0.40 0.025

Choice C: - - ¥ =2(0.12 0.02 0.5 0.30 ) / 0.30 0.1833

Choice D: - - ¥ =2(0.13 0.04 0.5 0.40 ) / 0.40 0.025

Choice E: 2(0.14 0.05 0.5 0.45 ) / 0.45 0.025

Since the expression is negative for Choice E, the parameterization in Choice E implies the probability that the time T stock price is less than the initial price is greater than 50%.

Solution 5.16

A Median Value of Future Stock Price

The expected stock price is:

a d-= ( )0[ ] t

tE S S e

Page 134: ActuarialBrew.com Exam MFE / 3F Actuarial Models …actuarialbrew.com/data/documents/MFE-Solutions-2014-1st-ed.pdf · Exam MFE / 3F Actuarial Models Financial Economics Segment Solutions

Exam MFE/3F Solutions Chapter 5 – Lognormally Distributed Prices

© ActuarialBrew.com 2014 Page 5.10

The median stock price is:

a d s- -=2( 0.5 )

0Median tS e

The median stock price can be written in terms of the expected stock price:

a d s s- - -= =2 2( ) 0.5 0.5

0Median [ ]t t ttS e e E S e

Setting = 5t , the median is:

- -= = ¥ =0.5(0.16)5 0.405Median [ ] 130 87.14E S e e

Solution 5.17

C One Standard Deviation Move

The stock price can be expressed in terms of a standard normal random variable z:

a d s s- - - + -= 2( 0.5 )( ) where: (0,1)T t z T t

T tS S e z N

A one standard deviation move up is obtained by setting = 1z :

a d s s- - - + -

- - ¥ +

=

==

2

2

( 0.5 )( ) ( )

(0.13 0.02 0.5 0.25 )0.5 0.25 0.5

One standard deviation move up for

100124.13

T t T tT tS S e

e

Solution 5.18

B One Standard Deviation Move

The stock price can be expressed in terms of a standard normal random variable z:

a d s s- - - + -= 2( 0.5 )( ) where: (0,1)T t z T t

T tS S e z N

We can use the prices from the table to solve for a and s . Let’s use the prices from the bottom row that correspond to 1 standard deviation moves up and down:

a s s

a s s

- -

- +

=

=

2

2

( 0.5 )5 5

( 0.5 )5 5

74.39 100

284.57 100

e

e

Dividing the second equation by the first equation, we can solve for s and then for a :

s

a

s

a- ¥ -

= fi =

= fi =2

2 5

( 0.5 0.3 )5 0.3 5

284.570.30

74.39

74.39 100 0.12

e

e

Page 135: ActuarialBrew.com Exam MFE / 3F Actuarial Models …actuarialbrew.com/data/documents/MFE-Solutions-2014-1st-ed.pdf · Exam MFE / 3F Actuarial Models Financial Economics Segment Solutions

Exam MFE/3F Solutions Chapter 5 – Lognormally Distributed Prices

© ActuarialBrew.com 2014 Page 5.11

A one standard deviation move down over 1 year is obtained by setting = -1z and - =( ) 1T t :

a d s s- - - - -

- - ¥ -

=

==

2

2

( 0.5 )( ) ( )

(0.12 0.00 0.5 0.30 )1 0.30 1

One standard deviation move down for

10079.85

T t T tT tS S e

e

Solution 5.19

C Two Standard Deviation Move

Notice that 1 month is expressed as 0.0849 of one year. For Exam MFE/3F we usually express one month as the following quantity of years:

1 0.083312

=

But in this question, we use:

31 0.0849365

=

The stock price can be expressed in terms of a standard normal random variable z:

a d s s- - - + -= 2( 0.5 )( ) where: (0,1)T t z T t

T tS S e z N

We can use the prices from the table to solve for a and s . Let’s use the prices from the bottom row that correspond to 1 standard deviation moves up and down:

a s s

a s s

- -

- +

=

=

2

2

( 0.5 )5 5

( 0.5 )5 5

74.39 100

284.57 100

e

e

Dividing the second equation by the first equation, we can solve for s and then for a :

s

a

s

a- ¥ -

= fi =

= fi =2

2 5

( 0.5 0.3 )5 0.3 5

284.570.30

74.39

74.39 100 0.12

e

e

A two standard deviation move down over one month is obtained by setting = -2z and - =( ) 31 / 365T t :

a d s s- - - - -

- - ¥ ¥ - ¥

=

==

2

2 31 31365 365

( 0.5 )( ) 2 ( )

(0.12 0.00 0.5 0.30 ) 2 0.30

Two standard deviation move down for

10084.49

T t T tT tS S e

e

Page 136: ActuarialBrew.com Exam MFE / 3F Actuarial Models …actuarialbrew.com/data/documents/MFE-Solutions-2014-1st-ed.pdf · Exam MFE / 3F Actuarial Models Financial Economics Segment Solutions

Exam MFE/3F Solutions Chapter 5 – Lognormally Distributed Prices

© ActuarialBrew.com 2014 Page 5.12

Solution 5.20

D Two Standard Deviation Move

From the table we observe that if a 2 standard deviation move down occurs after one day, then the new stock price will be $96.93 at the end of one day.

For the stock price to be less than $96.93 at the end of one day, the stock price must move by more than 2 standard deviations down. This is equivalent to z moving more than 2 standard deviations down in the following expression:

a d s s- - +

+ = 2 1 1

365 3651

365

( 0.5 ) where: (0,1)

zttS S e z N

The probability of z moving more than two standard deviations down is:

Prob( 2.00000) ( 2.00000) 0.02275z N< - = - =

Solution 5.21

E Effect of Increasing Time to Maturity

The probability that the put option expires in the money is:

[ ]< = - =2 0ˆProb ( ) where: rT

TS K N d K S e

The value of 2d is:

a sa d s

s sa s

s

Ê ˆÊ ˆ + - -Á ˜+ - -Á ˜Ë ¯ Ë ¯= =

- -=

2020

02

2

ln ( 0 0.5 )ln ( 0.5 )ˆ

[( ) 0.5 ]

rT

SS TTS eK

dT T

r T

The probability that the put option expires in the money is:

s a

sÊ ˆ- -È ˘< = - = Á ˜Î ˚ Ë ¯

2

0 20.5 ( )ˆProb ( )rT

Tr

S S e N d N T

The question tells us that the risk premium is greater than s 20.5 :

s a s a< - fi - - <2 20.5 0.5 ( ) 0r r

Therefore - 2d is negative:

s a

s- - <

20.5 ( )0

rT

This implies that - 2ˆ( )N d is less than 50%. Therefore, Statement E is false.

We could stop here, but let’s go through the other choices as well.

Page 137: ActuarialBrew.com Exam MFE / 3F Actuarial Models …actuarialbrew.com/data/documents/MFE-Solutions-2014-1st-ed.pdf · Exam MFE / 3F Actuarial Models Financial Economics Segment Solutions

Exam MFE/3F Solutions Chapter 5 – Lognormally Distributed Prices

© ActuarialBrew.com 2014 Page 5.13

We have established that the numerator in the fraction below is negative:

s a

sÊ ˆ- -È ˘< = Á ˜Î ˚ Ë ¯

2

00.5 ( )

Prob rTT

rS S e N T

Therefore, the value in the parentheses becomes more negative as T increases, meaning that an increase in T decreases the probability that the put option expires in the money. Therefore Statement A is true.

The value in the parenthesis becomes more negative as a increases, so an increase in a decreases the probability that the put option expires in the money. Therefore, Statement D is true.

The risk-neutral probability is obtained by substituting r for a :

s s

s sÊ ˆ Ê ˆ- - =Á ˜ Á ˜Ë ¯ Ë ¯

2 20.5 ( ) 0.5r rN T N T

The value in the parentheses is now positive, meaning that an increase in T increases the risk-neutral probability that the put option expires in the money. Therefore, Statement C is true.

Statement B is true because options on nondividend-paying stocks increase in value as their time to maturity increases when the strike price grows at the risk-free interest rate. This was covered in Chapter 2 of the Study Manual.

Solution 5.22

A Comparing Stock with a Risk-free Bond

The probability that Pete’s investment outperforms Jim’s investment is equal to the probability that a risk-free investment outperforms the stock:

[ ]< = - =2ˆProb ( ) where: 100 rT

TS K N d K e

The value of 2d is:

a sa d s

s sa s

s

Ê ˆÊ ˆ + - -+ - -Á ˜ Á ˜Ë ¯ Ë ¯= =

- -=

220

2

2

100ln ( 0 0.5 )ln ( 0.5 )

100ˆ

[( ) 0.5 ]

rTS

TTK ed

T T

r T

The probability that Pete’s investment outperforms Jim’s investment is:

s a

sÊ ˆ- -È ˘< = - = Á ˜Î ˚ Ë ¯

2

0 20.5 ( )ˆProb ( )rT

Tr

S S e N d N T

Page 138: ActuarialBrew.com Exam MFE / 3F Actuarial Models …actuarialbrew.com/data/documents/MFE-Solutions-2014-1st-ed.pdf · Exam MFE / 3F Actuarial Models Financial Economics Segment Solutions

Exam MFE/3F Solutions Chapter 5 – Lognormally Distributed Prices

© ActuarialBrew.com 2014 Page 5.14

The question tells us that this probability is equal to 50%:

s a

sÊ ˆ- - =Á ˜Ë ¯

20.5 ( )50%

rN T

This implies that the value in parentheses above is 0:

2

2

2

2

0.5 ( )0

0.5 ( ) 0

0.5 (0.10 0.08) 0

0.040.20

rT

r

Solution 5.23

D Conditional and Partial Expectations

The expected value of the call option at time 1 is:

[ ] { } ( )È ˘= > - ¥ >Î ˚1 1 1 1Call 102 102 Prob 102E E S S S

The probability that the final stock price is above 102 is found using the relationship below:

( )

( )

È ˘>Î ˚È ˘> =Î ˚ >

È ˘>Î ˚> = = =È ˘>Î ˚

1 11

1 1

Prob

102 68.98Prob 102 0.51986

132.69102

T TT T

T

PE S S KE S S K

S K

PE S SS

E S S

The expected value of the call at time 1 is therefore:

[ ] { } ( )È ˘= > - ¥ > = - ¥Î ˚=

1 1 1 1Call 102 102 Prob 102 {132.69 102} 0.51986

15.9545

E E S S S

The initial value of the call is:

[ ] - -= = ¥ =0.258 0.2580 1 15.9545 12.33Call E Call e e

Solution 5.24

E Conditional and Partial Expectations

The risk-neutral partial expectation of the stock price, conditional on the stock price being below 102 is:

È ˘< = = =Î ˚ Ú Ú102

* * *1 1 0.5 1

0 0

( ; ) ( ; ) 45.94K

T T T T t TPE S S K S g S S dS S g S S dS

Page 139: ActuarialBrew.com Exam MFE / 3F Actuarial Models …actuarialbrew.com/data/documents/MFE-Solutions-2014-1st-ed.pdf · Exam MFE / 3F Actuarial Models Financial Economics Segment Solutions

Exam MFE/3F Solutions Chapter 5 – Lognormally Distributed Prices

© ActuarialBrew.com 2014 Page 5.15

The risk-neutral probability that the final stock price is below 102 is found using the relationship below:

( )

( )

È ˘<Î ˚È ˘< =Î ˚ <

È ˘<Î ˚< = = =È ˘<Î ˚

**

*

*1 1*

1 *1 1

Prob

102 45.94Prob 102 0.5325

86.27102

T TT T

T

PE S S KE S S K

S K

PE S SS

E S S

The risk-neutral expected value of the put at time 1 is therefore:

[ ] { } ( )

{ }È ˘= - < ¥ <Î ˚

= - ¥ =

* * *Put Payoff Prob

102 86.27 0.5325 8.3764

T T TE K E S S K S K

Since we used the risk-neutral probability to obtain this expected value, we can use the risk-free rate of return to discount it back to time 0.5:

- ¥¥ =0.09 0.58.3764 8.01e

Solution 5.25

B Partial Expectation

The partial expectation of the stock price, conditional on the stock price being at least $48 is:

a d- - - ¥È ˘> = = =Î ˚( )( ) (0.13 0.04) 0.5

1 1 1ˆ ˆ ˆ( ) 50 ( ) 52.3014 ( )T t

T T tPE S S K S e N d e N d N d

The value of 1d (rounded to two places) is:

2 2

1

50ln ( 0.5 )( ) ln (0.13 0.04 0.5(0.3) )(0.5)

48ˆ 0.510630.3 0.5

tST t

Kd

T t

a d s

s

Ê ˆ Ê ˆ+ - + - + - +Á ˜ Á ˜Ë ¯ Ë ¯= = =

-

The value of 1( )N d is:

1( ) (0.51063) 0.69519N d N= =

The partial expectation is:

152.3014 ( ) 52.3014 0.69519 36.3594N d = ¥ =

Page 140: ActuarialBrew.com Exam MFE / 3F Actuarial Models …actuarialbrew.com/data/documents/MFE-Solutions-2014-1st-ed.pdf · Exam MFE / 3F Actuarial Models Financial Economics Segment Solutions

Exam MFE/3F Solutions Chapter 5 – Lognormally Distributed Prices

© ActuarialBrew.com 2014 Page 5.16

Solution 5.26

B Conditional Expectation

The values of 1d (rounded to two places) and 1( )N d are:

2 2

1

1

50ln ( 0.5 )( ) ln (0.13 0.04 0.5(0.3) )(0.5)

48ˆ 0.510630.3 0.5

ˆ( ) (0.51063) 0.69519

tST t

Kd

T t

N d N

a d s

s

Ê ˆ Ê ˆ+ - + - + - +Á ˜ Á ˜Ë ¯ Ë ¯= = =

-

= =

The values of 2d and 2ˆ( )N d are:

2 2

2

2

50ln ( 0.5 )( ) ln (0.13 0.04 0.5(0.3) )(0.5)

48ˆ 0.298500.3 0.5

ˆ( ) (0.29850) 0.61734

tST t

Kd

T t

N d N

a d s

s

Ê ˆ Ê ˆ+ - - - + - -Á ˜ Á ˜Ë ¯ Ë ¯= = =

-

= =

The conditional expectation is:

( )( ) (0.13 0.04) 0.5

1

2

ˆ( ) 50 0.6951948 58.8969

ˆ 0.61734( )

T tt

T TS e N d e

E S SN d

a d- - - ¥ ¥È ˘> = = =Î ˚

Solution 5.27

C Partial Expectation

The values of 1d (rounded to two places) and 1( )N d are:

202

01

1

ln ( 0.5 )( )0 (0.09 0.03 0.5(0.25) )(3)ˆ 0.63220

0.25 3ˆ( ) (0.63220) 0.73637

ST t

Sd

T t

N d N

a d s

s

Ê ˆ+ - + -Á ˜Ë ¯ + - += = =

-

= =

We can use the formula for the partial expectation to solve for the current stock price:

( )( )1

(0.09 0.03)33 3 0 0 1

(0.09 0.03)30

0

ˆ( )

ˆ( )

85.88 0.73637

97.4144

T tT T tPE S S K S e N d

PE S S S S e N d

S e

S

Page 141: ActuarialBrew.com Exam MFE / 3F Actuarial Models …actuarialbrew.com/data/documents/MFE-Solutions-2014-1st-ed.pdf · Exam MFE / 3F Actuarial Models Financial Economics Segment Solutions

Exam MFE/3F Solutions Chapter 5 – Lognormally Distributed Prices

© ActuarialBrew.com 2014 Page 5.17

Solution 5.28

A Conditional and Partial Expectations

The partial expectation of the stock price in 3 months, conditional on the stock price being less than K is the conditional expectation times the probability of the condition:

( )È ˘ È ˘< = < ¥ < = ¥ -Î ˚ Î ˚=

0.25 0.25 0.25 0.25 0.25Prob 67.69 (1 0.60)

27.076

PE S S K E S S K S K

Since the conditions are mutually exclusive and exhaustive, we can add the two partial expectations to obtain the expected value of the stock price in 3 months:

[ ] ( ) ( )< >

È ˘ È ˘= < ¥ < + > ¥ >Î ˚ Î ˚

= +=

Partial expectation conditional on Partial expectation conditional on

Prob Prob

27.076 52.0179.086

T T

T T T T T T T

S K S K

E S E S S K S K E S S K S K

The expected return on the stock is 17%, so:

¥¥

-

-

=

=

= ¥=

0.02 0.250.17 0.25

00.0375

00.0375

0

0

[ ]

[ ]

79.086

76.18

T

T

e E Se

S

S e E S

S e

S

Solution 5.29

D The Lognormal Distribution

Products of lognormal random variables are lognormally distributed, if the logs of the random variables are jointly normal. The sums of lognormal random variables are not lognormally distributed.

Solution 5.30

E The Normal Distribution

In the binomial model, the continuously compounded return approaches normality as the number of steps becomes large.

Page 142: ActuarialBrew.com Exam MFE / 3F Actuarial Models …actuarialbrew.com/data/documents/MFE-Solutions-2014-1st-ed.pdf · Exam MFE / 3F Actuarial Models Financial Economics Segment Solutions

Exam MFE/3F Solutions Chapter 5 – Lognormally Distributed Prices

© ActuarialBrew.com 2014 Page 5.18

Solution 5.31

B Probability of Future Stock Price

The stock will expire in the money if its price exceeds $125 in 2 years. The probability that the call will expire in the money is:

2

2

2

ln( / ) ( 0.5 )ˆ( ) ( )

ln(100 /125) (0.15 0.03 0.5(0.3) )2

0.3 2

0.17240

0.43156

tt

S K tP S K N d N

t

N

N

Solution 5.32

D Conditional Expectation

Given that the call is in the money at expiration, the expected stock price is:

100

3 1Prob( ) 105 125 55

8 8t

t tS

S S

This is the partial expectation of the stock price conditional upon the stock price at expiration being greater than $100. The conditional expectation of the stock price assuming the call is in the money at expiration is:

100

1 1Prob( ) 55 110

Prob( 100) 0.5t

t tt S

S SS

Solution 5.33

B Confidence Intervals

Since t = 0.5, the semiannual continuously compounded mean return is:

2 20.5 0.12 0.0 0.5 0.25 0.5 0.044375t

The semiannual standard deviation is:

0.25 0.5 0.176777t

Page 143: ActuarialBrew.com Exam MFE / 3F Actuarial Models …actuarialbrew.com/data/documents/MFE-Solutions-2014-1st-ed.pdf · Exam MFE / 3F Actuarial Models Financial Economics Segment Solutions

Exam MFE/3F Solutions Chapter 5 – Lognormally Distributed Prices

© ActuarialBrew.com 2014 Page 5.19

We use the standard normal calculator to determine Uz :

( )( )

( )( )

0.052

0.025

1 0.025

0.975

1.95996

U

U

U

U

U

P z z

P z z

P z z

P z z

z

> =

> =

- < =

< =

=

For a standard normal distribution, there is a 95% chance of drawing a number in the interval (–1.95996, 1.95996). Over a six month horizon, there is a 95% chance that the return will be:

0.044375 1.95996 0.176777 0.302100 and 0.390850

The 6-month return has a 95% confidence interval of (–0.302100, 0.390850). We need to determine the 95% confidence interval for the stock price in 6 months, so we exponentiate these returns and multiply them by the initial stock price of $75:

0.5

0.302100 0.3908500.5

0.5

ln( )0.302100 0.390850

75

75 75

55.4448 110.8678

S

e S e

S

Solution 5.34

A Covariance of tS and TS

To answer this question, we use the following formulas:

a d

a d s

- -

- - -

È ˘=Í ˙

Î ˚Ê ˆÈ ˘ = -Î ˚ Ë ¯

È ˘È ˘= Í ˙ Î ˚

Î ˚

2

( )( )

2 2( )( ) ( )

0

1

[ , ]

T tT

t

T t T tT t t

Tt T t

t

SE e

S

Var S S S e e

SCov S S E Var S S

S

The variance of (2)A is:

[ ] { }= + +1(2) [ (1) [ (2)] 2 [ (1), (2)]

4Var A Var S Var S Cov S S

Page 144: ActuarialBrew.com Exam MFE / 3F Actuarial Models …actuarialbrew.com/data/documents/MFE-Solutions-2014-1st-ed.pdf · Exam MFE / 3F Actuarial Models Financial Economics Segment Solutions

Exam MFE/3F Solutions Chapter 5 – Lognormally Distributed Prices

© ActuarialBrew.com 2014 Page 5.20

The variances of (1)S and (2)S are:

- -

- -

Ê ˆÈ ˘ = - =Î ˚ Ë ¯

Ê ˆÈ ˘ = - =Î ˚ Ë ¯

2

2

2 2(0.04)(1 0) 0.4 (1 0)1 0

2 2(0.04)(2 0) 0.4 (2 0)2 0

10 1 18.7962

10 1 44.2564

Var S S e e

Var S S e e

The covariance is:

-È ˘È ˘= = ¥ =Í ˙ Î ˚

Î ˚0.04(2 1)2

1 2 1 01

[ , ] 18.7962 19.5633S

Cov S S E Var S S eS

We can now find the variance of (2)A :

[ ] { }

{ }

= + +

= + + ¥ =

1(2) [ (1) [ (2)] 2 [ (1), (2)]

41

18.7962 44.2564 2 19.5633 25.54484

Var A Var S Var S Cov S S

Solution 5.35 B Covariance of tS and TS

To answer this question, we use the following formulas:

a d

a d s

- -

- - -

È ˘=Í ˙

Î ˚Ê ˆÈ ˘ = -Î ˚ Ë ¯

È ˘È ˘= Í ˙ Î ˚

Î ˚

2

( )( )

2 2( )( ) ( )

0

1

[ , ]

T tT

t

T t T tT t t

Tt T t

t

SE e

S

Var S S S e e

SCov S S E Var S S

S

The variance of X is:

[ ] = + + ¥ ¥ -16 [ (2)] [ (3)] 2 4 ( 1) [ (2), (3)]Var X Var S Var S Cov S S

The variances of (2)S and (3)S are:

- -

- -

Ê ˆÈ ˘ = - =Î ˚ Ë ¯

Ê ˆÈ ˘ = - =Î ˚ Ë ¯

2

2

2 2(0.06)(2 0) 0.3 (2 0)2 0

2 2(0.06)(3 0) 0.3 (3 0)3 0

2 1 1.0028

2 1 1.7771

Var S S e e

Var S S e e

The covariance is:

-È ˘È ˘= = ¥ =Í ˙ Î ˚

Î ˚0.06(3 2)3

2 3 2 02

[ , ] 1.0028 1.0649S

Cov S S E Var S S eS

We can now find the variance of X :

[ ] = + + ¥ ¥ -= ¥ + - ¥ =

16 [ (2) [ (3)] 2 4 ( 1) [ (2), (3)]

16 1.0028 1.7771 8 1.0659 9.3038

Var X Var S Var S Cov S S

Page 145: ActuarialBrew.com Exam MFE / 3F Actuarial Models …actuarialbrew.com/data/documents/MFE-Solutions-2014-1st-ed.pdf · Exam MFE / 3F Actuarial Models Financial Economics Segment Solutions

Exam MFE/3F Solutions Chapter 6 – Histograms and Normal Probability Plots

© ActuarialBrew.com 2014 Page 6.01

Chapter 6 – Solutions

Solution 6.01

D Order Statistics

The order statistics are the result of sorting the data. The order statistics are {2, 3, 5, 8, 10} . The fourth order statistic is 8.

Solution 6.02

A Quantiles

The second draw from {8, 2, 10, 5, 3} is 2.

The order statistics are the result of sorting the data. The order statistics are {2, 3, 5, 8, 10} .

We have 5 observed data points, so the probability ranges are of the following size:

=100%20%

5

The ranges are therefore 0 – 20%, 20% – 40%, 40% – 60%, 60% – 80%, and 80% – 100%. We use the midpoint of each range as a cumulative probability and assign it to its corresponding order statistic:

Order Statistics (Quantiles) Cumulative Probability 2 10% 3 30% 5 50% 8 70% 10 90%

Although the quantiles are the order statistics, the Derivatives Markets textbook uses the term “quantile value” to refer to the probability associated with a quantile. Therefore, the quantile value associated with 2 is 10%.

Solution 6.03

E Quantiles

The order statistics are the result of sorting the data. The order statistics are {2, 3, 5, 8, 10} .

We have 5 observed data points, so the probability ranges are of the following size:

=100%20%

5

Page 146: ActuarialBrew.com Exam MFE / 3F Actuarial Models …actuarialbrew.com/data/documents/MFE-Solutions-2014-1st-ed.pdf · Exam MFE / 3F Actuarial Models Financial Economics Segment Solutions

Exam MFE/3F Solutions Chapter 6 – Histograms and Normal Probability Plots

© ActuarialBrew.com 2014 Page 6.02

The ranges are therefore 0 – 20%, 20% – 40%, 40% – 60%, 60% – 80%, and 80% – 100%. We use the midpoint of each range as a cumulative probability and assign it to its corresponding order statistic:

Order Statistics (Quantiles) Cumulative Probability 2 10% 3 30% 5 50% 8 70% 10 90%

The 90% quantile is 10.

Solution 6.04

C Black-Scholes Model

Statement II is false. There are too many large returns in histograms and normal probability plots for daily and weekly compounded returns when compared to normally distributed returns.

Solution 6.05

A Quantiles

The 20% quantile is the value such that there is a 20% chance that a draw from the standard normal distribution is less than that number.

This question can be answered very quickly using the online normal distribution calculator. We see that P(z < x) = N-1(x) = 20%, so the 20% quantile is x = –0.84162.

Using the printed normal distribution table, we notice that the bottom of the MFE/3F standard normal table provides values of z for selected values of P(Z < z). From the bottom of the table, we see that P(Z < 0.8420) = 80%, so the 80% quantile is 0.8420. Due to the symmetric nature of the normal distribution, we have P(Z < –0.8420) = 20%, so the 20% quantile is –0.8420.

Alternatively, we can also determine the answer from the body of the MFE/3F printed standard normal table. Scanning the body of the table for 20%, we see that only values greater then 50% are provided. Since the normal distribution is symmetric, we look for 80% instead. We see that the closest we can get to 80% in the table is 0.7995, which corresponds with a z of 0.84. This means that P(Z < 0.84) = 79.95%, i.e., the 79.95% quantile is 0.84. Due to symmetry, P(Z < –0.84) = 1 – 0.7995 = 0.2005. So the 20.05% quantile is –0.84, which matches one of the answer choices.

Page 147: ActuarialBrew.com Exam MFE / 3F Actuarial Models …actuarialbrew.com/data/documents/MFE-Solutions-2014-1st-ed.pdf · Exam MFE / 3F Actuarial Models Financial Economics Segment Solutions

Exam MFE/3F Solutions Chapter 7 – The Black-Scholes Formula

© ActuarialBrew.com 2014 Page 7.01

Chapter 7 – Solutions

Solution 7.01

D Black-Scholes Call Price

The question doesn’t specifically tell us to use the Black-Scholes formula, but we can’t use the binomial model since we are not told the number of binomial periods. Therefore, we must use the Black-Scholes formula. This situation is not common, but it has appeared on the exam in the past. See, for example, Question #5 of the Spring 2007 Exam.

The first step is to calculate 1d and 2d :

2 2

1

2 1

ln( / ) ( 0.5 ) ln(46 / 45) (0.08 0.02 0.5 0.35 ) 0.25

0.35 0.250.29881

0.29881 0.35 0.25 0.12381

S K r Td

T

d d T

d ss

s

+ - + + - + ¥ ¥= =

=

= - = - =

In the Formula document for the MFE/3F exam, the SOA states that students should use 5 decimal places for both the inputs and the outputs of the online normal distribution calculator, so we will follow that convention in our Solutions.

We have:

1( ) (0.29881) 0.61746N d N

2( ) (0.12381) 0.54927N d N

The value of the European call option is:

1 20.02(0.25) 0.08(0.25)

( ) ( )

46 0.61746 45 0.54927 4.03378

T rTEurC Se N d Ke N d

e e

Solution 7.02

B Black-Scholes Put Price

The first step is to calculate 1d and 2d :

2 2

1

2 1

ln( / ) ( 0.5 ) ln(46 / 45) (0.08 0.02 0.5 0.35 ) 0.25

0.35 0.250.29881

0.29881 0.35 0.25 0.12381

S K r Td

T

d d T

d ss

s

+ - + + - + ¥ ¥= =

=

= - = - =

We have:

1( ) (0.29981) 0.61746N d N

2( ) (0.12381) 0.54927N d N

Page 148: ActuarialBrew.com Exam MFE / 3F Actuarial Models …actuarialbrew.com/data/documents/MFE-Solutions-2014-1st-ed.pdf · Exam MFE / 3F Actuarial Models Financial Economics Segment Solutions

Exam MFE/3F Solutions Chapter 7 – The Black-Scholes Formula

© ActuarialBrew.com 2014 Page 7.02

The value of the European put option is:

2 10.08(0.25) 0.02(0.25)

( ) ( )

45 (1 0.54927) 46 (1 0.61746)2.37215

rT TEurP Ke N d Se N d

e e

Solution 7.03

A Black-Scholes Formula Using Prepaid Forward Prices

The prepaid forward price of the stock is:

( 0.08 1) /120, 0 0,( ) ( ) 46 5 41.0332P

T TF S S PV Div e

The volatility of the prepaid forward price is:

s = 0.3924PF

The values of 1d and 2d are:

0, 2 20.08(0.25)0,

1

2 1

( ) 41.0332ln 0.5 ln 0.5 0.3924 0.25( ) 450.3924 0.25

0.27030

0.27030 0.3924 0.25 0.46650

PT

PFPT

PF

PF

F ST

F K edT

d d T

s

s

s

-

Ê ˆÊ ˆ+Á ˜ + ¥ ¥Á ˜ Á ˜Ë ¯ Ë ¯

= =

= -

= - = - - = -

We have:

1( ) ( 0.27030) 0.39346N d N

2( ) ( 0.46650) 0.32043N d N

The value of the European call option is:

0, 1 0, 2( ) ( ) ( ) ( ) 41.0332 0.39346 44.1089 0.32043

2.0111

P PEur T TC F S N d F K N d

Solution 7.04

B Options on Currencies

The values of 1d and 2d are:

2 20

1

2 1

ln( / ) ( 0.5 ) ln(1.25 /1.3) (0.06 0.035 0.5 0.11 )(0.5)

0.11 0.50.30464

0.30464 0.11 0.5 0.38243

fx K r r Td

T

d d T

Page 149: ActuarialBrew.com Exam MFE / 3F Actuarial Models …actuarialbrew.com/data/documents/MFE-Solutions-2014-1st-ed.pdf · Exam MFE / 3F Actuarial Models Financial Economics Segment Solutions

Exam MFE/3F Solutions Chapter 7 – The Black-Scholes Formula

© ActuarialBrew.com 2014 Page 7.03

We have:

1( ) ( 0.30464) 0.38032N d N

2( ) ( 0.38243) 0.35107N d N

The value of the call option is:

0 1 20.035(0.5) 0.06(0.5)

( ) ( )

1.25 0.38032 1.30 0.35107 0.024245

fr T rTEurC x e N d Ke N d

e e

Solution 7.05

C Options on Currencies

The values of 1d and 2d are:

2 20

1

2 1

ln( / ) ( 0.5 ) ln(1.25 /1.3) (0.06 0.035 0.5 0.11 )(0.5)

0.11 0.50.30464

0.30464 0.11 0.5 0.38243

fx K r r Td

T

d d T

We have:

1( ) ( 0.30464) 0.38032N d N

2( ) ( 0.38243) 0.35107N d N

The put price can be determined directly:

2 0 10.06(0.5) 0.035(0.5)

( ) ( )

1.30 (1 0.35107) 1.25 (1 0.38032) 0.05751

fr TrTEurP Ke N d x e N d

e e

Alternatively, we can find the value of the otherwise equivalent call option and then use put-call parity to find the value of the put option. The value of the call option is:

0 1 20.035(0.5) 0.06(0.5)

( ) ( )

1.25 0.38032 1.30 0.35107 0.024245

fr T rTEurC x e N d Ke N d

e e

We can now use put-call parity to find the value of the put option:

00.06(0.5) 0.035(0.5)0.024245 1.30 1.25

0.05751

fr TrTEur EurP C Ke x e

e e

Page 150: ActuarialBrew.com Exam MFE / 3F Actuarial Models …actuarialbrew.com/data/documents/MFE-Solutions-2014-1st-ed.pdf · Exam MFE / 3F Actuarial Models Financial Economics Segment Solutions

Exam MFE/3F Solutions Chapter 7 – The Black-Scholes Formula

© ActuarialBrew.com 2014 Page 7.04

Solution 7.06

C Options on Currencies

Since the put option is euro-denominated, we use the euro as the base currency. This means that the current exchange rate is:

01

0.801.25

x = =

Since the option is at-the-money, the strike price is also 0.80 €/$:

0.80K =

We also have:

0.050.08f

rr==

The values of 1d and 2d are:

2 20

1

2 1

ln( / ) ( 0.5 ) ln(0.80 / 0.80) (0.05 0.08 0.5 0.10 ) 1

0.10 10.25000

0.25000 0.10 1 0.35000

fx K r r Td

T

d d T

The values of 1( )N d and 2( )N d are:

1

2

( ) ( 0.25000) 0.40129

( ) ( 0.35000) 0.36317

N d N

N d N

= - == - =

The put price can be determined directly:

2 0 10.05(1) 0.08(1)

( ) ( )

0.80 (1 0.36317) 0.80 (1 0.40129) 0.04247

fr TrTEurP Ke N d x e N d

e e

Alternatively, we can find the value of the otherwise equivalent call option and then use put-call parity to find the value of the put option. The value of the call option is:

0 1 20.08(1) 0.05(1)

( ) ( )

0.80 0.40129 0.80 0.36317 0.01998

fr T rTEurC x e N d Ke N d

e e

We can now use put-call parity to find the value of the put option:

00.05(1) 0.08(1)0.01998 0.80 0.80

0.04247

fr TrTEur EurP C Ke x e

e e

--

- -

= + -

= + -=

Page 151: ActuarialBrew.com Exam MFE / 3F Actuarial Models …actuarialbrew.com/data/documents/MFE-Solutions-2014-1st-ed.pdf · Exam MFE / 3F Actuarial Models Financial Economics Segment Solutions

Exam MFE/3F Solutions Chapter 7 – The Black-Scholes Formula

© ActuarialBrew.com 2014 Page 7.05

Solution 7.07

C Options on Futures

The 1-year futures price is $60:

0,1 60F =

The values of 1d and 2d are:

2 20,

1

2 1

ln / 0.5 ln 60 / 60 0.5(0.25) (1)0.12500

0.25 1

0.12500 0.25 1 0.12500

FTF K Td

T

d d T

We have:

1( ) (0.12500) 0.54974N d N

2( ) ( 0.12500) 0.45026N d N

The value of the call option is:

0, 0, 1 2

0.08(1) 0.08(1)

( , , , , , ) ( ) ( )

60 0.54974 60 0.450265.5099

F FrT rT

Eur T TC F K r T r F e N d Ke N d

e e

Solution 7.08

B Options on Futures

The 1-year futures price is $60:

0,1 60F =

The values of 1d and 2d are:

2 20,

1

2 1

ln / 0.5 ln 60 / 60 0.5(0.25) (1)0.12500

0.25 1

0.12500 0.25 1 0.12500

FTF K Td

T

d d T

We have:

1( ) (0.12500) 0.54974N d N

2( ) ( 0.12500) 0.45026N d N

The put price is:

0, 2 0, 1

0.08(1) 0.08(1)

( , , , , , ) ( ) ( )

60 (1 0.45026) 60 (1 0.54974)5.5099

F FrT rT

Eur T TP F K r T r Ke N d F e N d

e e

Page 152: ActuarialBrew.com Exam MFE / 3F Actuarial Models …actuarialbrew.com/data/documents/MFE-Solutions-2014-1st-ed.pdf · Exam MFE / 3F Actuarial Models Financial Economics Segment Solutions

Exam MFE/3F Solutions Chapter 7 – The Black-Scholes Formula

© ActuarialBrew.com 2014 Page 7.06

Alternatively, we can find the value of the otherwise equivalent call option and then use put-call parity to find the value of the put option. The value of the call option is:

0, 0, 1 2

0.08(1) 0.08(1)

( , , , , , ) ( ) ( )

60 0.54974 60 0.450265.5099

F FrT rT

Eur T TC F K r T r F e N d Ke N d

e e

We now can use put-call parity to find the value of the put option:

0, 0, 0,

0.08(1) 0.08(1)

( , , , , , ) ( , , , , , )

5.5099 60 60 5.5099

F F FrT rT

Eur T Eur T TP F K r T r C F K r T r Ke F e

e e

When the strike price is equal to the futures price, the call price and the put price are the same.

Solution 7.09

B Options on Futures

The 6-month futures price is:

( )( )

,

(0.09 0.05)(0.5)0,0.5 70 71.4141

r T tt T tF S e

F e

d- -

-

=

= =

The values of 1d and 2d are:

2 2

0,1

2 1

ln / 0.5 ln 71.4141/ 65 0.5(0.30) (0.5)0.54970

0.30 0.5

0.54970 0.30 0.5 0.33756

FTF K Td

T

d d T

We have:

1( ) (0.54970) 0.70874N d N

2( ) (0.33756) 0.63215N d N

The value of the call option is:

0, 0, 1 2

0.09(0.5) 0.09(0.5)

( , , , , , ) ( ) ( )

71.4141 0.70874 65 0.632159.10518

F FrT rT

Eur T TC F K r T r F e N d Ke N d

e e

Page 153: ActuarialBrew.com Exam MFE / 3F Actuarial Models …actuarialbrew.com/data/documents/MFE-Solutions-2014-1st-ed.pdf · Exam MFE / 3F Actuarial Models Financial Economics Segment Solutions

Exam MFE/3F Solutions Chapter 7 – The Black-Scholes Formula

© ActuarialBrew.com 2014 Page 7.07

Solution 7.10

A Options on Futures

In this case, the option expires before the underlying futures contract:

0.5

1.0F

TT

==

The 1-year futures price is:

( )( )

,

(0.09 0.05)(1)0,1 70 72.8568

r T tt T tF S e

F e

d- -

-

=

= =

The values of 1d and 2d are:

2 2

0,1

2 1

ln / 0.5 ln 72.8568 / 65 0.5(0.30) (0.5)0.64398

0.30 0.5

0.64398 0.30 0.5 0.43184

FTF K Td

T

d d T

We have:

1( ) ( 0.64398) 0.25979N d N

2( ) ( 0.43184) 0.33293N d N

The value of the put option is:

0, 2 0, 1

0.09(0.5) 0.09(0.5)

( , , , , , ) ( ) ( )

65 0.33293 72.8568 0.259792.5936

F FrT rT

Eur T TP F K r T r Ke N d F e N d

e

Solution 7.11

D Options on Currencies

The values of 1d and 2d are:

2 20

1

2 1

ln( / ) ( 0.5 ) ln(1.40 /1.50) (0.05 0.025 0.5 0.10 )(1)

0.10 10.38993

0.38993 0.10 1 0.48993

fx K r r Td

T

d d T

s

s

s

+ - + + - + ¥= =

= -

= - = - - = -

From the standard normal table, we have:

1

2

( ) (0.38993) 0.65171

( ) (0.48993) 0.68791

N d N

N d N

- = =- = =

Page 154: ActuarialBrew.com Exam MFE / 3F Actuarial Models …actuarialbrew.com/data/documents/MFE-Solutions-2014-1st-ed.pdf · Exam MFE / 3F Actuarial Models Financial Economics Segment Solutions

Exam MFE/3F Solutions Chapter 7 – The Black-Scholes Formula

© ActuarialBrew.com 2014 Page 7.08

The value of one of the put options is:

2 0 10.05(1) 0.025(1)

( ) ( )

1.50 0.68791 1.40 0.65171 0.09167

fr TrTEurP Ke N d x e N d

e e

The value of 1,000 of the options is:

¥ =1,000 0.09167 91.67

Solution 7.12

A Holding Period Profit

The holding period profit is the current value of the position minus the cost of the position, including interest.

At the end of 6 months, the 1-year put option becomes a 6-month put option, and its value is shown in the 6-month column of the table. Since the final stock price is $38, the value of this option from the table is $3.50.

The 6-month option expires at the end of 6 months, and since the final stock price is $38 and the strike price is $40, the 6-month option has a payoff of $2.

The cost of establishing the position is the cost of the 12-month option minus the proceeds from writing (i.e., selling) the 6-month option, accumulated for interest:

0.095(0.5)(3.25 2.67) 0.6082e- =

The holding period profit is the value of the 12-month option minus the value of the 6-month option minus the cost of establishing the position, all evaluated at the end of 6 months:

3.50 2.00 0.6082 0.8918- - =

Solution 7.13

D Black-Scholes Call Price

As the number of steps in the binomial model becomes large, the resulting price for an option approaches the Black-Scholes price. 1,000 is a very large number of steps, so the binomial model price, rounded to two decimal places, is the same as the price obtained with the Black-Scholes formula.

The first step is to calculate 1d and 2d :

2 2

1

2 1

ln( / ) ( 0.5 ) ln(40 / 35) (0.10 0.02 0.5 0.30 ) 1

0.30 10.86177

0.86177 0.30 1 0.56177

S K r Td

T

d d T

d ss

s

+ - + + - + ¥ ¥= =

=

= - = - =

Page 155: ActuarialBrew.com Exam MFE / 3F Actuarial Models …actuarialbrew.com/data/documents/MFE-Solutions-2014-1st-ed.pdf · Exam MFE / 3F Actuarial Models Financial Economics Segment Solutions

Exam MFE/3F Solutions Chapter 7 – The Black-Scholes Formula

© ActuarialBrew.com 2014 Page 7.09

We have:

1( ) (0.86177) 0.80559N d N

2( ) (0.56177) 0.71286N d N

The value of the European call option is:

1 20.02(1) 0.10(1)

( ) ( )

40 0.80559 35 0.71286 9.00975

T rTEurC Se N d Ke N d

e e

Solution 7.14

B Black-Scholes Put Price

The dividend of $9 at the end of 9 months occurs after the option expires, so it does not affect the calculation of the price of the 6-month option.

The prepaid forward price of the stock is:

0.10(0.25)0, 0 0,( ) ( ) 77 7 70.1728P

T TF S S PV Div e-= - = - =

The volatility of the prepaid forward price is:

s = 0.2743PF

The values of 1d and 2d are:

0, 2 20.10(0.5)0,

1

2 1

( ) 70.1728ln 0.5 ln 0.5 0.2743 0.5( ) 73 0.151120.2743 0.5

0.15112 0.2743 0.5 0.04284

PT

PFPT

PF

PF

F ST

F K edT

d d T

We have:

1( ) ( 0.15112) 0.43974N d N

2( ) (0.04284) 0.51709N d N

The value of the put option is:

2 0 0, 1

0.10(0.5)

( ) ( ) ( )

73 0.51709 70.1728 0.43994 5.0348

rTEur TP Ke N d S PV Div N d

e

Solution 7.15

E Black-Scholes Formula Using Prepaid Forward Prices

The key to this question is recognizing that the ratio of the prepaid forward price of the stock to the present value of the strike price is the same for both options.

Page 156: ActuarialBrew.com Exam MFE / 3F Actuarial Models …actuarialbrew.com/data/documents/MFE-Solutions-2014-1st-ed.pdf · Exam MFE / 3F Actuarial Models Financial Economics Segment Solutions

Exam MFE/3F Solutions Chapter 7 – The Black-Scholes Formula

© ActuarialBrew.com 2014 Page 7.10

For the option on Stock X:

0.03(0.75)

0, (0.75)(0.75)

0,

( ) 801.11743

( ) 70

PT r

P rT

F S ee

F K e

For the option on Stock Y:

0.013(0.75)

0, (0.75)(0.75)

0,

( ) 118.481.11743

( ) 105

PT r

P rT

F S ee

F K e

The values of 1d and 2d are based on that ratio, s , and T:

0, 2

0,1

2 1

( )ln 0.5

( )

PT

PT

F ST

F Kd

T

d d T

s

ss

Ê ˆ+Á ˜

Á ˜Ë ¯=

= -

Therefore, both options have the same values of 1d and 2d , and they also have the same

values of 1( )N d and 2( )N d .

The formula for the price of a European call option is:

( )0, 0, 0, 1 0, 2( ), ( ), , ( ) ( ) ( ) ( )P P P PEur T T T TC F S F K T F S N d F K N ds = -

Therefore, for the option on Stock X, we have:

0.03(0.75) (0.75)

1 2(0.75)

1 2

16.05 80 ( ) 70 ( )

16.05 78.220 ( ) 70 ( )

r

r

e N d e N d

N d e N d

- -

-

= -

= -

For the option on Stock Y, we have:

0.013(0.75) (0.75)

1 2(0.75)

1 2

118.48 ( ) 105 ( )

117.330 ( ) 105 ( )

rEur

rEur

C e N d e N d

C N d e N d

- -

-

= -

= -

Notice that the strike price of the option on Stock Y is 150% of the Strike price of the option on Stock X:

105

1.5000070

=

Furthermore, the prepaid forward price of Stock Y is 150% of the prepaid forward price of Stock X:

117.330

1.5000078.220

=

Page 157: ActuarialBrew.com Exam MFE / 3F Actuarial Models …actuarialbrew.com/data/documents/MFE-Solutions-2014-1st-ed.pdf · Exam MFE / 3F Actuarial Models Financial Economics Segment Solutions

Exam MFE/3F Solutions Chapter 7 – The Black-Scholes Formula

© ActuarialBrew.com 2014 Page 7.11

If we multiply both sides of the Black-Scholes formula for the option on Stock X by 1.5, we have the value of the option on Stock Y:

(0.75)

1 2

(0.75)1 2

1.5 16.05 1.5 78.220 ( ) 70 ( )

24.075 117.330 ( ) 105 ( )

r

r

N d e N d

N d e N d

-

-

È ˘¥ = ¥ -Î ˚

= -

The right side of the equation above is the Black-Scholes formula for the value of the call option on Stock Y. Therefore, the value of the call option on Stock Y is $24.075.

Solution 7.16

D Calendar Spread

A calendar spread position involves buying and selling options with different expiration dates. Since Option 1 and Option 2 have the same expiration dates, Choices A and B cannot be correct. Since Option 3 and Option 4 have the same expiration dates, Choice E cannot be correct either.

This leaves us with Choice C and Choice D as possible correct answers. Theta is the time decay measure, which means that the lower the value of theta (i.e., the more negative), the more the option’s price decreases over time, assuming the stock price doesn’t change. Since the investor does not expect the stock price to change, the investor purchases an option with a high theta and sells an option with a low theta. Since the values of theta are negative, this means buying the option with the less negative value of theta and selling the option with the more negative value of theta. Therefore, the investor buys Option 4 and sells Option 2. The correct answer is Choice D.

Solution 7.17

C Options on Currencies

Since the put option is yen-denominated, we use the yen as the base currency. This means that the current exchange rate is:

0 132x =

We also have:

0.01

0.04f

r

r

==

The values of 1d and 2d are:

2 20

1

2 1

ln( / ) ( 0.5 ) ln(132 /125) (0.01 0.04 0.5 0.12 ) 0.5

0.12 0.50.50780

0.50780 0.12 0.5 0.42295

fx K r r Td

T

d d T

Page 158: ActuarialBrew.com Exam MFE / 3F Actuarial Models …actuarialbrew.com/data/documents/MFE-Solutions-2014-1st-ed.pdf · Exam MFE / 3F Actuarial Models Financial Economics Segment Solutions

Exam MFE/3F Solutions Chapter 7 – The Black-Scholes Formula

© ActuarialBrew.com 2014 Page 7.12

We have:

1( ) (0.50780) 0.69420N d N

2( ) (0.42295) 0.66383N d N

The value of the put option is:

2 0 10.01(0.5) 0.04(0.5)

( ) ( )

125 (1 0.66383) 132 (1 0.69420) 2.2454

fr TrTEurP Ke N d x e N d

e e

Alternatively, we can find the value of the otherwise equivalent call option and then use put-call parity to find the value of the put option. The value of the call option is:

0 1 20.04(0.5) 0.01(0.5)

( ) ( )

132 0.69420 125 0.66383 7.2550

fr T rTEurC x e N d Ke N d

e e

We can now use put-call parity to find the value of the put option:

00.01(0.5) 0.04(0.5)7.2550 125 132 2.2454

fr TrTEur EurP C Ke x e

e e

Solution 7.18

A Options on Currencies

Since the call option is euro-denominated, we use the euro as the base currency. This means that the current exchange rate is:

01

132x =

We also have:

0.040.01f

rr==

The values of 1d and 2d are:

22

01

2 1

1 /132ln (0.04 0.01 0.5 0.12 ) 0.5ln( / ) ( 0.5 ) 0.008

0.12 0.50.42295

0.42295 0.12 0.5 0.50780

fx K r r Td

T

d d T

We have:

1( ) ( 0.42295) 0.33617N d N

2( ) ( 0.50780) 0.30580N d N

Page 159: ActuarialBrew.com Exam MFE / 3F Actuarial Models …actuarialbrew.com/data/documents/MFE-Solutions-2014-1st-ed.pdf · Exam MFE / 3F Actuarial Models Financial Economics Segment Solutions

Exam MFE/3F Solutions Chapter 7 – The Black-Scholes Formula

© ActuarialBrew.com 2014 Page 7.13

The value of the call option is:

0 1 2

0.01(0.5) 0.04(0.5)

( ) ( )

10.33617 0.008 0.30580 0.00014

132

fr T rTEurC x e N d Ke N d

e e

Solution 7.19

D Options on Futures Contracts

The 1-year futures price is:

( )( )

,

(0.07 0.00)(1)0,1 100 107.2508

r T tt T tF S e

F e

d- -

-

=

= =

The values of 1d and 2d are:

2 2

0,1

2 1

ln / 0.5 ln 107.2508 /105 0.5(0.35) (1)0.23560

0.35 1

0.23560 0.35 1 0.11440

FTF K Td

T

d d T

We have:

1( ) (0.23560) 0.59313N d N

2( ) ( 0.11440) 0.45446N d N

The value of the call option is:

0, 0, 1 2

0.07(1) 0.07(1)

( , , , , , ) ( ) ( )

107.2508 0.59313 105 0.45446

14.8208

F FrT rT

Eur T TC F K r T r F e N d Ke N d

e e

Solution 7.20

C Black-Scholes Put Price

The first step is to calculate 1d and 2d :

2

1

2

2 1

ln( / ) ( 0.5 )

ln(100 /103) (0.06 0.025 0.5 0.5 ) 0.50.14267

0.5 0.5

0.14267 0.5 0.5 0.21088

S K r Td

T

d d T

d ss

s

+ - +=

+ - + ¥ ¥= =

= - = - = -

We have:

1( ) ( 0.14267) 0.44328N d N

Page 160: ActuarialBrew.com Exam MFE / 3F Actuarial Models …actuarialbrew.com/data/documents/MFE-Solutions-2014-1st-ed.pdf · Exam MFE / 3F Actuarial Models Financial Economics Segment Solutions

Exam MFE/3F Solutions Chapter 7 – The Black-Scholes Formula

© ActuarialBrew.com 2014 Page 7.14

2( ) (0.21088) 0.58351N d N

The value of the European put option is:

2 10.06(0.5) 0.025(0.5)

( ) ( )

103 0.58351 100 0.4432814.5479

rT TEurP Ke N d Se N d

e e

Solution 7.21

A Black-Scholes Call Price

It is never optimal to exercise the American call option early because the stock does not pay dividends. Therefore, its value is equal to an otherwise equivalent European call option, and we can use the Black-Scholes formula to find its price.

The values of 1d and 2d are:

2 2

1

2 1

ln( / ) ( 0.5 ) ln(78 / 82) (0.10 0.00 0.5 0.27 ) 0.25

0.27 0.250.11776

0.11776 0.27 0.25 0.25276

S K r Td

T

d d T

d ss

s

+ - + + - + ¥ ¥= =

= -

= - = - - = -

We have:

1( ) ( 0.11776) 0.45313N d N

2( ) ( 0.25276) 0.40023N d N

The value of the call option is:

1 20.00(0.25) 0.10(0.25)

( ) ( )

78 0.45313 82 0.40023 3.3356

T rTEurC Se N d Ke N d

e e

Solution 7.22

C Black-Scholes Formula

The annual volatility is:

[ ]s = = =ln ( ) 0.5

0.5Var S t t

t t

Page 161: ActuarialBrew.com Exam MFE / 3F Actuarial Models …actuarialbrew.com/data/documents/MFE-Solutions-2014-1st-ed.pdf · Exam MFE / 3F Actuarial Models Financial Economics Segment Solutions

Exam MFE/3F Solutions Chapter 7 – The Black-Scholes Formula

© ActuarialBrew.com 2014 Page 7.15

We can use the version of the Black-Scholes formula that is based on prepaid forward prices to find the value of the call option:

0, 2

0,1

2 1

1

2

( ) 50ln 0.5 ln 0.5 0.5 8( ) 501.00000

0.5 8

1.00000 0.5 8 1.00000

( ) (1.00000) 0.84134

( ) ( 1.00000) 0.15866

PT

PT

F ST

F Kd

T

d d T

N d N

N d N

s

ss

Ê ˆÊ ˆ+Á ˜ + ¥ ¥Á ˜ Á ˜Ë ¯ Ë ¯

= = =¥

= - = - ¥ = -= == - =

The price of the call option is:

0, 0, 0, 1 0, 2( ), ( ), , ( ) ( ) ( ) ( )

50 0.84134 50 0.1586634.134

P P P PEur T T T TC F S F K T F S N d F K N d

Solution 7.23

D Black-Scholes Formula Using Prepaid Forward Prices

The prepaid forward prices of the stock and the strike price are:

-

-

= - =

= =

0.07(4 /12)0,

0.07(0.5)0,

( ) 100 4.00 96.0923

( ) 105 101.3886

PT

PT

F S e

F K e

The volatility of the prepaid forward price is:

sÈ ˘Î ˚= = =

,0.5ln 0.26020.2602

tPF

Var F tt t

We use the prepaid forward volatility in the Black-Scholes Formula:

0, 2 20,

1

2 1

( ) 96.0923ln 0.5 ln 0.5 0.2602 0.5( ) 101.38860.2602 0.5

0.19961

0.19961 0.2602 0.5 0.38360

PT

PFPT

PF

PF

F ST

F Kd

T

d d T

s

s

s

Ê ˆÊ ˆ+Á ˜ + ¥ ¥Á ˜ Á ˜Ë ¯ Ë ¯

= =

= -

= - = - - = -

We have:

1( ) (0.19961) 0.57911N d N

2( ) (0.38360) 0.64936N d N

Page 162: ActuarialBrew.com Exam MFE / 3F Actuarial Models …actuarialbrew.com/data/documents/MFE-Solutions-2014-1st-ed.pdf · Exam MFE / 3F Actuarial Models Financial Economics Segment Solutions

Exam MFE/3F Solutions Chapter 7 – The Black-Scholes Formula

© ActuarialBrew.com 2014 Page 7.16

The value of the European put option is:

0, 0, 2 0 0, 1( ), ( ), , ( ) ( ) ( )

101.3886 0.64936 96.0923 0.5791110.1897

P P rTEur T T TP F S F K T Ke N d S PV Div N d

Solution 7.24

D Black-Scholes Formula Using Prepaid Forward Prices

The prepaid forward price of the stock is:

0.10 10, 0 0,( ) ( ) 82 4 78.3807P

T TF S S PV Div e

The volatility of the prepaid forward price is:

s = 0.2615PF

The values of 1d and 2d are:

0, 2 20.10 20,

1

2 1

( ) 78.3807ln 0.5 ln 0.5 0.2615 2( ) 870.2615 2

0.44360

0.4436 0.2615 2 0.07378

PT

PFPT

PF

PF

F ST

F K edT

d d T

s

s

s

- ¥

Ê ˆÊ ˆ+Á ˜ + ¥ ¥Á ˜ Á ˜Ë ¯ Ë ¯

= =

=

= - = - =

We have:

1( ) ( 0.44360) 0.32867N d N

2( ) ( 0.07378) 0.47059N d N

The value of the European put option is:

0.10 20, 2 0, 1( ) ( ) ( ) ( ) 87 0.47059 78.3807 0.32867

71.2296 0.47059 78.3807 0.32867 7.7586

P PEur T TP F K N d F S N d e

Solution 7.25

E Currency Options and Black-Scholes

This question is similar to Question #7 of the SOA MFE/3F Sample Exam Questions.

Page 163: ActuarialBrew.com Exam MFE / 3F Actuarial Models …actuarialbrew.com/data/documents/MFE-Solutions-2014-1st-ed.pdf · Exam MFE / 3F Actuarial Models Financial Economics Segment Solutions

Exam MFE/3F Solutions Chapter 7 – The Black-Scholes Formula

© ActuarialBrew.com 2014 Page 7.17

Statement (vii) contains a lot of information:

Let’s define x(t) to be the dollar per ruble exchange rate. Then (vii) tells us that:

a s a sÈ ˘= + fi = - -È ˘Í ˙ Î ˚

Î ˚

1ln dt dZ(t ) ln x(t ) dt dZ(t )x(t )

Since –Z(t) is a standard Brownian motion, the logarithm of the dollar per ruble exchange rate can be said to have the same volatility as the ruble per dollar

exchange rate. For both È ˘Í ˙Î ˚

1lnx(t )

and È ˘Î ˚ln x(t ) , the standard deviation is s t .

Therefore:

s s= fi =10.366397% 0.07

365

Since the logarithm of the dollar per ruble exchange rate is an arithmetic Brownian motion, the Black-Scholes framework applies. This means that put and call options on the ruble can be priced using the Black-Scholes formula.

The currency option is a put option with the ruble as its underlying asset. The domestic currency is dollars, and the current value of the one ruble is:

01

dollars26

x =

Since the option is at-the-money, the strike price is equal to the value of one ruble:

1

dollars26

K =

The domestic interest rate is 5%, and the foreign interest rate is 8%:

5%8%f

rr==

The volatility of the ruble per dollar exchange rate is equal to the volatility of the dollar per ruble exchange rate. We convert the daily volatility into annual volatility:

0.00366397

0.00366397 365 0.070001

365

h

h

ss = = = ¥ =

The values of 1d and 2d are:

2 20

1

2 1

ln( / ) ( 0.5 ) ln(1) (0.05 0.08 0.5 0.07 )0.25

0.07 0.250.19679

0.19679 0.07 0.25 0.23179

fx K r r Td

T

d d T

ss

s

+ - + + - + ¥= =

= -

= - = - - = -

Page 164: ActuarialBrew.com Exam MFE / 3F Actuarial Models …actuarialbrew.com/data/documents/MFE-Solutions-2014-1st-ed.pdf · Exam MFE / 3F Actuarial Models Financial Economics Segment Solutions

Exam MFE/3F Solutions Chapter 7 – The Black-Scholes Formula

© ActuarialBrew.com 2014 Page 7.18

We round 1d and 2d when using the normal distribution table:

1

2

( ) (0.19679) 0.57800

( ) (0.23179) 0.59165

N d N

N d N

- = =- = =

The value of a put option on one ruble is:

2 1

0.05(0.25) 0.08(0.25)

( , , , , , ) ( ) ( )

1 1(0.59165) (0.57800)

26 260.000682522

fr TrTEurP S K r T Ke N d Se N d

e e

Since the option is for 250,000,000 rubles the value of the put option in dollars is:

250,000,000 0.000682522 170,630.50¥ =

Solution 7.26

B Options on Currencies

Sam wants to give up $125,000 and receive €100,000 at the end of 1 year. This can be accomplished with 2 different purchases:

100,000 dollar-denominated call options on euros, each with a strike price of $1.25. 125,000 euro-denominated put options on dollars, each with a strike price of

€0.80.

Therefore, the correct answer must be either Choice B or Choice E.

Let’s find the price of the dollar-denominated call options.

The values of 1d and 2d are:

2 20

1

2 1

ln( / ) ( 0.5 ) ln(1.25 /1.25) (0.08 0.05 0.5 0.10 )(1)

0.10 10.35000

0.35000 0.10 1 0.25000

fx K r r Td

T

d d T

We have:

1( ) (0.35000) 0.63683N d N

2( ) (0.25000) 0.59871N d N

The value of the call option is:

0 1 20.05(1) 0.08(1)

( ) ( )

1.25 0.63683 1.25 0.59871 0.06637

fr T rTEurC x e N d Ke N d

e e

The value of 100,000 of the dollar-denominated call options is:

$0.06637 100,000 $6,637

Page 165: ActuarialBrew.com Exam MFE / 3F Actuarial Models …actuarialbrew.com/data/documents/MFE-Solutions-2014-1st-ed.pdf · Exam MFE / 3F Actuarial Models Financial Economics Segment Solutions

Exam MFE/3F Solutions Chapter 7 – The Black-Scholes Formula

© ActuarialBrew.com 2014 Page 7.19

If we had begun by finding the value of the 125,000 euro-denominated put options, we would have obtained a value of €5,309, implying that Choice E is not correct.

Solution 7.27

E Options on Currencies

Sam wants to give up $125,000 and receive €100,000 at the end of 1 year. This can be accomplished with 2 different purchases:

100,000 dollar-denominated call options on euros, each with a strike price of $1.25. 125,000 euro-denominated put options on dollars, each with a strike price of

€0.80.

Therefore, the correct answer must be either Choice B or Choice E.

Let’s find the price of the dollar-denominated call options.

The values of 1d and 2d are:

2 20

1

2 1

ln( / ) ( 0.5 ) ln(1.25 /1.25) (0.08 0.05 0.5 0.10 )(1)

0.10 10.35000

0.35000 0.10 1 0.25000

fx K r r Td

T

d d T

We have:

1( ) (0.35000) 0.63683N d N

2( ) (0.25000) 0.59871N d N

The value of the call option is:

0 1 20.05(1) 0.08(1)

( ) ( )

1.25 0.63683 1.25 0.59871 0.06637

fr T rTEurC x e N d Ke N d

e e

The value of 100,000 of the dollar-denominated call options is:

$0.06637 100,000 $6,637

Therefore, Choice B is not correct.

The 100,000 dollar-denominated calls with a strike price of $1.25 (described in Choice B) allow their owner the right to give up $125,000 to obtain €100,000. Likewise, 125,000 euro-denominated puts with a strike price of €0.80 (described in Choice E) allow their owner the right to give up $125,000 to obtain €100,000. Since both option positions have the same payoff, they must have the same current cost:

=$6,637€5,309

$1.25 / €

Therefore, Choice E is correct.

Page 166: ActuarialBrew.com Exam MFE / 3F Actuarial Models …actuarialbrew.com/data/documents/MFE-Solutions-2014-1st-ed.pdf · Exam MFE / 3F Actuarial Models Financial Economics Segment Solutions

Exam MFE/3F Solutions Chapter 7 – The Black-Scholes Formula

© ActuarialBrew.com 2014 Page 7.20

Solution 7.28

D Options on Futures

At time 0, the values of 1d and 2d are:

2 2

0,1

2 1

ln / 0.5 ln 30 / 30 0.50.5

0.5 0.5

FTF K T Td T

T T

d d T T T T

We can find - 1( )N d in terms of - 2( )N d :

( )( )

s

s

- = -

- =

- = - -

1

2

1 2

( ) 0.5

( ) 0.5

( ) 1 ( )

N d N T

N d N T

N d N d

We can substitute this value of - 1( )N d into the time 0 formula for the price in order to find 2d :

0, 2 0, 1

0.08 0.75 0.08 0.752 1

0.062 2

0.062

2

2

2

( , , , , , ) ( ) ( )

3.40 30 ( ) 30 ( )

3.40 30 ( ) 1 ( )

3.402 ( ) 1

30( ) 0.56017

0.15140

0.15140

F FrT rT

Eur T TP F K r T r Ke N d F e N d

e N d e N d

e N d N d

e N d

N d

d

d

We can now find s :

2 0.5

0.15140 0.5 0.750.34964

d T

After 3 months, the new values of 1d and 2d are:

2 20,

1

2 1

ln / 0.5 ln 27 / 30 0.5 0.34964 0.50.30254

0.34964 0.5

0.30254 0.34964 0.5 0.54975

FTF K Td

T

d d T

We can look up the values of - 1( )N d and - 2( )N d from the normal distribution table:

1

2

( ) (0.30254) 0.61888

( ) (0.54975) 0.70876

N d N

N d N

- = =- = =

Page 167: ActuarialBrew.com Exam MFE / 3F Actuarial Models …actuarialbrew.com/data/documents/MFE-Solutions-2014-1st-ed.pdf · Exam MFE / 3F Actuarial Models Financial Economics Segment Solutions

Exam MFE/3F Solutions Chapter 7 – The Black-Scholes Formula

© ActuarialBrew.com 2014 Page 7.21

After 3 months, the value of the put option is:

0, 2 0, 1

0.08 0.5 0.08 0.5

( , , , , , ) ( ) ( )

30 0.70876 27 0.618884.3745

F FrT rT

Eur T TP F K r T r Ke N d F e N d

e e

s - -

- ¥ - ¥

= - - -

= ¥ - ¥=

Solution 7.29

D Options on Futures

Put-call parity implies that the put and call options must be at-the-money:

s s- -

- ¥ - ¥

+ = +

+ = +=

0, 0, 0,

0.08 0.75 0.08 0.75

( , , , , , ) ( , , , , , )

5.61 50 5.6150

F F FrT rT

Eur T T Eur TC F K r T r Ke F e P F K r T r

Ke eK

At time 0, the values of 1d and 2d are:

2 2

0,1

2 1

ln / 0.5 ln 50 / 50 0.50.5

0.5 0.5

FTF K T Td T

T T

d d T T T T

We can find 1( )N d in terms of 2( )N d :

( )( )

s

s

=

= -

= -

1

2

1 2

( ) 0.5

( ) 0.5

( ) 1 ( )

N d N T

N d N T

N d N d

We can substitute this value of 1( )N d into the time 0 formula for the price of the call option in order to find 2d :

0, 0, 1 2

0.08 0.75 0.08 0.751 2

0.061 2

0.062 2

0.06

2

2

2

( , , , , , ) ( ) ( )

5.61 50 ( ) 50 ( )

5.61 50 ( ) ( )

5.61 50 1 ( ) ( )

5.611 2 ( )

50( ) 0.44043

0.14988

F FrT rT

Eur T TC F K r T r F e N d Ke N d

e N d e N d

e N d N d

e N d N d

eN d

N d

d

Page 168: ActuarialBrew.com Exam MFE / 3F Actuarial Models …actuarialbrew.com/data/documents/MFE-Solutions-2014-1st-ed.pdf · Exam MFE / 3F Actuarial Models Financial Economics Segment Solutions

Exam MFE/3F Solutions Chapter 7 – The Black-Scholes Formula

© ActuarialBrew.com 2014 Page 7.22

We can now find s :

2 0.5

0.14988 0.5 0.750.34613

d T

After 3 months, the new values of 1d and 2d are:

2 20,

1

2 1

ln / 0.5 ln 52 / 50 0.5 0.34613 0.50.28262

0.34613 0.5

0.28262 0.34613 0.5 0.03787

FTF K Td

T

d d T

We can look up the values of 1( )N d and 2( )N d from the normal distribution table:

1

2

( ) (0.28262) 0.61127

( ) (0.03787) (0.04) 0.51510

N d N

N d N N

= == ª =

After 3 months, the value of the call option is:

0, 0, 1 2

0.08 0.5 0.08 0.5

( , , , , , ) ( ) ( )

52 0.61127 50 0.515105.7946

F FrT rT

Eur T TC F K r T r F e N d Ke N d

e e

s - -

- ¥ - ¥

= -

= ¥ - ¥=

Solution 7.30

A Holding Period Profit

We begin by finding the value of the put option today. The first step is to calculate 1d

and 2d :

2 2

1

2 1

ln( / ) ( 0.5 ) ln(80 / 90) (0.06 0 0.5 0.28 ) 4 /12

0.28 4 /120.52405

0.52405 0.28 4 /12 0.68571

S K r Td

T

d d T

We have:

1( ) (0.52405) 0.69988N d N

2( ) (0.68571) 0.75355N d N

The value of the European put option today is:

2 10.06(4 /12) 0.00(4 /12)

( ) ( )

90 0.75355 80 0.69988 10.4862

rT TEurP Ke N d Se N d

e e

Page 169: ActuarialBrew.com Exam MFE / 3F Actuarial Models …actuarialbrew.com/data/documents/MFE-Solutions-2014-1st-ed.pdf · Exam MFE / 3F Actuarial Models Financial Economics Segment Solutions

Exam MFE/3F Solutions Chapter 7 – The Black-Scholes Formula

© ActuarialBrew.com 2014 Page 7.23

The holding period profit is current value of the position minus the cost of the position, including interest:

0.06(8 /12)10.4862 7 3.2005e- =

Solution 7.31

B Black-Scholes Formula

The prepaid forward volatility is:

,0.5ln ( ) 0.04

0.20

Pt

PF

Var F S tt t

The prepaid forward prices of the stock and the strike price are:

-

-

= - =

= =

0.07(4 /12)0,

0.07(0.5)0,

( ) 70 4.00 66.0923

( ) 75 72.4204

PT

PT

F S e

F K e

We use the prepaid forward volatility in the Black-Scholes Formula:

0, 2 20,

1

2 1

( ) 66.0923ln 0.5 ln 0.5 0.20 0.5( ) 72.42040.20 0.5

0.57584

0.57584 0.20 0.5 0.71726

PT

PFPT

PF

PF

F ST

F Kd

T

d d T

s

s

s

Ê ˆÊ ˆ+Á ˜ + ¥ ¥Á ˜ Á ˜Ë ¯ Ë ¯

= =

= -

= - = - - = -

We have:

1( ) ( 0.57584) 0.28236N d N

2( ) ( 0.71726) 0.23661N d N

The value of the European call option is:

0, 0, 0 0, 1 2( ), ( ), , ( ) ( ) ( )

66.0923 0.28236 72.4204 0.236611.5264

P P rTEur T T TC F S F K T S PV Div N d Ke N d

Page 170: ActuarialBrew.com Exam MFE / 3F Actuarial Models …actuarialbrew.com/data/documents/MFE-Solutions-2014-1st-ed.pdf · Exam MFE / 3F Actuarial Models Financial Economics Segment Solutions

Exam MFE/3F Solutions Chapter 7 – The Black-Scholes Formula

© ActuarialBrew.com 2014 Page 7.24

Solution 7.32

D Black-Scholes Formula

The prepaid forward price can be written in terms of the forward price:

( ) ( ),, , ,( ) ( )P r T t r T t P

t Tt T t T t TF S e F F e F S

The variance of the prepaid forward price is equal to the variance of the forward price:

(0.75 ),0.75 ,0.75

(0.75 ),0.75

,0.75

ln ( ) ln ( )

ln ln ( )

0 ln ( ) 0.04

P r tt t

r tt

t

Var F S Var e F S

Var e F S

Var F S t

The prepaid forward volatility is:

,0.75ln ( ) 0.04

0.20

Pt

PF

Var F S tt t

The prepaid forward prices of the stock and the strike price are:

-

-

= - =

= =

0.08(6 /12)0,

0.08(0.75)0,

( ) 70 5.00 65.1961

( ) 75 70.6323

PT

PT

F S e

F K e

We use the prepaid forward volatility in the Black-Scholes Formula:

0, 2 20,

1

2 1

( ) 65.1961ln 0.5 ln 0.5 0.20 0.75( ) 70.63230.20 0.75

0.37579

0.37579 0.20 0.75 0.54900

PT

PFPT

PF

PF

F ST

F Kd

T

d d T

s

s

s

Ê ˆÊ ˆ+Á ˜ + ¥ ¥Á ˜ Á ˜Ë ¯ Ë ¯

= =

= -

= - = - - = -

We have:

1( ) (0.37579) 0.64646N d N

2( ) (0.54900) 0.70850N d N

The value of the European put option is:

2 0 0, 10, 0,( ), ( ), , ( ) ( ) ( )

70.6323 0.70850 65.1961 0.64646

7.8964

P P rTEur TT TP F S F K T Ke N d S PV Div N d

Page 171: ActuarialBrew.com Exam MFE / 3F Actuarial Models …actuarialbrew.com/data/documents/MFE-Solutions-2014-1st-ed.pdf · Exam MFE / 3F Actuarial Models Financial Economics Segment Solutions

Exam MFE/3F Solutions Chapter 8 – The Greeks and Other Measures

© ActuarialBrew.com 2014 Page 8.01

Chapter 8 – Solutions

Solution 8.01

B Greek Measures for Portfolios

A call bull spread consists of a position in which a low-strike call is purchased and a high-strike call is sold. The low-strike call has the higher premium, so Option 1 is the one that is purchased and Option 2 is the one that is sold.

The quantity of Option 1 in the portfolio is 1q , and the quantity of Option 2 in the

portfolio is 2q :

1

2

1

1

q

q

== -

The value of vega for the bull spread is:

1

1 0.1329 1 0.1646 0.0317n

Port i ii

Vega q Vega=

= = ¥ - ¥ = -Â

Solution 8.02

E Delta

The value of 1d is:

2 2

1ln( / ) ( 0.5 ) ln(86 / 90) (0.095 0.03 0.5 0.35 ) 0.75

0.35 0.750.16240

S K r Td

T

d ss

+ - + + - + ¥ ¥= =

=

We have:

1( ) (0.16240) 0.56450N d N

The delta of the put option is:

0.03(0.75)1( ) (1 0.56450) 0.42581T

Put e N d e

Solution 8.03

C Delta

The value of 1d is:

2 2

1ln( / ) ( 0.5 ) ln(86 / 90) (0.095 0.03 0.5 0.35 ) 0.75

0.35 0.750.16240

S K r Td

T

d ss

+ - + + - + ¥ ¥= =

=

Page 172: ActuarialBrew.com Exam MFE / 3F Actuarial Models …actuarialbrew.com/data/documents/MFE-Solutions-2014-1st-ed.pdf · Exam MFE / 3F Actuarial Models Financial Economics Segment Solutions

Exam MFE/3F Solutions Chapter 8 – The Greeks and Other Measures

© ActuarialBrew.com 2014 Page 8.02

We have:

1( ) (0.16240) 0.56450N d N

The delta of the call option is:

0.03(0.75)1( ) 0.56450 0.55194T

Call e N d e

The delta of the put option is:

0.03(0.75)0.55194 0.42581TPut Call e e

The delta of the position is:

1

100 0.55194 50 ( 0.42581) 33.9035n

Port i ii

q

Solution 8.04

B Elasticity

The values of 1d and 2d are:

2 2

1

2 1

ln( / ) ( 0.5 ) ln(86 / 90) (0.095 0.03 0.5 0.35 ) 0.75

0.35 0.750.16240

0.16240 0.35 0.75 0.14071

S K r Td

T

d d T

d ss

s

+ - + + - + ¥ ¥= =

=

= - = - = -

We have:

1( ) ( 0.16240) 0.43550N d N

2( ) (0.14071) 0.55595N d N

The value of the European put option is:

2 10.095(0.75) 0.03(0.75)

( ) ( )

90 0.55595 86 0.43550 9.9749

rT TEurP Ke N d Se N d

e e

The delta of the put option is:

0.03(0.75)1( ) 0.43550 0.42581T

Put e N d e

The elasticity of the put option is:

86 ( 0.42581)

3.67129.9749

SV

Page 173: ActuarialBrew.com Exam MFE / 3F Actuarial Models …actuarialbrew.com/data/documents/MFE-Solutions-2014-1st-ed.pdf · Exam MFE / 3F Actuarial Models Financial Economics Segment Solutions

Exam MFE/3F Solutions Chapter 8 – The Greeks and Other Measures

© ActuarialBrew.com 2014 Page 8.03

Solution 8.05

E Greek Measures for Portfolios

The quantity of each option purchased is determined below:

1 1

2 2

3 3

2010 40% 50 2

1020

20 40% 50 12010

5 20% 50 25

q q

q q

q q

¥ = ¥ fi = =

¥ = ¥ fi = =

¥ = ¥ fi = =

The value of rho for the portfolio is:

1

2 0.2754 1 0.3641 2 ( 0.2087) 0.4975n

Port i ii

qr r=

= = ¥ + ¥ + ¥ - =Â

Solution 8.06

E Greek Measures for Portfolios

Let’s consider each of the choices.

The value of delta for a call option is always positive, so Choice A cannot be correct.

The value of vega for a put option is always positive, so Choice B cannot be correct.

The value of rho is always negative for a put option, so Choice C cannot be correct.

The value of psi is always negative for a call option, so Choice D cannot be correct.

The value of theta is usually negative, but it can be positive for a deep-in-the-money put option. This is the pattern in the graph, so Choice E is the correct answer.

Solution 8.07

B Elasticity

The risk premium of the option is:

( ) 4.5 (0.15 0.07) 0.36r rg a- = W ¥ - = ¥ - =

Solution 8.08

E Elasticity

The value of the option is:

V S B= D +

Page 174: ActuarialBrew.com Exam MFE / 3F Actuarial Models …actuarialbrew.com/data/documents/MFE-Solutions-2014-1st-ed.pdf · Exam MFE / 3F Actuarial Models Financial Economics Segment Solutions

Exam MFE/3F Solutions Chapter 8 – The Greeks and Other Measures

© ActuarialBrew.com 2014 Page 8.04

The percentage of the value of the option that is invested in the stock is the elasticity of the stock:

5.35S

VD = W = -

The amount invested in the stock is:

5.35 4.17 22.3095S VD = W = - ¥ = -

We can now solve for the amount that is lent at the risk-free rate of return:

4.17 22.309526.4795

V S BB

B

= D += - +

=

Solution 8.09

A Sharpe Ratio

The Sharpe ratio for any asset is the ratio of the asset’s risk premium to its volatility:

( ) 4.377(0.15 0.06)

Sharpe ratio for call option 0.281381.40

Call

Call

r rg as s

- W ¥ - -= = = =¥ W

Solution 8.10

C Elasticity

If someone writes an option, that person receives the price of the option in exchange for agreeing to provide the payoff if the option is exercised. Writing an option is essentially the same as selling the option.

The investor purchases 1 each of Option A and Option B, and the investor sells 1 of Option C:

1 1 1A B Cq q q= = = -

The value of the portfolio is:

1 9.986 1 7.985 1 6.307 11.664Port = ¥ + ¥ - ¥ =

The elasticity of the portfolio is:

1

9.986 7.985 6.3074.367 4.794 5.227 4.1943

11.664 11.664 11.664

n

Port i ii

w=

W = W = ¥ + ¥ - ¥ =Â

Page 175: ActuarialBrew.com Exam MFE / 3F Actuarial Models …actuarialbrew.com/data/documents/MFE-Solutions-2014-1st-ed.pdf · Exam MFE / 3F Actuarial Models Financial Economics Segment Solutions

Exam MFE/3F Solutions Chapter 8 – The Greeks and Other Measures

© ActuarialBrew.com 2014 Page 8.05

Solution 8.11

B Option Elasticity

The values of 1d and 2d are:

2 2

1

2 1

ln( / ) ( 0.5 ) ln(25 / 26) (0.06 0.01 0.5 0.20 )(2)

0.20 20.35631

0.35631 0.20 2 0.07347

S K r Td

T

d d T

d ss

s

+ - + + - + ¥= =

=

= - = - =

From the standard normal table, we have:

1

2

( ) ( 0.35631) 0.36080

( ) ( 0.07347) 0.47072

N d N

N d N

- = - =- = - =

The value of the put option is:

2 10.06(2) 0.01(2)

( ) ( )

26 0.47072 25 0.36080 2.01338

rT TEurP Ke N d Se N d

e e

The delta of the put option is:

0.01(2)1( ) (0.36080) 0.35366T

Put e N d ed- -D = - - = - = -

The elasticity of the put option is:

25 ( 0.35366)

4.39132.01338

SVD ¥ -W = = = -

Solution 8.12

D Elasticity of a Portfolio

Using put-call parity, we can determine the cost to establish the portfolio:

d

d

- -

- -

- -

+ = +

- = -

- = -- =

0

00.05(0.5) 0.13(0.5)60 62

0.42041

rT TEur Eur

T rTEur Eur

Eur Eur

Eur Eur

C Ke S e P

C P S e Ke

C P e e

C P

The value of the portfolio is therefore:

= - = 0.42041Port Eur EurV C P

The delta of the portfolio is the delta of the call minus the delta of the put:

( )d- -

=D = D = ¥ D - ¥ D = D - D - = =Â

20.05(0.5)

1

1 1 0.97531TPort i i Call Put Call Call

i

q e e

Page 176: ActuarialBrew.com Exam MFE / 3F Actuarial Models …actuarialbrew.com/data/documents/MFE-Solutions-2014-1st-ed.pdf · Exam MFE / 3F Actuarial Models Financial Economics Segment Solutions

Exam MFE/3F Solutions Chapter 8 – The Greeks and Other Measures

© ActuarialBrew.com 2014 Page 8.06

The elasticity of the portfolio is:

D ¥W = = =60 0.97531

139.20.42041

PortPort

Port

SV

Solution 8.13

B Risk Premium of a Portfolio

We can use put-call parity to determine the value of the put option:

00.06(0.5) 0.06(0.5)5.10 60 60

5.10

rT TEur Eur

Eur

Eur

C Ke S e P

e e P

P

The value of the portfolio is:

= ¥ + ¥ = ¥ + ¥ =2 1 2 5.10 1 5.10 15.30Port Eur EurV C P

The delta of the put option is:

d- -D = D - = - = -0.06(0.5)0.5277 0.4427TPut Call e e

The delta of the portfolio is:

=

D = D = ¥ D + ¥ D = ¥ + ¥ - =Â2

1

2 1 2 0.5277 1 ( 0.4427) 0.6127Port i i Call Puti

q

The elasticity of the portfolio is:

D ¥W = = =60 0.6127

2.402615.30

PortPort

Port

SV

The expected return of the portfolio is:

g a= - W + = - + =( ) (0.22 0.06)(2.4026) 0.06 0.4444Port Portr r

Solution 8.14

E Sharpe Ratio

The Sharpe ratio of a call option is the same as the Sharpe ratio of the underlying stock:

g as s

- -= =Sharpe ratio for call option Call

Call

r r

The Sharpe ratio for the underlying stock is:

as- -= =0.08 0.04

0.160.25

r

Therefore, the Sharpe ratio for the call option is 0.16.

Page 177: ActuarialBrew.com Exam MFE / 3F Actuarial Models …actuarialbrew.com/data/documents/MFE-Solutions-2014-1st-ed.pdf · Exam MFE / 3F Actuarial Models Financial Economics Segment Solutions

Exam MFE/3F Solutions Chapter 8 – The Greeks and Other Measures

© ActuarialBrew.com 2014 Page 8.07

Solution 8.15

A General

The page references below refer to the Second Edition of Derivatives Markets.

Choice A is false. From the third sentence from the bottom of page 391: the elasticity decreases as the strike price decreases. This is because the elasticity is highest for out-of-the-money options. As the strike price decreases, a call option becomes more in-the-money.

Choice B is true. On page 394, the last paragraph of the risk premium section: the expected return of a call option goes down as the price of the underlying stock goes up.

Choice C is true. On page 394, the last sentence of the risk premium section: the expected return of a put option is less than that of the underlying stock.

Choice D is true. Refer to the last paragraph of page 391. Since the risk premium for a put option is less than or equal to zero, it's elasticity is also less than or equal to zero.

Choice E is true. On page 394, the last full sentence of the page: the Sharpe ratio for a call equals the Sharpe ratio for the underlying stock.

Solution 8.16

B Greek Measures for Portfolios

A calendar spread consists of buying and selling options that have different expiration dates. Since the investor believes that the stock price will not change, the investor focuses on the theta measure when deciding which option to buy and which one to sell. Both options decline in value over time, but Option 1 declines more quickly than Option 2, since Option 1’s theta is more negative than Option 2’s theta. Therefore, the investor sells Option 1 and buys Option 2.

The quantity of Option 1 in the portfolio is 1q , and the quantity of Option 2 in the

portfolio is 2q :

1

2

1

1

q

q

= -=

The value of psi for the calendar spread position is:

1

1 ( 0.0184) 1 ( 0.2704) 0.2520n

Port i ii

qy y=

= = - ¥ - + ¥ - = -Â

Page 178: ActuarialBrew.com Exam MFE / 3F Actuarial Models …actuarialbrew.com/data/documents/MFE-Solutions-2014-1st-ed.pdf · Exam MFE / 3F Actuarial Models Financial Economics Segment Solutions

Exam MFE/3F Solutions Chapter 8 – The Greeks and Other Measures

© ActuarialBrew.com 2014 Page 8.08

Solution 8.17

C Greek Measures for Portfolios

A put bear spread consists of a position in which a high-strike put is purchased and a low-strike put is sold. The high-strike put has the higher price, so Option 2 is the high-strike put, and Option 1 is the low-strike put. Therefore, Option 2 is purchased, and Option 1 is sold.

The quantity of Option 1 in the portfolio is 1q , and the quantity of Option 2 in the

portfolio is 2q :

1

2

1

1

q

q

= -=

The value of gamma for the put bear spread position is:

1

1 (0.0181) 1 (0.0225) 0.0044n

Port i ii

q=

G = G = - ¥ + ¥ =Â

Solution 8.18

A Black-Scholes and Delta

Since the stock does not pay dividends, the price of the American call option is equal to the price of the otherwise equivalent European call option.

Since the stock does not pay dividends, delta is equal to 1( )N d :

1

1

( ) 0.48405

0.03999

N d

d

== -

The formula for 1d can be solved for the risk-free rate of return:

2

1

2

ln( / ) ( 0.5 )

ln(40 / 42) ( 0 0.5 0.36 )0.250.03999

0.36 0.250.10157

S K r Td

T

r

r

d ss

+ - +=

+ - + ¥- =

=

The value of 2d is:

2 1 0.03999 0.36 0.25 0.21999d d T

Page 179: ActuarialBrew.com Exam MFE / 3F Actuarial Models …actuarialbrew.com/data/documents/MFE-Solutions-2014-1st-ed.pdf · Exam MFE / 3F Actuarial Models Financial Economics Segment Solutions

Exam MFE/3F Solutions Chapter 8 – The Greeks and Other Measures

© ActuarialBrew.com 2014 Page 8.09

The value of the call option is:

2

2

1 20.10157 0.25

0.21999 0.5

0.22 0.5

( , , , , , ) ( ) ( )

40(0.48405) 42 ( 0.21999)

19.36 40.9470 1 (0.21999)

21.5850 40.9470 (0.21999)1

40.9470 21.58502

16.3355 21.5

T rTEur

x

x

C S K r T Se N d Ke N d

e N

N

N

e dx

e dx 850

The correct answer is A.

Solution 8.19

C Volatility of an Option

The first step is to calculate 1d and 2d :

2

1

2

2 1

ln( / ) ( 0.5 )

ln(90 / 95) (0.065 0.00 0.5 0.45 ) 10.24930

0.45 1

0.24930 0.45 1 0.20070

S K r Td

T

d d T

d ss

s

+ - +=

+ - + ¥ ¥= =

= - = - = -

We have:

1( ) (0.24930) 0.59844N d N

2( ) ( 0.20070) 0.42047N d N

The value of the call option is:

1 20(1) 0.065(1)

( , , , , , ) ( ) ( )

90 0.59844 95 0.4204716.4288

T rTEurC S K r T Se N d Ke N d

e e

ds d - -

- -

= -

= ¥ - ¥=

The value of delta is:

0.0(1)1( ) 0.59844 0.59844T

Call e N d ed- -D = = ¥ =

The elasticity of the option is:

90 0.59844

3.278416.4288

SVD ¥W = = =

The volatility of the call option is:

0.45 3.2784 1.4753V Stock Vs s= ¥ W = ¥ =

Page 180: ActuarialBrew.com Exam MFE / 3F Actuarial Models …actuarialbrew.com/data/documents/MFE-Solutions-2014-1st-ed.pdf · Exam MFE / 3F Actuarial Models Financial Economics Segment Solutions

Exam MFE/3F Solutions Chapter 8 – The Greeks and Other Measures

© ActuarialBrew.com 2014 Page 8.10

Solution 8.20

C Portfolio Delta and Elasticity

The elasticity of portfolio A is the weighted average of the elasticities of its components:

w=

W = W

¥= W + W¥ + ¥ +

= W + W

Â1

2 10 52.82

2 10 5 2 10 570.5 20 5

n

Port i ii

Call Put

Call Put

We can express the delta of an option in terms of the elasticity of the option:

DW = fi D = WS V

V S

The delta of portfolio B is the sum of the deltas of its components:

=D = D

= D - D

= W - W

= W - W

Â1

3.50 4 5

10 53.50 4 5

86 86301 40 25

n

Port i ii

Call Put

Call Put

Call Put

q

We now have system of 2 equations with 2 unknowns:

= W + W= W - W

70.5 20 5

301 40 25Call Put

Call Put

Let’s subtract the second equation from twice the first equation:

¥ - = ¥ W - W + ¥ W - - W- = WW = -

2 70.5 301 2 20 40 2 5 ( 25 )

160 35

4.5714

Call Call Put Put

Put

Put

Solution 8.21

D Delta

The delta of a bear spread is the same regardless of whether it is constructed of calls or puts. Let’s assume that the bear spread consists of calls.

Since the bear spread consists of purchasing the higher-strike call and selling the lower-strike call, the delta of the bear spread is:

D - D50 45

Page 181: ActuarialBrew.com Exam MFE / 3F Actuarial Models …actuarialbrew.com/data/documents/MFE-Solutions-2014-1st-ed.pdf · Exam MFE / 3F Actuarial Models Financial Economics Segment Solutions

Exam MFE/3F Solutions Chapter 8 – The Greeks and Other Measures

© ActuarialBrew.com 2014 Page 8.11

We can find the values of delta using:

d-D = 1( )TCall e N d and

d ss

+ - +=2

1ln( / ) ( 0.5 )S K r T

dT

With = 0.25T and = 50K , we have:

( )2

1

0.02 0.2550

ln(51/ 50) 0.06 0.02 0.5(0.25) (0.25)0.30092

0.25 0.25(0.30092) 0.61826

(0.61826) 0.61518

d

N

e- ¥

+ - += =

=

D = =

With = 0.25T and = 45K , we have:

( )2

1

0.02 0.2545

ln(51/ 45) 0.06 0.02 0.5(0.25) (0.25)1.14381

0.25 0.25(1.14381) 0.87365

(0.87365) 0.86929

d

N

e- ¥

+ - += =

=

D = =

Therefore, the delta of the bear spread is initially:

50 45 0.61518 0.86929 0.25412D - D = - = -

With = 2 /12T and = 50K , we have:

( )2

1

0.02 2 /1250

2ln(51/ 50) 0.06 0.02 0.5(0.25)

12 0.310382

0.2512

(0.31038) 0.62186

(0.62186) 0.61979

d

N

e- ¥

+ - += =

=

D = =

With = 2 /12T and = 45K , we have:

( )2

1

0.02 2 /1245

2ln(51/ 45) 0.06 0.02 0.5(0.25)

12 1.342692

0.2512

(1.34269) 0.91031

(0.91031) 0.90728

d

N

e- ¥

+ - += =

=

D = =

Therefore, the delta of the bear spread after 1 month is:

50 45 0.61979 0.90728 0.28749D - D = - = -

The change in the delta of the bear spread is:

0.28749 ( 0.25412) 0.03337- - - = -

Page 182: ActuarialBrew.com Exam MFE / 3F Actuarial Models …actuarialbrew.com/data/documents/MFE-Solutions-2014-1st-ed.pdf · Exam MFE / 3F Actuarial Models Financial Economics Segment Solutions

Exam MFE/3F Solutions Chapter 8 – The Greeks and Other Measures

© ActuarialBrew.com 2014 Page 8.12

Solution 8.22

D Elasticity

The contingent claim can be replicated with a portfolio consisting of the present value of $85 and a short position in a European put option with a strike price of $85.

To find the value of this contingent claim, we must find the value of the put option:

The first step is to calculate 1d and 2d :

2 2

1

2 1

ln( / ) ( 0.5 ) ln(90 / 85) (0.08 0.05 0.5 0.35 ) 0.5

0.35 0.50.41531

0.41531 0.35 0.5 0.16782

S K r Td

T

d d T

We have:

1( ) ( 0.41531) 0.33896N d N

2( ) ( 0.16782) 0.43336N d N

The value of the European put option is:

2 10.08 0.5 0.05 0.5

(85) ( ) ( )

85 0.43336 90 0.33896 5.6381

rT TP Ke N d Se N d

e e

The current value of the contingent claim is the present value of $85 minus the value of the put:

0.08 0.585 5.6381 76.0290V e

The delta of the put option is:

0.05 0.51( ) 0.33896 0.33059T

Put e N d e

The delta of the contingent claim is the delta of the present value of $85 (i.e., zero) minus the delta of the put:

0 ( 0.33059) 0.33059

The elasticity of the contingent claim is:

90 0.33059

0.391376.0290

SV

Solution 8.23

C Elasticity

The contingent claim can be replicated with a portfolio consisting of the present value of $85 and a short position in a European call option with a strike price of $85.

Page 183: ActuarialBrew.com Exam MFE / 3F Actuarial Models …actuarialbrew.com/data/documents/MFE-Solutions-2014-1st-ed.pdf · Exam MFE / 3F Actuarial Models Financial Economics Segment Solutions

Exam MFE/3F Solutions Chapter 8 – The Greeks and Other Measures

© ActuarialBrew.com 2014 Page 8.13

To find the value of this contingent claim, we must find the value of the call option:

The first step is to calculate 1d and 2d :

2 2

1

2 1

ln( / ) ( 0.5 ) ln(90 / 85) (0.08 0.05 0.5 0.35 ) 0.5

0.35 0.50.41531

0.41531 0.35 0.5 0.16782

S K r Td

T

d d T

We have:

1( ) (0.41531) 0.66104N d N= =

2( ) (0.16782) 0.56664N d N

The value of the European call option is:

1 20.05 0.5 0.08 0.5

(85) ( ) ( )

90 0.66104 85 0.56664 11.74885

T rTC Se N d Ke N d

e e

The current value of the contingent claim is the present value of $85 minus the value of the call:

0.08 0.585 11.74885 69.91825V e

The delta of the call option is:

0.05 0.51( ) 0.66104 0.64472T

Call e N d e

The delta of the contingent claim is the delta of the present value of $85 (i.e., zero) minus the delta of the call:

0 0.64472 0.64472

The elasticity of the contingent claim is:

90 ( 0.64472)

0.829969.91825

SV

Solution 8.24

A Call Option Delta

The value of delta can be used to determine 1d :

10.02 1

1

1

1

1

( )

0.6 ( )

0.61212 ( )

( ) 0.61212

0.28485

TCall e N d

e N d

N d

N d

d

Page 184: ActuarialBrew.com Exam MFE / 3F Actuarial Models …actuarialbrew.com/data/documents/MFE-Solutions-2014-1st-ed.pdf · Exam MFE / 3F Actuarial Models Financial Economics Segment Solutions

Exam MFE/3F Solutions Chapter 8 – The Greeks and Other Measures

© ActuarialBrew.com 2014 Page 8.14

The formula for 1d is used to find a quadratic equation in terms of s :

2

1

2

2

2

2

2

ln( / ) ( 0.5 )

ln( / ) (0.0494 0.02 0.5 ) 10.28485

1

(0.0294 0.5 )0.28485

0.28485 0.0294 0.5

0.5 0.28485 0.0294 0

0.5697 0.0588 0

S K r Td

T

S S

The two solutions to the quadratic equation are found below:

20.5697 ( 0.5697) 4(1)(0.0588)2

0.13539 or 0.43431

Shortcut: The higher the value of s , the higher the call price. Since (i) tells us that the call price is less than 10% of the stock price, we can guess that the volatility will be the lower of the two values, so s = 13.539% . Below, we show that this guess is correct.

The value of 2( )N d depends on the value of s :

2 1

2

2 1

2

0.13539 0.28485 0.13539 0.14946

( ) 0.55940

0.43431 0.28485 0.43431 0.14946

( ) 1 0.55940 0.44060

d d T

N d

d d T

N d

s s

s s

= fi = - = - =fi =

= fi = - = - = -fi = - =

For an at-the-money option, K = S. We are given that the call price is less than 10% of the stock price:

1 2

0.02 0.04941 2

0.02 0.04941 2

0.04942

0.04942

2

( ) ( )

( ) ( )

( ) ( ) 0.10

0.6 ( ) 0.10

0.50 ( )

0.52532 ( )

T rTC Se N d Ke N d

Ce N d e N d

S

e N d e N d

e N d

e N d

N d

d- -

- -

- -

-

-

= -

= -

- <

- <

<<

Page 185: ActuarialBrew.com Exam MFE / 3F Actuarial Models …actuarialbrew.com/data/documents/MFE-Solutions-2014-1st-ed.pdf · Exam MFE / 3F Actuarial Models Financial Economics Segment Solutions

Exam MFE/3F Solutions Chapter 8 – The Greeks and Other Measures

© ActuarialBrew.com 2014 Page 8.15

For the inequality above to be satisfied, it must be the case that:

0.13539s =

Solution 8.25

D Theta

We can take the derivative of both sides of the put-call parity equation and solve for the theta of the put option:

d

d

d

q q

q d q

qq

- - - -

- - - -

- - - -

- - - -

+ = +

È ˘ È ˘∂ ∂Î ˚ Î ˚+ = +∂ ∂

+ = +

- + ¥ = ¥ += -

( ) ( )

( ) ( )

( ) ( )

0.10(1 0) 0.05(1 0)7.16 0.10 105 0.05 100

2.4154

r T t T tEur t Eur

r T t T tt

Call Put

r T t T tCall t Put

Put

Put

C Ke S e P

Ke S e

t t

rKe S e

e e

A straddle consists of a put and a call. Therefore, the theta of the straddle is the sum of the theta of the put plus the theta of the call:

q q+ = - - = -2.4154 7.16 9.5754Put Call

Solution 8.26

A Option Volatility

The delta of the put option can be used to solve for - 1( )N d :

10.07 9

1

1

( )

0.1514 ( )

( ) 0.28427

TPut e N d

e N d

N d

d-

- ¥

D = - -

- = - -- =

We can use the standard normal table to find the value of 1d :

1

1

( ) 1 0.28427 0.71573

0.57020

N d

d

= - ==

The formula for 1d can be used to find s :

2

1

2

ln( / ) ( 0.5 )

0 (0.07 0.07 0.5 )90.57020

90.57020 1.5

0.380133

S K r Td

T

d ss

ss

ss

+ - +=

+ - +=

==

Page 186: ActuarialBrew.com Exam MFE / 3F Actuarial Models …actuarialbrew.com/data/documents/MFE-Solutions-2014-1st-ed.pdf · Exam MFE / 3F Actuarial Models Financial Economics Segment Solutions

Exam MFE/3F Solutions Chapter 8 – The Greeks and Other Measures

© ActuarialBrew.com 2014 Page 8.16

We can now find the value of 2d :

2 1 0.57020 0.380133 9 0.57020d d Ts= - = - = -

Next, we find the elasticity of the option:

0.07 91 1

0.07 9 0.07 92 1 2 1

0.63

( ) ( )

( ) ( ) ( ) ( )

0.15140.65886

0.71573 0.1514

T

rT TS e N d e N dS

V Ke N d Se N d e N d e N d

e

d

d

- - ¥

- - - ¥ - ¥

-

¥ - - - -DW = = =- - - - - -

-= = -¥ -

The volatility of the option is:

0.380133 0.65886 0.25045Option Stock Options s= ¥ W = ¥ - =

Solution 8.27

E Option Volatility

The expected return on the stock is equal to the rate of price appreciation plus the dividend yield:

a = + =0.08 0.07 0.15

Since the Sharpe ratio of the call option is equal to the call option of the stock, we can find sStock :

as

sss

- =

- =

==

0.24

0.15 0.090.24

0.25

0.25Stock

r

The values of 1d and 2d are:

d ss

s

+ - + + - + ¥= = =

= - = - =

2 2

1

2 1

ln( / ) ( 0.5 ) ln(105 /100) (0.09 0.07 0.5 0.25 )10.40016

0.25 1

0.40016 0.25 1 0.15016

S K r Td

T

d d T

We use the cumulative normal distribution calculator to obtain:

1

1

( ) (0.40016) 0.65548

( ) (0.15016) 0.55968

N d N

N d N

= == =

The value of the call option is:

d- - - -= - = ¥ - ¥

=

0.07 0.091 2( ) ( ) 105 0.65548 100 0.55968

13.02148

T rTEurC Se N d Ke N d e e

Page 187: ActuarialBrew.com Exam MFE / 3F Actuarial Models …actuarialbrew.com/data/documents/MFE-Solutions-2014-1st-ed.pdf · Exam MFE / 3F Actuarial Models Financial Economics Segment Solutions

Exam MFE/3F Solutions Chapter 8 – The Greeks and Other Measures

© ActuarialBrew.com 2014 Page 8.17

The value of the put option is:

[ ] [ ]d d- - - -

- -

= - - - = - - -

= ¥ - - ¥ -=

2 1 2 1

0.09 0.07

( ) ( ) 1 ( ) 1 ( )

100 (1 0.55968) 105 (1 0.65548)6.51324

rT T rT TEurP Ke N d Se N d Ke N d Se N d

e e

The deltas of the call and put options are:

d

d

- -

- -

D = = ¥ =

D = D - = - = -

0.071

0.07

( ) 0.65548 0.61117

0.61117 0.32123

TCall

TPut Call

e N d e

e e

The standard deviations are:

s s

s s

D ¥= ¥ W = ¥ = ¥ =

D ¥ -= ¥ W = ¥ = ¥ =

105 0.611170.25 0.25 1.23205

13.02148

105 ( 0.32123)0.25 0.25 1.29463

6.51324

CallCall Stock Call

Eur

PutPut Stock Put

Eur

SC

SP

The ranked standard deviations are:

s s s

> >> >

1.29463 1.23205 0.25

Put Call Stock

Solution 8.28

B Elasticity and Risk Premium

Since the Black-Scholes framework applies, we have:

a d s sÊ ˆ È ˘- - - -Á ˜ Î ˚Ë ¯

2 2ln ( 0.5 )( ), ( )T

t

SN T t T t

S

Therefore, from (iii), we have:

a d s- - =20.5 0.14

Under the risk-neutral probability measure, the expected return is r, and so from (iv), we have:

d s

d s

- - - =

- - =

2

2

( 0.5 )(4 2) 0.10

0.5 0.05

r

r

Subtracting the expression above from the one obtained from (iii), we have the risk premium for the stock:

a - = - =0.14 0.05 0.09r

Page 188: ActuarialBrew.com Exam MFE / 3F Actuarial Models …actuarialbrew.com/data/documents/MFE-Solutions-2014-1st-ed.pdf · Exam MFE / 3F Actuarial Models Financial Economics Segment Solutions

Exam MFE/3F Solutions Chapter 8 – The Greeks and Other Measures

© ActuarialBrew.com 2014 Page 8.18

This question is modified from the MFE/3F Sample Exam Questions, and the somewhat confusing language in (vii) is the same as appeared there. In (vii), we are not told if the put option being hedged has been purchased or sold. Therefore, we know that one of the following is true:

D = - D =42.43 or 42.43S S

Since the D of a put is negative, the second expression above must be true. Therefore, we have:

D = -42.43S

We can now find the elasticity of the put option:

D -W = = = -42.43

2.828715

SV

The formula for the risk premium of the option allows us to solve for the absolute value of its expected return:

g aggg

- = W ¥ -- = - ¥= -=

( )

0.07 2.8287 (0.09)0.18458

0.18458

r r

Solution 8.29

E Convex Positions

If a position has positive gamma, then it is convex with respect to the stock’s price. If it has negative gamma, then it is not convex.

Call options and put options have positive values for gamma. The gamma of the stock is zero. Therefore, writing a call and writing a put results in a negative gamma, so Edward has established a position that is not convex.

Page 189: ActuarialBrew.com Exam MFE / 3F Actuarial Models …actuarialbrew.com/data/documents/MFE-Solutions-2014-1st-ed.pdf · Exam MFE / 3F Actuarial Models Financial Economics Segment Solutions

Exam MFE/3F Solutions Chapter 9 –Delta-Hedging

© ActuarialBrew.com 2014 Page 9.01

Chapter 9 – Solutions

Solution 9.01

A Delta-Hedging

The required investment can consist of the market-maker’s capital, or it can be borrowed.

The delta of the position resulting from the sale of the 100 call options is:

Delta of a short position in 100 calls 100 (0.4998) 49.98= - ¥ = -

To delta-hedge the position, the market-maker purchases 49.98 shares of stock. The cost of this position is:

49.98 60 2,998.80¥ =

The investment required is the cost of the stock minus the proceeds received from selling the options:

2,998.80 100 3.76 2,622.80- ¥ =

Solution 9.02

C Market-Maker Profit

The market-maker sells 100 of the call options. From the market-maker’s perspective, the value of this short position is:

100 3.90 390.00- ¥ = -

The delta of the position, from the perspective of the market-maker, is:

Delta of a short position in 100 calls 100 (0.5743) 57.43= - ¥ = -

To delta-hedge the position, the market-maker purchases 57.43 shares of stock. The value of this position is:

¥ =57.43 50 2,871.50

Since the market-maker received only $390.00 from the sale of the calls, the differential must be borrowed. The value of the loan, from the perspective of the market-maker, is:

390.00 2,871.50 2,481.50- = -

The initial position, from the perspective of the market-maker, is:

Component Value Options –390.0000 Shares 2,871.5000 Risk-Free Asset –2,481.5000 Net 0.0000

After 1 day, the value of the options has changed by:

100 (4.32 3.90) 42.00- ¥ - = -

Page 190: ActuarialBrew.com Exam MFE / 3F Actuarial Models …actuarialbrew.com/data/documents/MFE-Solutions-2014-1st-ed.pdf · Exam MFE / 3F Actuarial Models Financial Economics Segment Solutions

Exam MFE/3F Solutions Chapter 9 –Delta-Hedging

© ActuarialBrew.com 2014 Page 9.02

After 1 day, the value of the shares of stock has changed by:

57.43 (50.75 50.00) 43.0725¥ - =

After 1 day, the value of the funds that were borrowed at the risk-free rate has changed by:

( )0.07 / 3652,481.50 1 0.4759e- - = -

The sum of these changes is the overnight profit.

Component Change Gain on Options –42.0000 Gain on Stock 43.0725 Interest –0.4759 Overnight Profit 0.5966

The change in the value of the position is $0.60, and this is the overnight profit.

Solution 9.03

E Market-Maker Profit

The market-maker sells 100 of the put options. From the perspective of the market-maker, the value of this position is:

100 3.04 304.00- ¥ = -

The delta of the position, from the perspective of the market-maker, is:

Delta of a short position in 100 puts 100 ( 0.4257) 42.57= - ¥ - =

To delta-hedge the position, the market-maker sells 42.57 shares of stock. From the perspective of the market-maker, the value of this position is:

42.57 50 2,128.50- ¥ = -

These proceeds plus the $304.00 received from the sale of the puts are lent at the risk-free rate:

+ =2,128.50 304.00 2,432.50

The initial position, from the perspective of the market-maker, is:

Component Value Options –304.0000 Shares –2,128.5000 Risk-Free Asset 2,432.5000 Net 0.0000

After 1 day, the value of the options has changed by:

100 (2.71 3.04) 33.00- ¥ - =

After 1 day, the value of the shares of stock has changed by:

Page 191: ActuarialBrew.com Exam MFE / 3F Actuarial Models …actuarialbrew.com/data/documents/MFE-Solutions-2014-1st-ed.pdf · Exam MFE / 3F Actuarial Models Financial Economics Segment Solutions

Exam MFE/3F Solutions Chapter 9 –Delta-Hedging

© ActuarialBrew.com 2014 Page 9.03

42.57 (50.75 50.00) 31.9275- ¥ - = -

After 1 day, the value of the funds that were lent at the risk-free rate has changed by:

( )0.07 / 3652,432.50 1 0.4666e - =

The sum of these changes is the overnight profit.

Component Change Gain on Options 33.0000 Gain on Stock –31.9275 Interest 0.4666 Overnight Profit 1.5391

The overnight profit is $1.54.

Solution 9.04

A Delta-Hedging

A ratio spread consists of a long position in one option and a short position in another option. The only difference between the two options is that they have different strike prices.

The delta of the position to be hedged is:

100 0.4043 300 0.5854 135.19¥ - ¥ = -

To delta-hedge the position, the market-maker purchases 135.19 shares of stock. The cost of this position is:

135.19 60 8,111.40¥ =

The investment required is the cost of the stock minus the proceeds received from setting up the call ratio spread:

( )8,111.40 300 4.83 100 2.75 6,937.40- ¥ - ¥ =

Solution 9.05

B Market-Maker Profit

From the market-maker’s perspective, the initial value of the position to be hedged is:

300 4.83 100 2.75 1,174.00- ¥ + ¥ = -

The delta of the position to be hedged, from the perspective of the market-maker, is:

300 0.5854 100 0.4043 135.19- ¥ + ¥ = -

To delta-hedge the position, the market-maker purchases 135.19 shares of stock. The value of this position is:

135.19 60 8,111.40¥ =

Page 192: ActuarialBrew.com Exam MFE / 3F Actuarial Models …actuarialbrew.com/data/documents/MFE-Solutions-2014-1st-ed.pdf · Exam MFE / 3F Actuarial Models Financial Economics Segment Solutions

Exam MFE/3F Solutions Chapter 9 –Delta-Hedging

© ActuarialBrew.com 2014 Page 9.04

Since the market-maker received only $1,174.00 for entering into the call ratio spread, the differential must be borrowed. The value of the loan, from the perspective of the market-maker, is:

1,174.00 8,111.40 6,937.40- = -

The initial position, from the perspective of the market-maker, is:

Component Value Options –1,174.0000 Shares 8,111.4000 Risk-Free Asset –6,937.4000 Net 0.0000

After 1 day, the value of the options has changed by:

300 (6.04 4.83) 100 (3.61 2.75) 277.00- ¥ - + ¥ - = -

After 1 day, the value of the shares of stock has changed by:

135.19 (62.00 60.00) 270.38¥ - =

After 1 day, the value of the funds that were borrowed at the risk-free rate has changed by:

( )0.09 / 3656,937.40 1 1.7108e- - = -

The sum of these changes is the overnight profit.

Component Change Gain on Options –277.0000 Gain on Stock 270.3800 Interest –1.7108 Overnight Profit –8.3308

The position experienced an overnight loss of $8.33.

Solution 9.06

C Market-Maker Profit

From the market-maker’s perspective, the initial value of the position to be hedged is:

100 1.60 200 3.50 540.00¥ - ¥ = -

The delta of the position to be hedged, from the perspective of the market-maker, is:

100 ( 0.2377) 200 ( 0.4146) 59.15¥ - - ¥ - =

To delta-hedge the position, the market-maker sells 59.15 shares of stock. The value of this position is:

59.15 60 3,549.00- ¥ = -

Page 193: ActuarialBrew.com Exam MFE / 3F Actuarial Models …actuarialbrew.com/data/documents/MFE-Solutions-2014-1st-ed.pdf · Exam MFE / 3F Actuarial Models Financial Economics Segment Solutions

Exam MFE/3F Solutions Chapter 9 –Delta-Hedging

© ActuarialBrew.com 2014 Page 9.05

The market-maker receives $540.00 for entering into the put ratio spread. The market-maker also receives $3,549 for the stock that it sells. The sum of the proceeds is lent at the risk-free rate of return:

540 3,549 4,089+ =

The initial position, from the perspective of the market-maker, is:

Component Value Options –540.0000 Shares –3,549.0000 Risk-Free Asset 4,089.0000 Net 0.0000

After 1 day, the value of the options has changed by:

100 (1.17 1.60) 200 (2.73 3.50) 111.00¥ - - ¥ - =

After 1 day, the value of the shares of stock has changed by:

59.15 (62.00 60.00) 118.30

After 1 day, the value of the funds that were lent at the risk-free rate has changed by:

0.09 / 3654,089.00 1 1.0084e

The sum of these changes is the overnight profit.

Component Change Gain on Options 111.0000 Gain on Stock –118.3000 Interest 1.0084 Overnight Profit –6.2916

The position experienced an overnight loss of $6.29.

Solution 9.07

E Market-Maker Profit

The market-maker purchases 100 of the call options. From the market-maker’s perspective, the value of this position is:

100 4.69 469.00¥ =

The delta of the position, from the perspective of the market-maker, is:

Delta of long position in 100 calls 100 (0.5218) 52.18= ¥ =

To delta-hedge the position, the market-maker sells 52.18 shares of stock. The value of this position is:

52.18 70 3,652.60- ¥ = -

Page 194: ActuarialBrew.com Exam MFE / 3F Actuarial Models …actuarialbrew.com/data/documents/MFE-Solutions-2014-1st-ed.pdf · Exam MFE / 3F Actuarial Models Financial Economics Segment Solutions

Exam MFE/3F Solutions Chapter 9 –Delta-Hedging

© ActuarialBrew.com 2014 Page 9.06

The market-maker receives $3,652.60 from the sale of the stock and has an outlay of $469 to purchase the calls. Therefore, the amount available to lend is:

3,652.60 469.00 3,183.60- =

The initial position, from the perspective of the market-maker, is:

Component Value Options 469.0000 Shares –3,652.6000 Risk-Free Asset 3,183.6000 Net 0.0000

After 1 day, the value of the options has changed by:

100 (3.68 4.69) 101.00¥ - = -

After 1 day, the value of the shares of stock has changed by:

52.18 (68.00 70.00) 104.36- ¥ - =

After 1 day, the value of the funds that were lent at the risk-free rate has changed by:

( )0.09 / 3653,183.60 1 0.7851e - =

The sum of these changes is the overnight profit.

Component Change Gain on Options –101.0000 Gain on Stock 104.3600 Interest 0.7851 Overnight Profit 4.1451

The overnight profit is $4.15.

Solution 9.08

D Delta

A call bull spread is described in Chapter 2 of the Study Manual.

A call bull spread consists of buying the low-strike call and selling the high-strike call. Therefore, the delta of the speculator’s position is:

1 0.5218 1 0.3994 0.1224¥ - ¥ =

Solution 9.09

A Market-Maker Profit

There is no need to calculate the first day’s profit to answer this question. It might help to envision the market-maker liquidating the position at the end of the first day and then immediately re-selling the 100 calls and delta-hedging them.

Page 195: ActuarialBrew.com Exam MFE / 3F Actuarial Models …actuarialbrew.com/data/documents/MFE-Solutions-2014-1st-ed.pdf · Exam MFE / 3F Actuarial Models Financial Economics Segment Solutions

Exam MFE/3F Solutions Chapter 9 –Delta-Hedging

© ActuarialBrew.com 2014 Page 9.07

At the end of the first day, the market-maker is short 100 call options. From the market-maker’s perspective, the value of this position is:

100 3.68 368.00- ¥ = -

The delta of the position, from the perspective of the market-maker, is:

Delta of short position in 100 calls 100 0.4545 45.45= - ¥ = -

To delta-hedge the position, the market-maker holds 45.45 shares of stock. The value of this position is:

45.45 68.00 3,090.60¥ =

Rebalancing is equivalent to liquidating the position, re-selling the call options, and delta-hedging the position. Since the cost of the stock is greater than the proceeds from selling the call options, the market-maker must borrow (or equivalently, invest its own funds). The value of the loan, from the market-maker’s perspective, is:

368.00 3,090.60 2,722.60- = -

After rebalancing, the position at the end of Day 1 is shown in the rightmost column below:

Day 0 Day 1 Component Value Value Options –469.0000 –368.0000 Shares 3,652.6000 3,090.6000 Risk-Free Asset –3,183.6000 –2,722.6000 Net 0.0000 0.0000

We’ve shown the Day 0 values for information only. They do not need to be calculated to answer this question. For illustrative purposes, the Day 0 values are calculated below:

469.00 100 4.693,652.60 100 0.5218 703,183.60 469.00 3,652.60

During the second day, the value of the options has changed by:

100 (4.62 3.68) 94.00- ¥ - = -

During the second day, the value of the shares of stock has changed by:

45.45 (70.00 68.00) 90.90¥ - =

During the second day, the value of the funds that were borrowed at the risk-free rate has changed by:

( )0.09 / 3652,722.60 1 0.6714e- - = -

Page 196: ActuarialBrew.com Exam MFE / 3F Actuarial Models …actuarialbrew.com/data/documents/MFE-Solutions-2014-1st-ed.pdf · Exam MFE / 3F Actuarial Models Financial Economics Segment Solutions

Exam MFE/3F Solutions Chapter 9 –Delta-Hedging

© ActuarialBrew.com 2014 Page 9.08

The sum of these changes is the overnight profit shown in the Day 2 column below:

Day 1 Day 2 Component Change Change Gain on Options 101.0000 –94.0000 Gain on Stock –104.3600 90.9000 Interest –0.7851 –0.6714 Overnight Profit –4.1451 –3.7714

We’ve shown the Day 1 values for information only. They do not need to be calculated to answer this question. For illustrative purposes, the Day 1 Changes are determined below:

0.09 / 365

101.00 368.00 ( 469.00)104.36 0.5218 100 68 3,652.60

0.7851 3,183.60 e 1

The market-maker experienced a loss of $3.77 during the second day.

Solution 9.10

D Delta Approximation

The delta approximation is:

( ) ( ) 6.13 2 0.5910 7.312tV t h V t

Solution 9.11

D Delta-Gamma Approximation

The delta-gamma approximation is:

2 22

( ) ( ) 6.13 2 0.5910 0.0296 7.3712 2t tV t h V t

Solution 9.12

C Delta-Gamma-Theta Approximation

The delta-gamma-theta approximation is:

2 22 1( ) ( ) 6.13 2 0.5910 0.0296 ( 14.0317)

2 2 3657.333

t t tV t h V t h

Page 197: ActuarialBrew.com Exam MFE / 3F Actuarial Models …actuarialbrew.com/data/documents/MFE-Solutions-2014-1st-ed.pdf · Exam MFE / 3F Actuarial Models Financial Economics Segment Solutions

Exam MFE/3F Solutions Chapter 9 –Delta-Hedging

© ActuarialBrew.com 2014 Page 9.09

Solution 9.13

B Market-Maker Profit

The market-maker’s profit is:

[ ]e qª - G - - D -

-= - - - ¥ -

= -

2

2

Market-maker's profit ( )2

2 ( 14.0317) 0.10(0.0296) (75 0.5910 6.13)

2 365 3650.0312

t t t th rh S V t

Solution 9.14

B Market-Maker Profit

The market-maker’s profit is:

[ ]e qª - G - - D -2

Market-maker's profit ( )2 t t t th rh S V t

We can use the formula above to analyze each of the statements.

A high value of gamma leads to a lower (i.e., more negative) value of 2

2 te- G , so Choice A

is false.

A small stock price movement leads to a higher (i.e., less negative) value of 2

2 te- G , so

Choice B is true.

A high value of theta leads to a lower value of thq- , so Choice C is false. For example, if

theta is –12, then the market-maker’s profit is less than it would be if theta were –13. Therefore, the relatively high value of –12 leads to a lower profit than the relatively low value of –13.

A high interest rate makes the term [ ]- D - ( )t trh S V t smaller, since for a call option,

[ ]D - ( )t tS V t is positive. Therefore, Choice D is false.

The direction of the stock price change does not affect the market-maker’s profit. It is the magnitude of the change that affects the market-maker’s profit. Therefore Choice E is false.

Page 198: ActuarialBrew.com Exam MFE / 3F Actuarial Models …actuarialbrew.com/data/documents/MFE-Solutions-2014-1st-ed.pdf · Exam MFE / 3F Actuarial Models Financial Economics Segment Solutions

Exam MFE/3F Solutions Chapter 9 –Delta-Hedging

© ActuarialBrew.com 2014 Page 9.10

Solution 9.15

D Market-Maker Profit

The market-maker’s profit is:

[ ]e qª - G - - D -2

Market-maker's profit ( )2 t t t th rh S V t

We can use the formula above to analyze each of the statements.

A large stock price movement leads to a lower value of 2

2 te- G , so Choice A is false.

A high value of gamma leads to a lower value of 2

2 te- G , so Choice B is false.

A high value of theta leads to a lower value of thq- , so Choice C is false. For example, if

theta is –12, then the market-maker’s profit is less than it would be if theta were –13. Therefore, the relatively high value of –12 leads to a lower profit than the relatively low value of –13.

A high interest rate makes the term [ ]- D - ( )t trh S V t larger, since the put option’s

negative delta implies that [ ]D - ( )t tS V t is negative. Therefore, Choice D is true.

The direction of the stock price change does not affect the market-maker’s profit. It is the magnitude of the change that affects the market-maker’s profit. Therefore Choice E is false.

Solution 9.16

B Black-Scholes Equation

The Black-Scholes equation can be used to find the price of the option:

s qG + D + =

+ - =

==

2 2

2 2

( )2

0.3 300.0866 (0.08)(30)(0.4118) 4.3974 0.08 ( )

20.09822 0.08 ( )

( ) 1.22775

tt t t t

SrS rV t

V t

V tV t

The Black-Scholes equation does not require the time until maturity of the option to determine the price of the option.

Page 199: ActuarialBrew.com Exam MFE / 3F Actuarial Models …actuarialbrew.com/data/documents/MFE-Solutions-2014-1st-ed.pdf · Exam MFE / 3F Actuarial Models Financial Economics Segment Solutions

Exam MFE/3F Solutions Chapter 9 –Delta-Hedging

© ActuarialBrew.com 2014 Page 9.11

Solution 9.17

E Black-Scholes Equation

The Black-Scholes equation works for put options as well as call options.

The Black-Scholes equation can be used to find the price of the option:

s qG + D + =

+ - - =

==

2 2

2 2

( )2

0.3 300.0866 (0.08)(30)( 0.5882) 1.8880 0.08 ( )

20.20762 0.08 ( )

( ) 2.59525

tt t t t

SrS rV t

V t

V tV t

Solution 9.18

B Black-Scholes Equation

The Black-Scholes equation can be used to find the price of the call option:

2 2

2 2

( )2

0.32 500.0243 (0.08)(50)(0.5901) 4.9231 0.08 ( )

20.5477 0.08 ( )

( ) 6.84625

tt t t t

SrS rC t

C t

C tC t

Put-call parity can be used to find the value of the put option:

00.08(1)

( ) ( )

6.84625 53 50 ( )( ) 5.77

rTC t Ke S P t

e P tP t

Solution 9.19

B Frequency of Re-Hedging

The variance of the hourly returns of the delta-hedged position for one call option is:

[ ] ( )s

¥

= G

Ê ˆÈ ˘ = ¥ ¥ ¥Á ˜Í ˙ Ë ¯¥Î ˚=

1365 24

22 2

22 2

12

1 150 0.30 0.0521

2 365 240.00000089537

hVar R S h

Var R

Page 200: ActuarialBrew.com Exam MFE / 3F Actuarial Models …actuarialbrew.com/data/documents/MFE-Solutions-2014-1st-ed.pdf · Exam MFE / 3F Actuarial Models Financial Economics Segment Solutions

Exam MFE/3F Solutions Chapter 9 –Delta-Hedging

© ActuarialBrew.com 2014 Page 9.12

The standard deviation of the hourly returns for one call option is:

¥

È ˘ = =Í ˙Î ˚1

365 240.00000089537 0.00094624Var R

The standard deviation of the hourly returns for 100 of the call options is just 100 times the standard deviation for one call option:

¥ =100 0.00094624 0.094624

Solution 9.20

D Frequency of Re-Hedging

The variance of the daily returns of the delta-hedged position is the square of the standard deviation:

( )È ˘ = =Í ˙Î ˚1

365

22.271 5.157441Var R

If the market-maker delta-hedges hourly, then the daily variance of the returns is:

[ ]

È ˘Í ˙Î ˚= = ¥ =

1365 1

5.157441 0.21489324 24

hVar RVar R

n

Therefore, with hourly re-hedging, the daily standard deviation is:

=0.214893 0.46357

Solution 9.21

D Frequency of Re-Hedging

Since the market-maker delta-hedges hourly, the daily variance of the returns for one call option is:

[ ] ( )s= ¥ G

È ˘Í ˙ Ê ˆÎ ˚ = ¥ ¥ ¥ ¥ =Á ˜Ë ¯

1365

22 2

22 2

1 12

1 1 160 0.30 0.0434 0.000030920

24 24 2 365

hVar RS h

n n

Var R

The daily standard deviation of the returns for one call option is:

=0.0000309202 0.00556059

The standard deviation for 100 of the call options is just 100 times the standard deviation for one call option:

¥ =100 0.00556059 0.556059

Page 201: ActuarialBrew.com Exam MFE / 3F Actuarial Models …actuarialbrew.com/data/documents/MFE-Solutions-2014-1st-ed.pdf · Exam MFE / 3F Actuarial Models Financial Economics Segment Solutions

Exam MFE/3F Solutions Chapter 9 –Delta-Hedging

© ActuarialBrew.com 2014 Page 9.13

Solution 9.22

B Frequency of Re-Hedging

The gamma the 100 calls is:

100 0.0434 4.340¥ =

The return on the delta-hedged position is:

s= G -2 2 21(1 )

2hR S x h

We can solve for the random values that produce a profit greater than $1.00:

2 2 2

2

2

2

1 11.00 60 0.30 4.34 (1 )

2 365

1.00 1.926247(1 )

0.51914 1

0.48086

0.48086 and 0.480860.69344 and 0.69344

x

x

x

x

x xx x

Since x is a standard normal random variable, the probability that both of the inequalities above are satisfied is:

[ ](0.69344) 1 (0.69344)N N- -

We have:

(0.69344) 0.75598N

The probability of the profit exceeding $1.00 is therefore:

(0.69344) 1 (0.69344) 0.75598 (1 0.75598) 0.51193N N

Solution 9.23

D Frequency of Re-Hedging

The question does not mention re-hedging, so we assume that the market-maker does not re-hedge the position after purchasing it.

We do not need to know the stock price, volatility, gamma, or interval length to answer this question. Regardless of those parameters, a market-maker expects to have profits about two-thirds of the time.

The return on the delta-hedged position is:

s= G -2 2 21(1 )

2hR S x h

Page 202: ActuarialBrew.com Exam MFE / 3F Actuarial Models …actuarialbrew.com/data/documents/MFE-Solutions-2014-1st-ed.pdf · Exam MFE / 3F Actuarial Models Financial Economics Segment Solutions

Exam MFE/3F Solutions Chapter 9 –Delta-Hedging

© ActuarialBrew.com 2014 Page 9.14

We can solve for the random values that produce a positive profit:

s< G -

- <

<< > -

2 2 2

2

2

10.00 (1 )

2

( 1) 0.00

1.001.00 and 1.00

S x h

x

xx x

Since x is a standard normal random variable, the probability that both of the inequalities above are satisfied is:

[ ](1.0000) ( 1.00000) (1.00000) 1 (1.00000) 2 (1.00000) 1

2 0.84134 1 0.68268

N N N N N- - = - - = ¥ -

= ¥ - =

Solution 9.24

B Frequency of Re-Hedging

The variance of the 2-day return of the delta-hedged position for one call option is:

[ ] ( )s

s

= ¥ G

È ˘Í ˙ Ê ˆ Ê ˆÎ ˚ = ¥ G = ¥ ¥ ¥ ¥Á ˜ Á ˜Ë ¯ Ë ¯

=

2365

22 2

2 22 2 2 2

1 12

1 1 2 1 1 280 0.30 0.02058

2 2 2 365 2 2 365

0.0010548

hVar RS h

n n

Var RS

The standard deviation of the 2-day return of the delta-hedged position for one call option is:

=0.0010548 0.032477

The standard deviation for 100 of the call options is just 100 times the standard deviation for one call option:

¥ =100 0.032477 3.2477

Solution 9.25

D Delta-Gamma Hedging

The gamma of the position to be hedged is:

100 0.0521 5.21- ¥ = -

We can now solve for the quantity, Q, of the $55-strike call option that must be purchased to bring the hedged portfolio’s gamma to zero:

5.21 0.0441 0.00

118.1406Q

Q- + ==

Page 203: ActuarialBrew.com Exam MFE / 3F Actuarial Models …actuarialbrew.com/data/documents/MFE-Solutions-2014-1st-ed.pdf · Exam MFE / 3F Actuarial Models Financial Economics Segment Solutions

Exam MFE/3F Solutions Chapter 9 –Delta-Hedging

© ActuarialBrew.com 2014 Page 9.15

Solution 9.26

A Delta-Gamma Hedging

The gamma of the position to be hedged is:

100 0.0521 5.21- ¥ = -

We can solve for the quantity, Q, of the $55-strike call option that must be purchased to bring the hedged portfolio’s gamma to zero:

5.21 0.0441 0.00

118.1406Q

Q- + ==

The delta of the position becomes:

100 0.5824 118.1406 0.3769 13.7128- ¥ + ¥ = -

The quantity of underlying stock that must be purchased, SQ , is the opposite of the delta

of the position being hedged:

13.7128SQ =

Solution 9.27

C Delta-Gamma Hedging

The gamma of the position to be hedged is:

100 0.0521 5.21- ¥ = -

We can solve for the quantity, Q, of the $55-strike call option that must be purchased to bring the hedged portfolio’s gamma to zero:

5.21 0.0441 0.00

118.1406Q

Q

- + ==

The delta of the position becomes:

100 0.5824 118.1406 0.3769 13.7128- ¥ + ¥ = -

The quantity of underlying stock that must be purchased, SQ , is the opposite of the delta

of the position being hedged:

13.7128SQ =

The cost of purchasing the $55-strike options and the stock is offset by the value of the $50-strike options that are sold. The resulting cost of establishing the delta-hedged position is:

13.7128 50 118.1406 2.05 100 3.48 579.83¥ + ¥ - ¥ =

Page 204: ActuarialBrew.com Exam MFE / 3F Actuarial Models …actuarialbrew.com/data/documents/MFE-Solutions-2014-1st-ed.pdf · Exam MFE / 3F Actuarial Models Financial Economics Segment Solutions

Exam MFE/3F Solutions Chapter 9 –Delta-Hedging

© ActuarialBrew.com 2014 Page 9.16

Solution 9.28

E Delta-Gamma Hedging

The gamma of the position to be hedged is:

100 0.0521 5.21- ¥ = -

We can solve for the quantity, Q, of the $55-strike call option that must be purchased to bring the hedged portfolio’s gamma to zero:

5.21 0.0441 0.00

118.1406Q

Q- + ==

The delta of the position becomes:

100 0.5824 118.1406 0.3769 13.7128- ¥ + ¥ = -

The quantity of underlying stock that must be purchased, SQ , is the opposite of the delta

of the position being hedged:

13.7128SQ =

The cost of purchasing the $55-strike options and the stock is offset by the value of the $50-strike options that are sold. The resulting cost of establishing the delta-hedged position is:

13.7128 50 118.1406 2.05 100 3.48 579.8288¥ + ¥ - ¥ =

The cost of establishing the position earns the risk-free rate of interest, so we treat it as borrowing.

After 1 day, the value of the $50-strike options has changed by:

100 (4.06 3.48) 58.00- ¥ - = -

After 1 day, the value of the $55-strike options has changed by:

118.1406 (2.43 2.05) 44.8934¥ - =

After 1 day, the value of the shares of stock has changed by:

13.7128 (51.00 50.00) 13.7128¥ - =

After 1 day, the value of the funds that were borrowed at the risk-free rate has changed by:

0.08 / 365579.8288 1 0.1271e

Page 205: ActuarialBrew.com Exam MFE / 3F Actuarial Models …actuarialbrew.com/data/documents/MFE-Solutions-2014-1st-ed.pdf · Exam MFE / 3F Actuarial Models Financial Economics Segment Solutions

Exam MFE/3F Solutions Chapter 9 –Delta-Hedging

© ActuarialBrew.com 2014 Page 9.17

The sum of these changes is the overnight profit.

Component Change Gain $50-Strike Call –58.0000 Gain on $55-Strike Call 44.8934 Gain on Stock 13.7128 Interest –0.1271 Overnight Profit 0.4791

The overnight profit is $0.4791.

Solution 9.29

B Delta-Gamma Hedging

The gamma of the position to be hedged is:

100 0.05212 5.212¥ =

We can solve for the quantity, Q, of the $55-strike put option that must be purchased to bring the hedged portfolio’s gamma to zero:

5.212 0.04860 0.00

107.2428

Q

Q

+ == -

The delta of the position becomes:

100 0.58240 107.2428 ( 0.66576) 129.6380¥ - ¥ - =

The quantity of underlying stock that must be purchased, SQ , is the opposite of the delta

of the position being hedged:

129.6380SQ = -

The proceeds from selling the $55-strike put options and the stock more than cover the cost of the $50-strike call options, so the cost (i.e., investment) to create this position is negative:

129.6380 50 107.2428 5.44 100 3.48 6,717.2991

Solution 9.30

A Delta-Rho Hedging

The rho of the position to be hedged is:

100 0.08229 8.229- ¥ - =

We can now solve for the quantity, Q, of the $60-strike call option that must be purchased to bring the hedged portfolio’s rho to zero:

8.229 0.10083 0.00

81.6126Q

Q

+ == -

Page 206: ActuarialBrew.com Exam MFE / 3F Actuarial Models …actuarialbrew.com/data/documents/MFE-Solutions-2014-1st-ed.pdf · Exam MFE / 3F Actuarial Models Financial Economics Segment Solutions

Exam MFE/3F Solutions Chapter 9 –Delta-Hedging

© ActuarialBrew.com 2014 Page 9.18

Since Q is negative, 81.6126 of the calls are sold.

The delta of the position becomes:

100 ( 0.45837) 81.6126 0.73855 14.43800

The quantity of underlying stock that must be purchased, SQ , is the opposite of the delta

of the position being hedged:

14.43800SQ

Solution 9.31

E Delta-Rho Hedging

The rho of the position to be hedged is:

100 0.08229 8.229- ¥ - =

We can solve for the quantity, Q, of the $60-strike call option that must be purchased to bring the hedged portfolio’s rho to zero:

8.229 0.10083 0.00

81.6126

Q

Q

+ == -

Since Q is negative, 81.6126 of the calls are sold.

The delta of the position becomes:

100 ( 0.45837) 81.6126 0.73855 14.4380

The quantity of underlying stock that must be purchased, SQ , is the opposite of the delta

of the position being hedged:

14.4380SQ =

The proceeds from selling the puts and the calls are greater than the cost of purchasing the stock. Therefore the following amount is available to lend at the risk-free rate:

100 3.67 81.6126 6.83 14.4380 64 0.3824¥ + ¥ - ¥ =

After 1 day, the change in the value of the put options is:

100 (3.22 3.67) 45.00- ¥ - =

After 1 day, the change in the value of the call options is:

81.6126 (7.55 6.83) 58.7611- ¥ - = -

After 1 day, the change in the value of the stock is:

14.4380 (65.00 64.00) 14.4380¥ - =

After 1 day, the change in the risk-free asset is:

( )0.08 / 3650.3824 1 0.0001e¥ - =

Page 207: ActuarialBrew.com Exam MFE / 3F Actuarial Models …actuarialbrew.com/data/documents/MFE-Solutions-2014-1st-ed.pdf · Exam MFE / 3F Actuarial Models Financial Economics Segment Solutions

Exam MFE/3F Solutions Chapter 9 –Delta-Hedging

© ActuarialBrew.com 2014 Page 9.19

The sum of these changes is the overnight profit.

Component Change Gain on $65-strike Put Options 45.0000 Gain on $60-strike Call Options –58.7611 Gain on Stock 14.4380 Interest 0.0001 Overnight Profit 0.6770

The overnight profit is $0.6770

Alternatively, we can find the value of the overnight profit by calculating the new value of the hedged position:

+ + +

= - ¥ - ¥ + ¥ + ¥ =0.08 / 365

Puts Calls Stock Lending

100 3.22 81.6126 7.55 14.4380 65.00 0.3824 0.6770e

Solution 9.32

C Delta-Gamma-Rho Hedging

The rho of the position to be hedged is:

1 0.08951 0.08951¥ =

The gamma of the position to be hedged is:

1 0.03723 0.03723¥ =

We must solve for the quantity of the $80-strike call options, 80Q , to purchase and the

quantity of the $65-strike put options, 65Q , to purchase in order to offset the rho and

gamma of the $70-strike call option:

80 65

80 65

0.11018 0.21484 0.08951

0.02552 0.01524 0.03723

Q Q

Q Q

- = -+ = -

The solution to this system of equations is:

80

65

1.307290

0.253804

Q

Q

= -= -

Since the values above are negative, both options are sold.

The delta of the position becomes:

1.307290 0.36647 0.253804 ( 0.25344) 1 0.58240 0.167642- ¥ - ¥ - + ¥ =

To offset this delta, the market-maker must sell 0.167642 shares. The value of these shares is:

0.167642 70 11.7349¥ =

Page 208: ActuarialBrew.com Exam MFE / 3F Actuarial Models …actuarialbrew.com/data/documents/MFE-Solutions-2014-1st-ed.pdf · Exam MFE / 3F Actuarial Models Financial Economics Segment Solutions

Exam MFE/3F Solutions Chapter 9 –Delta-Hedging

© ActuarialBrew.com 2014 Page 9.20

Solution 9.33

A Delta-Gamma-Vega Hedging

The vega of the position to be hedged is:

1 0.13645 0.13645¥ =

The gamma of the position to be hedged is:

1 0.03723 0.03723¥ =

We must solve for the quantity of the $80-strike call options, 80Q , to purchase and the

quantity of the $65-strike put options, 65Q , to purchase in order offset the vega and

gamma of the $70-strike call option:

80 65

80 65

0.18502 0.22406 0.13645

0.02552 0.01524 0.03723

Q Q

Q Q

+ = -+ = -

The solution to this system of equations is:

80

65

2.160660

1.175200

Q

Q

= -=

The hedge is constructed by selling 2.160660 of the $80-strike call options and purchasing 1.175200 of the $65-strike put options.

The delta of the position becomes:

2.160660 0.36647 1.175200 ( 0.25344) 1 0.58240 0.507260- ¥ + ¥ - + ¥ = -

To offset this delta, the market-maker must purchase 0.507260 shares.

Solution 9.34

A Delta-Gamma-Rho-Vega Hedging

European options on the same stock that have the same maturity date have the same ratio of gamma to vega. This implies that if the option to be hedged has the same time until maturity as the options used in the hedge, then hedging gamma is equivalent to hedging vega.

The rho of the position to be hedged is:

1 0.1280 0.1280- ¥ = -

The vega of the position to be hedged is:

1 0.1360 0.1360- ¥ = -

The gamma of the position to be hedged is:

1 0.0272 0.0272- ¥ = -

Page 209: ActuarialBrew.com Exam MFE / 3F Actuarial Models …actuarialbrew.com/data/documents/MFE-Solutions-2014-1st-ed.pdf · Exam MFE / 3F Actuarial Models Financial Economics Segment Solutions

Exam MFE/3F Solutions Chapter 9 –Delta-Hedging

© ActuarialBrew.com 2014 Page 9.21

We must solve for the quantity of the $85-strike call options, 85Q , to purchase and the

quantity of the $90-strike put options, 90Q , to purchase in order to offset the rho and

gamma of the $70-strike call option:

85 90

85 90

85 90

rho: 0.1033 0.1432 0.1280

vega: 0.1565 0.1540 0.1360

gamma: 0.0313 0.0308 0.0272

Q Q

Q Q

Q Q

- =+ =+ =

We appear to have 3 equations and 2 unknowns in the system of equations, but notice that the second equation is 5 times the third equation. Therefore, any solution that satisfies the third equation also satisfies the second equation, so we can drop the second equation from the system:

85 90

85 90

rho: 0.1033 0.1432 0.1280

gamma: 0.0313 0.0308 0.0272

Q Q

Q Q

- =+ =

The solution to this system of equations is:

85

90

1.022657

0.156142

Q

Q

== -

The hedge is constructed by purchasing 1.022657 of the $85-strike call options and selling 0.156142 of the $90-strike put options. The delta of the position becomes:

1.022657 0.5570 0.156142 ( 0.5942) 1 0.7082 0.045800¥ - ¥ - - ¥ = -

To offset this delta, the market-maker must purchase 0.045800 shares. The value of these shares is:

0.045800 84.18 3.86¥ =

Solution 9.35

E Delta-Gamma-Rho-Vega Hedging

The rho of the position to be hedged is:

1 0.1280 0.1280- ¥ = -

The vega of the position to be hedged is:

1 0.1360 0.1360- ¥ = -

The gamma of the position to be hedged is:

1 0.0272 0.0272- ¥ = -

Page 210: ActuarialBrew.com Exam MFE / 3F Actuarial Models …actuarialbrew.com/data/documents/MFE-Solutions-2014-1st-ed.pdf · Exam MFE / 3F Actuarial Models Financial Economics Segment Solutions

Exam MFE/3F Solutions Chapter 9 –Delta-Hedging

© ActuarialBrew.com 2014 Page 9.22

We must solve for the quantity of the $85-strike call options, 85Q , to purchase and the

quantity of the $90-strike put options, 90Q , to purchase in order to offset the rho and

gamma of the $70-strike call option:

85 90

85 90

85 90

rho: 0.1033 0.1432 0.1280

vega: 0.1565 0.1540 0.1360

gamma: 0.0313 0.0308 0.0272

Q Q

Q Q

Q Q

- =+ =+ =

We appear to have 3 equations and 2 unknowns in the system of equations, but notice that the second equation is 5 times the third equation. Therefore, any solution that satisfies the third equation also satisfies the second equation, so we can drop the second equation from the system:

85 90

85 90

rho: 0.1033 0.1432 0.1280

gamma: 0.0313 0.0308 0.0272

Q Q

Q Q

- =+ =

The solution to this system of equations is:

85

90

1.022657

0.156142

Q

Q

== -

The hedge is constructed by purchasing 1.022657 of the $85-strike call options and selling 0.156142 of the $90-strike put options. The delta of the position becomes:

1.022657 0.5570 0.156142 ( 0.5942) 1 0.7082 0.045800¥ - ¥ - - ¥ = -

To offset this delta, the market-maker must purchase 0.045800 shares.

The funds raised by selling the $80-strike call option and the $90-strike put option are a bit more than the funds required to buy the $85-strike call option and the stock:

1 8.27 1.022657 5.44 0.156142 7.42 0.045800 84.18 0.009876¥ - ¥ + ¥ - ¥ =

Therefore, 0.009876 is lent at the risk-free rate.

After 1 day, the change in the value of the $80-strike call option is:

1 (14.48 8.27) 6.21- ¥ - = -

After 1 day, the change in the value of the 1.022657 $85-strike calls is:

1.022657 (10.64 5.44) 5.3178¥ - =

After 1 day, the change in the value of the –0.156142 $90-strike puts is:

0.156142 (3.68 7.42) 0.5840- ¥ - =

After 1 day, the change in the value of the 0.045800 shares of stock is:

0.045800 (92.00 84.18) 0.3582¥ - =

Page 211: ActuarialBrew.com Exam MFE / 3F Actuarial Models …actuarialbrew.com/data/documents/MFE-Solutions-2014-1st-ed.pdf · Exam MFE / 3F Actuarial Models Financial Economics Segment Solutions

Exam MFE/3F Solutions Chapter 9 –Delta-Hedging

© ActuarialBrew.com 2014 Page 9.23

After 1 day, the change in the risk-free asset is quite small:

( )0.08 / 3650.009876 1 0.000002e¥ - =

The sum of these changes is the overnight profit.

Component Change Gain on $80-strike call –6.2100 Gain on $85-strike call 5.3178 Gain on $90-strike put 0.5840 Gain on Stock 0.3582 Interest 0.0000 Overnight Profit 0.0499

The overnight profit is $0.0499.

Solution 9.36

C Delta-Hedging and the Black-Scholes Equation

The page references below refer to the second edition of Derivatives Markets textbook.

Let’s consider each of the statements.

Statement A is true. It is a restatement of the first sentence of the “Delta-Hedging of American Options” section on page 430 of the textbook. The Black-Scholes Equation holds for all derivatives.

Statement B is true. It is a restatement of the sentence following the Example 13.3 on page 432. The mean return is:

[ ] 2 2 2 2 2 2

2 2

1 1E Market-maker's profit (1 ) (1 )

2 2

1(1 1) 0

2

E S x h S hE x

S h

s s

s

È ˘ È ˘= G - = G -Í ˙ Î ˚Î ˚

= G - =

Statement C is false. It is contradicted by the last sentence of the second paragraph of page 416. Delta understates the increase in the value of the option when the price of the underlying asset increases. That's why including the gamma term in the delta-gamma approximation always results in a higher, more accurate, price than would be obtained by using delta alone.

Statement D is true. It is a restatement of the first sentence of the third paragraph on page 416. Delta overstates the decline in the value of the option when the price of the underlying asset decreases. That's why including the gamma term in the delta-gamma approximation always results in a higher, more accurate, price than would be obtained by using delta alone.

Page 212: ActuarialBrew.com Exam MFE / 3F Actuarial Models …actuarialbrew.com/data/documents/MFE-Solutions-2014-1st-ed.pdf · Exam MFE / 3F Actuarial Models Financial Economics Segment Solutions

Exam MFE/3F Solutions Chapter 9 –Delta-Hedging

© ActuarialBrew.com 2014 Page 9.24

Statement E is true. It is a restatement of the second paragraph on page 429. The market-maker will experience profits about two-thirds of the time, and the market-maker will experience losses about one-third of the time. The profits tend to be smaller than the losses, and overall the market-maker expects to break even over time, since the expected value of the market-maker’s profit is zero.

Solution 9.37

E Static Option Replication

Static option replication is a strategy that requires little if any rebalancing. Put-call parity tells us that a call option is equivalent to a share of stock, plus a put option, plus borrowing at the risk-free rate:

0rT

Eur EurC S P Ke-= + -

Since the market-maker has written 100 calls, the components of a static hedge include purchasing 100 shares of the stock and 100 of the $85-strike put options. The market-maker may choose to borrow to fund this position or to invest its own funds.

Solution 9.38

A Delta-Hedging

The table in the question is similar to Table 13.2 of the second edition of the Derivatives Markets textbook.

For writing the 100 options, the market-maker receives:

100 8.79 879.00¥ =

The market-maker wrote 100 of the call options, so the initial delta of the position is:

100 0.7493 74.93- ¥ = -

The market-maker hedges this position by purchasing 74.93 shares of the stock for:

74.93 80.00 5,994.40¥ =

Since the cost of the stock exceeds the price of the options, the market-maker borrows:

5,994.40 879.00 5,115.40- =

At the end of Day 1, the change in the value of the options is:

100 (9.51 8.79) 72.00- ¥ - = -

At the end of Day 1, the change in the value of the stock is:

74.93 (81.00 80.00) 74.93¥ - =

At the end of Day 1, the change in the value of the borrowed funds is:

0.10 / 3655,115.40 1 1.4017e

Page 213: ActuarialBrew.com Exam MFE / 3F Actuarial Models …actuarialbrew.com/data/documents/MFE-Solutions-2014-1st-ed.pdf · Exam MFE / 3F Actuarial Models Financial Economics Segment Solutions

Exam MFE/3F Solutions Chapter 9 –Delta-Hedging

© ActuarialBrew.com 2014 Page 9.25

The first day’s profit is the sum of theses changes:

Day 1 Component Change Gain on Options –72.0000 Gain on Stock 74.9300 Interest –1.4017 Daily Profit 1.5283

The first day’s profit is $1.5283.

Solution 9.39

A Delta-Hedging

The table in the question is similar to Table 13.2 of the second edition of the Derivatives Markets textbook.

We can treat each day’s re-hedging as a liquidation of the position, followed by the recreation of the position. The liquidation produces each day’s profits or losses. The recreation of the position creates no cash flows at the outset since any funds needed are borrowed.

For writing the 100 options at the end of Day 4, the market-maker receives:

100 10.20 1,020.00¥ =

The market-maker wrote 100 of the call options, so the delta at the end of Day 4 is:

100 0.8010 80.10- ¥ = -

The market-maker hedges this position by purchasing 80.10 shares of the stock for:

80.10 82.00 6,568.20¥ =

Since the cost of the stock exceeds the price of the options, the market-maker borrows:

6,568.20 1,020.00 5,548.20- =

At the end of Day 5, the change in the value of the options is:

100 (10.98 10.20) 78.00- ¥ - = -

At the end of Day 5, the change in the value of the stock is:

80.10 (83.00 82.00) 80.10¥ - =

At the end of Day 5, the change in the value of the borrowed funds is:

( )0.10 / 3655,548.20 1 1.5203e- ¥ - = -

Page 214: ActuarialBrew.com Exam MFE / 3F Actuarial Models …actuarialbrew.com/data/documents/MFE-Solutions-2014-1st-ed.pdf · Exam MFE / 3F Actuarial Models Financial Economics Segment Solutions

Exam MFE/3F Solutions Chapter 9 –Delta-Hedging

© ActuarialBrew.com 2014 Page 9.26

The fifth day’s profit is the sum of theses changes:

Day 5 Component Change Gain on Options –78.0000 Gain on Stock 80.1000 Interest –1.5203 Daily Profit 0.5797

The fifth day’s profit is $0.5797.

Solution 9.40

C Delta-Gamma Hedging

A butterfly spread can be created by buying a $90-strike call, selling 2 $95-strike calls, and buying a $100-strike call. In this case, the market-maker writes (or sells) the butterfly spread, so the market-maker sells the $90-strike call, buys 2 $95-strike calls, and sells a $100-strike call.

The gamma of the position to be hedged is:

1 0.02502 2 0.02807 1 0.02774 0.00338- ¥ + ¥ - ¥ =

We can now solve for the quantity, Y, of the 180-day call option that must be purchased to bring the hedged portfolio’s gamma to zero:

+ =

= -0.00338 0.01956 0.00

0.172802Y

Y

The delta of the position becomes:

1 0.69088 2 0.55464 1 0.41878 0.172802 0.59575 0.103327- ¥ + ¥ - ¥ - ¥ = -

The quantity of underlying stock that must be purchased, X , is the opposite of the delta of the position being hedged:

= 0.103327X

The ratio is:

= = --0.103327

0.5979490.172802

XY

Solution 9.41

B Delta-Hedging

We are not told the interval of time over which the stock price increases. Instead, we are told that the stock price “quickly jumps.” We can take this to mean that the stock price increase is instantaneous. Therefore there is no need to consider the impact from borrowing or lending at the risk-free rate.

Page 215: ActuarialBrew.com Exam MFE / 3F Actuarial Models …actuarialbrew.com/data/documents/MFE-Solutions-2014-1st-ed.pdf · Exam MFE / 3F Actuarial Models Financial Economics Segment Solutions

Exam MFE/3F Solutions Chapter 9 –Delta-Hedging

© ActuarialBrew.com 2014 Page 9.27

The delta of a call option covering 1 share is:

0.05 1 0.05 11( ) (0.41000) 0.65910 0.626955T

Call e N d e N ed- - ¥ - ¥D = = = ¥ =

The market-maker sold 100 call options, each of which covers 100 shares. Therefore, from the perspective of the market-maker, the delta of the position to be hedged is:

100 100 0.626955 6,269.55- ¥ ¥ = -

To delta-hedge the position, the market-maker purchases 6,269.55 shares of stock. When the stock price increases, the market-maker has a gain on the stock and a loss on the call options:

Gain on Stock: 6,269.55 (52 51) 6,269.55Gain on Calls: 100 63.90 6,390.00

Net Profit: 120.45

¥ - =- ¥ = -

-

The market-maker has a net loss of 120.45.

Solution 9.42

B Market-Maker Profit

The market-maker’s profit is zero if the stock price movement is one standard deviation:

One-standard-deviation move tS hs=

To answer the question, we must determine the value of s . We can determine the value of 1( )N d :

10.0(0.25)

1

1

( )

0.591 ( )

( ) 0.59100

TCall e N d

e N d

N d

From the normal distribution table, this implies that:

1 0.23012d =

The formula for 1d can be used to solve for the value of s :

2

1

2

2

2

ln( / ) ( 0.5 )

ln(50 / 50) (0.10 0 0.5 )0.250.23012

0.25

0.11506 0.025 0.125

0.125 0.11506 0.025 0

S K r Td

T

Page 216: ActuarialBrew.com Exam MFE / 3F Actuarial Models …actuarialbrew.com/data/documents/MFE-Solutions-2014-1st-ed.pdf · Exam MFE / 3F Actuarial Models Financial Economics Segment Solutions

Exam MFE/3F Solutions Chapter 9 –Delta-Hedging

© ActuarialBrew.com 2014 Page 9.28

We use the quadratic formula:

20.11506 ( 0.11506) 4(0.125)(0.025)2(0.125)

0.568964 or 0.351516

s

s s

± - -=

= =

We are told that the volatility is less than 0.50, so 0.351516s = .

The price of the stock moves by:

1

0.351516 50 0.91996365tS h

The stock price moves either up or down by 0.92.

Solution 9.43

E Frequency of Re-Hedging

If the market-maker re-hedges only once per year, then the variance of the annual profit is:

[ ] ( ) ( )s= G = ¥ ¥ ¥ =2 22 2 2 2

11 1

65 0.14 0.0834 1 23.84892 2

Var R S h

If the market-maker re-hedges n times per year, then the variance of the annual profit is:

[ ]

==1 23.8489Var R

n n

The standard deviation of the annual profit does not exceed $2.184:

£

££

2

23.84892.184

23.8489 2.1844.9999

n

nn

Therefore rebalancing must occur at least 4.9999 times per year. The number of days

between rebalancing is = 365X

n. Since n must be greater than or equal to 4.9999, X

must be less than or equal to 73:

£

£

£

£

£

4.99991 1

4.9999365 365

4.9999365

73

73

n

n

n

nX

Page 217: ActuarialBrew.com Exam MFE / 3F Actuarial Models …actuarialbrew.com/data/documents/MFE-Solutions-2014-1st-ed.pdf · Exam MFE / 3F Actuarial Models Financial Economics Segment Solutions

Exam MFE/3F Solutions Chapter 9 –Delta-Hedging

© ActuarialBrew.com 2014 Page 9.29

Solution 9.44

D Frequency of Re-Hedging

If the re-hedging occurs once per month, then the variance of the monthly profit for one option is:

( )s Ê ˆÈ ˘ = G = ¥ ¥ ¥ =Á ˜Í ˙ Ë ¯Î ˚1

12

222 2 2 21 1 165 0.14 0.0834 0.165617

2 2 12Var R S h

If the market-maker re-hedges n times per month, then the variance of the monthly profit is for one option:

È ˘Í ˙Î ˚ =

112 0.165617

Var R

n n

The standard deviation of the monthly return for 100 of the options is 100 times the standard deviation of the monthly return for one option:

¥ =0.165617 40.6961100

n n

The market-maker chooses the re-hedging frequency so that the monthly returns have a standard deviation of no more than $18.20:

£

£

£

2

2

40.696118.20

40.6961

18.204.9999

n

n

n

Therefore rebalancing must occur at least 4.9999 times per month. The number of days

between rebalancing is = 30X

n. Since n must be greater than or equal to 4.9999, X must

be less than or equal to 6:

£

£

£

£

£

4.99991 1

4.999930 30

4.999930

6

6

n

n

n

nX

Page 218: ActuarialBrew.com Exam MFE / 3F Actuarial Models …actuarialbrew.com/data/documents/MFE-Solutions-2014-1st-ed.pdf · Exam MFE / 3F Actuarial Models Financial Economics Segment Solutions

Exam MFE/3F Solutions Chapter 9 –Delta-Hedging

© ActuarialBrew.com 2014 Page 9.30

Solution 9.45

D Delta-Gamma Hedging

The gamma of the position to be hedged is:

1,000 0.0422 42.20- ¥ = -

We can solve for the quantity, Q, of the other call option that must be purchased to bring the hedged portfolio’s gamma to zero:

42.2 0.0414 0.00

1,019.3Q

Q

Since only choice D specifies the purchase of 1,019.3 units of Call-II, we already have enough information to see that Choice D must be the correct answer.

The delta of the position becomes:

1,000 0.4125 1,019.3 0.6159 215.3

The quantity of underlying stock that must be purchased, SQ , is the opposite of the delta

of the position being hedged:

215.3SQ = -

Therefore, in order to delta-hedge and gamma-hedge the position, we must sell 215.3 units of stock and purchase 1,019.3 units of Call-II.

Solution 9.46

B Frequency of Re-Hedging

If the market-maker re-hedges only once per day, then the variance of the daily profit for one option is:

( )s Ê ˆÈ ˘ = G = ¥ ¥ ¥ =Á ˜Í ˙ Ë ¯Î ˚1

365

222 2 2 21 1 185 0.25 0.0187 0.000267611

2 2 365Var R S h

If the market-maker re-hedges n times per day, then the variance of the daily profit for one option is:

È ˘Í ˙È ˘ Î ˚= =Í ˙Î ˚

1365

1365

0.000267611 re-hedgings

Var RVar R n

n n

If the market-maker re-hedges n times per day, then the standard deviation of the daily profit for 1,000 options is 1,000 times the standard deviation for one option:

¥ =0.000267611 16.35881,000

n n

Page 219: ActuarialBrew.com Exam MFE / 3F Actuarial Models …actuarialbrew.com/data/documents/MFE-Solutions-2014-1st-ed.pdf · Exam MFE / 3F Actuarial Models Financial Economics Segment Solutions

Exam MFE/3F Solutions Chapter 9 –Delta-Hedging

© ActuarialBrew.com 2014 Page 9.31

The market-maker chooses the re-hedging frequency so that the daily returns have a standard deviation of no more than $5.79:

2

2

16.35885.79

16.3588

5.797.9826

n

n

n

Therefore rebalancing must occur at least 7.9826 times per day. The number of hours

between rebalancing is = 24X

n. Since n must be greater than or equal to 7.9826, X must

be less than or equal to 3.0065:

£

£

£

£

£

7.98261 1

7.982624 24

7.982624

3.0065

3.0065

n

n

n

nX

Solution 9.47

A Market-Maker Profit

Let’s assume that the options expire at time T and that the current time is time t. We can use put-call parity to obtain a system of 2 equations.

- -

-

+ = +

+ = +

( )8.85 47 5.66

12.13 50 4.53

r T t

rT

Ke

Ke

This can be solved to find rte :

- -

-

¸= Ô fi = =˝= Ô

( ) 43.81 43.811.03325

42.4042.40

r T trt

rT

Kee

Ke

The market-maker sold 100 of the call options. From the market-maker’s perspective, the value of this short position was:

- ¥ = -100 12.13 1,213.00

The delta of the position, from the perspective of the market-maker, was:

= - ¥ = -Delta of a short position in 100 calls 100 (0.726) 72.6

Page 220: ActuarialBrew.com Exam MFE / 3F Actuarial Models …actuarialbrew.com/data/documents/MFE-Solutions-2014-1st-ed.pdf · Exam MFE / 3F Actuarial Models Financial Economics Segment Solutions

Exam MFE/3F Solutions Chapter 9 –Delta-Hedging

© ActuarialBrew.com 2014 Page 9.32

To delta-hedge the position, the market-maker purchased 72.6 shares of stock. The value of this position was:

¥ =72.6 50 3,630.00

Since the market-maker received only $1,213.00 from the sale of the calls, the differential was borrowed. The value of the loan, from the perspective of the market-maker, was:

- + = -3,630.00 1,213.00 2,417.00

The initial position, from the perspective of the market-maker, was:

Component Value Options –1,213.00 Shares 3,630.00 Risk-Free Asset –2,417.00 Net 0.00

After t years elapsed, the value of the options changed by:

- ¥ - =100 (8.85 12.13) 328.00

After t years elapsed, the value of the shares of stock changed by:

¥ - = -72.6 (47.00 50.00) 217.80

After t years elapsed, the value of the funds that were borrowed at the risk-free rate changed by:

( ) ( )- - = - - = -2,417.00 1 2,417.00 1.03325 1 80.38rte

The sum of these changes is the profit.

Component Change Gain on Options 328.00 Gain on Stock –217.80 Interest –80.38 Overnight Profit 29.82

The change in the value of the position is $29.82, and this is the profit.

Solution 9.48

D Market-Maker Profit

We do not have enough information to use the standard formula for the market-maker’s profit:

[ ]e qª - G - - D -2

Market-maker's profit ( )2 t t t th rh S V t

Page 221: ActuarialBrew.com Exam MFE / 3F Actuarial Models …actuarialbrew.com/data/documents/MFE-Solutions-2014-1st-ed.pdf · Exam MFE / 3F Actuarial Models Financial Economics Segment Solutions

Exam MFE/3F Solutions Chapter 9 –Delta-Hedging

© ActuarialBrew.com 2014 Page 9.33

We can use the Black-Scholes Equation to obtain an expression that is equal to zero:

s qÈ ˘

G + D + - =Í ˙Í ˙Î ˚

2 2( ) 0

2t

t t t tS

h rS rV t

Let’s add this expression (which is equal to zero) to the Market-maker’s profit to obtain a new formula for the market-maker’s profit:

[ ] se q q

s e

È ˘Í ˙ª - G - - D - + G + D + -Í ˙Î ˚

= G - G

2 22

2 2 2

Market-maker's profit ( ) ( )2 2

2 2

tt t t t t t t t

tt t

Sh rh S V t h rS rV t

Sh

The market-maker’s profit is:

s eª G - G = -

= -

2 2 2 2 2 20.35 75 1 2Market-maker's profit (0.0296) (0.0296)

2 2 2 365 20.03126

tt t

Sh

Solution 9.49

C Market-Maker Profit

Let’s assume that “several months ago” was time 0. Further, let’s assume that the options expire at time T and that the current time is time t. We can use put-call parity to obtain a system of 2 equations.

( )

21.94 100 4.19

27.82 110 1.95

rT

r T t

Ke

Ke

This can be solved to find rte :

( )

82.25 84.1382.2584.13

rTrt

r T t

Kee

Ke

The market-maker sold 50 of the call options. From the market-maker’s perspective, the value of this short position was:

50 21.94 1,097.00

The delta of the position, from the perspective of the market-maker, was:

Delta of a short position in 50 calls 50 (0.788) 39.4

To delta-hedge the position, the market-maker purchased 39.4 shares of stock. The value of this position was:

39.4 100 3,940.00

Page 222: ActuarialBrew.com Exam MFE / 3F Actuarial Models …actuarialbrew.com/data/documents/MFE-Solutions-2014-1st-ed.pdf · Exam MFE / 3F Actuarial Models Financial Economics Segment Solutions

Exam MFE/3F Solutions Chapter 9 –Delta-Hedging

© ActuarialBrew.com 2014 Page 9.34

Since the market-maker received only $1,097 from the sale of the calls, the differential was borrowed. The value of the loan, from the perspective of the market-maker, was:

1,097.00 3,940.00 2,843.00

The initial position, from the perspective of the market-maker, was:

Component Value Options –1,097.00 Shares 3,940.00 Risk-Free Asset –2,843.00 Net 0.00

After t years elapsed, the value of the options changed by:

50 (27.82 21.94) 294.00

After t years elapsed, the value of the shares of stock changed by:

39.4 (110.00 100.00) 394.00

After t years elapsed, the value of the funds that were borrowed at the risk-free rate changed by:

84.132,843.00 1 2,843.00 1 64.9829

82.25rte

The sum of these changes is the profit.

Component Change Gain on Options –294.00 Gain on Stock 394.00 Interest –64.98 Overnight Profit 35.02

The change in the value of the position is $35.02, and this is the profit.

Solution 9.50

C Market-Maker Profit

Question 47 of the Sample Exam Questions used a constant risk-free interest rate, but in the solution provided by the Society of Actuaries a “remark” described how the question could still be answered if the risk-free interest rate is deterministic but not necessarily constant.

Let’s assume that “several months ago” was time 0. Further, let’s assume that the options expire at time T and that the current time is time t.

Page 223: ActuarialBrew.com Exam MFE / 3F Actuarial Models …actuarialbrew.com/data/documents/MFE-Solutions-2014-1st-ed.pdf · Exam MFE / 3F Actuarial Models Financial Economics Segment Solutions

Exam MFE/3F Solutions Chapter 9 –Delta-Hedging

© ActuarialBrew.com 2014 Page 9.35

The interest rates are not necessarily constant, so they can vary over time. Therefore, we replace the usual discount factors as described below:

--

-- -

Ú

Ú

0 ( )

( )( )

is replaced by

is replaced by

T

Tt

r s dsrT

r s dsr T t

e e

e e

We can use put-call parity to obtain a system of 2 equations:

0 ( )

( )

7.86 50 4.01

14.26 60 1.31

T

Tt

r s ds

r s ds

K e

Ke

This can be solved to find Ú ( )Tt

r s dse :

0 0

0

( ) ( ) ( )( )

( ) ( )

46.15 46.15 47.05

46.1547.05 47.05

T t Ttt

T Tt t

r s ds r s ds r s dsr s ds

r s ds r s ds

Ke Ke ee

Ke Ke

The market-maker purchased 100 of the put options. From the market-maker’s perspective, the value of this position was:

100 4.01 401.00

The delta of the call option can be used to determine the delta of the put option:

d- - ¥D = D - = - = - = -00.662 0.662 1 0.338T TPut Call e e

The delta of the position, from the perspective of the market-maker, was:

Delta of a long position in 100 puts 100 ( 0.338) 33.8

To delta-hedge the position, the market-maker purchased 33.8 shares of stock. The value of this position was:

33.8 50 1,690.00

The market maker borrowed to purchase the puts and the stock. The value of the loan, from the perspective of the market-maker, was:

401.00 1,690.00 2,091

The initial position, from the perspective of the market-maker, was:

Component Value Options 401.00 Shares 1,690.00 Risk-Free Asset –2,091.00 Net 0.00

Page 224: ActuarialBrew.com Exam MFE / 3F Actuarial Models …actuarialbrew.com/data/documents/MFE-Solutions-2014-1st-ed.pdf · Exam MFE / 3F Actuarial Models Financial Economics Segment Solutions

Exam MFE/3F Solutions Chapter 9 –Delta-Hedging

© ActuarialBrew.com 2014 Page 9.36

After t years elapsed, the value of the options changed by:

100 (1.31 4.01) 270.00

After t years elapsed, the value of the shares of stock changed by:

33.8 (60.00 50.00) 338.00

After t years elapsed, the value of the funds that were borrowed at the risk-free rate changed by:

0 ( ) 47.052,091.00 1 2,091.00 1 40.7779

46.15

t r s dse

The sum of these changes is the profit.

Component Change Gain on Options –270.00 Gain on Stock 338.00 Interest –40.78 Overnight Profit 27.22

The change in the value of the position is $27.22, and this is the profit.

Solution 9.51

C Delta-Gamma Approximation

The delta-gamma approximation is:

2

( ) ( )2t tV t h V t

The approximation above can be used to create a quadratic equation:

2

2

2

6.08 5.92 0.323 (0.015)2

0 0.0075 0.323 0.16

0.323 ( 0.323) 4(0.0075)( 0.16)2(0.0075)

0.4898 or 43.5565

The definition of e can be rearranged to obtain an expression for 0S :

e e= - fi = -0 0h hS S S S

Since we have 2 possible values for e , we have 2 possible values for 0S :

ee= - fi = - - == fi = - =

0

0

0.4898 86 ( 0.4898) 86.4898

43.5565 86 43.5565 42.4435

S

S

Page 225: ActuarialBrew.com Exam MFE / 3F Actuarial Models …actuarialbrew.com/data/documents/MFE-Solutions-2014-1st-ed.pdf · Exam MFE / 3F Actuarial Models Financial Economics Segment Solutions

Exam MFE/3F Solutions Chapter 9 –Delta-Hedging

© ActuarialBrew.com 2014 Page 9.37

The value of 42.4435 is less than 75, and Statement (i) in the question tells us that the original stock price is greater than 75. Therefore, the other possibility must be correct, and the initial stock price is 86.4898.

Solution 9.52

B Delta-Gamma Hedging

The gamma of the position to be hedged is:

- ¥ = -1,000 0.0237 23.7

We can solve for the quantity, Q, of the other put option that must be purchased to bring the hedged portfolio’s gamma to zero:

23.7 0.0486 0.00

487.6543

Q

Q

Although the market-maker purchases stock to bring the delta of the position to zero, the vega of the stock is zero. Therefore, the vega of the position is not affected by the quantity of stock that the market-maker holds.

The vega of the position is determined by the quantities of Put-I and Put-II that the market-maker holds:

- ¥ + ¥ = -1,000 0.1774 487.6543 0.0909 133.0722

Page 226: ActuarialBrew.com Exam MFE / 3F Actuarial Models …actuarialbrew.com/data/documents/MFE-Solutions-2014-1st-ed.pdf · Exam MFE / 3F Actuarial Models Financial Economics Segment Solutions

Exam MFE/3F Solutions Chapter 9 –Delta-Hedging

© ActuarialBrew.com 2014 Page 9.38

This page is intentionally blank.

Page 227: ActuarialBrew.com Exam MFE / 3F Actuarial Models …actuarialbrew.com/data/documents/MFE-Solutions-2014-1st-ed.pdf · Exam MFE / 3F Actuarial Models Financial Economics Segment Solutions

Exam MFE/3F Solutions Chapter 10 – Exotic Options: Part I

© ActuarialBrew.com 2014 Page 10.01

Chapter 10 – Solutions

Solution 10.01

B Asian Options

The syllabus doesn’t include the version of the Black-Scholes formula that prices Asian options. This means that the examiners need to rely on the binomial model to calculate the price of an Asian option.

From the information provided, we have:

5050

0.060.30

10.5

20

SK

r

Th

n

This allows us to solve for u and d:

d s

d s

- + - +

- - - -

= = =

= = =

( ) (0.06 0.00)(0.5) 0.3 0.5

( ) (0.06 0.00)(0.5) 0.3 0.5

1.2740

0.8335

r h h

r h h

u e e

d e e

We next solve for the risk-neutral probability of an upward movement:

d- -- -= = =- -

( ) (0.06 0.00)0.5 0.83350.4472

1.2740 0.8335

r he d ep

u d

The tree of stock prices below includes the payoff under each path:

Arithmetic Call Avg. Price Payoff Probability 81.15 72.42 22.42 0.2000 63.70 53.09 58.39 8.39 0.2472

50.00 53.09 47.38 0.00 0.2472 41.67 34.74 38.20 0.00 0.3056

Each average is calculated as an arithmetic average. For example, the first arithmetic average price is:

+ =63.70 81.15

72.422

Page 228: ActuarialBrew.com Exam MFE / 3F Actuarial Models …actuarialbrew.com/data/documents/MFE-Solutions-2014-1st-ed.pdf · Exam MFE / 3F Actuarial Models Financial Economics Segment Solutions

Exam MFE/3F Solutions Chapter 10 – Exotic Options: Part I

© ActuarialBrew.com 2014 Page 10.02

Each call payoff is determined using the arithmetic average price. For example, the first call payoff value is:

72.42 50,0 22.42Max

Each probability is determined using the risk-neutral probabilities. For example, the first probability, which represents two upward movements, is:

0.4472 0.4472 0.2000

The arithmetic average price exceeds 50 for only 2 of the 4 possible paths. The present value of the option is:

0.06 122.42(0.2000) 8.39(0.2472) 0(0.2472) 0(0.3056) 6.18e

Solution 10.02

C Asian Options

From the information provided, we have:

500.060.30

10.5

20

Sr

Th

n

This allows us to solve for u and d:

d s

d s

- + - +

- - - -

= = =

= = =

( ) (0.06 0.00)(0.5) 0.3 0.5

( ) (0.06 0.00)(0.5) 0.3 0.5

1.2740

0.8335

r h h

r h h

u e e

d e e

We next solve for the risk-neutral probability of an upward movement:

d- -- -= = =- -

( ) (0.06 0.00)0.5 0.83350.4472

1.2740 0.8335

r he d ep

u d

The tree of stock prices below includes the payoff under each path:

Geometric Call Avg. Strike Payoff Probability

81.15 71.90 9.25 0.2000 63.70 53.09 58.15 0.00 0.2472

50.00 53.09 47.04 6.05 0.2472 41.67 34.74 38.05 0.00 0.3056

Page 229: ActuarialBrew.com Exam MFE / 3F Actuarial Models …actuarialbrew.com/data/documents/MFE-Solutions-2014-1st-ed.pdf · Exam MFE / 3F Actuarial Models Financial Economics Segment Solutions

Exam MFE/3F Solutions Chapter 10 – Exotic Options: Part I

© ActuarialBrew.com 2014 Page 10.03

Each average is calculated as a geometric average. For example, the first geometric average strike is:

( )¥ =0.563.70 81.15 71.90

Each call payoff is determined using the geometric average strike. For example, the first call payoff value is:

81.15 71.90,0Max

The geometric average strike is less than the final stock price for only 2 of the 4 possible paths. The present value of the option is:

0.06 19.25(0.2000) 0(0.2472) 6.05(0.2472) 0(0.3056) 3.15e

Solution 10.03

A Asian Options

From the information provided, we have:

500.060.30

10.5

20

Sr

Th

n

This allows us to solve for u and d:

d s

d s

- + - +

- - - -

= = =

= = =

( ) (0.06 0.00)(0.5) 0.3 0.5

( ) (0.06 0.00)(0.5) 0.3 0.5

1.2740

0.8335

r h h

r h h

u e e

d e e

We next solve for the risk-neutral probability of an upward movement:

d- -- -= = =- -

( ) (0.06 0.00)0.5 0.83350.4472

1.2740 0.8335

r he d ep

u d

The tree of stock prices below includes the payoff under each path:

Geometric Call Avg. Strike Payoff Probability

81.15 71.90 9.25 0.2000 63.70 53.09 58.15 0.00 0.2472

50.00 53.09 47.04 6.05 0.2472 41.67 34.74 38.05 0.00 0.3056

Page 230: ActuarialBrew.com Exam MFE / 3F Actuarial Models …actuarialbrew.com/data/documents/MFE-Solutions-2014-1st-ed.pdf · Exam MFE / 3F Actuarial Models Financial Economics Segment Solutions

Exam MFE/3F Solutions Chapter 10 – Exotic Options: Part I

© ActuarialBrew.com 2014 Page 10.04

The corresponding tree of option prices is:

9.25 4.02 0.00

3.15 6.05 2.63 0.00

It is not necessary to calculate the option price of 3.15, but we included it here for completeness.

The option prices are obtained by working from right to left. For example, the option price after one upward movement is:

[ ]- ¥ ¥ + - ¥ =0.06 0.5 0.4472 9.25 (1 0.4472) 0 4.02e

The value of delta is:

d- - ¥- -D = = =- -

0 0.5 4.02 2.63( ,0) 0.0630

63.70 41.67h u dV V

S e eSu Sd

Solution 10.04

B Barrier Options

From the information provided, we have:

5050

0.060.30

10.5

20

SK

r

Th

n

This allows us to solve for u and d:

d s

d s

- + - +

- - - -

= = =

= = =

( ) (0.06 0.00)(0.5) 0.3 0.5

( ) (0.06 0.00)(0.5) 0.3 0.5

1.2740

0.8335

r h h

r h h

u e e

d e e

We next solve for the risk-neutral probability of an upward movement:

d- -- -= = =- -

( ) (0.06 0.00)0.5 0.83350.4472

1.2740 0.8335

r he d ep

u d

Page 231: ActuarialBrew.com Exam MFE / 3F Actuarial Models …actuarialbrew.com/data/documents/MFE-Solutions-2014-1st-ed.pdf · Exam MFE / 3F Actuarial Models Financial Economics Segment Solutions

Exam MFE/3F Solutions Chapter 10 – Exotic Options: Part I

© ActuarialBrew.com 2014 Page 10.05

The tree of stock prices below includes the payoff under each path:

Call Payoff Probability

81.15 31.15 0.2000 63.70 53.09 3.09 0.2472

50.00 53.09 0.00 0.2472 41.67 34.74 0.00 0.3056

The barrier of $60 is above the current stock price, so the call option is an up-and-in call option. Therefore the call option pays off only if the stock price exceeds $60, which occurs only if the stock moves up in the first period. For example, the first call payoff value is:

81.15 50.00,0 31.15Max

The present value of the option is:

0.06 131.15(0.2000) 3.09(0.2472) 0(0.2472) 0(0.3056) 6.59e

Solution 10.05

E Asian options

When calculating the averages, we do not use the initial stock price.

The arithmetic average is:

+ + + + += =5 7 8 12 4 3

(1.5) 6.56

A

The geometric average is:

= ¥ ¥ ¥ ¥ ¥ =1 / 6(1.5) (5 7 8 12 4 3) 5.86G

The payoff of the arithmetic average price Asian call is:

- = - =Average price Asian call option payoff = Max[ ( ) ,0] 6.5 5 1.5A T K

The payoff of the geometric average strike Asian put is:

- = - =Average strike Asian put option payoff = Max[ ( ) ,0] 5.86 3 2.86TG T S

The total payoff on 7/1/2008 is:

+ =1.5 2.86 4.36

Page 232: ActuarialBrew.com Exam MFE / 3F Actuarial Models …actuarialbrew.com/data/documents/MFE-Solutions-2014-1st-ed.pdf · Exam MFE / 3F Actuarial Models Financial Economics Segment Solutions

Exam MFE/3F Solutions Chapter 10 – Exotic Options: Part I

© ActuarialBrew.com 2014 Page 10.06

Solution 10.06

D Asian Options

Consider the payoffs of each of the available Asian options:

Arithmetic average price Asian call: [ ]-Max ( ) , 0A T K

Geometric average price Asian call: [ ]-Max ( ) , 0G T K

Arithmetic average strike Asian call: [ ]-Max ( ), 0TS A T

Arithmetic average price Asian put: [ ]-Max ( ), 0K A T

Arithmetic average strike Asian put: [ ]-Max ( ) , 0TA T S

The arithmetic average price Asian call and the geometric average price Asian call are inappropriate since their payoffs decline yen falls in value.

The arithmetic average strike Asian call will not provide a positive payoff if the yen’s value is low at the end of the 2 years.

The payoff of the arithmetic average price Asian put increases as the average value of the yen falls. Therefore, answer choice D is the correct answer.

The arithmetic average strike Asian put does not provide a positive payoff if the yen’s value is low for most of the 2 years but rises at the very end.

Solution 10.07

E Barrier options

Only the first row of the table is needed to solve this problem. The rest of the table was included to make the question seem more difficult than it really is.

The stock price at issuance is less than $63, so the option is an up-and-in call option.

The up-and-in call option and the ordinary call option both have the same payoff:

[ ]-Max 65, 0TS

Since a positive payoff requires that TS be greater than $65, the barrier of $63 is reached in all of the positive payoff scenarios. Therefore, the barrier does not reduce the value of the up-and-in call relative to the ordinary call. So, the price of this call is $7.15.

Page 233: ActuarialBrew.com Exam MFE / 3F Actuarial Models …actuarialbrew.com/data/documents/MFE-Solutions-2014-1st-ed.pdf · Exam MFE / 3F Actuarial Models Financial Economics Segment Solutions

Exam MFE/3F Solutions Chapter 10 – Exotic Options: Part I

© ActuarialBrew.com 2014 Page 10.07

Solution 10.08

C Barrier Options

The payoffs of each of the options are: Payoff

A: A knock-in call with a barrier of $105 and a strike of $100 10

B: A knock-in call with a barrier of $95 and a strike of $105 0

C: A knock-out call with a barrier of $95 and a strike of $105 5

D: A knock-in put with a barrier of $105 and a strike of $105 0

E: A knock-out put with a barrier of $95 and a strike of $120 10

Solution 10.09

E Barrier Options

For a given barrier, the parity relationship can be used to find the value of the ordinary call option. Since we have the value of both the knock-in call and the knock-out call option when the barrier is $57:

Knock-in option + Knock-out option = Ordinary option

+ =10.35 5.14 15.49

Therefore an ordinary call option with a strike price of $50 has a value of $15.49.

We can apply the parity relationship again, this time with the barrier set to $54:

Knock-in option + Knock-out option = Ordinary option

6.40 + Knock-out option = 15.49

Knock-out option = - =15.49 6.40 9.09

Solution 10.10

D Barrier Options

The first step is the use the Black-Scholes formula to find the value of an ordinary call option.

The values of 1d and 2d are:

2 0.0 1 2

0.08 1

1

2 1

75 0.30 1ln ln

2 2750.41667

0.30 1

0.41667 0.3 1 0.11667

T

rTSe T e

Ke ed

T

d d T

We have:

1( ) (0.41667) 0.66154N d N

2( ) (0.11667) 0.54644N d N

Page 234: ActuarialBrew.com Exam MFE / 3F Actuarial Models …actuarialbrew.com/data/documents/MFE-Solutions-2014-1st-ed.pdf · Exam MFE / 3F Actuarial Models Financial Economics Segment Solutions

Exam MFE/3F Solutions Chapter 10 – Exotic Options: Part I

© ActuarialBrew.com 2014 Page 10.08

Using Black-Scholes, the value of the ordinary call option is:

1 20.08

( ) ( )

75(0.66154) 75 (0.54644) 11.7834

t rTEurC Se N d Ke N d

e

The value of the knock-out call option can be found using the parity relation for barrier options:

Knock-in option Knock-out option Ordinary option

3.56 Knock-out option 11.7834

Knock-out option 11.7834 3.56 8.22

Solution 10.11

A Barrier Options

Since the current stock price of $75 is already above the barrier of $60, the up-and-in put is the same as an ordinary put option. Therefore, the ordinary put option has a price of $8.31.

We can now use the parity relationship for barrier options to find the value of the down-and-out put:

Down-and-in put + Down-and-out put = Ordinary put

7.13 Down-and-out put 8.31Down-and-out put 8.31 7.13 1.18

Solution 10.12

A Barrier Options

We don’t need the table to answer this question. The table was included to make the question seem more difficult than it really is.

The stock price upon issuance of the option is less than the barrier of $63, so the option is an up-and-out call option.

For the up-and-out call option to have a positive payoff, the final stock price must exceed the strike price of $65. But that would imply that the final stock price is also above $63, meaning that the option has been “knocked-out.” Consequently, there is no possibility that the option will produce a positive payoff. Therefore, the price of the up-and-out call option is $0.

Page 235: ActuarialBrew.com Exam MFE / 3F Actuarial Models …actuarialbrew.com/data/documents/MFE-Solutions-2014-1st-ed.pdf · Exam MFE / 3F Actuarial Models Financial Economics Segment Solutions

Exam MFE/3F Solutions Chapter 10 – Exotic Options: Part I

© ActuarialBrew.com 2014 Page 10.09

Solution 10.13

A Compound Options

We are asked to find the value of a put on a put option. We can use put-call parity to find the value of the put on a put:

-

-

- = -

- = -=

1

0.05(1)4.17 6.51 3

0.51

rtEurCallOnPut PutOnPut P xe

PutOnPut e

PutOnPut

Solution 10.14

C Compound Options

The value of the American call option is:

-= - +10 0( , , ) rt

AmerC S K T S Ke CallOnPut

The call on a put expires at time 1t and has a strike price of:

( ) ( )- - - -= - - = - - =1( ) 0.07(0.5 0.25)1 5 83 1 3.56r T tx D K e e

The underlying put option has Stock Z as its underlying asset, a strike price of $83, and matures in 6 months.

From the table, we observe that the price of the call on the put is $4.04 when the strike is $3.56.

Therefore, the price of the American option is:

- -= - + = - + =1 0.07(0.25)0 0( , , ) 80 83 4.04 2.48rt

AmerC S K T S Ke CallOnPut e

Solution 10.15

B Compound Options

We know that the value of an American call option is:

-= - +10 0( , , ) rt

AmerC S K T S Ke CallOnPut

The call on a put expires at time 1t and has a strike price of:

( )- -= - - 1( )1 r T tx D K e

In this case, we have:

( ) ( )- - - -= - - = - - =1( ) 0.08(1 0.25)1 6 65 1 2.215r T tx D K e e

Page 236: ActuarialBrew.com Exam MFE / 3F Actuarial Models …actuarialbrew.com/data/documents/MFE-Solutions-2014-1st-ed.pdf · Exam MFE / 3F Actuarial Models Financial Economics Segment Solutions

Exam MFE/3F Solutions Chapter 10 – Exotic Options: Part I

© ActuarialBrew.com 2014 Page 10.10

Therefore the value of the compound call with a strike price of $2.215 can be found as follows:

-

-

= - +

= - +=

10 0

0.08(0.25)

( , , )

4.49 65 653.20

rtAmerC S K T S Ke CallOnPut

e CallOnPutCallOnPut

Solution 10.16

D Compound Options

Using the formula for an American call option, we can find the value of the call on a put. Then we can use compound option parity to find the value of the ordinary put option.

We know that the value of an American call option is:

-= - +10 0( , , ) rt

AmerC S K T S Ke CallOnPut

The call on a put expires at time 1t and has a strike price of:

( )- -= - - 1( )1 r T tx D K e

In this case, we have:

( ) ( )- - - -= - - = - - =1( ) 0.08(1 0.25)1 6 57 1 2.68r T tx D K e e

Therefore the value of the compound call with a strike price of $2.68 can be found as follows:

-

-

= - +

= - +=

10 0

0.08(0.25)

( , , )

9.85 65 570.7213

rtAmerC S K T S Ke CallOnPut

e CallOnPutCallOnPut

We can now use the parity relationship to find the value of the European put option:

-

-

- = -

- = -=

1

0.08(0.25)0.7213 0.88 2.682.468

rtCallOnPut PutOnPut Put xe

Put ePut

Solution 10.17

B Compound Options

We know that the value of an American call option is:

-= - +10 0( , , ) rt

AmerC S K T S Ke CallOnPut

Page 237: ActuarialBrew.com Exam MFE / 3F Actuarial Models …actuarialbrew.com/data/documents/MFE-Solutions-2014-1st-ed.pdf · Exam MFE / 3F Actuarial Models Financial Economics Segment Solutions

Exam MFE/3F Solutions Chapter 10 – Exotic Options: Part I

© ActuarialBrew.com 2014 Page 10.11

The call on a put expires at time 1t and has a strike price of:

( )- -= - - 1( )1 r T tx D K e

In this case, we have:

( ) ( )- - - -= - - = - - =1( ) 0.08(1 0.25)1 6 65 1 2.215r T tx D K e e

Therefore the value of the compound call with a strike price of $2.215 can be found as follows:

10 0

0.08(0.25)

( , , )

4.49 65 65

3.2029

rtAmerC S K T S Ke CallOnPut

e CallOnPut

CallOnPut

We can now use the parity relationship to find the value of the put on a put option:

-

-

- = -

- = -=

1

0.08(0.25)3.2029 5.21 2.215

0.16

rtCallOnPut PutOnPut Put xe

PutOnPut e

PutOnPut

Solution 10.18

B Compound Options

This question is potentially confusing because we might be tempted to try to apply the formula involving an American call option and a call on a put. But that formula is applicable only when there is a single discrete dividend, and Stock X does not pay any dividends.

At the end of 9 months, the American call option has 3 months until expiration. Since Stock X does not pay dividends, the value of the American call option is equal to the value of an otherwise equivalent European call option:

=(100,90,0.25) 13.37EurC

We can use put-call parity to find the value of the underlying European put option:

d- -

-

- = -

- = -=

00.08(0.25)

(100,90,0.25) (100,90,0.25)

13.37 (100,90,0.25) 100 90

(100,90,0.25) 1.588

T rTEur Eur

Eur

Eur

C P S e Ke

P e

P

The right to buy the European put option for $1 is worth:

- =1.588 1.00 0.588

Page 238: ActuarialBrew.com Exam MFE / 3F Actuarial Models …actuarialbrew.com/data/documents/MFE-Solutions-2014-1st-ed.pdf · Exam MFE / 3F Actuarial Models Financial Economics Segment Solutions

Exam MFE/3F Solutions Chapter 10 – Exotic Options: Part I

© ActuarialBrew.com 2014 Page 10.12

Solution 10.19

D Gap Options

The strike and trigger prices are:

= == =

1

2

Strike price 29

Trigger price 31

K

K

The values of 1d and 2d are:

d s

ss

- -

- -

Ê ˆ Ê ˆ+ +Á ˜ Á ˜

Ë ¯ Ë ¯= = = -

= - = - - = -

2 0.03(0.5) 2

0.06(0.5)2

1

2 1

30 0.20 (0.5)ln ln

2 2310.05508

0.20 0.5

0.05508 0.2 0.5 0.19650

T

rTSe T e

K e ed

T

d d T

We have:

1( ) ( 0.05508) 0.47804N d N

2( ) ( 0.19650) 0.42211N d N

Using Black-Scholes, the value of the gap call option is:

1 1 2

0.03(0.5) 0.06(0.5)

( ) ( )

30 (0.47804) 29 (0.42211) 2.2483

t rTGapC Se N d K e N d

e e

Solution 10.20

C Gap Options

Let’s consider each statement separately:

A When the strike is equal to the trigger, the premium for a gap option is the same as for an ordinary option with the same strike.

True. When the strike is equal to the trigger, the payoffs for the gap option are the same as the payoffs for an ordinary option.

B If the trigger exceeds the strike for a gap put, then increasing the trigger reduces the premium.

True. If the trigger exceeds the strike for a gap put, then negative payoffs are possible. Increasing the trigger makes even more negative payoffs possible.

C If the trigger is fixed for a gap put, then increasing the strike price decreases the premium.

False. Increasing the strike for a gap put makes the payoffs larger, so it increases the premium.

Page 239: ActuarialBrew.com Exam MFE / 3F Actuarial Models …actuarialbrew.com/data/documents/MFE-Solutions-2014-1st-ed.pdf · Exam MFE / 3F Actuarial Models Financial Economics Segment Solutions

Exam MFE/3F Solutions Chapter 10 – Exotic Options: Part I

© ActuarialBrew.com 2014 Page 10.13

D If the strike is equal to the trigger, then increasing the trigger reduces the premium of a gap call.

True. Increasing the trigger above the strike removes some of the positive payoffs that would otherwise occur when the final stock price is between the strike and the trigger.

E For any positive strike, there is a trigger that makes the premium for a gap put equal to zero.

True. The premium is positive if the trigger is equal to the strike. From there, the trigger can be increased until the premium falls to zero.

Since statement C is false, answer choice C is the answer to the question.

Solution 10.21

A Gap Options

Option A must have a higher price than Option B because if < <50 60TS , then Option A has positive payoffs while Option B has zero payoffs.

Option A must have a higher price than Option C because if > 50TS , then the payoffs to Option C are $10 less than the payoffs the Option A.

We can use put-call parity to show that the price of Option A must be higher than the price of Option E. Notice that in the put-call parity relationship below, Option A is an ordinary call and Option E is an ordinary put:

d- -

- -

- = -

- = -- =

fi >

00.04 0.08

(Option A) (Option E) 50 50

(Option A) (Option E) 1.88

(Option A) (Option E)

T rTEur EurC P S e Ke

e e

Option E must have a higher price than Option D because if < <40 50TS , then Option E has positive payoffs while Option D has zero payoffs.

Since we have already shown that Option A has a higher price than Option E, Option A must have the highest price.

Solution 10.22

B Gap Options

Gap options are not path-dependent, so we can ignore all of the prices except for the final one. If these options had been either Asian or Barrier options, then we would have needed to know the path to answer the question.

Page 240: ActuarialBrew.com Exam MFE / 3F Actuarial Models …actuarialbrew.com/data/documents/MFE-Solutions-2014-1st-ed.pdf · Exam MFE / 3F Actuarial Models Financial Economics Segment Solutions

Exam MFE/3F Solutions Chapter 10 – Exotic Options: Part I

© ActuarialBrew.com 2014 Page 10.14

We recall that the gap call option payoff is the final stock price less the strike price if the final stock price is above the trigger, but the gap call option payoff is zero if the final stock price is less than or equal to the trigger:

1 2

2

if Gap call option payoff

0 if T T

T

S K S K

S K

Similarly, the gap put option payoff is the strike price less the final stock price if the final stock price is less than the trigger, but the gap put option payoff is zero if the final stock price is greater than or equal to the trigger:

1 2

2

if Gap put option payoff

0 if T T

T

K S S K

S K

Based on the final stock price of $55, the payoffs to each option are shown in the right-most column below:

Option Type Strike Trigger Payoff

1 Gap call 50 50 5

2 Gap call 50 60 0

3 Gap call 60 50 –5

4 Gap put 50 40 0

5 Gap put 50 60 –5

The total payoff is:

+ - + - = -5 0 5 0 5 5

Solution 10.23

C Gap Options

This problem isn’t as time-consuming as it may first appear because we can use the same values of 1N(d ) and 2N(d ) for each of the options.

Since each of the options has the same trigger =2( 82)K , we will use the same values of

1( )N d and 2( )N d for all three options.

The values of 1d and 2d are:

2 0.0 1 2

0.08 12

1

2 1

80 0.41 1ln ln2 282

0.339900.41 1

0.33990 0.41 1 0.07010

T

rTSe T e

K e ed

T

d d T

Page 241: ActuarialBrew.com Exam MFE / 3F Actuarial Models …actuarialbrew.com/data/documents/MFE-Solutions-2014-1st-ed.pdf · Exam MFE / 3F Actuarial Models Financial Economics Segment Solutions

Exam MFE/3F Solutions Chapter 10 – Exotic Options: Part I

© ActuarialBrew.com 2014 Page 10.15

We have:

1

2

( ) (0.33990) 0.63303

( ) ( 0.07010) 0.47206

N d N

N d N

The value of the European call option is:

1 1 20.08

(80,82,82) ( ) ( )

80(0.63303) 82 (0.47206) 14.9096

t rTC Se N d K e N d

e

d- -

-

= -

= - =

The value of the gap option having a strike price of $84 is:

1 1 20.08

(80,84,82) ( ) ( )

80(0.63303) 84 (0.47206) 14.0380

t rTC Se N d K e N d

e

d- -

-

= -

= - =

The value of the gap option having a strike price of $120 is:

1 1 20.08

(80,120,82) ( ) ( )

80(0.63303) 120 (0.47206) 1.6496

t rTC Se N d K e N d

e

d- -

-

= -

= - = -

The total value of the three options is:

14.9096 14.0380 1.6496 27.2980+ - =

Solution 10.24

C Gap Options

The strike and trigger prices are:

= == =

1

2

Strike price 1.00

Trigger price 0.90

K

K

The values of 1d and 2d are:

2 0.03(1) 2

0.06(1)2

1

2 1

0.90 0.10 (1)ln ln

2 20.900.35000

0.10 1

0.35000 0.1 1 0.25000

T

rTSe T e

K e ed

T

d d T

d s

ss

- -

- -

Ê ˆ Ê ˆ+ +Á ˜ Á ˜

Ë ¯ Ë ¯= = =

= - = - =

The values of 1( )N d and 2( )N d are:

1( ) (0.35000) 0.63683N d N= =

2( ) (0.25000) 0.59871N d N= =

The value of the gap call option is:

1 1 2

0.03(1) 0.06(1)

( ) ( )

0.90 (0.63683) 1.00 (0.59871) 0.007636

t rTGapC Se N d K e N d

e e

d- -

- -

= -

= - = -

Page 242: ActuarialBrew.com Exam MFE / 3F Actuarial Models …actuarialbrew.com/data/documents/MFE-Solutions-2014-1st-ed.pdf · Exam MFE / 3F Actuarial Models Financial Economics Segment Solutions

Exam MFE/3F Solutions Chapter 10 – Exotic Options: Part I

© ActuarialBrew.com 2014 Page 10.16

Solution 10.25

D Gap Options

The price of a gap option is linearly related to its strike price. This means that if the only variable changing is the strike price, then we can use linear interpolation (or extrapolation when appropriate) to find the new value of a gap option corresponding to a new strike price.

First, let’s use linear interpolation to solve this problem.

Since the only difference between the three gap call options is their strike price, we can use linear interpolation to find the value of the third option:

-+ - =-

50 484.98 (1.90 4.98) 4.21

56 48

Second, there is another (somewhat longer) way to solve this problem.

The formula for the price of a gap call option is:

d- -= -1 1 2( ) ( )t rTGapC Se N d K e N d

This suggests the following system of 2 equations:

d

d

- -

- -

= -

= -1 2

1 2

4.98 ( ) 48 ( )

1.90 ( ) 56 ( )

t rT

t rT

Se N d e N d

Se N d e N d

Subtracting the second equation from the first equation, we have:

2

2

3.08 8 ( )

( ) 0.385

rT

rT

e N d

e N d

This allows us to solve for the first term of the pricing formula:

d

d

-

-

= -

=1

1

4.98 ( ) 48(0.385)

( ) 23.46

t

t

Se N d

Se N d

We can now find the price of the gap call option having a strike price of $50:

d- -= -

= - =1 1 2( ) ( )

23.46 (50)(0.385) 4.21

t rTGapC Se N d K e N d

Solution 10.26

C Gap Options

The strike and trigger prices are:

= == =

1

2

Strike price 65

Trigger price 52

K

K

Page 243: ActuarialBrew.com Exam MFE / 3F Actuarial Models …actuarialbrew.com/data/documents/MFE-Solutions-2014-1st-ed.pdf · Exam MFE / 3F Actuarial Models Financial Economics Segment Solutions

Exam MFE/3F Solutions Chapter 10 – Exotic Options: Part I

© ActuarialBrew.com 2014 Page 10.17

The values of 1d and 2d are:

d s

ss

- -

- -

Ê ˆ Ê ˆ+ +Á ˜ Á ˜

Ë ¯ Ë ¯= = = -

= - = - - = -

2 0.04(0.25) 2

0.07(0.25)2

1

2 1

50 0.40 (0.25)ln ln

2 2520.05860

0.40 0.25

0.05860 0.4 0.25 0.25860

T

rTSe T e

K e ed

T

d d T

We have:

1( ) ( 0.05860) 0.47664N d N

2( ) ( 0.25860) 0.39797N d N

The value of the gap call option is:

1 1 2

0.04(0.25) 0.07(0.25)

( ) ( )

50 (0.47664) 65 (0.39797) 1.8244

t rTGapC Se N d K e N d

e e

Solution 10.27

E Asian options

When calculating the averages, we do not use the initial stock price.

The arithmetic average is:

+ + + + += =8 5 2 4 3 2

(1.5) 46

A

The geometric average is:

1 / 6(1.5) (8 5 2 4 3 2) 3.5255G

The payoff of the arithmetic average strike Asian put is:

- = - =Average strike Asian put option payoff = Max[ ( ) ,0] 4 2 2.0TA T S

The payoff of the geometric average price Asian put is:

Average price Asian put option payoff = Max[ ( ),0] 5 3.5255 1.4745K G T

The total payoff is:

2.0 1.4745 3.4745

Solution 10.28

A Compound Options

The dividend on Stock Y does not occur until a year has elapsed. By then, the options have expired, so the dividend does not affect our valuation of the options.

Page 244: ActuarialBrew.com Exam MFE / 3F Actuarial Models …actuarialbrew.com/data/documents/MFE-Solutions-2014-1st-ed.pdf · Exam MFE / 3F Actuarial Models Financial Economics Segment Solutions

Exam MFE/3F Solutions Chapter 10 – Exotic Options: Part I

© ActuarialBrew.com 2014 Page 10.18

Since the American call option expires before any dividends are paid, it is equivalent to a European call option expiring in 6 months:

6.75EurC

We are asked to find the value of a put on a call option. We can use put-call parity to find the value of the put on a call:

-

-

- = -

- = -=

1

0.08(0.25)4.13 6.75 3.450.76

rtEurCallOnCall PutOnCall C xe

PutOnCall ePutOnCall

Solution 10.29

B Asian Options

From the information provided, we have:

500.060.30

10.5

20

Sr

Th

n

This allows us to solve for u and d:

d s

d s

- + - +

- - - -

= = =

= = =

( ) (0.06 0.00)(0.5) 0.3 0.5

( ) (0.06 0.00)(0.5) 0.3 0.5

1.2740

0.8335

r h h

r h h

u e e

d e e

We next solve for the risk-neutral probability of an upward movement:

d- -- -= = =- -

( ) (0.06 0.00)0.5 0.83350.4472

1.2740 0.8335

r he d ep

u d

The tree of stock prices below includes the payoff under each path:

Geometric Put

Avg. Strike Payoff Probability

81.15 71.90 0.00 0.2000

63.70

53.09 58.15 5.06 0.2472

50.00

53.09 47.04 0.00 0.2472

41.67

34.74 38.05 3.31 0.3056

Page 245: ActuarialBrew.com Exam MFE / 3F Actuarial Models …actuarialbrew.com/data/documents/MFE-Solutions-2014-1st-ed.pdf · Exam MFE / 3F Actuarial Models Financial Economics Segment Solutions

Exam MFE/3F Solutions Chapter 10 – Exotic Options: Part I

© ActuarialBrew.com 2014 Page 10.19

Each average is calculated as a geometric average. For example, the first geometric average strike is:

( )¥ =0.563.70 81.15 71.90

Each put payoff is determined using the geometric average strike. For example, the first put payoff is:

71.90 81.15,0 0Max

The geometric average strike is less than the final stock price for only 2 of the 4 possible paths. The present value of the option is:

0.06 10(0.2000) 5.06(0.2472) 0(0.2472) 3.31(0.3056) 2.13e

Solution 10.30

C Barrier Options

All of the options have the same strike price. We can use the parity relationship for barrier options to find the value of a regular option. Making use of the down-and-out and down-and-in options that have a barrier of $90, we have:

Knock-in option + Knock-out option = Ordinary option

+ =9.61 5.70 15.31

Therefore, the value of the ordinary option is $15.31. We can use the parity relationship again, this time with the up-and-in and up-and-out options that have a barrier of $125:

Knock-in option + Knock-out option = Ordinary option

+ ==

1.48 15.3113.83

XX

The value of the up-and-in option with a barrier of $125 is $13.83.

Solution 10.31

B Gap Options

For the gap option, we have:

==

1

2

100

110

K

K

The delta of the regular call option is:

d-D = =1( ) 0.1563TCall e N d

Page 246: ActuarialBrew.com Exam MFE / 3F Actuarial Models …actuarialbrew.com/data/documents/MFE-Solutions-2014-1st-ed.pdf · Exam MFE / 3F Actuarial Models Financial Economics Segment Solutions

Exam MFE/3F Solutions Chapter 10 – Exotic Options: Part I

© ActuarialBrew.com 2014 Page 10.20

For the regular European call option, we have:

d- -

-

-

= -

= -

=

1 2

2

2

1.32 ( ) ( )

1.32 85(0.1563) 110 ( )

( ) 0.108777

T rT

rT

rT

Se N d Ke N d

e N d

e N d

Since the regular call option and the gap call option have the same values for 1d and 2d ,

we can substitute the final line above into the equation for the value of the gap call option:

d- -= -= -=

1 1 2Gap call price ( ) ( )

85(0.1563) 100(0.108777)2.41

T rTSe N d K e N d

Solution 10.32

E Asian Options

Statement I is true. If there were only one sampling period, then an average strike option would be certain to pay zero. As the number of sampling periods increases, the strike price has a greater chance of being different from the final price of the stock, thereby increasing the value of the average strike Asian call.

Statement II is false. As the frequency of the sampling is increased, the average price exhibits less volatility, thereby decreasing the value of the average price Asian call.

Statement III is true. The arithmetic average is always greater than or equal to the geometric average, so the value of a geometric average price Asian call option is always less than the value of an otherwise equivalent arithmetic average price Asian call option.

Solution 10.33

C Compound Options

Since Stock Y does not pay dividends, an American call option on Stock Y has the same value as a European call option on Stock Y. Therefore, we can use the Black-Scholes formula to find the value of the call option at the end of 3 months, at which time the call option has 9 months remaining until it matures.

The first step is to calculate 1d and 2d at the end of time 3 months:

2 2

1

2 1

ln( / ) ( 0.5 ) ln(87 / 90) (0.07 0.00 0.5 0.30 ) 0.75

0.30 0.750.20149

0.20149 0.30 0.75 0.05832

S K r Td

T

d d T

d ss

s

+ - + + - + ¥ ¥= =

=

= - = - = -

We have:

1( ) (0.20149) 0.57984N d N

Page 247: ActuarialBrew.com Exam MFE / 3F Actuarial Models …actuarialbrew.com/data/documents/MFE-Solutions-2014-1st-ed.pdf · Exam MFE / 3F Actuarial Models Financial Economics Segment Solutions

Exam MFE/3F Solutions Chapter 10 – Exotic Options: Part I

© ActuarialBrew.com 2014 Page 10.21

2( ) ( 0.05832) 0.47675N d N

The value of the call option is:

1 20.00(0.75) 0.07(0.75)

( ) ( )

87 0.57984 90 0.47675 9.73311

T rTEurC Se N d Ke N d

e e

The payoff of the put on a call is:

[ ]1 1Compound put payoff ( , , ), 0

12 9.73311,0

2.2669

tMax x V S K T t

Max

È ˘= - -Î ˚= -=

Solution 10.34

A Compound Options

The strike price of the call on a put is:

1( ) 0.08(152 91) / 3651 7 90 1 5.80r T tx D K e e

Therefore, the decision to exercise the call on a put option is equivalent to deciding not to exercise the American call option:

¤1 1

Don't Exercise American Exercise call on

call option at time put at time t t

The American call option is not exercised at the end of 91 days if the payoff from early exercise of the American call option is less than the value of the unexercised option, which is a European call option that expires in 61 days:

Don’t exercise American call early if 1 1 1( , , )t Eur tS D K C S K T t

The table below summarizes the calculations and the resulting decisions regarding exercise of the options:

Ex-Dividend

Price: 91S 1tS D K

European Call Maturing in 61 days:

1 1( , , )Eur tC S K T t

Exercise American Call

early?

Exercise Call on

Put?

$86 + - =86 7 90 3 $3.02 No Yes

$87 + - =87 7 90 4 $3.46 Yes No

$88 + - =88 7 90 5 $3.93 Yes No

$89 + - =89 7 90 6 $4.45 Yes No

$90 + - =90 7 90 7 $5.00 Yes No

Page 248: ActuarialBrew.com Exam MFE / 3F Actuarial Models …actuarialbrew.com/data/documents/MFE-Solutions-2014-1st-ed.pdf · Exam MFE / 3F Actuarial Models Financial Economics Segment Solutions

Exam MFE/3F Solutions Chapter 10 – Exotic Options: Part I

© ActuarialBrew.com 2014 Page 10.22

The call on a put is exercised if the ex-dividend stock price is $86 in 91 days, but if the stock price is higher than $86, then the call on a put expires unexercised.

Although we have shown the full table above, it is not necessary to check all of the ex-dividend prices to answer this question. If it is optimal to exercise the American call option for some stock price, then it is also optimal to exercise the American call option at higher prices.

Alternate Solution:

Another approach is to use put-call parity to find the value of the put option at the end of 91 days. The call on the put is exercised only if the put option is worth more than $5.80.

Ex-Dividend Price: 91S

European Call Maturing in 61 days

European Put Maturing in 61 days

Exercise Call on Put?

$86 $3.02 -+ - =0.08(61 / 365)3.02 90 86 5.82e Yes

$87 $3.46 -+ - =0.08(61 / 365)3.46 90 87 5.26e No

$88 $3.93 -+ - =0.08(61 / 365)3.93 90 88 4.73e No

$89 $4.45 -+ - =0.08(61 / 365)4.45 90 89 4.25e No

$90 $5.00 -+ - =0.08(61 / 365)5.00 90 90 3.80e No

Although the full table is provided above, it is not necessary to calculate the value of the put option for every stock price. The value of the put option declines as the stock price increases, so since the put option’s value is less than $5.80 when the stock price is $87, we can conclude that the put option’s value is also less than $5.80 if the stock price is greater than $87.

Solution 10.35

C Path-Dependent Options

The arithmetic average price is:

+ + + + + + + + + + += =95 120 115 110 115 110 100 95 120 125 110 105

11012

S

The payoff of Option (i) is:

- = - =Max[ ,0] Max[100 110,0] 0K S

The barrier of 130 for Option (ii) is not reached, so the option is not knocked out. Therefore, the payoff of Option (ii) is:

- = - =[ ,0] [120 105,0] 15Max K S Max

Page 249: ActuarialBrew.com Exam MFE / 3F Actuarial Models …actuarialbrew.com/data/documents/MFE-Solutions-2014-1st-ed.pdf · Exam MFE / 3F Actuarial Models Financial Economics Segment Solutions

Exam MFE/3F Solutions Chapter 10 – Exotic Options: Part I

© ActuarialBrew.com 2014 Page 10.23

The barrier of 120 for Option (iii) is reached on February 28, 2009, so the option is knocked in. Therefore, the payoff of Option (iii) is:

- = - =Max[ ,0] Max[115 105,0] 10K S

The barrier of 125 for Option (iv) is reached on October 31, 2009, so the option is knocked out. Therefore, the payoff for Option (iv) is 0.

The highest payoff is 15 from Option (ii). The lowest payoff is 0 from Options (i) and (iv). The difference is:

- =15 0 15

Solution 10.36

C Gap Options

We can use put-call parity for gap options to answer this question:

d- -

- -

+ = +

¥ + = +¥ - =

=

10.08(1) 0.03(1)2.7 90 100

2.7 13.9641

8.2142

rT TGapCall K e Se GapPut

GapPut e e GapPut

GapPut GapPut

GapPut

Solution 10.37

C Barrier Options

As the barrier of an up-and-out call approaches infinity, the price of the up-and-out call approaches the price of a regular call.

Therefore, a regular call option with a strike price of $85 can be read from the far right cell of the first table:

(85) 12.6264C

The value of the corresponding up-and-in calls can be determined with the parity relationship for barrier options:

Up-and-in call + Up-and-out call = Ordinary call

For the $95 and $105 barriers, we have:

H Up-and-out Call Up-and-in Call

95 0.0239 12.6264 – 0.0239 = 12.6025

105 0.2072 12.6264 – 0.2072 = 12.4192

Let’s use (1)Max to denote the running maximum of the stock price from time 0 to time 1:

£ £

=0 1

(1) Max ( )t

Max S t

Page 250: ActuarialBrew.com Exam MFE / 3F Actuarial Models …actuarialbrew.com/data/documents/MFE-Solutions-2014-1st-ed.pdf · Exam MFE / 3F Actuarial Models Financial Economics Segment Solutions

Exam MFE/3F Solutions Chapter 10 – Exotic Options: Part I

© ActuarialBrew.com 2014 Page 10.24

The payoff to the special option depends on the running maximum of the stock price:

£ < fi = ¥ -£ fi = ¥ -

95 (1) 105 Payoff 3 [ (1) 85,0]105 (1) Payoff 2 [ (1) 85,0]

Max Max SMax Max S

If the running maximum is between $95 and $105, then the payoff is the same as the payoff of 3 calls. Over this range, we can replicate the payoff by purchasing 3 $95-barrier up-and-in calls. But if the running maximum reaches $105, then the payoff is the same as 2 calls. So we need to reduce the payoff by one call if the running maximum reaches $105. We can do this by selling a $105-barrier up-and-in call.

Therefore, the special option consists of 3 of the $95-barrier up-and-in calls and a short position in 1 of the $105-barrier up-and-in calls. The price of the special option is equal to the cost of establishing this position:

3 12.6025 1 12.4192 25.3883

Solution 10.38

C Barrier Options

Since the current price of the stock is $65, the first four knock-out puts are down-and-out puts. The remaining put (with H of ∞ and knock-out put price of 8.4825) is an up-and-out put.

When the barrier of a knock-out option is infinity, it is certain that the barrier will not be reached and the option will not be knocked out. Therefore, the price of a regular put option with a strike price of $60 can be read from the far right cell of the first table:

(60) 8.4825P

The value of the corresponding down-and-in puts can be determined with the parity relationship for barrier options:

Down-and-in put + Down-and-out put = Ordinary put

For the $30 and $40 barriers, we have:

H Down-and-out Put Down-and-in Put

30 4.1830 8.4825 – 4.1830 = 4.2995

40 1.1175 8.4825 – 1.1175 = 7.3650

Let’s use (1)Min to denote the running minimum of the stock price from time 0 to time 1:

£ £

=0 1

(1) Min ( )t

Min S t

The payoff to the special option depends on the running minimum of the stock price at the end of one year:

< £ fi = ¥ -

£ fi = ¥ -30 (1) 40 Payoff 5 [60 (1),0]

(1) 30 Payoff 3 [60 (1),0]Min Max S

Min Max S

Page 251: ActuarialBrew.com Exam MFE / 3F Actuarial Models …actuarialbrew.com/data/documents/MFE-Solutions-2014-1st-ed.pdf · Exam MFE / 3F Actuarial Models Financial Economics Segment Solutions

Exam MFE/3F Solutions Chapter 10 – Exotic Options: Part I

© ActuarialBrew.com 2014 Page 10.25

If the running minimum is between $30 and $40, then the payoff is the same as the payoff of 5 puts. Over this range, we can replicate the payoff by purchasing 5 $40-barrier down-and-in puts. But if the running minimum is less than or equal to $30, then the payoff is the same as 3 puts. So we need to reduce the payoff by two puts if the running minimum is less than $30. We can do this by selling two $30-barrier down-and-in puts.

Therefore, the special option consists of 5 of the $40-barrier down-and-in puts and a short position in 2 of the $30-barrier down-and-in puts. The price of the special option is equal to the cost of establishing this position:

5 7.3650 2 4.2995 28.2260

Solution 10.39

A American Call on Dividend-Paying Stock

The trick to this question is to recognize that the American call option is never called early, and therefore its value must be the same as that of a corresponding European call option.

At time 0.5, the American call is exercised only if the cum-dividend stock price minus the strike price is greater than the value of retaining the option as a European call option. That is, if the American call option is exercised at time 0.5, then the following condition must be satisfied:

1 1 1( , , )t Eur tS D K C S K T t

Let’s put in the values provided in the question and make use of put-call parity to rewrite the value of the European call option at time 0.5:

1

1

1

0.5

0.05 0.50.5 0.5

2 82 ( ,82,0.5)

2 82 82 ( ,82,0.5)

0.02459 ( ,82,0.5)

Eur t

Eur t

Eur t

S C S

S e S P S

P S

Since the minimum possible value of the put option at time 0.5 is zero, it is impossible for the inequality above to be satisfied at time 0.5. Therefore, the American call option is never called early, and its value is the same as that of a corresponding European call option.

Here’s another way to reach the conclusion that the American call option will not be exercised early. At time 0.5, the present value of the dividends is less than the present value of the interest cost of paying the strike price early:

r(T t )0.5 ,1

0.05(0.5 )

PV (div) K Ke

2 82 82e2 2.0246

- -

-

< -

< -<

Therefore, the following necessary but not sufficient condition for early exercise (which appears in Chapter 2 of the Study Manual) is not satisfied:

r(T t )t ,TPV (div) K Ke- -> -

Page 252: ActuarialBrew.com Exam MFE / 3F Actuarial Models …actuarialbrew.com/data/documents/MFE-Solutions-2014-1st-ed.pdf · Exam MFE / 3F Actuarial Models Financial Economics Segment Solutions

Exam MFE/3F Solutions Chapter 10 – Exotic Options: Part I

© ActuarialBrew.com 2014 Page 10.26

The prepaid forward price of the stock is:

( 0.05 0.5)0 0,0, ( ) ( ) 80 2 78.04938P

TTF S S PV Div e

The volatility of the prepaid forward price is equal to the volatility of the forward price:

( )

sÈ ˘Í ˙Î ˚= = = = =

,1 ,1ln ( ) [ln ] 0.0625

0.0625 0.25

Pt t

PF

Var F S Var F tt t t

The values of 1d and 2d are:

0, 2 20.05 10,

1

2 1

( ) 78.04938ln 0.5 ln 0.5 0.25 1( ) 82 0.127490.25 1

0.12749 0.25 1 0.12251

PT

PFPT

PF

PF

F ST

F K edT

d d T

s

s

s

- ¥

Ê ˆÊ ˆ+Á ˜ + ¥ ¥Á ˜ Á ˜Ë ¯ Ë ¯

= = =

= - = - = -

We have:

1

2

( ) (0.12749) 0.55072

( ) ( 0.12251) 0.45125

N d N

N d N

The value of the call option is:

0.050, 1 0, 2( ) ( ) ( ) ( ) 78.04938 0.55072 82 0.45125

7.7855

P PEur T TC F S N d F K N d e

Solution 10.40

B Barrier Options

The value of the corresponding up-and-in calls can be determined with the parity relationship for barrier options:

Up-and-in call + Up-and-out call = Ordinary call

Let’s use EurC to denote the price of an ordinary call. For the $95, $105, and $115

barriers, we have the following prices:

H Up-and-out Call Up-and-in Call

95 0.0239 - 0.0239EurC

105 0.2072 - 0.2072EurC

115 0.6650 - 0.6650EurC

Let’s use (1)Max to denote the running maximum of the stock price from time 0 to time 1:

£ £

=0 1

(1) Max ( )t

Max S t

Page 253: ActuarialBrew.com Exam MFE / 3F Actuarial Models …actuarialbrew.com/data/documents/MFE-Solutions-2014-1st-ed.pdf · Exam MFE / 3F Actuarial Models Financial Economics Segment Solutions

Exam MFE/3F Solutions Chapter 10 – Exotic Options: Part I

© ActuarialBrew.com 2014 Page 10.27

The payoff to the special option depends on the running maximum of the stock price:

£ < fi = ¥ -£ < fi = ¥ -£ fi =

95 (1) 105 Payoff 3 [ (1) 85,0]105 (1) 115 Payoff 2 [ (1) 85,0]115 (1) Payoff 0

Max Max SMax Max SMax

Let’s consider each possibility for the running maximum:

If the running maximum is between $95 and $105, then the payoff is the same as the payoff of 3 calls. Over this range, we can replicate the payoff by purchasing 3 $95-barrier up-and-in calls.

If the running maximum is between $105 and $115, then the payoff is the same as 2 calls. So we need to reduce the payoff obtained above by one call if the running maximum is between $105 and $115. We can do this by selling a $105-barrier up-and-in call.

If the running maximum is above $115, then the payoff is zero. So we need to reduce the payoff obtained above by 2 calls if the running maximum is above $115. We can do this by selling 2 $115-barrier up-and-in calls.

Therefore, the special option consists of 3 of the $95-barrier up-and-in calls, a short position in 1 of the $105-barrier up-and-in calls, and a short position in 2 of the $115-barrier up-and-in calls. The price of the special option is equal to the cost of establishing this position:

3 0.0239 1 0.2072 2 0.6650 1.4655Eur Eur EurC C C

Solution 10.41

A Asian Options

Newly issued average price Asian options are worth less than otherwise equivalent ordinary options because the average stock price exhibits less volatility than the final stock price. Therefore, we can rule out answer choices C, D, and E.

Making use of put-call parity, the ordinary call option is worth more than the ordinary put option:

d- -

-

- = -

- = -- = fi >

00.03(2)

100 100

5.82

T rTEur Eur

Eur Eur

Eur Eur Eur Eur

C P S e Ke

C P e

C P C P

Solution 10.42

C Put-Call Parity For Gap Options

Option B must have a higher price than Option A, because although they have the same trigger price, Option B has a higher strike price. Therefore Option A cannot have the highest price.

Page 254: ActuarialBrew.com Exam MFE / 3F Actuarial Models …actuarialbrew.com/data/documents/MFE-Solutions-2014-1st-ed.pdf · Exam MFE / 3F Actuarial Models Financial Economics Segment Solutions

Exam MFE/3F Solutions Chapter 10 – Exotic Options: Part I

© ActuarialBrew.com 2014 Page 10.28

Option C must have a higher price than Option B, because although they have the same strike price, if < <84 85TS , then Option C has a positive payoff while Option B has a zero payoff. Therefore, Option B cannot have the highest price.

Option D must have a higher price than Option E, because although they have the same trigger price, Option D has a lower strike price. Therefore, Option E cannot have the highest price.

The correct answer must be either Option C or Option D. We can use gap put-call parity to compare the two options:

1

10.04(1) 0.05(1)(Option D) (Option C) 85 86

(Option D) (Option C) 0.1386

rT T

T rT

GapCall K e Se GapPut

GapCall GapPut Se K e

e e

Since the difference above is negative, Option C must have a higher price than Option D.

Page 255: ActuarialBrew.com Exam MFE / 3F Actuarial Models …actuarialbrew.com/data/documents/MFE-Solutions-2014-1st-ed.pdf · Exam MFE / 3F Actuarial Models Financial Economics Segment Solutions

Exam MFE/3F Solutions Chapter 11 – Exotic Options: Part II

© ActuarialBrew.com 2014 Page 11.01

Chapter 11 – Solutions

Solution 11.01

E Exchange Options

The underlying asset is one share of Stock A. The strike asset is 4 shares of Stock B. Therefore, we have:

== ¥ =

424 10 40

SK

The volatility of ln( / )S K is:

s s s rs s= + - = + - = =2 2 2 22 0.3 0.5 2(0.4)(0.3)(0.5) 0.22 0.46904S KKS

The values of 1d and 2d are:

2 0.0(1) 2

0.04(1)

1

42 0.46904 (1)ln ln2 240

0.423820.46904(1)

S

K

T

TSe T e

Ke ed

T

s= - = - = -2 1 0.42382 0.46904 0.04522d d T

We have:

1( ) (0.42382) 0.66415N d N

2( ) ( 0.04522) 0.48197N d N

The value of the exchange call is:

1 20.04(1)

ExchangeCallPrice ( ) ( )

42(0.66415) 40 (0.48197) 9.3714

S KT TSe N d Ke N d

e

Solution 11.02

C Exchange Options

We don’t need to know the risk-free rate of return to answer this question. It was provided solely to make the question appear more difficult.

The underlying asset is one share of Stock R. The strike asset is 1 share of Stock Q. Therefore, we have:

==

7575

SK

Page 256: ActuarialBrew.com Exam MFE / 3F Actuarial Models …actuarialbrew.com/data/documents/MFE-Solutions-2014-1st-ed.pdf · Exam MFE / 3F Actuarial Models Financial Economics Segment Solutions

Exam MFE/3F Solutions Chapter 11 – Exotic Options: Part II

© ActuarialBrew.com 2014 Page 11.02

The volatility of ln( / )S K is:

s s s rs s= + - = + - = =2 2 2 22 0.3 0.25 2(1)(0.3)(0.25) 0.0025 0.05S KKS

The values of 1d and 2d are:

2 2

1

75 0.05 (1)ln ln2 75 20.02500

0.05(1)

S

K

T

TSe T

Ked

T

d

ds

s

-

-

Ê ˆÊ ˆ+Á ˜ +Á ˜Ë ¯ Ë ¯

= = =

2 1 0.02500 0.05(1) 0.02500d d Ts= - = - = -

We have:

1( ) (0.02500) 0.50997N d N

2( ) ( 0.02500) 0.49003N d N

The value of the exchange call is:

1 2ExchangeCallPrice ( ) ( )

75(0.50997) 75(0.49003) 1.4955

S KT TSe N d Ke N d

Solution 11.03

E Exchange Options

The underlying asset is one share of Stock D. The strike asset is 0.5 shares of Stock C. Therefore, we have:

400.5 70 35

SK

The volatility of ln( / )S K is:

s s s rs s= + - = + - - = =2 2 2 22 0.3 0.5 2( 0.4)(0.3)(0.5) 0.46 0.67823S KKS

The values of 1d and 2d are:

2 0.0(0.75) 2

0.03(0.75)

1

40 0.67823 (0.75)ln ln

2 2350.55933

0.67823 0.75

S

K

T

TSe T e

Ke ed

T

s= - = - = -2 1 0.55933 0.67823 0.75 0.02804d d T

We have:

1( ) (0.55933) 0.71203N d N

2( ) ( 0.02804) 0.48882N d N

Page 257: ActuarialBrew.com Exam MFE / 3F Actuarial Models …actuarialbrew.com/data/documents/MFE-Solutions-2014-1st-ed.pdf · Exam MFE / 3F Actuarial Models Financial Economics Segment Solutions

Exam MFE/3F Solutions Chapter 11 – Exotic Options: Part II

© ActuarialBrew.com 2014 Page 11.03

The value of the exchange call is:

1 20.03(0.75)

ExchangeCallPrice ( ) ( )

40(0.71203) 35 (0.48882) 11.7531

S KT TSe N d Ke N d

e

Solution 11.04

A Exchange Options

We are asked to find the price of an exchange call option having one share of Stock Y as the underlying asset and 1/3 share of Stock X as the strike asset. We can find this price using the standard formula:

S KT T1 2ExchangeCallPrice Se N(d ) Ke N(d )d d- -= -

But it is quicker to use put-call parity.

Let’s define the underlying asset as 1 share of Stock X and the strike asset as 3 shares of Stock Y.

The first option in the question is then an exchange call option. It gives its owner the right to acquire 1 share of Stock X for 3 shares of Stock Y.

Using put-call parity, we have:

, ,

0.04(0.75) 0.03(0.75)

( , , ) ( , , ) ( ) ( )

17.97 ( , , ) 100 3(30)

( , , ) 8.923

P Pt t t t t T t T

t t

t t

C S K T t P S K T t F S F K

P S K T t e e

P S K T t

The value of an option giving its owner the right to give up 1 share of Stock X for 3 shares of Stock Y is therefore $8.923.

Since the option to exchange 1 share of Stock X for 3 shares of Stock Y has a value of $8.923, the value of an option allowing its owner to exchange 1/3 shares of Stock X for 1 share of Stock Y is:

=8.9232.97

3

Solution 11.05

E Exchange Options

Let’s define Stock E as the strike asset and Stock F as the underlying asset.

The first option described in the question gives its owner the right to acquire stock F in exchange for giving up Stock E, so this exchange option is a call option.

We are asked to find the value of an exchange option giving its owner the right to give up Stock F in exchange for Stock E. Since we defined Stock F as the underlying asset, this exchange option is a put option.

Page 258: ActuarialBrew.com Exam MFE / 3F Actuarial Models …actuarialbrew.com/data/documents/MFE-Solutions-2014-1st-ed.pdf · Exam MFE / 3F Actuarial Models Financial Economics Segment Solutions

Exam MFE/3F Solutions Chapter 11 – Exotic Options: Part II

© ActuarialBrew.com 2014 Page 11.04

Since both the call and put options have the same strike asset and same underlying asset, we can use put-call parity to find the value of the put option:

-

- - - = -

- - = -- =

, ,

0.07(1)

( , , ) ( , , ) ( ) ( )

2.16 ( , , ) 50 55

( , , ) 10.54

P Pt t t t t T t T

t t

t t

C S K T t P S K T t F S F K

P S K T t e

P S K T t

Solution 11.06

B Exchange Options

Let’s define the underlying asset to be Stock Y and the strike asset to be 4 shares of Stock X. This makes the option an exchange call option with:

== ¥ =

424 10 40

SK

The volatility of ln( / )S K is:

[ ] [ ] [ ]s = + - = + -

= =

2 21 1 1 1ln( ) ln( ) 2 ln( ),ln( ) 0.4 0.3 2(0.06)

0.13 0.36056

Var S Var K Cov S K

The values of 1d and 2d are:

2 0.02(1) 2

0.0(1)

1

42 0.36056 (1)ln ln2 240

0.260130.36056 1

S

K

T

TSe T e

Ke ed

T

s= - = - = -2 1 0.26013 0.36056 1 0.10043d d T

We have:

1( ) (0.26013) 0.60262N d N= =

2( ) ( 0.10043) 0.46000N d N= - =

The value of the exchange call is:

1 20.02(1)

ExchangeCallPrice ( ) ( )

42 (0.60262) 40(0.46000) 6.4089

S KT TSe N d Ke N d

e

Solution 11.07

C Exchange Options

Let’s define the underlying asset to be 1 euro and the strike asset to be 1.4 Canadian dollars. This makes the option an exchange call option with:

== ¥ =1.251.4 0.9 1.26

SK

Page 259: ActuarialBrew.com Exam MFE / 3F Actuarial Models …actuarialbrew.com/data/documents/MFE-Solutions-2014-1st-ed.pdf · Exam MFE / 3F Actuarial Models Financial Economics Segment Solutions

Exam MFE/3F Solutions Chapter 11 – Exotic Options: Part II

© ActuarialBrew.com 2014 Page 11.05

The volatility of ln( / )S K is:

s s s rs s= + - = + - = =2 2 2 22 0.2 0.3 2(0.605)(0.2)(0.3) 0.0574 0.23958S KKS

The values of 1d and 2d are:

d

ds

s

- -

- -

Ê ˆ Ê ˆ+ +Á ˜ Á ˜

Ë ¯ Ë ¯= = =

2 0.06(1) 2

0.08(1)

1

1.25 0.23958 (1)ln ln

2 21.260.17001

0.23958 1

S

K

T

TSe T e

Ke ed

T

s= - = - = -2 1 0.17001 0.23958 1 0.06957d d T

We have:

1( ) (0.17001) 0.56750N d N= =

2( ) ( 0.06957) 0.47227N d N= - =

The value of the exchange call is:

1 20.06(1) 0.08(1)

ExchangeCallPrice ( ) ( )

1.25 (0.56750) 1.26 (0.47227) 0.11875

S KT TSe N d Ke N d

e e

Solution 11.08

D Exchange Options

For questions like this, it is a good idea to scan all the answer choices first, and then begin where you are most confident either choosing one of the answer choices as the correct answer or eliminating answer choices until the correct answer choice remains.

Answer choice C is the option to give up 1 share of Stock X in exchange for 0.5 shares of Stock Y. Answer choice D is the option to give up 2 shares of Stock X for 1 Share of Stock Y. Therefore, D is essentially 2 of C. So, the value of D is twice that of C, and the answer cannot be C.

Answer choice B is the option to give up 0.5 shares of Stock Y in exchange for 1 share of Stock X. Answer choice E is the option to give up 1 share of Stock Y in exchange for 2 shares of Stock X. Therefore, E is essentially 2 of B. So, the value of E is twice that of B, and the answer cannot be B.

At this point, the possible answers are A, D, and E. Let’s compare A with E. The underlying asset is 1 share of Stock Y, and the strike asset is 2 shares of Stock X:

== ¥ =

802 40 80

SK

Page 260: ActuarialBrew.com Exam MFE / 3F Actuarial Models …actuarialbrew.com/data/documents/MFE-Solutions-2014-1st-ed.pdf · Exam MFE / 3F Actuarial Models Financial Economics Segment Solutions

Exam MFE/3F Solutions Chapter 11 – Exotic Options: Part II

© ActuarialBrew.com 2014 Page 11.06

From put call parity, we have:

-

- - - = -

- - - = -- - - =

, ,

0.04

( , , ) ( , , ) ( ) ( )

( , , ) ( , , ) 80 80

( , , ) ( , , ) 3.137

P Pt t t t t T t T

t t t t

t t t t

C S K T t P S K T t F S F K

C S K T t P S K T t e

C S K T t P S K T t

So the value of the call option is greater than the value of the put option by $3.137. That means that the value of D is $3.137 greater than E, so E cannot be the correct answer.

Since the value of the put option cannot be less than zero, the value of D must be at least $3.137. Therefore the value of D is more than $3.10, and A cannot be the correct answer.

By the process of elimination, the value of D is highest.

Solution 11.09

D Exchange Options

Let’s define 0.5 shares of Stock N as the strike asset and 1 share of Stock M as the underlying asset:

== ¥ =

350.5 80 40

SK

The first option described in the question gives its owner the right to acquire Stock M in exchange for giving up 0.5 shares of Stock N, so this exchange option is a call option. The value of this call option is:

= =(35,40,0.5) 0.0353(35) 1.2355C

The value of an exchange put option giving its owner the right to give up 1 share of Stock M in exchange for 0.5 shares of Stock N can be found using put-call parity:

- -

- - - = -

- = -=

, ,

0.02(0.5) 0.04(0.5)

( , , ) ( , , ) ( ) ( )

1.2355 (35,40,0.5) 35 40(35,40,0.5) 5.7917

P Pt t t t t T t TC S K T t P S K T t F S F K

P e eP

Two of these exchange put options give their owner the right to give up 2 shares of Stock M in exchange for 1 share of Stock N, so the solution is:

¥ =2 5.7917 11.5834

Solution 11.10

A Barrier Options

Deferred rebate options are barrier options, and barrier options are path-dependent since their payoff depends on whether the barrier is reached.

Page 261: ActuarialBrew.com Exam MFE / 3F Actuarial Models …actuarialbrew.com/data/documents/MFE-Solutions-2014-1st-ed.pdf · Exam MFE / 3F Actuarial Models Financial Economics Segment Solutions

Exam MFE/3F Solutions Chapter 11 – Exotic Options: Part II

© ActuarialBrew.com 2014 Page 11.07

Solution 11.11

D Gap Options

A gap option must be exercised if the trigger condition is met. This can result in negative payoffs. As negative payoffs become more likely, the premium falls, and it can even fall below zero.

Solution 11.12

D Chooser Options

The formula for the value of the chooser option is:

( ) ( )d d- - - - -= + 1 1( ) ( )( )0 0 1Price of Chooser Option , , , ,T t r T t

Eur EurC S K T e P S Ke t

Since r and d are both equal to zero, this formula can be simplified and used to find the value of a 2-year European put option:

= +

=42 30 (100,90,2)

(100,90,2) 12Eur

Eur

P

P

We can now use put-call parity to find the value of the 2-year European call option:

d- -

- ¥ - ¥

+ = +

+ = +=

2 20

2 0 2 0

(100,90,2) (100,90,2)

(100,90,2) 90 100 12

(100,90,2) 22

rEur Eur

Eur

Eur

C Ke S e P

C e e

C

Solution 11.13

D Chooser Options

The formula for the value of the chooser option is:

( ) ( )d d- - - - -= + 1 1( ) ( )( )0 0 1Price of Chooser Option , , , ,T t r T t

Eur EurC S K T e P S Ke t

In the question, we are given the first term to the right of the equal sign:

( ) = =0 , , (50,55,0.75) 4.00EurC S K T C

The strike price of the put option in the second term is:

d- - - - -= =1( )( ) (0.10 0.04)(0.25)55 54.1812r T tKe e

We must use the Black-Scholes formula to find the price of the 6-month put option. The first step is to calculate 1d and 2d :

2 2

1

2 1

ln( / ) ( 0.5 ) ln(50 / 54.1812) (0.10 0.04 0.5 0.30 ) 0.5

0.30 0.50.13110

0.13110 0.30 0.5 0.34323

S K r Td

T

d d T

d ss

s

+ - + + - + ¥ ¥= =

= -

= - = - - = -

Page 262: ActuarialBrew.com Exam MFE / 3F Actuarial Models …actuarialbrew.com/data/documents/MFE-Solutions-2014-1st-ed.pdf · Exam MFE / 3F Actuarial Models Financial Economics Segment Solutions

Exam MFE/3F Solutions Chapter 11 – Exotic Options: Part II

© ActuarialBrew.com 2014 Page 11.08

We have:

1( ) (0.13110) 0.55215N d N

2( ) (0.34323) 0.63429N d N

The value of the European put option is:

2 10.10(0.5) 0.04(0.5)

54.1812 ( ) 50 ( )

54.1812 0.63429 50 0.552155.62965

rT TEurP e N d e N d

e e

We can now find the price of the chooser option:

( ) ( )1 1( ) ( )( )0 0 1

0.04(0.75 0.50)

Price of Chooser Option , , , ,

4.00 5.629659.5736

T t r T tEur EurC S K T e P S Ke t

e

d d- - - - -

- -

= +

= +=

Solution 11.14

A Forward Start Option

In one year, the value of the call option will be:

d- -

- -

= -

= -1 1 1 2

0.02 0.101 1 1 2

( ) ( ) ( )

( ) ( )

T rTEurC S S e N d Ke N d

S e N d S e N d

In one year, the values of 1d and 2d will be:

d ss

s

+ - + ¥+ - += =

=

= - = - =

221 1

1

2 1

ln( / ) (0.10 0.02 0.5 0.30 )(1)ln( / ) ( 0.5 )

0.30 10.41667

0.41667 0.30 1 0.11667

S SS K r Td

T

d d T

From the normal distribution table:

1

2

( ) (0.41667) 0.66154

( ) (0.11667) 0.54644

N d N

N d N

In one year, the value of the call option can be expressed in terms of the stock price at that time:

0.02 0.101 1 1 1 2

0.02 0.101 1

1

( ) ( ) ( )

0.66154 0.54644

0.154001

EurC S S e N d S e N d

S e S e

S

- -

- -

= -

= ¥ - ¥= ¥

In one year, the call option will be worth 0.154001 shares of stock.

Page 263: ActuarialBrew.com Exam MFE / 3F Actuarial Models …actuarialbrew.com/data/documents/MFE-Solutions-2014-1st-ed.pdf · Exam MFE / 3F Actuarial Models Financial Economics Segment Solutions

Exam MFE/3F Solutions Chapter 11 – Exotic Options: Part II

© ActuarialBrew.com 2014 Page 11.09

The prepaid forward price of one share of stock is:

-

-

-

=

=

= ¥

=

0,0,

0.100,1 0,1

0.100,1

0,1

( ) ( )

( ) ( )

( ) 50

( ) 45.2419

P rTTT

P

P

P

F S e F S

F S e F S

F S e

F S

The value today of 0.1540 shares of stock in one year is:

0.154001 45.2419 6.9673¥ =

Solution 11.15

D Forward Start Option

In one year, the value of the call option will be:

d- -

- -

= -

= -1 1 1 2

0.02 0.101 1 1 2

( ) ( ) ( )

( ) ( )

T rTEurC S S e N d Ke N d

S e N d S e N d

In one year, the values of 1d and 2d will be:

d ss

s

+ - + ¥+ - += =

=

= - = - =

221 1

1

2 1

ln( / ) (0.10 0.02 0.5 0.30 )(1)ln( / ) ( 0.5 )

0.30 10.41667

0.41667 0.30 1 0.11667

S SS K r Td

T

d d T

From the normal distribution table:

1

2

( ) (0.41667) 0.66154

( ) (0.11667) 0.54644

N d N

N d N

In one year, the value of the call option can be expressed in terms of the stock price at that time:

0.02 0.101 1 1 1 2

0.02 0.101 1

1

( ) ( ) ( )

0.66154 0.54644

0.154001

EurC S S e N d S e N d

S e S e

S

- -

- -

= -

= ¥ - ¥= ¥

In one year, the call option will be worth 0.154001 shares of stock. One share of stock in 1

year is equivalent to d-e shares of stock now. Therefore, 0.154001 shares of stock in 1 year is equivalent to the following quantity of shares now:

0.020.154001 0.154001 0.150952e ed- -= =

Page 264: ActuarialBrew.com Exam MFE / 3F Actuarial Models …actuarialbrew.com/data/documents/MFE-Solutions-2014-1st-ed.pdf · Exam MFE / 3F Actuarial Models Financial Economics Segment Solutions

Exam MFE/3F Solutions Chapter 11 – Exotic Options: Part II

© ActuarialBrew.com 2014 Page 11.10

The delta of 1 share of stock is 1.0, so the delta of 0.150952 shares of stock is:

0.150952 1.0 0.150952¥ =

Solution 11.16

A Forward Start Option

In one year, the value of the put option will be:

d- -

- -

= - - -

= - - -1 2 1

0.10 0.021 2 1 1

( ) ( ) ( )

( ) ( )

rT TEurP S Ke N d Se N d

S e N d S e N d

In one year, the values of 1d and 2d will be:

d ss

s

+ - + ¥+ - += =

=

= - = - =

221 1

1

2 1

ln( / ) (0.10 0.02 0.5 0.30 )(1)ln( / ) ( 0.5 )

0.30 10.41667

0.41667 0.30 1 0.11667

S SS K r Td

T

d d T

From the normal distribution table:

1

2

( ) ( 0.41667) 0.33846

( ) ( 0.11667) 0.45356

N d N

N d N

In one year, the value of the put option can be expressed in terms of the stock price at that time:

0.10 0.021 1 2 1 1

0.10 0.021 1

1

( ) ( ) ( )

0.45356 0.33846

0.078640

EurP S S e N d S e N d

S e S e

S

- -

- -

= - - -

= ¥ - ¥=

In one year, the put option will be worth 0.078640 shares of stock.

The prepaid forward price of one share of stock is:

-

-

-

=

=

= ¥

=

0,0,

0.100,1 0,1

0.100,1

0,1

( ) ( )

( ) ( )

( ) 50

( ) 45.2419

P rTTT

P

P

P

F S e F S

F S e F S

F S e

F S

The value today of 0.078640 shares of stock in one year is:

0.078640 45.2419 3.55782¥ =

Page 265: ActuarialBrew.com Exam MFE / 3F Actuarial Models …actuarialbrew.com/data/documents/MFE-Solutions-2014-1st-ed.pdf · Exam MFE / 3F Actuarial Models Financial Economics Segment Solutions

Exam MFE/3F Solutions Chapter 11 – Exotic Options: Part II

© ActuarialBrew.com 2014 Page 11.11

Solution 11.17

A Forward Start Option

At time 1, the value of the option is:

d- -

- -

= -

È ˘= -Î ˚

0.5 0.51 1 1 1 1 2

0.5(0.03) 0.5(0.11)1 1 2

( ,1.1 ,0.5) ( ) 1.10 ( )

( ) 1.10 ( )

rEurC S S S e N d S e N d

S e N d e N d

where:

212

11

2 1

ln (0.11 0.03 0.5 )(0.5)ln 1.10 (0.08 0.5 )(0.5)1.10

0.5 0.5

0.5

SS

d

d d

We would need to know the value of s to determine the values of 1( )N d and 2( )N d , but notice that these values are independent of 1S . Therefore, the ratio of the value of the call option at time 1 to the value of the underlying stock at time 1 is constant across all possible values of 1S :

- -= -0.5(0.03) 0.5(0.11)1 11 2

1

( ,1.1 ,0.5)( ) 1.10 ( )EurC S S

e N d e N dS

Let’s consider one particular possible value of 1S . If the stock price at time 1 is $150, then the strike price at time 1 will be ¥ =110% $150 $165 .

We can use put-call parity to find the value of a 6-month European call option with a strike price of $165 when the stock price is $150.

- ¥ - ¥

- - - = -

- = -=

, ,

0.03 0.5 0.11 0.5

( , , ) ( , , ) ( ) ( )

(150,165,0.5) 17.47 150 165(150,165,0.5) 9.0667

P Pt t t t t T t TC S K T t P S K T t F S F K

C e eC

We can now determine the ratio of the call price to the stock price at time 1:

- -

- -

- -

= -

= -

= -

0.5(0.03) 0.5(0.11)1 2

0.5(0.03) 0.5(0.11)1 2

0.5(0.03) 0.5(0.11)1 2

(150,165,0.5)( ) 1.10 ( )

1509.0667

( ) 1.10 ( )150

0.06044 ( ) 1.10 ( )

EurCe N d e N d

e N d e N d

e N d e N d

Therefore 0.06044 shares of stock at time 1 will have the same value as the call option at time 1. The prepaid forward price of 0.06044 shares delivered at time 1 is therefore equal to the value of the forward start option:

d- -= ¥ =0.0300.06044 0.06044 100 5.866e S e

Page 266: ActuarialBrew.com Exam MFE / 3F Actuarial Models …actuarialbrew.com/data/documents/MFE-Solutions-2014-1st-ed.pdf · Exam MFE / 3F Actuarial Models Financial Economics Segment Solutions

Exam MFE/3F Solutions Chapter 11 – Exotic Options: Part II

© ActuarialBrew.com 2014 Page 11.12

Solution 11.18

E Forward Start Option

The strategies described in this question are introduced in Problem 14.22 at the end of Chapter 14 of the second edition of the Derivatives Markets textbook.

The cost of Ron’s strategy is the cost of a 1-year European put option with a strike price of $95. The values of 1d and 2d are:

2 2

1

2 1

ln( / ) ( 0.5 ) ln(100 / 95) [0.10 0 0.5(0.25) ] 1

0.25 10.73017

0.73017 0.25 1 0.48017

S K r Td

T

d d T

d ss

s

+ - + + - + ¥= =

=

= - = - =

From the normal distribution table:

1

2

( ) ( 0.73017) 0.23264

( ) ( 0.48017) 0.31555

N d N

N d N

The cost of the 1-year European put option is:

0.10(100,95,1) 95 (0.31555) 100(0.23264)

3.86054EurP e-= -

=

Now let’s consider the cost of Wally’s strategy. We must compute new values of 1d and

2d for the 1-month option:

2 2

1

2 1

ln( / 0.95 ) ( 0.5 ) ln(0.95) (0.10 0.00 0.5 0.25 )(1 /12)

10.25

120.86230

10.86230 0.25 0.79013

12

S S r Td

T

d d T

d ss

s

+ - + - + - + ¥= =

=

= - = - =

From the normal distribution table:

1

2

( ) ( 0.86230) 0.19426

( ) ( 0.79013) 0.21473

N d N

N d N

The cost of a 1-month put option that has a strike price of 95% of the current stock price is a function of the then-current stock price:

2 10.10 /12 0.00 /12

2 10.10 /12

( ,0.95 ,1 /12) 0.95 ( ) ( )

0.95 ( ) ( )

0.95 (0.21473) 0.19426

0.00804

rT TEurP S S Se N d Se N d

Se N d Se N d

S e

S

d- -

- -

-

= - - -

= - - -

È ˘= -Î ˚=

Page 267: ActuarialBrew.com Exam MFE / 3F Actuarial Models …actuarialbrew.com/data/documents/MFE-Solutions-2014-1st-ed.pdf · Exam MFE / 3F Actuarial Models Financial Economics Segment Solutions

Exam MFE/3F Solutions Chapter 11 – Exotic Options: Part II

© ActuarialBrew.com 2014 Page 11.13

Therefore, each of the 1-month put options can be purchased with 0.00804 shares of stock. The cost of purchasing all 12 of the options now is equal to the cost of acquiring 12 0.00804 0.096487¥ = shares of stock. Since the current stock price is $100, Wally’s cost is:

0.096487 100 9.6487¥ =

Wally’s cost therefore exceeds Ron’s cost by:

9.6487 3.8605 5.7882- =

Solution 11.19

C Chooser Options

The price of the chooser option can be expressed in terms of a call option and put option:

( ) ( )d d- - - - -= + 1 1( ) ( )( )0 0 1Price of Chooser Option , , , ,T t r T t

Eur EurC S K T e P S Ke t

The risk-free interest rate and the dividend yield are equal, so the put option has the same strike price as the call option:

( ) ( )d d- - - - -

-

= +

= +

1 1( ) ( )( )0 0 1

0.05(2)

Price of Chooser Option , , , ,

Price of Chooser Option (95,100,3) (95,100,1)

T t r T tEur Eur

Eur Eur

C S K T e P S Ke t

C e P

We can use put-call parity to find the value of the 1-year put option:

0.05(1) 0.05(1)0

0.05(1) 0.05(1)

(95,100,1) (95,100,1)

9.20 100 95 (95,100,1)

(95,100,1) 13.96

Eur Eur

Eur

Eur

C Ke e S P

e e P

P

We can now solve for the price of the 3-year call option:

0.05(2)

0.05(2)

Price of Chooser Option (95,100,3) (95,100,1)

28.32 (95,100,3) 13.96

(95,100,3) 15.69

Eur Eur

Eur

Eur

C e P

C e

C

Solution 11.20

E Chooser Options and Delta

The price of the chooser option can be expressed in terms of a call option and put option:

( ) ( )( ) ( )( )

d d- - - - -

- - - - -

= +

= +

= +

1 1( ) ( )( )0 0 1

0.08(4 2) (0.05 0.08)(4 2)

Price of Chooser Option , , , ,

Price of Chooser Option 50,45,4 50,45 ,2

Price of Chooser Option 50,45,4 0.8521 50,47.7826

T t r T tEur Eur

Eur Eur

Eur Eur

C S K T e P S Ke t

C e P e

C P ( ),2

Page 268: ActuarialBrew.com Exam MFE / 3F Actuarial Models …actuarialbrew.com/data/documents/MFE-Solutions-2014-1st-ed.pdf · Exam MFE / 3F Actuarial Models Financial Economics Segment Solutions

Exam MFE/3F Solutions Chapter 11 – Exotic Options: Part II

© ActuarialBrew.com 2014 Page 11.14

From the equation above, we observe that owning a chooser option is equivalent to owning the 4-year call option and 0.8521 of the 2-year put options. To find the delta of the call option, we must calculate 1d with a 4-year maturity and = 45K :

2 2

1

1(0.08)4

1

ln( / ) ( 0.5 ) ln(50 / 45) (0.05 0.08 0.5(0.3) )40.27560

0.3 4( ) (0.27560) 0.60857

( ) 0.60857 0.44191TCall

S K r Td

TN d N

e N d ed

d ss

- -

+ - + + - += = =

= =

D = = ¥ =

To find the delta of the put option, we must calculate 1d with a 2-year maturity and = 47.7826K :

2 2

1

1(0.08)2

1

ln( / ) ( 0.5 ) ln(50 / 47.7826) (0.05 0.08 0.5(0.3) )20.17763

0.3 2( ) ( 0.17763) 0.42951

( ) 0.42951 0.36600TPut

S K r Td

TN d N

e N d ed

d ss

- -

+ - + + - += = =

- = - =

D = - - = - ¥ = -

Now we can calculate the delta of a portfolio consisting of one of the call options and 0.8521 of the put options. This is the delta of the chooser option:

1 0.8521 0.44191 0.8521 ( 0.36600) 0.13002Chooser Call PutD = ¥ D + D = + ¥ - =

Since we wrote 100 of the chooser options, the number of shares of stock that we must purchase to delta-hedge is:

100 0.13002 13.00¥ =

Solution 11.21

D Chooser Options

A formula for the value of the chooser option is:

( ) ( )d d- - - - -= + 1 1( ) ( )( )0 0 1Price of Chooser Option , , , ,T t r T t

Eur EurP S K T e C S Ke t

Note that:

d- - - - - -= =1( )( ) (0.08 0.02)(1 0.75)90 88.66r T tKe e

The price of the chooser option is:

( ) ( )- -

-

= +

= + =

0.02(1 0.75)

0.02(0.25)

Price of Chooser Option 85,90,1 85,88.66,0.75

9.80 8.76 18.5163

Eur EurP e C

e

Page 269: ActuarialBrew.com Exam MFE / 3F Actuarial Models …actuarialbrew.com/data/documents/MFE-Solutions-2014-1st-ed.pdf · Exam MFE / 3F Actuarial Models Financial Economics Segment Solutions

Exam MFE/3F Solutions Chapter 11 – Exotic Options: Part II

© ActuarialBrew.com 2014 Page 11.15

Solution 11.22

B Exchange Options

Let’s treat this option as a call option. The underlying asset is one share of Stock A. The

strike asset is ab

shares of Stock B. Therefore, we have:

aa b ab

=

= ¥ =

S

K

The volatility of ln( / )S K is:

s s s rs s= + - = + - = =2 2 2 22 0.3 0.5 2(0.4)(0.3)(0.5) 0.22 0.46904S KKS

The values of 1d and 2d are:

2 2

1

0.46904 (1)ln ln2 20.23452

0.46904(1)

S

K

T

TSe T

Ked

T

2 1 0.23452 0.46904 0.23452d d Ts= - = - = -

We have:

1( ) (0.23452) 0.59271N d N

2( ) ( 0.23452) 0.40729N d N

The value of the exchange call is:

1 2ExchangeCallPrice ( ) ( )

(0.59271) (0.40729)

0.18542

S KT TSe N d Ke N d

Solution 11.23

D Forward Start Options

The values of 1d and 2d for a 6-month put option with a strike price that is 85% of the

current stock price are:

2 2

1

2 1

ln( / 0.85 ) ( 0.5 ) ln(0.85) (0.06 0.00 0.5 0.4 )(0.5)

0.4 0.50.82208

0.82208 0.4 0.5 0.53924

S S r Td

T

d d T

Page 270: ActuarialBrew.com Exam MFE / 3F Actuarial Models …actuarialbrew.com/data/documents/MFE-Solutions-2014-1st-ed.pdf · Exam MFE / 3F Actuarial Models Financial Economics Segment Solutions

Exam MFE/3F Solutions Chapter 11 – Exotic Options: Part II

© ActuarialBrew.com 2014 Page 11.16

From the normal distribution table:

1

2

( ) ( 0.82208) 0.20552

( ) ( 0.53924) 0.29486

N d N

N d N

The cost of a 6-month put option that has a strike price of 85% of the current stock price is a function of the then-current stock price:

2 10.5 0.06 0.00

2 1

0.03

( ,0.85 ,0.5) 0.85 ( ) ( )

0.85 ( ) ( )

0.85 (0.29486) 0.20552

0.03770

rT TEurP S S Se N d Se N d

Se N d Se N d

S e

S

Therefore, each of the 6-month put options can be purchased with 0.03770 shares of stock. The cost of purchasing both of the options now is equal to the cost of acquiring 2 0.03770 0.07541 shares of stock. Since the current stock price is $75, this cost is:

0.07541 75 5.65556

Solution 11.24

C Forward Start Options

The values of 1d and 2d for a 6-month put option with a strike price that is 85% of the

current stock price are:

2 2

1

2 1

ln( / 0.85 ) ( 0.5 ) ln(0.85) (0.06 0.05 0.5 0.4 )(0.5)

0.4 0.50.73369

0.73369 0.4 0.5 0.45085

S S r Td

T

d d T

d ss

s

+ - + - + - + ¥= =

=

= - = - =

From the normal distribution table:

1

2

( ) ( 0.73369) 0.23157

( ) ( 0.45085) 0.32605

N d N

N d N

The cost of a 6-month put option that has a strike price of 85% of the current stock price is a function of the then-current stock price:

2 10.5 0.06 0.5 0.05

2 1

0.03 0.025

( ,0.85 ,0.5) 0.85 ( ) ( )

0.85 ( ) ( )

0.85 (0.32605) (0.23157)

0.043099

rT TEurP S S Se N d Se N d

Se N d Se N d

S e e

S

Page 271: ActuarialBrew.com Exam MFE / 3F Actuarial Models …actuarialbrew.com/data/documents/MFE-Solutions-2014-1st-ed.pdf · Exam MFE / 3F Actuarial Models Financial Economics Segment Solutions

Exam MFE/3F Solutions Chapter 11 – Exotic Options: Part II

© ActuarialBrew.com 2014 Page 11.17

Therefore, the first put option can be purchased with 0.043099 shares of stock now, and the value of this purchase price is:

0.043099 0.043099 75 3.23244S

The second option can be purchased with 0.043099 shares of stock in 6 months. The current value of a share of stock 6 months from now is its prepaid forward price, so the time 0 value of this purchase price is:

0.5 0.050,0.50.043099 ( ) 0.043099 75 3.15263PF S e- ¥= ¥ =

The broker is paid the sum of the two prices:

3.23244 3.15263 6.38507+ =

Solution 11.25

A Exchange Options

Consider an asset, which we will call Asset X, which has a payoff of:

1 1 2[ (1),2 (1)]X Min S S

From statement (vi), we know that the price of a European call option on Asset X, with a strike price of 22, is 0.237. The payoff of this call option is:

1 2 1(1),2 (1) 22,0 22,0Max Min S S Max X

We are asked to find the price of an option that has the payoff described below:

1 2 122 (1),2 (1) ,0 22 ,0Max Min S S Max X

We recognize this as a European put option on Asset X. From put-call parity, we have:

( )

0.06 100.237 22

r T tEur t Eur

Eur

C Ke S P

e X P

If we can find the current value of Asset X, then we can solve for the value of the put option. Asset X can be replicated by purchasing 1 share of Stock 1, and selling an exchange call option that allows its owner to exchange 2 shares of Stock 2 for 1 share of Stock 1:

1 1 2 1 1 2[ (1),2 (1)] (1) [ (1) 2 (1),0]X Min S S S Max S S

The current value of Asset X is the value of Stock 1 minus the value of the exchange call:

0 0,1 1 2 0,1 1 1 2

1

[ (1),2 (1)] (0) (0),2 (0),1

(0) ExchangeCallPrice

P PEurX F Min S S F S C S S

S

Page 272: ActuarialBrew.com Exam MFE / 3F Actuarial Models …actuarialbrew.com/data/documents/MFE-Solutions-2014-1st-ed.pdf · Exam MFE / 3F Actuarial Models Financial Economics Segment Solutions

Exam MFE/3F Solutions Chapter 11 – Exotic Options: Part II

© ActuarialBrew.com 2014 Page 11.18

The exchange call option is an exchange call option with 1 share of Stock 1 as the underlying asset and 2 shares of Stock 2 as the strike asset. To find the value of this exchange call option, we find the appropriate volatility parameter:

s s s rs s= + - = + - - =2 2 2 22 0.15 0.20 2( 0.30)(0.15)(0.20) 0.28373S KKS

The values of 1d and 2d are:

d

ds

ss

- - ¥

- - ¥

Ê ˆ Ê ˆ ¥+ +Á ˜ Á ˜¥Ë ¯ Ë ¯

= = = -

= - = - - = -

2 0 1 2

0 1

1

2 1

20 0.28373 1ln ln

2 2(2 11)0.19406

0.28373 1

0.19406 0.28373 1 0.47779

S

K

T

TSe T e

Ke ed

T

d d T

From the normal table, we have:

1

2

( ) ( 0.19406) 0.42306

( ) ( 0.47779) 0.31640

N d N

N d N

= - == - =

The value of the exchange call option is:

1 20 1 0 1

ExchangeCallPrice ( ) ( )

20 (0.42306) 2 11 (0.31640)

1.5004

S KT TSe N d Ke N d

e e

The current value of Asset X is:

0 1(0) ExchangeCallPrice 20 1.5004 18.4996X S

We can now use put-call parity to find the value of the put option:

0.06 10

0.06

0.237 22

0.237 22 18.4996

2.4562

Eur

Eur

Eur

e X P

e P

P

- ¥

-

+ = +

+ = +=

Solution 11.26

D Exchange Options

Consider an asset, which we will call Asset X, which has a payoff of:

1 1 2[ (1),2 (1)]X Max S S

From statement (vi), we know that the price of a European put option on Asset X, with a strike price of 22, is 0.29. The payoff of this put option is:

1 2 122 (1),2 (1) ,0 22 ,0Max Max S S Max X

We are asked to find the price of an option that has the payoff described below:

1 2 1(1),2 (1) 22,0 22,0Max Max S S Max X

Page 273: ActuarialBrew.com Exam MFE / 3F Actuarial Models …actuarialbrew.com/data/documents/MFE-Solutions-2014-1st-ed.pdf · Exam MFE / 3F Actuarial Models Financial Economics Segment Solutions

Exam MFE/3F Solutions Chapter 11 – Exotic Options: Part II

© ActuarialBrew.com 2014 Page 11.19

We recognize this as a European call option on Asset X. From put-call parity, we have:

- -

- ¥

+ = +

+ = +

( )

0.06 1022 0.29

r T tEur t Eur

Eur

C Ke S P

C e X

If we can find the current value of Asset X, then we can solve for the value of the call option. Asset X can be replicated by purchasing 2 shares of Stock 2, and buying an exchange call option that allows its owner to exchange 2 shares of Stock 2 for 1 share of Stock 1:

1 1 2 2 1 2[ (1),2 (1)] 2 (1) [ (1) 2 (1),0]X Max S S S Max S S

The current value of Asset X is the value of 2 shares of Stock 2 plus the value of the exchange call:

0 0,1 1 2 0,1 2 1 2

2

[ (1),2 (1)] 2 (0) (0),2 (0),1

2 (0) ExchangeCallPrice

P PEurX F Max S S F S C S S

S

The exchange call option is an exchange call option with 1 share of Stock 1 as the underlying asset and 2 shares of Stock 2 as the strike asset. To find the value of this exchange call option, we find the appropriate volatility parameter:

s s s rs s= + - = + - - =2 2 2 22 0.15 0.20 2( 0.30)(0.15)(0.20) 0.28373S KKS

The values of 1d and 2d are:

d

ds

ss

- - ¥

- - ¥

Ê ˆ Ê ˆ ¥+ +Á ˜ Á ˜¥Ë ¯ Ë ¯

= = = -

= - = - - = -

2 0 1 2

0 1

1

2 1

20 0.28373 1ln ln

2 2(2 11)0.19406

0.28373 1

0.19406 0.28373 1 0.47779

S

K

T

TSe T e

Ke ed

T

d d T

From the normal table, we have:

1

2

( ) ( 0.19406) 0.42306

( ) ( 0.47779) 0.31640

N d N

N d N

= - == - =

The value of the exchange call option is:

1 20 1 0 1

ExchangeCallPrice ( ) ( )

20 (0.42306) 2 11 (0.31640)1.5004

S KT TSe N d Ke N d

e e

The current value of Asset X is:

0 22 (0) ExchangeCallPrice 2 11 1.5004 23.5004X S

Page 274: ActuarialBrew.com Exam MFE / 3F Actuarial Models …actuarialbrew.com/data/documents/MFE-Solutions-2014-1st-ed.pdf · Exam MFE / 3F Actuarial Models Financial Economics Segment Solutions

Exam MFE/3F Solutions Chapter 11 – Exotic Options: Part II

© ActuarialBrew.com 2014 Page 11.20

We can now use put-call parity to find the value of the call option:

0.06 10

0.06

22 0.29

22 23.5004 0.29

3.0716

Eur

Eur

Eur

C e X

C e

C

- ¥

-

+ = +

+ = +=

Solution 11.27

E Exchange Options

Consider an asset, which we will call Asset X, which has a payoff of:

1 1 2[ (1),2 (1)]X Max S S

From statement (vi), we know that the price of a European put option on Asset X, with a strike price of 22, is 0.29. The payoff of this put option is:

1 2 122 (1),2 (1) ,0 22 ,0Max Max S S Max X

We are asked to find the price of an option that has the payoff described below:

1 2 1(1),2 (1),22 ,22Max S S Max X

This payoff can be obtained by owning an asset that has a payoff of 1X and the put option:

1 1 1,22 22 ,0Max X X Max X

The present value of this payoff is the current value of Asset X plus the value of the European put option on Asset X:

0,1 1 0,1 1 0,1 1

0

,22 22 ,0

0.29

P P PF Max X F X F Max X

X

If we can find the current value of Asset X, then we can solve for the value of the rainbow option. Asset X can be replicated by purchasing 2 shares of Stock 2, and buying an exchange call option that allows its owner to exchange 2 shares of Stock 2 for 1 share of Stock 1:

1 1 2 2 1 2[ (1),2 (1)] 2 (1) [ (1) 2 (1),0]X Max S S S Max S S

The current value of Asset X is the value of 2 shares of Stock 2 plus the value of the exchange call:

0 0,1 1 2 0,1 2 1 2

2

[ (1),2 (1)] 2 (0) (0),2 (0),1

2 (0) ExchangeCallPrice

P PEurX F Max S S F S C S S

S

The exchange call option is an exchange call option with 1 share of Stock 1 as the underlying asset and 2 shares of Stock 2 as the strike asset. To find the value of this exchange call option, we find the appropriate volatility parameter:

s s s rs s= + - = + - - =2 2 2 22 0.15 0.20 2( 0.30)(0.15)(0.20) 0.28373S KKS

Page 275: ActuarialBrew.com Exam MFE / 3F Actuarial Models …actuarialbrew.com/data/documents/MFE-Solutions-2014-1st-ed.pdf · Exam MFE / 3F Actuarial Models Financial Economics Segment Solutions

Exam MFE/3F Solutions Chapter 11 – Exotic Options: Part II

© ActuarialBrew.com 2014 Page 11.21

The values of 1d and 2d are:

d

ds

ss

- - ¥

- - ¥

Ê ˆ Ê ˆ ¥+ +Á ˜ Á ˜¥Ë ¯ Ë ¯

= = = -

= - = - - = -

2 0 1 2

0 1

1

2 1

20 0.28373 1ln ln

2 2(2 11)0.19406

0.28373 1

0.19406 0.28373 1 0.47779

S

K

T

TSe T e

Ke ed

T

d d T

From the normal table, we have:

1

1

( ) ( 0.19406) 0.42306

( ) ( 0.47779) 0.31640

N d N

N d N

= - == - =

The value of the exchange call option is:

1 20 1 0 1

ExchangeCallPrice ( ) ( )

20 (0.42306) 2 11 (0.31640)1.5004

S KT TSe N d Ke N d

e e

The current value of Asset X is:

0 22 (0) ExchangeCallPrice 2 11 1.5004 23.5004X S

The value of the rainbow option is:

[ ]( )0,1 1 0,22 0.29 23.5004 0.29 23.7904PF Max X X= + = + =

Solution 11.28

A Cash Call Options

The payoff is the same as the payoff resulting from owning 50 cash calls. To find the value of the cash calls, we must find the values of 2d and 2( )N d :

( )

( )

2

2

2

2

ln / [ 0.5 ]( )

ln 50 / 52 [0.08 0.03 0.5(0.3) ](0.5)0.17310

0.3 0.5( ) ( 0.17310) 0.43129

tS K r T td

T t

N d N

d ss

+ - - -=

-

+ - -= = -

= - =

The value of 50 cash calls is:

( ) 0.08(0.5)250 ( ) 50 ( ) 50 (0.43129) 20.7189r T tCashCall K e N d e- - -¥ = = =

Page 276: ActuarialBrew.com Exam MFE / 3F Actuarial Models …actuarialbrew.com/data/documents/MFE-Solutions-2014-1st-ed.pdf · Exam MFE / 3F Actuarial Models Financial Economics Segment Solutions

Exam MFE/3F Solutions Chapter 11 – Exotic Options: Part II

© ActuarialBrew.com 2014 Page 11.22

Solution 11.29

D Asset Call Options

The payoff is the same as the payoff resulting from owning 5 asset calls. To find the value of the asset calls, we must find the values of 1d and 1( )N d :

( )

( )

2

1

2

1

ln / [ 0.5 ]( )

ln 50 / 52 [0.08 0.03 0.5(0.3) ](0.5)0.03903

0.3 0.5( ) (0.03903) 0.51557

tS K r T td

T t

N d N

d ss

+ - + -=

-

+ - += =

= =

The value of 5 asset calls is:

( ) 0.03(0.5)15 ( ) 5 ( ) 5 50 (0.51557) 126.9735T t

tAssetCall K S e N d ed- - -¥ = ¥ = ¥ =

Solution 11.30

B All-or-Nothing Options

We can use the value of the gap call to find the value of an asset call with a strike price of $52:

= - ¥= - ¥

=

2 1 2( ) ( )

3.43 (52) 53 0.41(52) 25.16

GapCall AssetCall K K CashCall K

AssetCallAssetCall

Since the combination of an asset call with a strike price of $52 and an asset put with a strike price of $52 is certain to have a payoff of 1 share of stock at time 0.5, the combination of their prices must be equal to the prepaid forward price of 1 share of stock:

0,0.5

0.03 0.5

(52) (52) ( )

25.16 (52) 50(52) 24.0956

PAssetCall AssetPut F S

AssetPut eAssetPut

Solution 11.31

B All-or-Nothing Options

We can use the value of the gap put to find the value of a cash put with a strike price of $52:

= ¥ -= ¥ -

=

1 2 2

2

( ) ( )

6.00 53 (52) 20.68( ) 0.5034

GapPut K CashPut K AssetPut K

CashPut

CashPut K

Page 277: ActuarialBrew.com Exam MFE / 3F Actuarial Models …actuarialbrew.com/data/documents/MFE-Solutions-2014-1st-ed.pdf · Exam MFE / 3F Actuarial Models Financial Economics Segment Solutions

Exam MFE/3F Solutions Chapter 11 – Exotic Options: Part II

© ActuarialBrew.com 2014 Page 11.23

Since the combination of a cash call and a cash put is certain to pay off for $1 at the end of the year, the current value of the combination must be the present value of $1:

( )

0.08 1

(52) (52)

(52) 0.5034(52) 0.4197

r T tCashCall CashPut e

CashCall e

CashCall

Solution 11.32

B All-or-Nothing Options

We can use the asset-or-nothing call price find the value of 1d :

d- -

-

=

==

=

( )1

0.03(1)1

1

1

( ) ( )

27.84 50 ( )

( ) 0.57376

0.18596

T ttAssetCall K S e N d

e N d

N d

d

The value of 2d is:

s= - - = - = -2 1 0.18596 0.3 1 0.11404d d T t

The Sharpe ratio of the asset call is equal to the Sharpe ratio of the stock:

as-=0.20

r

We can now find 2d :

a d s a d s

s s

d sa as ss

Ê ˆ Ê ˆ+ - - - + - + - - -Á ˜ Á ˜Ë ¯ Ë ¯= =

- -Ê ˆ + - - -Á ˜Ë ¯ - -= + - = + -

-= - + =

2 2

2

2

2

ln ( 0.5 )( ) ln ( 0.5 )( )ˆ

ln ( 0.5 )( )

0.11404 0.2 1 0.08596

t t

t

S ST t r r T t

K Kd

T t T tS

r T tK r r

T t d T tT t

The probability that the stock price is greater than $52 is equal to 2ˆ( )N d :

2ˆ( ) (0.08596) 0.53425N d N= =

Page 278: ActuarialBrew.com Exam MFE / 3F Actuarial Models …actuarialbrew.com/data/documents/MFE-Solutions-2014-1st-ed.pdf · Exam MFE / 3F Actuarial Models Financial Economics Segment Solutions

Exam MFE/3F Solutions Chapter 11 – Exotic Options: Part II

© ActuarialBrew.com 2014 Page 11.24

Solution 11.33

A All-or-Nothing Options

For Stock B, we can use the cash call price to determine K :

( )

- -

-

=

==

=

+ - -=

=

( )2

0.12(0.75)2

2

22

( ) ( )

0.49 ( )

( ) 0.53615

0.09074

ln 50 / [0.12 0.03 0.5(0.2) ](0.75)0.09074

0.2 0.7551.87341

r T tCashCall K e N d

e N d

N d

d

K

K

We can use K to obtain 1d for Stock A:

( )

( )

d ss

+ - + -=

-

+ - += =

2

1

2

ln / [ 0.5 ]( )

ln 50 / 51.87341 [0.12 0.07 0.5(0.2) ](0.75)0.09074

0.2 0.75

tS K r T td

T t

We can now determine the price of the asset put on Stock A:

d- -

-

-

= -

= -

= ¥=

( )1

0.07(0.75)

0.07(0.75)

( ) ( )

50 ( 0.09074)

50 0.4638522.00630

T ttAssetPut K S e N d

e N

e

Solution 11.34

B All-or-Nothing Options

For the square of the final stock price to be less than 100, the final stock price must be less than 10:

[ ] < ¤ <2(1) 100 (1) 10S S

Therefore, the option described in the question is 100 cash-or-nothing put options that have a strike price of 10. The current value of the option is:

0.03 12 2100 ( , , ) 100 ( ) 100 ( )rTCashPut S K T e N d e N d

To find the delta of the option, we must find the derivative of the price with respect to the stock price:

( ) ( )-

- -∂ ¥ - ∂ - ∂

= = - -∂ ∂ ∂

0.032 20.03 0.03 2

2

100 ( ) ( )100 100 '( )

e N d N d de e N d

S S S

Page 279: ActuarialBrew.com Exam MFE / 3F Actuarial Models …actuarialbrew.com/data/documents/MFE-Solutions-2014-1st-ed.pdf · Exam MFE / 3F Actuarial Models Financial Economics Segment Solutions

Exam MFE/3F Solutions Chapter 11 – Exotic Options: Part II

© ActuarialBrew.com 2014 Page 11.25

The derivative of 2d with respect to the stock price is:

( )

d

d

d

s

ss

s

s s

-

-

- -

- -

Ê ˆÊ ˆ+Á ˜Á ˜

Ë ¯Á ˜∂ -Á ˜Á ˜

¥Á ˜∂ - Ë ¯∂= = = =

∂ ∂ ∂

= =¥ ¥

2

12

ln2

1

1 110 0.30 1 3

T

rT

rT T

T rT

SeT

KeT

T Ke ed Td Se Ke

S S S T S T

The current values of 1d and 2d are:

2 0.02 2

0.03

1

2 1

10 0.3ln ln

2 2100.18333

0.30 1

0.18333 0.30 1 0.11667

T

rTSe e

TKe e

dT

d d T

d s

ss

- -

- -

Ê ˆ Ê ˆ+ +Á ˜ Á ˜

Ë ¯ Ë ¯= = =

= - = - = -

The density function for the standard normal random variable is:

p-=

20.51'( )

2xN x e

We can now calculate the delta of the option:

22

2

0.50.03 0.0322

0.03 0.5 ( 0.11667)

1 1100 '( ) 100

321 1

100 12.8175332

dde N d e e

S

e e

p

p

-- -

- - ¥ -

∂- - = - ¥∂

= - ¥ ¥ = -

Since the option was sold, the resulting delta of the position is:

1 ( 12.81753) 12.81753- ¥ - =

Therefore, 12.81753 shares of stock must be sold to hedge the position. Or, said differently, the quantity to be purchased is –12.81753.

Solution 11.35

A All-or-Nothing Options

The option can be replicated by purchasing 100 cash calls with a strike price of $100 and selling 50 cash calls with a strike price of $150. Therefore, the price of the option is:

100 (100,100,1) 50 (100,150,1)CashCall CashCall

Page 280: ActuarialBrew.com Exam MFE / 3F Actuarial Models …actuarialbrew.com/data/documents/MFE-Solutions-2014-1st-ed.pdf · Exam MFE / 3F Actuarial Models Financial Economics Segment Solutions

Exam MFE/3F Solutions Chapter 11 – Exotic Options: Part II

© ActuarialBrew.com 2014 Page 11.26

First we find the value of the cash call with a strike price of $100:

2 0.03 1 2

0.09 1

1

2 1

20.09 1

2

100 0.3ln ln 1

2 21000.35000

0.3 1

0.35000 0.3 1 0.05000

( ) (0.05000) 0.51994

(100,100,1) ( ) (0.51994) 0.47519

T

rT

rT

Se eT

Ke ed

T

d d T

N d N

CashCall e N d e

Next, we calculate the value of the cash call with a strike price of $150:

2 0.03 1 2

0.09 1

1

2 1

20.09 1

2

100 0.3ln ln 1

2 21501.00155

0.3 1

1.00155 0.3 1 1.30155

( ) ( 1.30155) 0.09654

(100,150,1) ( ) (0.09654) 0.088231

T

rT

rT

Se eT

Ke ed

T

d d T

N d N

CashCall e N d e

The value of the option described in the question is:

100 (100,100,1) 50 CashCall(100,150,1)

100 0.47519 50 0.088231 43.1074CashCall

Solution 11.36

C Collect-on-Delivery Call

The COD call consists of a regular call option and the sale of P cash calls. Let’s begin by finding the value of the regular call.

The first step is to calculate 1d and 2d :

2 2

1

2 1

ln( / ) ( 0.5 )( ) ln(100 /105) (0.07 0.02 0.5 0.2 ) 2

0.2 20.32248

0.32248 0.20 2 0.03964

S K r T td

T t

d d T

d ss

s

+ - + - + - + ¥ ¥= =-

=

= - = - =

We have:

1( ) (0.32248) 0.62646N d N

2( ) (0.03964) 0.51581N d N

Page 281: ActuarialBrew.com Exam MFE / 3F Actuarial Models …actuarialbrew.com/data/documents/MFE-Solutions-2014-1st-ed.pdf · Exam MFE / 3F Actuarial Models Financial Economics Segment Solutions

Exam MFE/3F Solutions Chapter 11 – Exotic Options: Part II

© ActuarialBrew.com 2014 Page 11.27

The value of the European call option is:

( ) ( )1 2

0.02(2) 0.07(2)

( ) ( )

100 0.62646 105 0.51581 13.10513

T t r T tEurC Se N d Ke N d

e e

The value of a cash call is:

( ) 0.07(2)2(105) ( ) 0.51581 0.44842r T tCashCall e N d e- - -= = ¥ =

The initial cost of a COD call is zero:

( ) 0

13.10513 0.44842 029.2249

EurC P CashCall K

PP

- ¥ =- ¥ =

=

If the final stock price is $130, then the payout is:

( ) 130 105 29.2249 4.2249S T K P- - = - - = -

Solution 11.37

D Collect-on-Delivery Call

The COD call consists of a regular call option and the sale of P cash calls. Let’s begin by finding the value of the regular call.

The first step is to calculate 1d and 2d :

2 2

1

2 1

ln( / ) ( 0.5 )( ) ln(100 /105) (0.07 0.02 0.5 0.2 ) 2

0.2 20.32248

0.32248 0.20 2 0.03964

S K r T td

T t

d d T

d ss

s

+ - + - + - + ¥ ¥= =-

=

= - = - =

We have:

1( ) (0.32248) 0.62646N d N

2( ) (0.03964) 0.51581N d N

The value of the European call option is:

( ) ( )1 2

0.02(2) 0.07(2)

( ) ( )

100 0.62646 105 0.51581 13.10513

T t r T tEurC Se N d Ke N d

e e

The value of a cash call is:

( ) 0.07(2)2(105) ( ) 0.51581 0.44842r T tCashCall e N d e- - -= = ¥ =

The initial cost of a COD call is zero:

( ) 0

13.10513 0.448424 029.2249

EurC P CashCall K

PP

- ¥ =- ¥ =

=

Page 282: ActuarialBrew.com Exam MFE / 3F Actuarial Models …actuarialbrew.com/data/documents/MFE-Solutions-2014-1st-ed.pdf · Exam MFE / 3F Actuarial Models Financial Economics Segment Solutions

Exam MFE/3F Solutions Chapter 11 – Exotic Options: Part II

© ActuarialBrew.com 2014 Page 11.28

The COD call consists of a regular call option and the sale of P cash calls.

The delta of the European call option is:

( ) 0.02(2)1( ) 0.62646 0.601896T t

Call e N d e

The delta of the cash call is:

22

2

( ) ( ) 22 2

0.5( )( )

0.07(2) 0.5(0.03964)

( ) ( ) '( )

1 1

21 1

2 100(0.2) 20.012252

r T t r T t

dr T t

dCashCall K e N d e N d

S S S

e eS T

e e

p s

p

- - - -

-- -

- -

∂∂ ∂ È ˘= =Î ˚∂ ∂ ∂

= ¥ ¥

= ¥ ¥

=

The delta of the COD call is the delta of the regular call minus P times the delta of the cash call:

0.601896 29.2249 0.012252 0.2438- ¥ =

Since the market-maker sold the COD call, the market-maker must purchase 0.2438 shares in order to delta-hedge the position.

Solution 11.38

C Asset-or-Nothing Options

The investor can replicate the payoff by purchasing an asset call and selling 100 cash puts. Therefore, the current value of the payoff is:

( ) ( )1 2

1 2

1 2

(100) 100 (100) ( ) 100 ( )

90 ( ) 100 ( )

100 ( ) ( ) (since 90 100)

T t r T tt

r r

r r

AssetCall CashPut S e N d e N d

e e N d N d

e N d N d e

Once again, we can make use of the substitution 90 100re in determining the values of 1d and 2d :

d d sd s

s sss s s s

-Ê ˆ

+ - + ¥ ¥Á ˜Ë ¯+ - + -= =-

=

= - = - = -

22 ( )1

1

2 1

90ln ( 0.5 ) 1

ln( / ) ( 0.5 )( ) 901

0.5

0.5 1 0.5

rr

S K r T t edT t

d d T

Therefore:

= -1 2d d

Page 283: ActuarialBrew.com Exam MFE / 3F Actuarial Models …actuarialbrew.com/data/documents/MFE-Solutions-2014-1st-ed.pdf · Exam MFE / 3F Actuarial Models Financial Economics Segment Solutions

Exam MFE/3F Solutions Chapter 11 – Exotic Options: Part II

© ActuarialBrew.com 2014 Page 11.29

The current value of the payoff is:

[ ] [ ]- -- - = - =1 2 1 1100 ( ) ( ) 100 ( ) ( ) 0r re N d N d e N d N d

Solution 11.39

B Cash-Or-Nothing Call Option

A European call option has the same gamma as an otherwise equivalent European put option. Therefore, the gamma of a call option with the payoff described below is 0.035:

- >60 if 60T TS S

The gap call option described in the question has a gamma of 0.03, and its payoff is:

- >55 if 60T TS S

The cash-or-nothing call option has the following payoff:

1,000 if 60TS

The cash-or-nothing call option can be replicated by purchasing 200 of the gap call options and selling 200 of the call options:

[ ] [ ]- = - - - = ¥ = >200 200 ( 55) ( 60) 200 5 1,000 if 60T T TGapCall Call S S S

The gamma of the position is the gamma of a position consisting of 200 long gap calls and 200 short calls is:

[ ]- = ¥ - = -200 0.03 0.035 200 ( 0.005) 1.00

Solution 11.40

A Cash-Or-Nothing Call Option

A European call option has the same vega as an otherwise equivalent European put option. Therefore, the vega of a call option with the payoff described below is 0.185:

- >60 if 60T TS S

The gap call option described in the question has a vega of 0.160, and its payoff is:

- >55 if 60T TS S

The cash-or-nothing call option has the following payoff:

1,000 if 60TS

The cash-or-nothing call option can be replicated by purchasing 200 of the gap call options and selling 200 of the call options:

[ ] [ ]- = - - - = ¥ = >200 200 ( 55) ( 60) 200 5 1,000 if 60T T TGapCall Call S S S

The vega of the position is the vega of a position consisting of 200 long gap calls and 200 short calls is:

[ ]- = ¥ - = -200 0.160 0.185 200 ( 0.025) 5.00

Page 284: ActuarialBrew.com Exam MFE / 3F Actuarial Models …actuarialbrew.com/data/documents/MFE-Solutions-2014-1st-ed.pdf · Exam MFE / 3F Actuarial Models Financial Economics Segment Solutions

Exam MFE/3F Solutions Chapter 11 – Exotic Options: Part II

© ActuarialBrew.com 2014 Page 11.30

Solution 11.41

D Cash-Or-Nothing Call Option

The cash-or-nothing put option pays:

<1,000 if 60TS

A gap put option with a strike price of 55 and a trigger price of 60 pays:

- <55 if 60T TS S

A regular put option with a strike price of 60 pays:

- <60 if 60T TS S

The cash-or-nothing put option can be replicated by purchasing 200 of the put options and selling 200 of the gap put options:

[ ] [ ]- = - - - = ¥ = <200 200 (60 ) (55 ) 200 5 1,000 if 60T T TPut GapPut S S S

We use put-call parity for gap calls and gap puts to find the theta of the gap put:

d

q q

q

- - - -

- - - -

- - - -

- - - -

+ = +

+ = +

+ = +

- + = +

( ) ( )1

0.12(0.75 ) 0.07(0.75 )

0.12(0.75 ) 0.07(0.75 )

0.12(0.75 ) 0.07(0.75 )

55 60

(0.12)55 (0.07)60

3.5 (0.12)55 (0.07)60

r T t T t

t t

t tGapCall GapPut

t tGap

GapCall K e Se GapPut

GapCall e e GapPut

e e

e e Put

Evaluating at time = 0t , we have:

q

q

- - - -- + = +

= -

0.12(0.75 0) 0.07(0.75 0)3.5 (0.12)55 (0.07)60

1.4532GapPut

GapPut

e e

The theta of the cash-or-nothing put is the theta of a position consisting of 200 long puts and 200 short gap puts:

200 200 1.0 ( 1.4532) 200 0.4532 90.65Put GapPut

Solution 11.42

C Cash-Or-Nothing Call Option

The call option has the payoff described below:

- >60 if 60T TS S

The gap call option has the payoff described below:

- >55 if 60T TS S

The cash-or-nothing call option has the following payoff:

1,000 if 60TS

Page 285: ActuarialBrew.com Exam MFE / 3F Actuarial Models …actuarialbrew.com/data/documents/MFE-Solutions-2014-1st-ed.pdf · Exam MFE / 3F Actuarial Models Financial Economics Segment Solutions

Exam MFE/3F Solutions Chapter 11 – Exotic Options: Part II

© ActuarialBrew.com 2014 Page 11.31

The cash-or-nothing call option can be replicated by purchasing 200 of the gap call options and selling 200 of the call options:

[ ] [ ]- = - - - = ¥ = >200 200 ( 55) ( 60) 200 5 1,000 if 60T T TGapCall Call S S S

Let C denote the cost of the regular call option. The value of the replicating portfolio is:

- =200(1.5 ) 100C C C

We can now determine the weights of the gap calls and calls in the portfolio. Note that the portfolio consists of a short position in the calls, so the calls have a negative weight:

w

w

= =

-= = -

200(1.5 )3

100200( )

2100

GapCall

Call

CC

CC

The elasticity of the replicating portfolio is:

w w¥ W + ¥ W = ¥ - ¥ = - =3 6 2 7 18 14 4GapCall GapCall Call Call

Solution 11.43

C Early Asset-or-Nothing Put

Since K = 1,000(1 – 0.35) = 650, the value of 1d is:

2

1

2

ln / [ 0.5 ]( )

ln 1,000 / 650 [0.12 0.05 0.5(0.22) ](1 0)

0.22 1 02.38629

tS K r T td

T t

The value of - 1( )N d is:

1( ) ( 2.38629) 0.00851N d N- = - =

The price of one of the asset-or-nothing put options is:

( ) 0.05(1 0)1( ) ( ) 1,000 0.00851 8.094962T t

tAssetPut K S e N d ed- - - -= - = ¥ =

Therefore, the price of one million of the options is:

1,000,000 8.094962 8,094,962¥ =

Page 286: ActuarialBrew.com Exam MFE / 3F Actuarial Models …actuarialbrew.com/data/documents/MFE-Solutions-2014-1st-ed.pdf · Exam MFE / 3F Actuarial Models Financial Economics Segment Solutions

Exam MFE/3F Solutions Chapter 11 – Exotic Options: Part II

© ActuarialBrew.com 2014 Page 11.32

Solution 11.44

B Delta-Hedging Gap Call Options

The payoff of each gap call can be written as the payoff of a regular call minus 20 cash calls:

= - >= - - >

1 1

1 1

Payoff 70 if 50

( 50) 20 if 50

GapCall S S

S S

Therefore, the price of a gap call option is equal to a price of a regular call minus the price of 20 cash calls:

= - ¥(50) 20 (50)GapCall Call CashCall

Therefore, the delta of one gap call option is:

D = D - D(50) (50)20GapCall Call CashCall

To find the deltas of the regular call and the cash call, we need to calculate 1d and 2d :

2 0 1 2

0.05 12

1 1

2 1

50 0.50ln ln 1

2 2500.35000 ( ) 0.63683

0.50 1

0.50 0.35000 0.50 1 0.15000

T

rTSe e

TK e e

d N dT

d d T

d s

s

- - ¥

- - ¥

Ê ˆ Ê ˆ+ + ¥Á ˜ Á ˜

Ë ¯ Ë ¯= = = fi =

= - = - = -

The deltas of the regular call and the cash call are:

2 22

( ) 0 11

0.5( )( ) ( ) 0.05 1 0.5( 0.15)

(50) 2

( ) (0.63683) 0.63683

'( ) 0.0150012 ( ) 50 0.5 2 1

T tCall

dr T t r T t

CashCall

e N d e

e e e e eN d

S T t S T t

Now we can calculate the delta of the gap call option:

(50) 20 (50) 0.63683 20 0.015001 0.33664GapCall Call CashCall= - ¥ = - ¥ =

Since the market-maker sells 1,000 of the gap call options, the market-maker multiplies the delta of one gap call option by 1,000 to determine the number of shares that must be purchased in order to delta-hedge the position:

1,000 0.33664 336.64¥ =

Solution 11.45

D Asset-or-Nothing Power Option

Since ( )S t is a geometric Brownian motion, so is [ ( )]S t a :

a d s a d s sÈ ˘= - + fi = - + - +Î ˚2( ) ( )

( ) ( ) ( ) 0.5 ( 1) ( )( )

dS t d Sdt dZ t dt dZ t

S t S

a

aa a a a

Page 287: ActuarialBrew.com Exam MFE / 3F Actuarial Models …actuarialbrew.com/data/documents/MFE-Solutions-2014-1st-ed.pdf · Exam MFE / 3F Actuarial Models Financial Economics Segment Solutions

Exam MFE/3F Solutions Chapter 11 – Exotic Options: Part II

© ActuarialBrew.com 2014 Page 11.33

The coefficient of ( )dZ t is the asset’s volatility. For 4[ ( )]S t , let’s call this volatility s * :

s s= = ¥ =* 4 0.32 1.28a

The dividend yield on 4[ ( )]S t is:

d d s= - - - - = - - - - = -2 2* ( ) 0.5 ( 1) 0.12 4(0.12 0.06) 0.5(4)(4 1)(0.32) 0.7344r ra a a

The initial price of 4[ ( )]S t is:

= =4 4[ (0)] 2 16S

We use 4[ (1)]S as the underlying asset, and the trigger price that corresponds to this underlying asset is 16:

> fi > fi =4(1) 2 [ (1)] 16 * 16S S K

The first step is to calculate 1d :

( )4 2

1

2

ln( / *) ( *) 0.5( *)

( *)

ln(16 /16) (0.12 ( 0.7344) 0.5 1.28 ) 11.30750

1.28 1

S K r Td

T

d s

s

+ - +=

+ - - + ¥ ¥= =

We have:

1( ) (1.30750) 0.90448N d N

The value of the asset-or-nothing power call option is:

4 ( *) ( 0.7344) 11(16) [ (0)] ( ) 16 0.90448 30.1623TAssetCall S e N d e

Solution 11.46

B Asset-Or-Nothing Call Option

A European call option has the same gamma as an otherwise equivalent European put option. Therefore, the gamma of a call option with the payoff described below is 0.035:

- >60 if 60T TS S

The gap call option described in the question has a gamma of 0.03, and its payoff is:

- >55 if 60T TS S

The asset-or-nothing call option has the following payoff:

if 60T TS S

We need to use the call and the gap call to build an option that pays off for some amount of stock when the final stock price is higher than 60. Purchasing 60 of the gap calls and selling 55 of the call options leaves us with a payout equal to that of 5 asset calls:

¥ - ¥ = - - - = >60 55 60( 55) 55( 60) 5 if 60T T T TGapCall Call S S S S

Page 288: ActuarialBrew.com Exam MFE / 3F Actuarial Models …actuarialbrew.com/data/documents/MFE-Solutions-2014-1st-ed.pdf · Exam MFE / 3F Actuarial Models Financial Economics Segment Solutions

Exam MFE/3F Solutions Chapter 11 – Exotic Options: Part II

© ActuarialBrew.com 2014 Page 11.34

Therefore, the gamma of 5 asset calls is:

¥ G - ¥ G = ¥ - ¥ = -60 55 60 0.03 55 0.035 0.125GapCall Call

The gamma of 1 asset call is found by dividing by 5:

- = -0.125

0.0255

Page 289: ActuarialBrew.com Exam MFE / 3F Actuarial Models …actuarialbrew.com/data/documents/MFE-Solutions-2014-1st-ed.pdf · Exam MFE / 3F Actuarial Models Financial Economics Segment Solutions

Exam MFE/3F Solutions Chapter 12 – Monte Carlo Simulation

© ActuarialBrew.com 2014 Page 12.01

Chapter 12 – Solutions

Solution 12.01

C Standard Deviation of Monte Carlo Estimate

The standard deviation of the Monte Carlo estimate is the standard deviation of the individual price estimates divided by the square root of the number of paths:

s

s = = =( )

3.460.0346

10,000V

V n n

Solution 12.02

B Standard Deviation of Monte Carlo Estimate

The standard deviation of the Monte Carlo estimate is the standard deviation of the individual price estimates divided by the square root of the number of paths:

s

s = =( )

3.46VV n n n

If we were given the actual price of the option, we would use it, but since we do not have the actual price, we use the Monte Carlo estimate as a reasonable approximation:

3.46

0.02 4,9062.47

n n

Solution 12.03

D Forward Price with Monte Carlo Valuation

The Monte Carlo estimate for the current price of the derivative is:

=

= ¥ =Â1

1( ) 24.43

n

ii

V V tn

In Chapter 1 of the study manual, we saw that the forward price for an asset that does not pay dividends is its current price accrued forward at the risk-free rate of return. Therefore the forward price of the derivative is:

¥¥ =0.08 0.2524.43 24.92e

Alternatively, we can obtain the same answer by finding the average of the undiscounted derivative prices. These prices are in the second column from the right side of the table, and the bottom row tells us that their average is $24.93.

The small difference between $24.92 obtained using the first method and $24.93 obtained using the second method is attributable to rounding.

Page 290: ActuarialBrew.com Exam MFE / 3F Actuarial Models …actuarialbrew.com/data/documents/MFE-Solutions-2014-1st-ed.pdf · Exam MFE / 3F Actuarial Models Financial Economics Segment Solutions

Exam MFE/3F Solutions Chapter 12 – Monte Carlo Simulation

© ActuarialBrew.com 2014 Page 12.02

Solution 12.04

B Monte Carlo Valuation in Binomial Model

The risk-neutral probability of up and down movements is:

( ) (0.06 0.05) 0.5 0.92770

* 0.4800 1 * 0.52001.08877 0.92770

r he d ep p

u d

The risk-neutral probabilities of two ups, an up and a down, and two downs are:

uu: 0.4800 0.4800 0.2304ud or du: 2 0.4800 0.5200 0.4992dd: 0.5200 0.5200 0.2704

The tree of stock prices, option payoffs, the associated probabilities, and the associated colors is:

Option Payoff

Risk-Neutral

Probability Color 59.2710 0.0000 0.2304 Red 54.4385

50.0000 50.5026 0.0000 0.4992 Blue 46.3850 43.0314 3.9686 0.2704 Green

The investor’s gambling wheel represents the three outcomes by associating a color with each probability. Only the green values provide an option payoff, and the estimate resulting from the Monte Carlo testing is:

( ) 0.06

1

( ) 31 0 70 0 39 3.9686 1.0412140

nr T t

ii

e eV V T

n

Solution 12.05

A Monte Carlo Valuation in the Binomial Model

The risk-neutral probability of up and down movements is:

( ) (0.06 0.05) 0.5 0.92770

* 0.4800 1 * 0.52001.08877 0.92770

r he d ep p

u d

The risk-neutral probabilities of two ups, an up and a down, and two downs are:

uu: 0.4800 0.4800 0.2304

ud or du: 2 0.4800 0.5200 0.4992dd: 0.5200 0.5200 0.2704

Page 291: ActuarialBrew.com Exam MFE / 3F Actuarial Models …actuarialbrew.com/data/documents/MFE-Solutions-2014-1st-ed.pdf · Exam MFE / 3F Actuarial Models Financial Economics Segment Solutions

Exam MFE/3F Solutions Chapter 12 – Monte Carlo Simulation

© ActuarialBrew.com 2014 Page 12.03

The tree of stock prices, option payoffs, the associated probabilities, and the associated colors is:

Option Payoff Risk-Neutral Probability Color

59.2710 12.2710 0.2304 Red 54.4385

50.0000 50.5026 3.5026 0.4992 Blue 46.3850 43.0314 0.0000 0.2704 Green

The investor’s gambling wheel represents the three outcomes by associating a color with each probability. Only the red and blue values provide option payoffs, and the estimate resulting from the Monte Carlo testing is:

( ) 0.06

1

( ) 31 12.2710 70 3.5026 39 0 4.2082140

nr T t

ii

e eV V T

n

Solution 12.06

A Sum of 12 Uniformly Distributed Random Variables

The sum of the uniform random variables is 5.863:

0.126 0.205 0.080 0.303 0.992 0.481 0.162 0.786 0.279 0.703 0.752 0.994 5.863

The estimate for the standard normal draw is therefore:

=

= - = - = -Â12

1

6 5.863 6 0.137ii

Z u

The Black-Scholes framework implies that the stock price follows a lognormal distribution, so:

a d s s- - - + - - - ¥ ¥ - + ¥ - -= =

=

2 2( 0.5 )( ) (0.08 0.03 0.5 0.3 ) (2 0) 0.3 ( 0.137) 2 0100

95.30

T t z T tT tS S e e

Solution 12.07

A Sum of 12 Uniformly Distributed Random Variables

The sum of the uniform random variables is 5.863:

0.126 0.205 0.080 0.303 0.992 0.481 0.162 0.786 0.279 0.703 0.752 0.994 5.863

Page 292: ActuarialBrew.com Exam MFE / 3F Actuarial Models …actuarialbrew.com/data/documents/MFE-Solutions-2014-1st-ed.pdf · Exam MFE / 3F Actuarial Models Financial Economics Segment Solutions

Exam MFE/3F Solutions Chapter 12 – Monte Carlo Simulation

© ActuarialBrew.com 2014 Page 12.04

The estimate for the standard normal draw is therefore:

=

= - = - = -Â12

1

6 5.863 6 0.137ii

Z u

For the risk-neutral distribution of the stock, we replace a with r. The Black-Scholes framework implies that the stock price follows a lognormal distribution, so:

d s s- - - + - - - ¥ ¥ - + ¥ - -= =

=

2 2( 0.5 )( ) (0.05 0.03 0.5 0.3 ) (2 0) 0.3 ( 0.137) 2 0100

89.75

r T t z T tT tS S e e

Solution 12.08

E Inverse Cumulative Normal Distribution

The steps to obtaining a standard normal random variable are:

1. Obtain an observation x of a random variable x. This observation is 4.6.

2. Find the probability that x is less than x :

= <ˆ ˆ( ) Prob( )F x x x

For a uniform distribution, we have:

-=-

( ) for ( , )x

F x U bb

aa

a

Therefore, the cumulative probability of 4.6 is:

-= =-

4.6 3(4.6) 0.8

5 3F

3. Find the value of z that is the ˆ( )F x quantile of the normal distribution:

1

ˆ ˆ( ) ( )ˆ( ) 0.80000

ˆ 0.80000 0.84162

N z F xN z

z N

Thus 0.84162 is the observation from the standard normal distribution.

For the risk-neutral distribution of the stock, we replace a with r. The Black-Scholes framework implies that the stock price follows a lognormal distribution, so:

2 2( 0.5 )( ) (0.05 0.03 0.5 0.3 ) (2 0) 0.3 (0.84162) 2 0100

135.9435

r T t z T tT tS S e ed s s- - - + - - - ¥ ¥ - + ¥ -= ==

Page 293: ActuarialBrew.com Exam MFE / 3F Actuarial Models …actuarialbrew.com/data/documents/MFE-Solutions-2014-1st-ed.pdf · Exam MFE / 3F Actuarial Models Financial Economics Segment Solutions

Exam MFE/3F Solutions Chapter 12 – Monte Carlo Simulation

© ActuarialBrew.com 2014 Page 12.05

Solution 12.09

B Inverse Cumulative Normal Distribution

The steps to obtaining a standard normal random variable are:

1. Obtain an observation x of a random variable x. This observation is 0.21072.

2. Find the probability that x is less than x :

= <ˆ ˆ( ) Prob( )F x x x

For the exponential distribution in this question, we have:

-= - 0.5( ) 1 xF x e

Therefore, the cumulative probability of 0.21072 is:

0.5 0.21072(0.21072) 1 0.10000F e- ¥= - =

3. Find the value of z that is the ˆ( )F x quantile of the normal distribution:

1

ˆ ˆ( ) ( )ˆ( ) 0.10000

(0.10000) 1.28155ˆ 1.28155

N z F xN z

Nz

Thus 1.28155- is the observation from the standard normal distribution.

For the risk-neutral distribution of the stock, we replace a with r. The Black-Scholes framework implies that the stock price follows a lognormal distribution, so:

2 2( 0.5 )( ) (0.05 0.03 0.5 0.3 ) (2 0) 0.3 ( 1.28155) 2 0100

55.2271

r T t z T tT tS S e ed s s- - - + - - - ¥ ¥ - + ¥ - -= ==

Solution 12.10

E Sequence of Stock Prices

We convert the draws from the uniform distribution into draws from the standard normal distribution:

11 1

12 2

ˆ ˆ(0.15) ( ) (0.15000) 1.03643 1.03643

ˆ ˆ(0.65) ( ) N (0.65000) = 0.38532 0.38532

F N z N z

F N z z

We use these draws to find the new stock prices:

21 2

2

( 0.5 ) ( )

(0.11 0.03 0.5 0.3 ) 2 0.5 0.3( 1.03643 0.38532) 0.5100 90.2018

nr nh z z z hT tS S e

e

Page 294: ActuarialBrew.com Exam MFE / 3F Actuarial Models …actuarialbrew.com/data/documents/MFE-Solutions-2014-1st-ed.pdf · Exam MFE / 3F Actuarial Models Financial Economics Segment Solutions

Exam MFE/3F Solutions Chapter 12 – Monte Carlo Simulation

© ActuarialBrew.com 2014 Page 12.06

Solution 12.11

A Geometric Average Strike Call Option

We convert the draws from the uniform distribution into draws from the standard normal distribution:

11 1

12 2

13 3

ˆ ˆ(0.12) ( ) (0.12000) 1.17499 1.17499

ˆ ˆ(0.87) ( ) (0.87000) 1.12639 1.12639

ˆ ˆ(0.50) ( ) (0.50000) 0.00000 0.00000

F N z N z

F N z N z

F N z N z

We use these draws to find the new stock prices:

2 21 1 1 113 3 3 3

13

2 21 1 1 123 3 3 3

2 13 3

2 1 133 3

23

( 0.5 ) (0.11 0.03 0.5 0.3 ) 0.30 ( 1.17499)0

( 0.5 ) (0.11 0.03 0.5 0.3 ) 0.30 (1.12639)

( 0.5 )1

100 82.54327

82.54327 101.50274

101.502

r z

r z

r z

S S e e

S S e e

S S e

2 1 1

3 3(0.11 0.03 0.5 0.3 ) 0.30 (0.00000)

74 102.69387e

The geometric average of the stock prices is:

13(82.54327 101.50274 102.69387) 95.11185S = ¥ ¥ =

The payoff of the geometric average strike Asian call is:

Average strike Asian call option payoff = Max[ ,0] 102.69387 95.11185

7.5820TS S- = -

=

Solution 12.12

C Asian Call Options

The fact that one trial out of 10,000 resulted in a price of $15.00 is of no use.

As the sampling frequency increases, the price of the Asian option declines. Since the sampling frequency of Option 4 is greater than the sampling frequency of Option 2, it must have a lower price than Option 2:

Option 4 Price 12.15

Since the sampling frequency of Option 4 is less than the sampling frequency of Option 3, it must have a higher price than Option 3:

Option 4 Price 9.71

Furthermore, an arithmetic average is always greater than a geometric average, so the price of Option 4 must be higher than the price of Option 1:

Option 4 Price 9.77

The only choice that satisfies these conditions is $10.30.

Page 295: ActuarialBrew.com Exam MFE / 3F Actuarial Models …actuarialbrew.com/data/documents/MFE-Solutions-2014-1st-ed.pdf · Exam MFE / 3F Actuarial Models Financial Economics Segment Solutions

Exam MFE/3F Solutions Chapter 12 – Monte Carlo Simulation

© ActuarialBrew.com 2014 Page 12.07

Solution 12.13

A Standard Deviation of Monte Carlo Estimate

The standard deviation of an individual option price is:

s = 13.15V

There are 5 x 500 = 2,500 draws, so the standard deviation of the Monte Carlo estimate is:

s

s = = =( )

13.150.263

2,500V

V n n

Solution 12.14

C Standard Deviation of Monte Carlo Estimate

The formula for the standard deviation of the Monte Carlo price estimate is:

( )V

V n n

We have:

(500)

(500)(4,000)

500

4,000 8 500 8

VV

VV VV

If we were given the standard deviation of an individual option price, then we would make use of it. But since we are not given the standard deviation, we begin by estimating the standard deviation of the estimate obtained using 500 draws. We have 5 observations of (500)V available to us, so we can estimate (500)V :

Using the TI-30X IIS, the procedure is:

[2nd][STAT] (Select 1-VAR) [ENTER]

[DATA]

X1= 9.44 ØØ (Hit the down arrow twice)

X2= 10.80 ØØ

X3= 9.78 ØØ

X4= 8.95 ØØ

X5= 9.80 [ENTER]

[STATVAR] Æ Æ (Arrow over to Sx)

The result is: 0.678586767

To exit the statistics mode: [2nd] [EXITSTAT] [ENTER]

Page 296: ActuarialBrew.com Exam MFE / 3F Actuarial Models …actuarialbrew.com/data/documents/MFE-Solutions-2014-1st-ed.pdf · Exam MFE / 3F Actuarial Models Financial Economics Segment Solutions

Exam MFE/3F Solutions Chapter 12 – Monte Carlo Simulation

© ActuarialBrew.com 2014 Page 12.08

The estimate for (500)V is therefore 0.678586767.

Alternatively, using the BA II Plus calculator, the procedure is:

[2nd][DATA] [2nd][CLR WORK]

X01= 9.440 [ENTER] ØØ (Hit the down arrow twice)

X02= 10.80 [ENTER] ØØ

X03= 9.78 [ENTER] ØØ

X04= 8.95 [ENTER] ØØ

X05= 9.80 [ENTER]

[2nd][STAT] ØØØ

The result is: 0.67858677

To exit the statistics mode: [2nd][QUIT]

The resulting solution is:

(500)(4,000)

0.678586770.2399

8 8V

V

Solution 12.15

E Standard Deviation of Monte Carlo Estimate

The standard deviation of the Monte Carlo estimate is the standard deviation of the individual price estimates divided by the square root of the number of paths:

s s

s = = =( )

11.40V CV n n n n

We can solve for the value of n that leads to a standard deviation that is 1% of the price:

= fi =

11.40

0.01 24,6577.26

n n

Page 297: ActuarialBrew.com Exam MFE / 3F Actuarial Models …actuarialbrew.com/data/documents/MFE-Solutions-2014-1st-ed.pdf · Exam MFE / 3F Actuarial Models Financial Economics Segment Solutions

Exam MFE/3F Solutions Chapter 12 – Monte Carlo Simulation

© ActuarialBrew.com 2014 Page 12.09

Solution 12.16

C Monte Carlo Valuation of a European Put

We convert the draws from the uniform distribution into draws from the standard normal distribution:

11 1

12 2

13 3

4

ˆ ˆ(0.90) ( ) (0.90000) 1.28155 1.28155

ˆ ˆ(0.74) ( ) (0.74000) 0.64335 0.64335

ˆ ˆ(0.21) ( ) (0.21000) 0.80642 0.80642

ˆ(0.48) ( )

F N z N z

F N z N z

F N z N z

F N z N 14ˆ(0.48000) 0.05015 0.05015z

When using the Monte Carlo method to value an option, we use the risk-neutral distribution of the stock. Therefore, the four stock prices are found using the following formula:

d s s- - - + -= 2( 0.5 )( ) where: (0,1)r T t z T t

T tS S e z N

The four stock prices are and the associated payoffs to the put option are:

2

2

2

1 (0.07 0.02 0.5 0.25 )(1) 0.25 1.28155 11 1

2 (0.07 0.02 0.5 0.25 )(1) 0.25 0.64335 11 2

3 (0.07 0.02 0.5 0.25 )(1) 0.25 ( 0.80642) 11 3

41

100 140.37363 (1) 0

100 119.67236 (1) 0

100 83.28889 (1) 18.7111

S e V

S e V

S e V

S

- - ¥ + ¥

- - ¥ + ¥

- - ¥ + ¥ -

= = =

= = =

= = =2(0.07 0.02 0.5 0.25 )(1) 0.25 ( 0.05015) 1

3100 100.62318 (1) 1.3768e V- - ¥ + ¥ -= = =

The Monte Carlo estimate is:

[ ]( ) 0.07(1 0)

1

( ) 0 0 18.7111 1.3768 4.68254

nr T t

ii

e eV V T

n

- - - -

== = + + + =Â

Solution 12.17

D Monte Carlo Valuation of an Asian Put

The arithmetic averages of the stock prices are:

i Arithmetic

Average 1 39.133 2 60.767 3 61.847 4 53.440

Page 298: ActuarialBrew.com Exam MFE / 3F Actuarial Models …actuarialbrew.com/data/documents/MFE-Solutions-2014-1st-ed.pdf · Exam MFE / 3F Actuarial Models Financial Economics Segment Solutions

Exam MFE/3F Solutions Chapter 12 – Monte Carlo Simulation

© ActuarialBrew.com 2014 Page 12.10

The payoff of an average strike Asian put option is the average stock price minus the final stock price, when that amount is greater than zero:

-Average strike Asian put option payoff = Max[ ,0]TS S

These payoffs are shown in the rightmost column below:

i Arithmetic

Average

Final Stock Price

Option Payoff

1 39.133 32.43 6.703 2 60.767 47.26 13.507 3 61.847 70.11 0.000 4 53.440 52.44 1.000

The Monte Carlo estimate is:

[ ]- - - ¥ -

== = + + + =Â

( ) 0.10 (1 0)

1

( ) 6.703 13.507 0.000 1.000 4.804

nr T t

ii

e eV V T

n

Solution 12.18

D Control Variate Valuation

The arithmetic and geometric averages of the stock prices are:

i Arithmetic

Average Geometric Average

1 39.133 37.423 2 60.767 59.675 3 61.847 61.353 4 53.440 53.425

The payoff of an average strike Asian put option is the average stock price minus the final stock price, when that amount is greater than zero:

-Average strike Asian put option payoff = Max[ ,0]TS S

These payoffs are shown in the rightmost columns below:

i Arithmetic

Average Geometric Average

Final Stock Price

Arithmetic Option Payoff

Geometric Option Payoff

1 39.133 37.423 32.43 6.703 4.993 2 60.767 59.675 47.26 13.507 12.415 3 61.847 61.353 70.11 0.000 0.000 4 53.440 53.425 52.44 1.000 0.985

The Monte Carlo estimate for the arithmetic average strike Asian put is:

[ ]- - - ¥ -

== = + + + =Â

( ) 0.10 (1 0)

1

( ) 6.703 13.507 0.000 1.000 4.79794

nr T t

ii

e eA V T

n

Page 299: ActuarialBrew.com Exam MFE / 3F Actuarial Models …actuarialbrew.com/data/documents/MFE-Solutions-2014-1st-ed.pdf · Exam MFE / 3F Actuarial Models Financial Economics Segment Solutions

Exam MFE/3F Solutions Chapter 12 – Monte Carlo Simulation

© ActuarialBrew.com 2014 Page 12.11

The Monte Carlo estimate for the geometric average strike Asian put is:

[ ]- - - ¥ -

== = + + + =Â

( ) 0.10 (1 0)

1

( ) 4.993 12.415 0.000 0.985 4.16054

nr T t

ii

e eG V T

n

The true price for the geometric option is $2.02, so the control variate price of the arithmetic average option is:

= + - = + - =* ( ) 4.7979 (2.02 4.1605) 2.6574A A G G

Solution 12.19

E Control Variate Valuation

The arithmetic and geometric averages of the stock prices are:

i Arithmetic

Average Geometric Average

1 39.133 37.423 2 60.767 59.675 3 61.847 61.353 4 53.440 53.425

The payoff of an average strike Asian put option is the average stock price minus the final stock price, when that amount is greater than zero:

-Average strike Asian put option payoff = Max[ ,0]TS S

These payoffs are shown in the rightmost columns below:

i Arithmetic

Average Geometric Average

Final Stock Price

0.10iY e

Arithmetic Option Payoff

0.10iX e

Geometric Option Payoff

1 39.133 37.423 32.43 6.703 4.993 2 60.767 59.675 47.26 13.507 12.415 3 61.847 61.353 70.11 0.000 0.000 4 53.440 53.425 52.44 1.000 0.985

The values in the two rightmost columns are the payoffs, which means that they are the discounted Monte Carlo prices, iY and iX , times rTe .

The estimate for b is:

( )

( )b = =

= =

- - - -

= =

- -

 Â

 Â

2

1 1

22 2

1 1

( )( ) ( )( )

( ) ( )

n nrT

i i i ii i

n nrT

i ii i

Y Y X X e Y Y X X

X X e X X

Page 300: ActuarialBrew.com Exam MFE / 3F Actuarial Models …actuarialbrew.com/data/documents/MFE-Solutions-2014-1st-ed.pdf · Exam MFE / 3F Actuarial Models Financial Economics Segment Solutions

Exam MFE/3F Solutions Chapter 12 – Monte Carlo Simulation

© ActuarialBrew.com 2014 Page 12.12

We get the same estimate for b regardless of whether we use the time 0 prices or the time 1 payoffs (as shown in the rightmost expression above). To save time, we use the time 1 payoffs from the table above.

We perform a regression using the sixth column in the table above as the x-values and the fifth column as the y-values. The resulting slope coefficient is:

b = 1.0959

During the exam, it is more efficient to let the calculator perform the regression and determine the slope coefficient.

Using the TI-30XS Multiview calculator, we first clear the data by pressing [data] [data] ↓↓↓ until Clear ALL is shown, and then press [enter].

Fill out the table as shown below:

L1 L2 L3 4.993 6.703 ----------- 12.415 13.507 0.000 0.000 0.985 1.000

Next, press:

[2nd] [stat] 2 (i.e., select 2-Var Stats)

Select L1 for x-data and press [enter].

Select L2 for y-data and press [enter].

Select CALC and then [enter]

Exit the data table by pressing [2nd] [quit].

Obtain access to the statistics by pressing [2nd] [stat] 3

Note the following statistics:

4.598255.30251.0959 (This is )

x

y

a

Alternatively, the TI-30X IIS or the BA II Plus can be used to obtain the values above. To use one of those calculators, follow the steps outlined at the end of this solution and then return to this point to continue the solution.

The Monte Carlo estimate for the arithmetic average strike Asian put is:

[ ]- -

- ¥ -

== = = =Â

( )0.10 (1 0)

1

( ) 5.3025 4.80nr T t

ii

eA Y V T e

n

Page 301: ActuarialBrew.com Exam MFE / 3F Actuarial Models …actuarialbrew.com/data/documents/MFE-Solutions-2014-1st-ed.pdf · Exam MFE / 3F Actuarial Models Financial Economics Segment Solutions

Exam MFE/3F Solutions Chapter 12 – Monte Carlo Simulation

© ActuarialBrew.com 2014 Page 12.13

The Monte Carlo estimate for the geometric average strike Asian put is:

( )

0.10 (1 0)

1

( ) 4.59825 4.16nr T t

ii

eG X V T e

n

The true price for the geometric option is $2.02, so the control variate price of the arithmetic average option is:

b= + - = + - =* ( ) 4.80 1.0959(2.02 4.16) 2.45A A G G

Alternative Calculators

Using the TI-30X IIS, the procedure is:

[2nd][STAT] (Select 2-VAR) [ENTER]

[DATA]

X1= 4.993 Ø (Hit the down arrow once)

Y1= 6.703 Ø

X2= 12.415 Ø

Y2= 13.507 Ø

X3= 0.000 Ø

Y3= 0.000 Ø

X4= 0.985 Ø

Y4= 1.000 [ENTER]

[STATVAR]

Press the right arrow and note the following statistics:

4.598255.30251.0959 (This is )

x

y

a

To exit the statistics mode: [2nd] [EXITSTAT] [ENTER]

Using the BA II Plus calculator, the procedure is:

[2nd][DATA] [2nd][CLR WORK]

X01= 4.993 [ENTER] Ø (Hit the down arrow once)

Y01= 6.703 [ENTER] Ø

X02= 12.415 [ENTER] Ø

Page 302: ActuarialBrew.com Exam MFE / 3F Actuarial Models …actuarialbrew.com/data/documents/MFE-Solutions-2014-1st-ed.pdf · Exam MFE / 3F Actuarial Models Financial Economics Segment Solutions

Exam MFE/3F Solutions Chapter 12 – Monte Carlo Simulation

© ActuarialBrew.com 2014 Page 12.14

Y02= 13.507 [ENTER] Ø

X03= 0.000 [ENTER] Ø

Y03= 0.000 [ENTER] Ø

X04= 0.985 [ENTER] Ø

Y04= 1.000 [ENTER]

[2nd][STAT]

Press the down arrow and note the following statistics:

4.598255.30251.0959 (This is )

x

y

b

To exit the statistics mode: [2nd][QUIT]

Solution 12.20

C Variance and the Control Variate Method

The covariance of the two naïve price estimates is:

r È ˘È ˘= ¥ = ¥ ¥ =Î ˚ Î ˚,[ , ] 0.8 3 5 3.0984A GCov A G Var A Var G

The variance of the control variate estimate for the price of option G is:

[ ] È ˘ È ˘È ˘= + - = + - ¥ =Î ˚Î ˚ Î ˚* 2 , 3 5 2 3.0984 1.8032Var G Var G Var A Cov A G

The reduction in the variance is:

[ ]È ˘ - = - =Î ˚ * 3 1.8032 1.1968Var G Var G

Solution 12.21

E Variance and the Control Variate Method

The variance of the control variate estimate is:

[ ] ( )rÈ ˘= - = - =Î ˚2 2

,* 1 2(1 0.75 ) 0.875

A BVar A Var A

Page 303: ActuarialBrew.com Exam MFE / 3F Actuarial Models …actuarialbrew.com/data/documents/MFE-Solutions-2014-1st-ed.pdf · Exam MFE / 3F Actuarial Models Financial Economics Segment Solutions

Exam MFE/3F Solutions Chapter 12 – Monte Carlo Simulation

© ActuarialBrew.com 2014 Page 12.15

Solution 12.22

B Antithetic Variate Method

The original estimate is based on the final stock prices and their corresponding payoffs:

i 0.5iS Option

Payoff 1 35.50 0.00 2 41.70 1.70 3 37.95 0.00 4 29.41 0.00

The original Monte Carlo estimate for the value of the call option is:

[ ]- - - ¥ -

== = + + + =Â

( ) 0.08 (0.5 0.0)

1

( ) 0 1.70 0 0 0.40834

nr T t

ii

e eV V T

n

The antithetic variate method utilizes each draw from the standard normal distribution twice. To infer the draws from the normal distribution, we use the following formula:

2( 0.5 )( )r T t z T t

T tS S e

For the first stock price of 35.50, we have:

21

1

(0.08 0.00 0.5 0.35 )(0.5 0.0) 0.35 0.5 0.0

0.009375 0.35 0.5

1

35.50 40

35.50 400.52011

z

z

e

ez

For the second stock price of 41.70, we have:

22

2

(0.08 0.00 0.5 0.35 )(0.5 0.0) 0.35 0.5 0.0

0.009375 0.35 0.5

2

41.70 40

41.70 400.13030

z

z

e

ez

For the third stock price of 37.95, we have:

23

3

(0.08 0.00 0.5 0.35 )(0.5 0.0) 0.35 0.5 0.0

0.009375 0.35 0.5

3

37.95 40

37.95 400.25046

z

z

e

ez

For the fourth stock price of 29.41, we have:

24

4

(0.08 0.00 0.5 0.35 )(0.5 0.0) 0.35 0.5 0.0

0.009375 0.35 0.5

4

29.41 40

29.41 401.28055

z

z

e

ez

Page 304: ActuarialBrew.com Exam MFE / 3F Actuarial Models …actuarialbrew.com/data/documents/MFE-Solutions-2014-1st-ed.pdf · Exam MFE / 3F Actuarial Models Financial Economics Segment Solutions

Exam MFE/3F Solutions Chapter 12 – Monte Carlo Simulation

© ActuarialBrew.com 2014 Page 12.16

We take the opposite of each of these values and generate new stock prices. The first new stock price is therefore:

2(0.08 0.00 0.5 0.35 )(0.5 0.0) 0.35(0.52011) 0.5 0.040 45.92e

The other 3 new prices are calculated in a similar manner. The new prices and their associated payoffs appear in the bottom 4 rows of the table below:

i iz 0.5iS Option

Payoff 1 -0.52 35.50 0.00 2 0.13 41.70 1.70 3 -0.25 37.95 0.00 4 -1.28 29.41 0.00 5 0.52 45.92 5.92 6 -0.13 39.10 0.00 7 0.25 42.96 2.96 8 1.28 55.43 15.43

The new antithetic variate estimate for the price of the option is:

( ) 0.08 (0.5 0.0)

1

( ) 0 1.70 0 0 5.92 0 2.96 15.438

3.124382

nr T t

ii

e eV V T

n

The antithetic variate price estimate exceeds the original price estimate by the following amount:

3.124382 0.4083 2.7160

Solution 12.23

C Control Variate Method

Let’s refer to the cash-or-nothing call option as Option Y. We have:

s

==0.5228

0.0046Y

Y

Let’s refer to the regular call option as Option X. We have:

s

=

==

13.36

13.4815

0.1916X

X

X

We can use the control variate price to solve for b :

* ( )

0.5209 0.5228 (13.36 13.4815)0.015638

Y Y X X

Page 305: ActuarialBrew.com Exam MFE / 3F Actuarial Models …actuarialbrew.com/data/documents/MFE-Solutions-2014-1st-ed.pdf · Exam MFE / 3F Actuarial Models Financial Economics Segment Solutions

Exam MFE/3F Solutions Chapter 12 – Monte Carlo Simulation

© ActuarialBrew.com 2014 Page 12.17

We can use b to find the covariance of the naïve price estimates:

2

,

,0.015638

0.1916

, 0.0005741

Cov X Y

Var X

Cov X Y

Cov X Y

bÈ ˘Î ˚=È ˘Î ˚

È ˘Î ˚=

È ˘ =Î ˚

The variance of the control variate estimate is:

[ ] b bÈ ˘ È ˘ È ˘= + -Î ˚ Î ˚ Î ˚

= + ¥ - ¥ ¥=

2

2 2 2

* 2 ,

0.0046 0.015638 0.1916 2 0.015638 0.00057410.0000121827

Var Y Var Y Var X Cov X Y

The standard deviation of the control estimate is the square root of the variance:

=0.0000121827 0.00349

Solution 12.24

D Stratified Sampling

The formula for stratified sampling is:

+ -

= £1

ˆ for ii

u iu i k

k

Since there are 3 intervals, we have = 3k . When i exceeds k, we start over with the second i in the numerator:

1 21 2

3 43 4

5 65 6

1 1 2 10.156 0.695 1ˆ ˆ0.052 0.565

3 3 3 33 1 1 10.788 2 0.188

ˆ ˆ0.929 0.0633 3 3 32 1 3 10.751 1 0.132 2

ˆ ˆ0.584 0.7113 3 3 3

u uu u

u uu u

u uu u

The sum of these numbers is:

0.052 0.565 0.929 0.063 0.584 0.711 2.903

Page 306: ActuarialBrew.com Exam MFE / 3F Actuarial Models …actuarialbrew.com/data/documents/MFE-Solutions-2014-1st-ed.pdf · Exam MFE / 3F Actuarial Models Financial Economics Segment Solutions

Exam MFE/3F Solutions Chapter 12 – Monte Carlo Simulation

© ActuarialBrew.com 2014 Page 12.18

Solution 12.25

B Stratified Sampling

We begin by stratifying the uniform random variables:

+ -= = =

+ - += = =

+ - += = =

+ - += = =

11

22

33

44

1 1 0.152ˆ 0.038

4 42 1 0.696 1

ˆ 0.4244 43 1 0.788 2

ˆ 0.6974 44 1 0.188 3

ˆ 0.7974 4

uu

uu

uu

uu

We convert these draws into draws from the standard normal distribution:

11 1

12 2

13 3

ˆ ˆ(0.038) ( ) (0.03800) 1.77438 1.77438

ˆ ˆ(0.424) ( ) (0.42400) 0.19167 0.19167

ˆ ˆ(0.697) ( ) (0.69700) 0.51579 0.51579

(0.797)

F N z N z

F N z N z

F N z N z

F 14 4ˆ ˆ( ) (0.79700) 0.83095 0.83095N z N z

We use these draws to find the stock prices along the path:

2 21 1 1 1

14 4 4 414

2 21 1 1 114 4 4 4

2 14 4

2 1 114 4

3 244

( 0.5 ) (0.08 0.00 0.5 0.3 ) 0.30 ( 1.77438)0

( 0.5 ) (0.08 0.00 0.5 0.3 ) 0.30 ( 0.19167)

( 0.5 )

40 30.92214

30.92214 30.30983

30.3098

r z

r z

r z

S S e e

S S e e

S S e

d s s

d s s

d s s

- - + - - ¥ + ¥ -

- - + - - ¥ + ¥ -

- - +

= = =

= = =

= =2 1 1

4 4

2 21 1 1 114 4 4 4

34

(0.08 0.00 0.5 0.3 ) 0.30 (0.51579)

( 0.5 ) (0.08 0.00 0.5 0.3 ) 0.30 (0.83095)1

3 33.03576

33.03576 37.7499r z

e

S S e ed s s

- - ¥ + ¥

- - + - - ¥ + ¥

=

= = =

The arithmetic average of the stock prices is:

30.92214 30.30983 33.03576 37.7499

33.00444

S

The payoff of the arithmetic average strike Asian call is:

Average strike Asian call option payoff = Max[ ,0] 37.7499 33.0044

4.7455TS S- = -

=

Page 307: ActuarialBrew.com Exam MFE / 3F Actuarial Models …actuarialbrew.com/data/documents/MFE-Solutions-2014-1st-ed.pdf · Exam MFE / 3F Actuarial Models Financial Economics Segment Solutions

Exam MFE/3F Solutions Chapter 12 – Monte Carlo Simulation

© ActuarialBrew.com 2014 Page 12.19

Solution 12.26

E Normal Random Variables as Quantiles

We convert the draws from the uniform distribution into draws from the standard normal distribution:

11 1

12 2

13 3

ˆ ˆ(0.209) ( ) (0.20900) 0.80990 0.80990

ˆ ˆ(0.881) ( ) (0.88100) 1.18000 1.18000

ˆ ˆ(0.025) ( ) (0.02500) 1.95996 1.95996

F N z N z

F N z N z

F N z N z

We use these draws to find the new stock prices:

2

2

2

2

( 0.5 )( )

(0.12 0.00 0.5 0.30 )3 0.30 ( 0.80990) 33

(0.12 0.00 0.5 0.30 )3 0.30 (1.18000) 33

(0.12 0.00 0.5 0.30 )3 0.30 ( 1.95996) 33

1. 50 41.10734

2. 50 115.60382

3. 50 22.614

T t z T tT tS S e

S e

S e

S e 65

The average of the stock prices is:

41.10734 115.60382 22.61465

59.77533

Solution 12.27

E Stratified Sampling Method

The stratified sampling method assigns the first and sixth uniform (0, 1) random numbers to the segment (0.00, 0.20), the second and seventh uniform (0, 1) random variables to the segment (0.20, 0.40), the third and eighth uniform (0, 1) random variables to the segment (0.40, 0.60), the fourth and the ninth uniform (0, 1) random variables to the segment (0.60, 0.80), and the fifth and the tenth uniform (0,1) random variables to the segment (0.80, 1.00).

The lowest simulated normal random variables will come from the segment (0.00, 0.20). The lower of the two values in this segment is the sixth one: 0.347, so we use it to find the corresponding standard normal random variable:

6

6 6

16

0.347

0.347 (1 1)ˆ 0.06940

5

(0.06940) 1.48027

u

u V

Z N -

=+ -= = =

= = -

Page 308: ActuarialBrew.com Exam MFE / 3F Actuarial Models …actuarialbrew.com/data/documents/MFE-Solutions-2014-1st-ed.pdf · Exam MFE / 3F Actuarial Models Financial Economics Segment Solutions

Exam MFE/3F Solutions Chapter 12 – Monte Carlo Simulation

© ActuarialBrew.com 2014 Page 12.20

The highest simulated normal random variable will come from the segment (0.80, 1.00). The higher of the two values in this segment is the fifth one: 0.782, so we use it to find the corresponding standard normal random variable:

5

5 5

15

0.782

0.782 (5 1)ˆ 0.95640

5

(0.95640) 1.71036

u

u V

Z N -

=+ -= = =

= =

The difference between the largest and smallest simulated normal random variates is:

1.71036 ( 1.48027) 3.19063- - =

Solution 12.28

C Control Variate Method

Let the variable Y be associated with the $52-strike option and the variable X be associated with the $50-strike option. In the regression analysis, Y will be the dependent variable and X will be the independent variable.

The payoffs depend on the simulated stock prices:

Payoff Payoff With With Stock Strike = 50 Strike = 52

Price ( )0.025iX e ( )0.025

iY e

43.30 0.00 0.00 47.30 0.00 0.00 52.00 2.00 0.00 54.00 4.00 2.00 58.90 8.90 6.90

The values in the two rightmost columns are the payoffs, which means that they are the discounted Monte Carlo prices, iX and iY , times 0.10 0.25e .

The estimate for b is:

( )

( )b = =

= =

- - - -

= =

- -

 Â

 Â

2

1 1

22 2

1 1

( )( ) ( )( )

( ) ( )

n nrT

i i i ii i

n nrT

i ii i

Y Y X X e Y Y X X

X X e X X

We get the same estimate for b regardless of whether we use the time 0 prices or the time 0.25 payoffs (as shown in the rightmost expression above). To save time, we use the time 0.25 payoffs from the table above.

Page 309: ActuarialBrew.com Exam MFE / 3F Actuarial Models …actuarialbrew.com/data/documents/MFE-Solutions-2014-1st-ed.pdf · Exam MFE / 3F Actuarial Models Financial Economics Segment Solutions

Exam MFE/3F Solutions Chapter 12 – Monte Carlo Simulation

© ActuarialBrew.com 2014 Page 12.21

We perform a regression using the second column in the table above as the x-values and the third column as the y-values. The resulting slope coefficient is:

b = 0.78251

During the exam, it is more efficient to let the calculator perform the regression and determine the slope coefficient.

Using the TI-30XS Multiview calculator, we first clear the data by pressing [data] [data] ↓↓↓ until Clear ALL is shown, and then press [enter].

Fill out the table as shown below:

L1 L2 L3 0.00 0.00 ----------- 0.00 0.00 2.00 0.00 4.00 2.00 8.90 6.90

Next, press:

[2nd] [stat] 2 (i.e., select 2-Var Stats)

Select L1 for x-data and press [enter].

Select L2 for y-data and press [enter].

Select CALC and then [enter]

Scroll down to: a = 0.7825135017.

Exit the statistics mode by pressing [2nd] [quit].

Since the $50-strike call is our control, we have:

-

-

+ + + += ¥ =

+ + + += ¥ =

0.025

0.025

0 0 2.00 4.00 8.902.9064

50 0 0 2.00 6.90

1.73615

X e

Y e

The estimate for the price of the $52-strike option is:

* (52) * ( ) 1.7361 0.78251(2.67 2.9064) 1.5510C Y Y X X

Solution 12.29

B Control Variate Method

Let the variable Y be associated with the $52-strike option and the variable X be associated with the $50-strike option. In the regression analysis, Y will be the dependent variable and X will be the independent variable.

Page 310: ActuarialBrew.com Exam MFE / 3F Actuarial Models …actuarialbrew.com/data/documents/MFE-Solutions-2014-1st-ed.pdf · Exam MFE / 3F Actuarial Models Financial Economics Segment Solutions

Exam MFE/3F Solutions Chapter 12 – Monte Carlo Simulation

© ActuarialBrew.com 2014 Page 12.22

The payoffs depend on the simulated stock prices:

Payoff Payoff With With Stock Strike = 50 Strike = 52

Price ( )0.025iX e ( )0.025

iY e

43.30 0.00 0.00 47.30 0.00 0.00 52.00 2.00 0.00 54.00 4.00 2.00 58.90 8.90 6.90

The values in the two rightmost columns are the payoffs, which means that they are the discounted Monte Carlo prices, iX and iY , times 0.10 0.25e .

Using the TI-30XS Multiview calculator, we first clear the data by pressing [data] [data] ↓↓↓ until Clear ALL is shown, and then pressing [enter].

Fill out the table as shown below:

L1 L2 L3 0.00 0.00 ----------- 0.00 0.00 2.00 0.00 4.00 2.00 8.90 6.90

Next, press:

[2nd] [stat] 2 (i.e., select 2-Var Stats)

Select L1 for x-data and press [enter].

Select L2 for y-data and press [enter].

Select CALC and then [enter]

Scroll down to obtain the estimates for the standard deviation of the 52-strike option’s payoffs and the correlation coefficient:

2.99032

0.968649

Syr

Exit the statistics mode by pressing [2nd] [quit].

Page 311: ActuarialBrew.com Exam MFE / 3F Actuarial Models …actuarialbrew.com/data/documents/MFE-Solutions-2014-1st-ed.pdf · Exam MFE / 3F Actuarial Models Financial Economics Segment Solutions

Exam MFE/3F Solutions Chapter 12 – Monte Carlo Simulation

© ActuarialBrew.com 2014 Page 12.23

To obtain the standard deviation of the individual values of the 52-strike option, we adjust for the fact that the payoffs occur at time 0.25. Then we find the standard deviation of the naïve estimate:

0.10 0.25 0.0252.99032 2.91649

2.916491.30429

5

Y

YY

Sy e e

n

The variance of the estimate is then:

2 2 2,

* 1 1.30429 1 0.968649 0.10499X Y

Var Y Var Y

The standard deviation of the estimate is:

=0.10499 0.32403

Solution 12.30

E Variance of Control Variate Estimate

The formula from the ActuarialBrew.com study manual that has X as the control variate and *Y as the control variate estimate is:

[ ] ( )rÈ ˘= -Î ˚2

,* 1

X YVar Y Var Y

In this question, however, the control variate is denoted by Y and the control variate estimate is denoted by *X , so we switch the roles of X and Y:

[ ] ( ) ( )rÈ ˘= - = - =Î ˚2 2 2

,* 1 5 1 0.4 21

X YVar X Var X

Page 312: ActuarialBrew.com Exam MFE / 3F Actuarial Models …actuarialbrew.com/data/documents/MFE-Solutions-2014-1st-ed.pdf · Exam MFE / 3F Actuarial Models Financial Economics Segment Solutions

Exam MFE/3F Solutions Chapter 12 – Monte Carlo Simulation

© ActuarialBrew.com 2014 Page 12.24

This page is intentionally blank.

Page 313: ActuarialBrew.com Exam MFE / 3F Actuarial Models …actuarialbrew.com/data/documents/MFE-Solutions-2014-1st-ed.pdf · Exam MFE / 3F Actuarial Models Financial Economics Segment Solutions

Exam MFE/3F Solutions Chapter 13 – Volatility

© ActuarialBrew.com 2014 Page 13.01

Chapter 13 – Solutions

Solution 13.01

C Exercise Boundaries

Answer A is not correct because an increase in the volatility of the stock increases the value of the implicit insurance, thereby making early exercise less attractive.

Answer B is not correct because an increase in the volatility of the stock increases the value of the implicit insurance, thereby making early exercise less attractive.

Answer C is correct because an increase in the volatility of the stock increases the value of the implicit insurance, making early exercise less attractive and increasing the exercise boundary that must be reached in order for early exercise to be optimal.

Answer D is not correct because as an American call option ages, the early-exercise criteria become less stringent, which means that the exercise boundary decreases.

Answer E is not correct because as an American put option ages, the early-exercise criteria become less stringent, which means that the exercise boundary increases.

Solution 13.02

E Exercise Boundaries

For each option, there are 3 effects to consider: the change in volatility, the new stock price, and the passage of time.

A call option is exercised only if the stock’s price is greater than the exercise boundary, and a put option is exercised only if the stock’s price is less than the exercise boundary.

Let’s consider the options in order.

American call option on Stock X

Higher volatility increases the exercise boundary: Less likely exercise

Unchanged price does not affect likelihood of exercise: No effect

Passage of time decreases the exercise boundary: More likely exercise

Net effect: Indeterminate

American call option on Stock Y

Lower volatility decreases the exercise boundary: More likely exercise

Lower price decreases the likelihood of exercise: Less likely exercise

Passage of time decreases the exercise boundary: More likely exercise

Net effect: Indeterminate

Page 314: ActuarialBrew.com Exam MFE / 3F Actuarial Models …actuarialbrew.com/data/documents/MFE-Solutions-2014-1st-ed.pdf · Exam MFE / 3F Actuarial Models Financial Economics Segment Solutions

Exam MFE/3F Solutions Chapter 13 – Volatility

© ActuarialBrew.com 2014 Page 13.02

American call option on Stock Z

Higher volatility increases the exercise boundary: Less likely exercise

Lower price decreases the likelihood of exercise: Less likely exercise

Passage of time decreases the exercise boundary: More likely exercise

Net effect: Indeterminate

American put option on Stock X

Higher volatility decreases the exercise boundary: Less likely exercise

Unchanged price does not affect likelihood of exercise: No effect

Passage of time increases the exercise boundary: More likely exercise

Net effect: Indeterminate

American put option on Stock Y

Lower volatility increases the exercise boundary: More likely exercise

Lower price increases the likelihood of exercise: More likely exercise

Passage of time increases the exercise boundary: More likely exercise

Net effect: More likely exercise

The only option which we can say for sure is more likely to be exercised now is the American put option on Stock Y. Since we are told that only one of the options is optimal to exercise now, that option must be the put option on Stock Y.

Solution 13.03

E Estimating Volatility

Since we have 6 weeks of data, we can calculate 5 weekly returns. Each weekly return is calculated as a continuously compounded rate:

-

Ê ˆ= Á ˜Ë ¯1

ln ii

i

Sr

S

The next step is to calculate the average of the returns:

1==Âk

ii

r

rk

Page 315: ActuarialBrew.com Exam MFE / 3F Actuarial Models …actuarialbrew.com/data/documents/MFE-Solutions-2014-1st-ed.pdf · Exam MFE / 3F Actuarial Models Financial Economics Segment Solutions

Exam MFE/3F Solutions Chapter 13 – Volatility

© ActuarialBrew.com 2014 Page 13.03

The returns and their average are shown in the third column below:

Date Price -

Ê ˆ= Á ˜Ë ¯1

ln ii

i

Sr

S 2( )-ir r

10/11/2006 85 10/18/2006 81 –0.048202 0.004381 10/25/2006 87 0.071459 0.002859 11/01/2006 80 –0.083881 0.010378 11/08/2006 86 0.072321 0.002952 11/15/2006 93 0.078252 0.003632

=r 0.017990

52

1

( )=

- =Â ii

r r 0.024201

The fourth column shows the squared deviations and the sum of squares.

The estimate for the standard deviation of the weekly returns is:

s

s

=-

=-

= =

Â

152

2

1

( )

ˆ1

0.024201ˆ 0.0777844

k

ii

h

r r

k

We adjust the weekly volatility to obtain the annual volatility:

s s= = =1ˆ ˆ 0.077784 52 0.561h h

This problem isn’t very difficult if you are familiar with the statistical function on your calculator. We recommend using the TI-30XS MultiView.

Using the TI-30XS MultiView, the procedure is:

[data] [data] 4 (to clear the data table)

(enter the data below)

L1 L2 L3 85 81 ----------- 81 87 87 80 80 86 86 93

(place cursor in the L3 column)

[data] Æ (to highlight FORMULA)

1 [ln] [data] 2 / [data] 1 ) [enter]

[2nd] [quit] [2nd] [stat] 1

DATA: (highlight L3) FRQ: (highlight one) (select CALC) [enter]

Page 316: ActuarialBrew.com Exam MFE / 3F Actuarial Models …actuarialbrew.com/data/documents/MFE-Solutions-2014-1st-ed.pdf · Exam MFE / 3F Actuarial Models Financial Economics Segment Solutions

Exam MFE/3F Solutions Chapter 13 – Volatility

© ActuarialBrew.com 2014 Page 13.04

3 (to obtain Sx)

¥ 52Sx [enter] The result is 0.560909261

On the TI-30X IIS, the steps are: [2nd][STAT] (Select 1-VAR) [ENTER] [DATA]

X1= 81

ln85

Ê ˆÁ ˜Ë ¯

[ENTER] ØØ (Hit the down arrow twice)

X2= 87

ln81

Ê ˆÁ ˜Ë ¯

[ENTER] ØØ

X3= 80

ln87

Ê ˆÁ ˜Ë ¯

[ENTER] ØØ

X4= 86

ln80

Ê ˆÁ ˜Ë ¯

[ENTER] ØØ

X5= 93

ln86

Ê ˆÁ ˜Ë ¯

[ENTER]

[STATVAR] Æ Æ (Arrow over to Sx)

* (52) [ENTER]

The result is: 0.560909261 To exit the statistics mode: [2nd] [EXITSTAT] [ENTER]

Solution 13.04

D Estimating Volatility

Since we have 7 months of data, we can calculate 6 monthly returns. Each monthly return is a calculated as a continuously compounded rate:

-

Ê ˆ= Á ˜Ë ¯1

ln ii

i

Sr

S

The next step is to calculate the average of the returns:

1==Âk

ii

r

rk

Page 317: ActuarialBrew.com Exam MFE / 3F Actuarial Models …actuarialbrew.com/data/documents/MFE-Solutions-2014-1st-ed.pdf · Exam MFE / 3F Actuarial Models Financial Economics Segment Solutions

Exam MFE/3F Solutions Chapter 13 – Volatility

© ActuarialBrew.com 2014 Page 13.05

The returns and their average are shown in the third column below:

Date Price -

Ê ˆ= Á ˜Ë ¯1

ln ii

i

Sr

S 2( )-ir r

1 100 2 110 0.095310 0.004215 3 112 0.018019 0.000153 4 105 –0.064539 0.009011 5 113 0.073427 0.001852 6 115 0.017544 0.000165 7 120 0.042560 0.000148

=r 0.030387 6

2

1

( )=

- =Â ii

r r 0.015544

The fourth column shows the squared deviations and the sum of squares.

The estimate for the standard deviation of the monthly returns is:

s

s

=-

=-

= =

Â

112

2

1

( )

ˆ1

0.015544ˆ 0.0557575

k

ii

h

r r

k

We adjust the monthly volatility to obtain the annual volatility:

s s= = =1ˆ ˆ 0.055757 12 0.193h h

This problem isn’t very difficult if you are familiar with the statistical function of your calculator.

On the TI-30X IIS, the steps are: [2nd][STAT] (Select 1-VAR) [ENTER] [DATA]

X1= 110

ln100Ê ˆÁ ˜Ë ¯

[ENTER] ØØ (Hit the down arrow twice)

X2= 112

ln110Ê ˆÁ ˜Ë ¯

[ENTER] ØØ

X3= 105

ln112Ê ˆÁ ˜Ë ¯

[ENTER] ØØ

X4= 113

ln105Ê ˆÁ ˜Ë ¯

[ENTER] ØØ

X5= 115

ln113Ê ˆÁ ˜Ë ¯

[ENTER] ØØ

Page 318: ActuarialBrew.com Exam MFE / 3F Actuarial Models …actuarialbrew.com/data/documents/MFE-Solutions-2014-1st-ed.pdf · Exam MFE / 3F Actuarial Models Financial Economics Segment Solutions

Exam MFE/3F Solutions Chapter 13 – Volatility

© ActuarialBrew.com 2014 Page 13.06

X6= 120

ln115Ê ˆÁ ˜Ë ¯

[ENTER]

[STATVAR] Æ Æ (Arrow over to Sx)

(12)¥ [ENTER]

The result is: 0.193149327 To exit the statistics mode: [2nd] [EXITSTAT] [ENTER]

On the BA II Plus calculator, the steps are: [2nd][DATA] [2nd][CLR WORK]

110/100 = LN [ENTER] ØØ (Hit the down arrow twice)

112/110 = LN [ENTER] ØØ

105/112 = LN [ENTER] ØØ

113/105 = LN [ENTER] ØØ

115/113 = LN [ENTER] ØØ

120/115 = LN [ENTER]

[2nd][STAT] (12)¥ =

The result is: 0.19314933

To exit the statistics mode: [2nd][QUIT]

Solution 13.05

D Estimated Standard Deviation

The quickest way to do this problem is to input the data into a calculator.

Using the TI-30X IIS, the procedure is:

[2nd][STAT] (Select 1-VAR) [ENTER]

[DATA]

X1= ( )ln 104 /100.00 [ENTER] ØØ (Hit the down arrow twice)

X2= ( )ln 97 /104 [ENTER] ØØ

X3= ( )ln 95 / 97 [ENTER] ØØ

X4= ( )ln 103 / 95 [ENTER]

[STATVAR] Æ Æ (Arrow over to Sx)

¥ 12 [ENTER]

The result is: 0.229314157

To exit the statistics mode: [2nd] [EXITSTAT] [ENTER]

Page 319: ActuarialBrew.com Exam MFE / 3F Actuarial Models …actuarialbrew.com/data/documents/MFE-Solutions-2014-1st-ed.pdf · Exam MFE / 3F Actuarial Models Financial Economics Segment Solutions

Exam MFE/3F Solutions Chapter 13 – Volatility

© ActuarialBrew.com 2014 Page 13.07

Alternatively, using the BA II Plus calculator, the procedure is:

[2nd][DATA] [2nd][CLR WORK]

X01= 104/100 = LN [ENTER] ØØ (Hit the down arrow twice)

X02= 97/104 = LN [ENTER] ØØ

X03= 95/97 = LN [ENTER] ØØ

X04= 103/95 = LN [ENTER]

[2nd][STAT] ØØØ ¥ 12 =

The result is: 0.22931416

To exit the statistics mode: [2nd][QUIT]

Solution 13.06

D Estimated Parameters of the Lognormal Distribution

The quickest way to do this problem is to input the data into a calculator.

Using the TI-30X IIS, the procedure is:

[2nd][STAT] (Select 1-VAR) [ENTER]

[DATA]

X1= ( )ln 104 /100.00 ØØ (Hit the down arrow twice)

X2= ( )ln 97 /104 ØØ

X3= ( )ln 95 / 97 ØØ

X4= ( )ln 103 / 95 [ENTER]

[STATVAR] Æ Æ (Arrow over to Sx)

¥ 12 [ENTER] (The result is 0.229314157)

[STATVAR] Æ (Arrow over to x )

¥ 12 [ENTER] (The result is 0.088676407)

To exit the statistics mode: [2nd] [EXITSTAT] [ENTER]

Alternatively, using the BA II Plus calculator, the procedure is:

[2nd][DATA] [2nd][CLR WORK]

X01= 104/100 = LN [ENTER] ØØ (Hit the down arrow twice)

X02= 97/104 = LN [ENTER] ØØ

X03= 95/97 = LN [ENTER] ØØ

Page 320: ActuarialBrew.com Exam MFE / 3F Actuarial Models …actuarialbrew.com/data/documents/MFE-Solutions-2014-1st-ed.pdf · Exam MFE / 3F Actuarial Models Financial Economics Segment Solutions

Exam MFE/3F Solutions Chapter 13 – Volatility

© ActuarialBrew.com 2014 Page 13.08

X04= 103/95 = LN [ENTER]

[2nd][STAT] ØØØ ¥ 12 = (The result is 0.22931416)

[2nd][STAT] ØØ ¥ 12 = (The result is 0.08867641)

To exit the statistics mode: [2nd][QUIT]

Therefore, the annualized estimates for the mean and standard deviation of the normal distribution are:

a d ss- - ==

2ˆ ˆ0.5 0.088676ˆ 0.22931

The estimate for the annualized expected return is:

a d s= + + = + + ¥ =2 2ˆ ˆ0.5 0.088676 0 0.5 (0.22931) 0.114969rh

We know that the stock price at the end of month 4 is $103. The expected value of the stock at the end of month 12 is:

a d- -

- - ¥

=

= = =412

( )( )

(0.114969 0)(1 4 /12) 0.114969 2 / 31

[ ]

[ ] 103 111.20

T tT tE S S e

E S S e e

Solution 13.07

B Estimated Parameters of the Lognormal Distribution

The quickest way to do this problem is to input the data into a calculator.

Using the TI-30XS MultiView, the procedure is:

[data] [data] 4 (to clear the data table)

(enter the data below)

L1 L2 L3 50.00 54.00 ----------- 54.00 48.00 48.00 55.00 55.00 61.00 61.00 56.00 56.00 54.00

(place cursor in the L3 column) [data] Æ (to highlight FORMULA)

1 [ln] [data] 2 / [data] 1 ) [enter]

[2nd] [quit] [2nd] [stat] 1

Data: (highlight L3) FRQ: (highlight one) [enter]

2 (to obtain x )

Page 321: ActuarialBrew.com Exam MFE / 3F Actuarial Models …actuarialbrew.com/data/documents/MFE-Solutions-2014-1st-ed.pdf · Exam MFE / 3F Actuarial Models Financial Economics Segment Solutions

Exam MFE/3F Solutions Chapter 13 – Volatility

© ActuarialBrew.com 2014 Page 13.09

¥12x [enter] The result is 0.153922082

[2nd] [stat] 3

3 (to obtain Sx)

¥ 12Sx [enter] The result is 0.368883615

Using the TI-30X IIS, the procedure is:

[2nd][STAT] (Select 1-VAR) [ENTER]

[DATA]

X1= ln(54/50) [ENTER] ØØ (Hit the down arrow twice)

X2= ln(48/54) [ENTER] ØØ

X3= ln(55/48) [ENTER] ØØ

X4= ln(61/55) [ENTER] ØØ

X5= ln(56/61) [ENTER] ØØ

X6= ln(54/56) [ENTER]

[STATVAR] Æ Æ (Arrow over to Sx)

¥ 12 [ENTER] (The result is 0.368883615)

[STATVAR] Æ (Arrow over to x )

¥ 12 [ENTER] (The result is 0.153922082)

To exit the statistics mode: [2nd] [EXITSTAT] [ENTER]

Alternatively, using the BA II Plus calculator, the procedure is:

[2nd][DATA] [2nd][CLR WORK]

X01= 54/50 = LN [ENTER] ØØ (Hit the down arrow twice)

X02= 48/54 = LN [ENTER] ØØ

X03= 55/48 = LN [ENTER] ØØ

X04= 61/55 = LN [ENTER] ØØ

X05= 56/61 = LN [ENTER] ØØ

X06= 54/56 = LN [ENTER]

[2nd][STAT] ØØØ ¥ 12 = (The result is 0.36888361)

[2nd][STAT] ØØ ¥ 12 = (The result is 0.15392208)

To exit the statistics mode: [2nd][QUIT]

Page 322: ActuarialBrew.com Exam MFE / 3F Actuarial Models …actuarialbrew.com/data/documents/MFE-Solutions-2014-1st-ed.pdf · Exam MFE / 3F Actuarial Models Financial Economics Segment Solutions

Exam MFE/3F Solutions Chapter 13 – Volatility

© ActuarialBrew.com 2014 Page 13.10

Therefore, the annualized estimates for the mean and standard deviation of the normal distribution are:

2ˆ ˆ0.5 0.153922ˆ 0.368884

The estimate for the annualized expected return is:

a d s= + + = + + ¥ =2 2ˆ ˆ0.5 0.153922 0 0.5 (0.368884) 0.221960rh

Solution 13.08

E Annualized Expected Return

First, we determine the weekly mean and the weekly standard deviation using a calculator, and then we determine the annualized standard deviation and the annualized expected return.

Using the TI-30X IIS calculator, press [2nd] [STAT] and choose 1-VAR and press [ENTER] [DATA]. Perform the following sequence:

X1 = LN(51.0/50.5) [ENTER] ↓↓ (Hit the down arrow twice)

X2 = LN(52.0/51.0) [ENTER] ↓↓

X3 = LN(51.5/52.0) [ENTER] ↓↓

X4 = LN(51.0/51.5) [ENTER] ↓↓

X5 = LN(52.0/51.0) [ENTER] ↓↓

X6 = LN(52.5/52.0) [ENTER]

Press [STATVAR] and → and the weekly mean is shown as 0.0064733. Press → again and the weekly standard deviation is shown as 0.01326559. Multiply this number by the square root of 52 to get the annualized standard deviation of 0.09566. To exit the statistics mode, press [2nd] [EXITSTAT] [ENTER].

To determine the annualized expected return, we multiply the weekly mean return by 52 and we add to that amount one half of the annualized variance:

20.006473 52 0.5 0.09566 0.34119

Page 323: ActuarialBrew.com Exam MFE / 3F Actuarial Models …actuarialbrew.com/data/documents/MFE-Solutions-2014-1st-ed.pdf · Exam MFE / 3F Actuarial Models Financial Economics Segment Solutions

Exam MFE/3F Solutions Chapter 13 – Volatility

© ActuarialBrew.com 2014 Page 13.11

Alternatively, we could use the BA II Plus Professional calculator. Press [2nd] [DATA] [2nd] [CLR WORK], and then perform the following sequence:

X01 = 51.0/50.5 = LN [ENTER] ↓↓

X02 = 52.0/51.0 = LN [ENTER] ↓↓

X03 = 51.5/52.0 = LN [ENTER] ↓↓

X04 = 51.0/51.5 = LN [ENTER] ↓↓

X05 = 52.0/51.0 = LN [ENTER] ↓↓

X06 = 52.5/52.0 = LN [ENTER]

Press [2nd] [STAT] and [2nd] [SET] until you see 1-V. Then press ↓ to see that 6 data points have been entered. Press ↓ again and the weekly mean is shown as 0.0064733. Press ↓ again and the weekly standard deviation is shown as 0.0132656. Multiply this number by the square root of 52 to get the annualized standard deviation of 0.09566. To exit the statistics mode, press [2nd] [QUIT].

To determine the annualized expected return, we multiply the weekly mean return by 52 and we add to that amount one half of the annualized variance:

20.006473 52 0.5 0.09566 0.34119

Solution 13.09

C Volatility Skew

There is no such thing as a volatility wink.

The speculator wants the volatility to be low for the option she is purchasing and high for the option she is selling. Therefore, she wants the volatility to be low when the strike price is high, and she wants the volatility to be high when the strike price is low. This describes a volatility smirk.

Solution 13.10

E Historical Volatility

There is not enough information available for us to determine r , so we cannot estimate the volatility as:

k2

ii 1

(r r )

ˆh(k 1)

s =

-

=-

Â

Therefore, we’ll use the alternative method explained in Chapter 13 of the ActuarialBrew.com Study Manual.

Page 324: ActuarialBrew.com Exam MFE / 3F Actuarial Models …actuarialbrew.com/data/documents/MFE-Solutions-2014-1st-ed.pdf · Exam MFE / 3F Actuarial Models Financial Economics Segment Solutions

Exam MFE/3F Solutions Chapter 13 – Volatility

© ActuarialBrew.com 2014 Page 13.12

The original volatility estimate is based on 10 prices, so there are 9 returns and = 9k :

s =

=

=

= ¥-

= ¥-

=

Â

Â

Â

2

1

92

1

92

1

( )1ˆ

1

( )1

0.229 11/ 252

( ) 0.0015365

k

ii

H

ii

ii

r

kh

r

r

After a week passes, the analyst can add 5 more returns into the sum of squared returns:

= = = == + = +

È ˘ È ˘ È ˘ È ˘ È ˘Ê ˆ Ê ˆ Ê ˆ Ê ˆ Ê ˆ= + + + + +Í ˙ Í ˙ Í ˙ Í ˙ Í ˙Á ˜ Á ˜ Á ˜ Á ˜ Á ˜Ë ¯ Ë ¯ Ë ¯ Ë ¯ Ë ¯Î ˚ Î ˚ Î ˚ Î ˚ Î ˚= +=

   Â14 9 14 14

2 2 2 2

1 1 10 102 2 2 2 2

( ) ( ) ( ) 0.0015365 ( )

102 104 106 105 1010.0015365 ln ln ln ln ln

100 102 104 106 105

0.0015365 0.00273040

i i i ii i i i

r r r r

.0042669

Now we can calculate the volatility using all 15 prices. There are 14 returns, so = 14k :

s = == ¥ = ¥ = ¥ =- - -

 Â14

2 2

1 1

( ) ( )1 1 1 0.0042669ˆ 0.2876

1 14 1 14 11/ 252 1/ 252

k

i ii i

H

r r

kh

Solution 13.11

D Implied Volatility

If the volatility is 30%, then 1( )N d- and 2( )N d- are calculated as shown below:

1( ) ( 0.21670) 0.41422N d N

2( ) (0.08330) 0.53319N d N

If the volatility is 30%, then the price of the put option is:

2 10.08(1) 0.06(1)

( , , , , , ) ( ) ( )

35 0.53319 35 0.41422 3.57345

rT TEurP S K r T Ke N d Se N d

e e

Since the calculated price of $3.57 matches the observed price of the option, the implied volatility is 30%.

Page 325: ActuarialBrew.com Exam MFE / 3F Actuarial Models …actuarialbrew.com/data/documents/MFE-Solutions-2014-1st-ed.pdf · Exam MFE / 3F Actuarial Models Financial Economics Segment Solutions

Exam MFE/3F Solutions Chapter 13 – Volatility

© ActuarialBrew.com 2014 Page 13.13

The prices for the rest of the volatilities are shown in the table below:

s 1d 2d 1( )N d- 2( )N d- Price 0.15 0.20833 0.05830 0.41750 0.47675 1.64 0.20 0.20000 0.00000 0.42074 0.50000 2.29 0.25 0.20500 –0.04500 0.41879 0.51795 2.93 0.30 0.21670 –0.08330 0.41422 0.53319 3.57 0.35 0.23210 –0.11790 0.40823 0.54693 4.21

To answer the question, it is not necessary to calculate more than 3 of the prices of the table. We can begin by calculating the price that corresponds to the middle volatility of 25%. Since that price is $2.93, which is too low, we choose a higher volatility next. That way, we don’t need to calculate the prices that correspond to the 15% and 20% volatilities.

Page 326: ActuarialBrew.com Exam MFE / 3F Actuarial Models …actuarialbrew.com/data/documents/MFE-Solutions-2014-1st-ed.pdf · Exam MFE / 3F Actuarial Models Financial Economics Segment Solutions

Exam MFE/3F Solutions Chapter 13 – Volatility

© ActuarialBrew.com 2014 Page 13.14

This page is intentionally blank.

Page 327: ActuarialBrew.com Exam MFE / 3F Actuarial Models …actuarialbrew.com/data/documents/MFE-Solutions-2014-1st-ed.pdf · Exam MFE / 3F Actuarial Models Financial Economics Segment Solutions

Exam MFE/3F Solutions Chapter 14 – Brownian Motion

© ActuarialBrew.com 2014 Page 14.01

Chapter 14 – Solutions

Solution 14.01

E Diffusion Process

A stochastic process is a diffusion process if its future values do not depend on the path taken to reach the current value. All of the processes listed in the question have this property.

Solution 14.02

B Multiplication Rules

The geometric Brownian motion process is described below:

= +( ) 0.05 ( ) 0.08 ( ) ( )dX t X t dt X t dZ t

We then square ( )dX t :

[ ] [ ]

[ ] [ ] [ ] [ ]= +

= + +

2 2

2 2 2 22

( ) 0.05 ( ) 0.08 ( ) ( )

0.0025 ( ) ( ) 0.008 ( ) ( ) 0.0064 ( ) ( )

dX t X t dt X t dZ t

X t dt X t dtdZ t X t dZ t

Now we use the multiplication rules.

The rule that =2( ) 0dt means that the first term is 0.

The rule that ¥ = 0dt dZ means that the second term is 0.

The rule that =2( )dZ dt applies to the third term.

Applying the rules, we have:

[ ] [ ]=2 2( ) 0.0064 ( )dX t X t dt

Solution 14.03

A Prepaid Forward Price of $1

We don’t need the information provided in statement (i) to answer this question.

The usual formula for a prepaid forward is:

d-=0, ( ) (0)P TTF S S e

Let’s find the prepaid forward price of $1, in euros. From the perspective of a euro-denominated investor, the initial asset price is:

=1 1(0) 1.40x

From the perspective of a euro-denominated investor, the dividend yield of $1 is 5%.

Page 328: ActuarialBrew.com Exam MFE / 3F Actuarial Models …actuarialbrew.com/data/documents/MFE-Solutions-2014-1st-ed.pdf · Exam MFE / 3F Actuarial Models Financial Economics Segment Solutions

Exam MFE/3F Solutions Chapter 14 – Brownian Motion

© ActuarialBrew.com 2014 Page 14.02

The prepaid forward price of one dollar, in euros, is therefore:

- ¥ - ¥= =0.05 3 0.05 31 10.61479

(0) 1.40e e

x

The prepaid forward price of $1,000, in euros, is:

¥ =0.61479 1,000 614.79

Solution 14.04

A Geometric Brownian Motion Equivalencies

When the price follows geometric Brownian motion, the natural log of the price follows arithmetic Brownian motion:

[ ] ( )a s a s s= + ¤ = - +2( ) ( ) ( ) ( ) ln ( ) 0.5dS t S t dt S t dZ t d S t dt dZ

We have:

0.5

2

2

ln 0.5ln

[ln ] 0.5 [ln ] 0.5 (0.08 0.5 0.18 ) 0.18 0.0319 0.09

(0.0319 0.5 0.09 ) 0.09 0.03595 0.09

C SC S

d C d S dt dZ dt dZ

dC Cdt CdZ Cdt CdZ

==

È ˘= = - ¥ + = +Î ˚

= + ¥ + = +

Alternative Solution

Since the stock is governed by geometric Brownian motion, we can use the following formula from Chapter 15:

2

2

( )( ) 0.5 ( 1)

( ) ( ) 0.5 ( 1)

d Sdt dZ

S

d S S dt S dZ

a

a

a a a

a a a a

a a a a

We have:

a d s= - = =0.5 0.08 0.18a

Putting these values into the formula gives us:

( ) ( )( )

= + - +

= - + = +

= +

0.5 2 0.5 0.50.5(0.08) 0.5(0.5)( 0.5)(0.18) (0.5)(0.18)

0.04 0.00405 0.09 0.03595 0.09

0.03595 0.09

d S S dt S dZ

Sdt SdZ Sdt SdZ

Cdt CdZ

Page 329: ActuarialBrew.com Exam MFE / 3F Actuarial Models …actuarialbrew.com/data/documents/MFE-Solutions-2014-1st-ed.pdf · Exam MFE / 3F Actuarial Models Financial Economics Segment Solutions

Exam MFE/3F Solutions Chapter 14 – Brownian Motion

© ActuarialBrew.com 2014 Page 14.03

Solution 14.05

E Geometric Brownian Motion Equivalencies

When the price follows geometric Brownian motion, the natural log of the price follows arithmetic Brownian motion:

[ ] ( )a s a s s= + ¤ = - +2( ) ( ) ( ) ( ) ln ( ) 0.5dS t S t dt S t dZ t d S t dt dZ

We have:

==

È ˘= = - ¥ + = +Î ˚

= + ¥ + = +

0.5

2

2

ln 0.5 ln

[ln ] 0.5 [ln ] 0.5 (0.07 0.5 0.20 ) 0.20 0.025 0.10

(0.025 0.5 0.10 ) 0.10 0.03 0.10

y Sy S

d y d S dt dZ dt dZ

dy ydt ydZ ydt ydZ

Solution 14.06

D Geometric Brownian Motion Equivalencies

A lognormal stock price implies that changes in the stock price follow geometric Brownian motion, and vice versa:

a d s s a d s- - += ¤ = - +2( 0.5 ) ( )( ) (0) ( ) ( ) ( ) ( ) ( )t Z tS t S e dS t S t dt S t dZ t

We can use the equation provided in the question to find values for a d-( ) and s :

s

a d s

a d

a d

+==

= - -

= - -

- = + =

0.035 0.3 ( )

2

2

2

( ) 200.3

0.035 0.5

0.035 0.5(0.3 )

0.035 0.5(0.3 ) 0.08

t Z tS t e

The solution is:

a d s= - + = +( ) ( ) ( ) ( ) ( ) 0.08 ( ) 0.3 ( ) ( )dS t S t dt S t dZ t S t dt S t dZ t

Solution 14.07

B Geometric Brownian Motion Equivalencies

A lognormal stock price implies that changes in the stock price follow geometric Brownian motion, and vice versa:

a d s s a d s- - += ¤ = - +2( 0.5 ) ( )( ) (0) ( ) ( ) ( ) ( ) ( )t Z tS t S e dS t S t dt S t dZ t

Page 330: ActuarialBrew.com Exam MFE / 3F Actuarial Models …actuarialbrew.com/data/documents/MFE-Solutions-2014-1st-ed.pdf · Exam MFE / 3F Actuarial Models Financial Economics Segment Solutions

Exam MFE/3F Solutions Chapter 14 – Brownian Motion

© ActuarialBrew.com 2014 Page 14.04

We can use the equation provided in the question to find values for a d-( ) and s :

s

a d s

a d

a d

+ += fi =

==

= - -

= - -

- = + =

0.20 0.05 0.4 ( ) 0.20 0.05 0.4 ( )

0.20

2

2

2

( ) ( )

(0)0.4

0.05 0.5

0.05 0.5(0.4 )

0.05 0.5(0.4 ) 0.13

t Z t t Z tS t e e S t e e

S e

The solution is:

( ) ( ) ( ) ( ) 0.13 ( ) 0.4 ( ) ( )dS t S t dt S t dZ S t dt S t dZ ta d s= - + = +

Solution 14.08

E Geometric Brownian Motion Equivalencies

When the price follows geometric Brownian motion, the natural log of the price follows arithmetic Brownian motion:

[ ] ( )a s a s s= + ¤ = - +2( ) ( ) ( ) ( ) ln ( ) 0.5dS t S t dt S t dZ t d S t dt dZ

We have:

È ˘= = - ¥ + = +Î ˚

= + ¥ + = +

2 2

2 2 2 2 2 2

[ln ] 2 [ln ] 2 (0.09 0.5 0.25 ) 0.25 0.1175 0.5

(0.1175 0.5 0.5 ) 0.5 0.2425 0.5

d S d S dt dZ dt dZ

dS S dt S dZ S dt S dZ

Solution 14.09

D Ornstein-Uhlenbeck Process

The general form of the Ornstein-Uhlenbeck process is:

l a s= ¥ - +( ) [ ( )] ( )dX t X t dt dZ t

With a = 0 , the Ornstein-Uhlenbeck process can be written as:

l s

sl

= ¥ - +

= - +

( ) [0 ( )] ( )( )

( )( ) ( )

dX t X t dt dZ tdX t

dt dZ tX t X t

Answer D is in this form with:

ls==

0.4

0.20

Page 331: ActuarialBrew.com Exam MFE / 3F Actuarial Models …actuarialbrew.com/data/documents/MFE-Solutions-2014-1st-ed.pdf · Exam MFE / 3F Actuarial Models Financial Economics Segment Solutions

Exam MFE/3F Solutions Chapter 14 – Brownian Motion

© ActuarialBrew.com 2014 Page 14.05

Solution 14.10

A Geometric Brownian Motion Equivalencies

This question can also be be answered using Itô’s Lemma, from Chapter 15 of the Study Manual.

Since we are given the risk-free rate in both the U.S. and Great Britain, we have:

- = - = -* 0.06 0.09 0.03r r

The forward price follows geometric Brownian motion:

- - - + - -

- - +

- +

= =

=

=

2

2

2

0.03( ) [0.05 0.5(0.30) ] 0.3 ( ) 0.03( )

0.03 [0.05 0.5(0.30) ] 0.3 ( ) 0.03

[0.08 0.5(0.30) ] 0.3 ( )

( ) ( ) (0)

(0)

(0)

T t t Z t T t

T t Z t t

t Z t

G t S t e S e e

S e e e

G e

Since the forward price follows geometric Brownian motion, we can use the following equivalency:

a d s sa d s - - += - + ¤ =2ˆˆ ˆ ˆ( 0.5 ) [ ( )]ˆˆ ˆ( ) ( ) ( ) ( ) ( ) G( ) (0) t Z tdG t G t dt G t dZ t t G e

We use the ^ symbol above to avoid confusion with the parameters for S(t).

Since we have the rightmost expression above, we can express the left side as:

[ ]= + = +( ) 0.08 ( ) 0.3 ( ) ( ) ( ) 0.08 0.3 ( )dG t G t dt G t dZ t G t dt dZ t

Alternative Solution (Using Itô’s Lemma)

The expression for ( )G t is therefore:

- -= 0.03( )( ) ( ) T tG t S t e

The partial derivatives are:

- -

-- - -

==

È ˘∂ Î ˚= = =∂

0.03( )

0.03 0.030.03 0.03 0.03( )

0

( )( ) (0.03) ( )(0.03)

T tS

SS

T tT t T t

t

G e

G

S t e eG S t e e S t e

t

Page 332: ActuarialBrew.com Exam MFE / 3F Actuarial Models …actuarialbrew.com/data/documents/MFE-Solutions-2014-1st-ed.pdf · Exam MFE / 3F Actuarial Models Financial Economics Segment Solutions

Exam MFE/3F Solutions Chapter 14 – Brownian Motion

© ActuarialBrew.com 2014 Page 14.06

From Itô’s Lemma, we have:

[ ][ ]

- - - -

- -

- -

= + +

= + +

= +

= + +

= + +

=

2

0.03( ) 2 0.03( )

0.03( )

0.03( )

1( ) ( ) [ ( )]

21

( ) (0)[ ( )] ( )(0.03)2

( ) ( )(0.03)

( ) 0.05 0.3 ( ) ( )(0.03)

( ) 0.05 0.3 ( ) ( )(0.03)

( ) 0

S SS t

T t T t

T t

T t

dG t G dS t G dS t G dt

e dS t dS t S t e dt

e dS t G t dt

e S t dt dZ t G t dt

G t dt dZ t G t dt

G t [ ]+.08 0.3 ( )dt dZ t

Solution 14.11

A Black-Scholes Framework

Statement (i) is true, because when the Black-Scholes framework applies, the stock prices are lognormally distributed:

( )a s s+ - - 2 2ln ( ) ln ( ) ( 0.5 ) , S t h S t N h h

If we are given ( )S t , this implies:

( )a s s

s

+ + -

È ˘+ =Î ˚

2 2

2

ln ( ) ln ( ) ( 0.5 ) ,

ln( ( ) ( )

S t h N S t h h

Var S t h S t h

Statement (ii) is false, because when the Black-Scholes framework applies, the stock prices follow geometric Brownian motion:

a s= +( )( )

( )dS t

dt dZ tS t

The variance is:

a s s sÈ ˘ È ˘ È ˘= + = + =Í ˙ Î ˚ Î ˚Î ˚2 2( )

( ) ( ) ( ) 0 ( ) ( )( )

dS tVar S t Var dt dZ t Z t Var dZ t Z t dt

S t

Statement (iii) is false, because when the Black-Scholes framework applies, the stock prices follow geometric Brownian motion:

a s= +( ) ( ) ( ) ( )dS t S t dt S t dZ t

Therefore, we have:

[ ][ ] [ ]a s s

s s

È ˘ È ˘ È ˘+ = + = +Î ˚ Î ˚ Î ˚

È ˘ È ˘= + = +Î ˚ Î ˚

= =

22

2 22 2

( ) ( ) ( ) ( ) ( ) 0 ( ) ( )

( ) ( ) ( ) ( ) 0 ( ) ( ) ( )

( ) ( )

Var S t dt S t Var S t dS t S t Var dS t S t

Var S t dt S t dZ t Z t S t Var dZ t Z t

S t dt S t dt

Page 333: ActuarialBrew.com Exam MFE / 3F Actuarial Models …actuarialbrew.com/data/documents/MFE-Solutions-2014-1st-ed.pdf · Exam MFE / 3F Actuarial Models Financial Economics Segment Solutions

Exam MFE/3F Solutions Chapter 14 – Brownian Motion

© ActuarialBrew.com 2014 Page 14.07

Solution 14.12

C Properties of Brownian Motion

Statement I is true. The quadratic variation of the pure Brownian motion process from time 0 to time T is equal to T.

Statement II is true. Since the quadratic variation is equal to T, it is finite.

Statement III is false. The total variation is infinite.

Solution 14.13

E Probability

The stock price follows geometric Brownian motion with:

as= - ==

0.10 0.02 0.080.34

The stock prices are lognormally distributed:

[ ]( )[ ]( )

( )

a s s+ -

+ - ¥ ¥

2 2

2 2

ln ( ) ln (0) ( 0.5 ) ,

ln (0.25) ln 30 (0.08 0.5 0.34 )(0.25), 0.34 0.25

ln (0.25) 3.4067, 0.0289

S t N S t t

S N

S N

The probability that (0.25)S is less than $32 is:

[ ] [ ]

[ ]

Prob (0.25) 32 Prob ln (0.25) ln32

ln (0.25) 3.4067 ln32 3.4067Prob

0.0289 0.0289

Prob 0.34699 0.63570

S S

S

Z

< = <

È ˘- -= <Í ˙Î ˚

= < =

Solution 14.14

C Geometric Brownian Motion Equivalencies

When a process follows geometric Brownian motion, the natural log of the process follows arithmetic Brownian motion:

[ ]a b a b b= + ¤ = - +2( ) ( ) ( ) ( ) ln ( ) ( 0.5 )dS t S t dt S t dZ t d S t dt dZ

We have:

( )

2

2 2

2 2 2

ln 2ln

ln 2 ln 2 ( 0.5 ) 2( 0.5 ) 2

[2( 0.5 ) 0.5 (2 ) ] 2 (2 ) 2

Y SY S

d Y d S dt dZ dt dZ

dYdt dZ dt dZ

Y

a b b a b b

a b b b a b b

==

È ˘= = - + = - +Î ˚

= - + ¥ + = + +

Page 334: ActuarialBrew.com Exam MFE / 3F Actuarial Models …actuarialbrew.com/data/documents/MFE-Solutions-2014-1st-ed.pdf · Exam MFE / 3F Actuarial Models Financial Economics Segment Solutions

Exam MFE/3F Solutions Chapter 14 – Brownian Motion

© ActuarialBrew.com 2014 Page 14.08

This implies that:

m a b s b= + =22 and 2

Consequently:

m a bs b

+=22

2

Solution 14.15

A Geometric Brownian Motion Equivalencies

When a process follows geometric Brownian motion, the natural log of the process follows arithmetic Brownian motion:

[ ]m s m s s= + ¤ = - +2( ) ( ) ( ) ( ) ln ( ) ( 0.5 )dX t X t dt X t dZ t d X t dt dZ

We have:

m s s m s s

m s s s m s s

==

È ˘= = - + = - +Î ˚

È ˘= - + ¥ + = + +Î ˚

3

2 2

2 2 2

ln 3ln

ln 3 [ln ] 3 ( 0.5 ) 3( 0.5 ) 3

3( 0.5 ) 0.5 (3 ) 3 (3 3 ) 3

Y XY X

d Y d X dt dZ dt dZ

dYdt dZ dt dZ

Y

Therefore:

m s m sa

b s s+ +

= =2 23 3

3

Solution 14.16

A Stochastic Differential Equations

The 2t within the integral can be pulled out of the integral:

= + = +Ú Ú3 2 3 3

0 0

( ) 6 ( ) 6 ( )t t

X t t t t sdZ s t t sdZ s

The first term of the differential is easily obtained:

È ˘Í ˙= +Í ˙Î ˚Ú2 3

0

( ) 18 ( )t

dX t t dt d t sdZ s

Page 335: ActuarialBrew.com Exam MFE / 3F Actuarial Models …actuarialbrew.com/data/documents/MFE-Solutions-2014-1st-ed.pdf · Exam MFE / 3F Actuarial Models Financial Economics Segment Solutions

Exam MFE/3F Solutions Chapter 14 – Brownian Motion

© ActuarialBrew.com 2014 Page 14.09

For the second term, we use the product rule:

[ ]È ˘ È ˘ È ˘Í ˙ Í ˙ Í ˙= + = +Í ˙ Í ˙ Í ˙Î ˚ Î ˚ Î ˚Ú Ú Ú3 2 3 2 4

0 0 0

( ) 3 ( ) ( ) 3 ( ) ( )t t t

d t sdZ s t dt sdZ s t tdZ t t dt sdZ s t dZ t

We can now write ( )dX t as:

È ˘ È ˘Í ˙ Í ˙= + = + +Í ˙ Í ˙Î ˚ Î ˚

Ï ¸ +Ô Ô= + + = + =Ì ˝Ô ÔÓ ˛

Ú Ú

Ú

2 3 2 2 4

0 0

53 3 4 4

0

( ) 18 ( ) 18 3 ( ) ( )

3 3 3 ( ) ( )6 ( ) ( ) ( ) ( )

t t

t

dX t t dt d t sdZ s t dt t dt sdZ s t dZ t

X t dt t dZ tt t sdZ s dt t dZ t X t dt t dZ t

t t t

Solution 14.17

D Geometric Brownian Motion and Mutual Funds

The instantaneous percentage increase of the mutual fund is the weighted average of the return on the stock (including its dividend yield) and the return on the risk-free asset, minus the dividend yield of the mutual fund:

[ ]

d dÈ ˘= + + - - ¥Í ˙Î ˚= + + + -= + + -= +

Mutual Fund( ) ( )

(1 )( ) ( )

0.70 0.07 0.20 ( ) 0.05 0.30(0.06) 0.03

0.084 0.14 ( ) 0.018 0.030.072 0.14 ( )

dW t dS tb dt b rdt dt

W t S t

dt dZ t dt dt dt

dt dZ t dt dtdt dZ t

This can be rewritten as:

= +( ) 0.072 ( ) 0.14 ( ) ( )dW t W t dt W t dZ t

The expression above matches Choice (D).

Solution 14.18

E Probability

Using the geometric Brownian motion equivalencies, we can express the Brownian motion for ( )Y t in the form at right below:

2 ( )ln ( ) 0.235 0.5 ( ) 0.235 ( )

( )dY t

d Y t dt dZ t dt dZ tY t

Page 336: ActuarialBrew.com Exam MFE / 3F Actuarial Models …actuarialbrew.com/data/documents/MFE-Solutions-2014-1st-ed.pdf · Exam MFE / 3F Actuarial Models Financial Economics Segment Solutions

Exam MFE/3F Solutions Chapter 14 – Brownian Motion

© ActuarialBrew.com 2014 Page 14.10

Both stocks have only ( )Z t as their source of randomness. Therefore, they have the same market price of risk:

2

2

2

0.4 0.085 0.235 0.0850.2

0.4 0.085 0.03

0.4 0.085 0.03 0

0.085 ( 0.085) 4(0.4)( 0.085)2 0.4

0.1875 or 0.40

bb

b b

b b

b

b b

- -=

- =

- - =

± - - -=

¥= - =

Since we are given that b > 0 , we have b = 0.40 . We can now express the processes for

( )X t and ( )Y t as:

( ) 0.16 ( ) 0.2 ( ) ( )( ) 0.235 ( ) 0.4 ( ) ( )

dX t X t dt X t dZ tdY t Y t dt Y t dZ t

The geometric Brownian motion equivalencies tell us that the stochastic differential equations above imply:

2

2

0.16 0.5(0.2) 0.2 ( ) 0.14 0.2 ( )

0.235 0.5(0.4) 0.4 ( ) 0.155 0.4 ( )

( ) (0) 10

( ) (0) 12

t Z t t Z t

t Z t t Z t

X t X e e

Y t Y e e

The probability that (2)X is greater than (2)Y is:

[ ]

[ ] [ ]

0.155 2 0.4 (2) 0.14 2 0.2 (2)

0.31 0.4 (2) 0.28 0.2 (2) 0.2 (2) 0.03

0.2 (2)

(2) (2) 12 10

1012 10

12

0.80870 0.2 (2) 0.21232 (2) 1.06161

Z Z

Z Z Z

Z

P Y X P e e

P e e P e e

P e P Z P Z

¥ + ¥ +

+ + -

È ˘< = <Î ˚È ˘È ˘= < = <Í ˙Î ˚ Î ˚

È ˘= < = < - = < -Î ˚

Since (2)Z has the same distribution as 2Z , where (0,1)Z N , we have:

[ ] [ ] [ ](2) (2) (2) 1.06161 2 1.06161 0.75067

( 0.75067) 0.22643

P Y X P Z P Z P Z

N

È ˘< = < - = < - = < -Î ˚= - =

Solution 14.19

D Probability

The stock price follows geometric Brownian motion with:

a ds- ==

0.080.32

Page 337: ActuarialBrew.com Exam MFE / 3F Actuarial Models …actuarialbrew.com/data/documents/MFE-Solutions-2014-1st-ed.pdf · Exam MFE / 3F Actuarial Models Financial Economics Segment Solutions

Exam MFE/3F Solutions Chapter 14 – Brownian Motion

© ActuarialBrew.com 2014 Page 14.11

The stock prices are lognormally distributed:

( )( )[ ]( )

( )

2 2

2 2

2 2

ln ( ) ln ( ) ( 0.5 ) ,

ln ( ) ln ( ) ( 0.5 ) ,

ln (6) ln (2) (0.08 0.5 0.32 )(4), 0.32 4

ln (6) 3.11093, 0.40960

S t h S t N h h

S t h N S t h h

S N S

S N

a d s s

a d s s

+ - - -

+ + - -

+ - ¥ ¥

Since the dividends are reinvested, at time 6 the value of the investment is:

- =0.04(6 2) 0.16(6) (6)e S e S

The probability that 0.16 (6)e S is greater than or equal to $25 is:

[ ][ ] [ ]

[ ] [ ]

0.16 0.16Prob (6) 25 Prob (6) 25 Prob (6) 21.30359

Prob ln (6) ln(21.30359) Prob ln (6) 3.05888

ln (6) 3.11093 3.05888 3.11093Prob

0.40960 0.40960

Prob 0.081338 1 Prob 0.081338

1 0.466

e S S e S

S S

S

Z Z

-È ˘ È ˘≥ = ≥ = ≥Î ˚ Î ˚= ≥ = ≥

È ˘- -= ≥Í ˙Î ˚

= ≥ - = - < -= - 77

0.53323=

Solution 14.20

A Ornstein-Uhlenbeck Process

This question is quite easy if we have memorized the differential and integral forms of the Ornstein-Uhlenbeck process. For the sake of thoroughness, we show how to derive the correct answer below.

The integral equation contains 3 terms. The first two terms do not contain random variables, so their differentials are easy to find. The third term will be more difficult:

l l ll la s- - - -È ˘Í ˙= - + +Í ˙Î ˚Ú ( )

0

( ) (0) ( )t

t t t sdX t X e dt e dt d e dZ s

The third term has a function of t in the integral. We can pull the t-dependent portion out of the integral, so that we are finding the differential of a product. We then use the following version of the product rule to find the differential:

[ ] = +( ) ( ) ( ) ( ) ( ) ( )d U t V t dU t V t U t dV t

Page 338: ActuarialBrew.com Exam MFE / 3F Actuarial Models …actuarialbrew.com/data/documents/MFE-Solutions-2014-1st-ed.pdf · Exam MFE / 3F Actuarial Models Financial Economics Segment Solutions

Exam MFE/3F Solutions Chapter 14 – Brownian Motion

© ActuarialBrew.com 2014 Page 14.12

The differential of the third term is:

l l l

l l l l

l l l l

l

s s

s s

ls s

ls s

- - -

- -

- -

- -

È ˘ È ˘Í ˙ Í ˙=Í ˙ Í ˙Î ˚ Î ˚

È ˘È ˘ Í ˙= +Î ˚ Í ˙

Î ˚Ê ˆ

= - +Á ˜Á ˜Ë ¯

Ê ˆ= - +Á ˜

Á ˜Ë ¯

Ú Ú

Ú Ú

Ú

Ú

( )

0 0

0 0

0

( )

0

( ) ( )

( ) ( )

( ) ( )

( ) ( )

t tt s t s

t tt s t s

tt s t t

tt s

d e dZ s d e e dZ s

d e e dZ s e d e dZ s

e e dZ s dt e e dZ t

e dZ s dt dZ t

Putting the three differentials together, we have:

l l l

l l l

l l l

l

l la ls s

l a s s

l a a s a s

l a

- - - -

- - - -

- - - -

-

Ê ˆ= - + - +Á ˜

Á ˜Ë ¯

È ˘Ê ˆÍ ˙= - - + +Á ˜Í ˙Á ˜Ë ¯Î ˚È ˘Ê ˆÍ ˙= - + - + - +Á ˜Í ˙Á ˜Ë ¯Î ˚

= - +

Ú

Ú

Ú

( )

0

( )

0

( )

0

( ) (0) ( ) ( )

(0) ( ) ( )

(0) ( ) ( )

(0)

tt t t s

tt t t s

tt t t s

t

dX t X e dt e dt e dZ s dt dZ t

X e e e dZ s dt dZ t

X e e e dZ s dt dZ t

X e

[ ][ ]

l ls a s

l a sl a s

- - -È ˘Ê ˆÍ ˙- + - +Á ˜Í ˙Á ˜Ë ¯Î ˚

= - - +

= - +

Ú ( )

0

(1 ) ( ) ( )

( ) ] ( )

( )] ( )

tt t se e dZ s dt dZ t

X t dt dZ t

X t dt dZ t

Solution 14.21

E Ornstein-Uhlenbeck Process

The trick to this question is to recognize that the process is an Ornstein-Uhlenbeck Process.

The differential form and its solution of an Ornstein-Uhlenbeck are:

( )l l l

l a s

a s- - - -

= ¥ - +

= + - + Ú ( )

0

( ) [ ( )] ( )

( ) (0) 1 ( )t

t t t s

dX t X t dt dZ t

X t X e e e dZ s

Page 339: ActuarialBrew.com Exam MFE / 3F Actuarial Models …actuarialbrew.com/data/documents/MFE-Solutions-2014-1st-ed.pdf · Exam MFE / 3F Actuarial Models Financial Economics Segment Solutions

Exam MFE/3F Solutions Chapter 14 – Brownian Motion

© ActuarialBrew.com 2014 Page 14.13

The process provided in the question can be re-written so that we can identify its parameters:

( ) 0.048 0.3 ( ) 0.4 ( )

[0.048 0.4 ( )] 0.3 ( )0.4[0.12 ( )] 0.3 ( )

dX t dt dZ t X t dtX t dt dZ t

X t dt dZ t

From the final version above, we see that the process is an Ornstein-Uhlenbeck process with:

l a s= = =0.4 0.12 0.3

Therefore, there is a solution of the form:

( )

( )

l l la s- - - -

- - - -

= + - +

= + - +

Ú

Ú

( )

0

0.4 0.4 0.4( )

0

( ) (0) 1 ( )

(0) 0.12 1 0.3 ( )

tt t t s

tt t t s

X t X e e e dZ s

X e e e dZ s

None of the answer choices are in this exact form, but a quick inspection shows that only

choice (E) can provide - -0.4( )t se within the integral.

Depending on the time available during the exam, we might want to select choice (E) as the answer at this point and move on to the next question. For completeness though, we continue with the solution below.

Let’s see if choice (E) can be put in the form shown above:

( )( )( )

( )

-

- - -

- - - - -

- - - -

- -

È ˘= + +Í ˙Î ˚

= + +

= + - + +

= + - +

= + -

Ú

ÚÚÚ

0.4 0.4 0.40

0.4 0.4( )0

0.4 0.4 0.4 0.4( )0

0.4 0.4 0.4( )0

0.4 0.

( ) 0.12 1 0.3 ( )

0.12 1 0.3 ( )

0.12 1 0.3 ( )

2(0.12) 0.12 1 0.3 ( )

0.24 0.12 1

tt t s

tt t s

tt t t t s

tt t t s

t

X t e e e dZ s

e e dZ s

e e e e dZ s

e e e dZ s

e e( ) - -+ Ú4 0.4( )0

0.3 ( )tt t se dZ s

Thus we see that choice (E) results in the correct form with =(0) 0.24.X

Page 340: ActuarialBrew.com Exam MFE / 3F Actuarial Models …actuarialbrew.com/data/documents/MFE-Solutions-2014-1st-ed.pdf · Exam MFE / 3F Actuarial Models Financial Economics Segment Solutions

Exam MFE/3F Solutions Chapter 14 – Brownian Motion

© ActuarialBrew.com 2014 Page 14.14

Solution 14.22

E Correlation Coefficient

Although we are not given the values of a and s , the arithmetic Brownian motion is described by:

a s- = +( ) (0) ( )X T X T Z T

The correlation coefficient is:

[ (8), (17)]

[ (8)] [ (17)]

Cov X X

Var X Var X

The variances in the denominator are:

s

s

=

=

2

2

[ (8)] 8

[ (17)] 17

Var X

Var X

Now let’s find the covariance:

2

2

2

[ (8), (17)]

(8) [ (8)] (17) [ (17)]

(8) (0) 8 (17) (0) 17

(8) (17)

(8) (17)

[8,17]

8

Cov X X

E X E X X E X

E X X X X

E Z Z

E Z Z

Min

The correlation coefficient is:

2

2 2

[ (8), (17)] 8 80.6860

[ (8)] [ (17)] 178 17

Cov X X

Var X Var X

Solution 14.23

E Geometric Brownian Motion and Mutual Funds

The instantaneous percentage increase of the mutual fund is the weighted average of the return on the stock (including its dividend yield) and the return on the risk-free asset:

[ ][ ]

d

a d s da s

È ˘= + + -Í ˙Î ˚= - + + + -

= + - +

( ) ( )(1 )

( ) ( )

( ) ( ) (1 )

(1 ) ( )

dW t dS tb dt b rdt

W t S t

b dt dZ t dt b rdt

b b r dt b dZ t

Page 341: ActuarialBrew.com Exam MFE / 3F Actuarial Models …actuarialbrew.com/data/documents/MFE-Solutions-2014-1st-ed.pdf · Exam MFE / 3F Actuarial Models Financial Economics Segment Solutions

Exam MFE/3F Solutions Chapter 14 – Brownian Motion

© ActuarialBrew.com 2014 Page 14.15

The expression above does not match Choice (A), because it does not include the d dt term. Therefore, we can rule out Choice (A).

The expression above does not match Choice (B) either since the final term in Choice (B) does not include the factor b. Therefore, we can rule out Choice (B).

As written above, we see that ( )W t is a geometric Brownian motion. Therefore, the price can be expressed as:

a s sÈ ˘+ - - +Î ˚=

2 2(1 ) 0.5 ( )( ) (0)

b b r b t b Z tW t W e

Although the expression above is close to Choice (C), it is slightly different, so we can rule out Choice (C). Therefore, we turn to Choices (D) and (E).

Since Choices (D) and (E) contain [ ( ) / (0)]bS t S , let’s find the value of this expression:

a d s s

a d s s

a d s s

- - +

- - +

- - +

=

È ˘ =Í ˙Î ˚

È ˘ =Í ˙Î ˚

2

2

2

( 0.5 ) ( )

( 0.5 ) ( )

( 0.5 ) ( )

( ) (0)

( )(0)

( )(0)

t Z t

bb t b Z t

bb b b t b Z t

S t S e

S te

S

S te

S

We can now describe the process of the mutual fund with:

a s s

a d s s d s s

d s s

d s

È ˘+ - - +Î ˚

È ˘ È ˘- - + + - - +Î ˚ Î ˚

È ˘+ - - +Î ˚

È ˘+ - + - +Î ˚

=

=

È ˘= Í ˙Î ˚

È ˘= Í ˙Î ˚

=

2 2

2 2 2 2

2 2 2

2

(1 ) 0.5 ( )

0.5 ( ) (1 ) 0.5 0.5

(1 ) 0.5 0.5

(1 ) 0.5 ( 1)

( ) (0)

(0)

( )(0)

(0)

( )(0)

(0)

(

b b r b t b Z t

b b b t b Z t b b r b b t

b b b r b b t

b b b r b b t

W t W e

W e e

S tW e

S

S tW e

S

W d s+ - +È ˘Í ˙Î ˚

2[ (1 )( 0.5 )]( )0)

(0)

bb b r b tS t

eS

This matches Choice (E).

Page 342: ActuarialBrew.com Exam MFE / 3F Actuarial Models …actuarialbrew.com/data/documents/MFE-Solutions-2014-1st-ed.pdf · Exam MFE / 3F Actuarial Models Financial Economics Segment Solutions

Exam MFE/3F Solutions Chapter 14 – Brownian Motion

© ActuarialBrew.com 2014 Page 14.16

Solution 14.24

B Geometric Brownian Motion and Mutual Funds

The instantaneous percentage increase of the mutual fund is the weighted average of the return on the stock (including its dividend yield) and the return on the risk-free asset:

[ ]

d

d d

È ˘= + + -Í ˙Î ˚= - + + -= + -= +

( ) ( )(1 )

( ) ( )

1.5 (0.10 ) 0.20 ( ) 0.5(0.04)

0.15 0.30 ( ) 0.020.13 0.3 ( )

dW t dS tb dt b rdt

W t S t

dt dZ t dt dt

dt dZ t dtdt dZ t

The expression above matches Choice (B).

Solution 14.25

D Geometric Brownian Motion and Mutual Funds

The stock’s price follows geometric Brownian motion, so:

a d s s- - + + -

- +

+

+ =

=

== -

2

2

( 0.5 ) [ ( ) ( )]

[0.07 0.5(0.20) ]4 0.20 (4)

0.20 0.20 (4)

( ) ( )

(4) (0)

100 110(4) 1.4766

h Z t h Z t

Z

Z

S t h S t e

S S e

eZ

We can use the value of the stock in the mutual fund at time 4 to determine the value of b:

¥ ==

=

100 % 80 96100 % 1.20

1.20

bb

b

The instantaneous percentage increase of the mutual fund is the weighted average of the return on the stock (including its dividend yield) and the return on the risk-free asset:

[ ][ ]

dÈ ˘= + + -Í ˙Î ˚= + + -

= + -= + -= +

( ) ( )(1 )

( ) ( )

1.2 0.07 0.20 ( ) 0.03 0.2(0.04)

1.2 0.10 0.20 ( ) 0.2(0.04)

0.12 0.24 ( ) 0.0080.112 0.24 ( )

dW t dS tb dt b rdt

W t S t

dt dZ t dt dt

dt dZ t dt

dt dZ t dtdt dZ t

Page 343: ActuarialBrew.com Exam MFE / 3F Actuarial Models …actuarialbrew.com/data/documents/MFE-Solutions-2014-1st-ed.pdf · Exam MFE / 3F Actuarial Models Financial Economics Segment Solutions

Exam MFE/3F Solutions Chapter 14 – Brownian Motion

© ActuarialBrew.com 2014 Page 14.17

As written above, we see that ( )W t is a geometric Brownian motion. Therefore, the price can be expressed as:

È ˘- +Î ˚

+

+

+ -

=

=

=

==

20.112 (0.5)(0.24) 0.24 ( )

0.0832 0.24 ( )

0.0832(4) 0.24 (4)

0.0832(4) 0.24( 1.4766)

( ) (0)

( ) (0)

(4) (0)

80 (0)(0) 81.7445

t Z t

t Z t

Z

W t W e

W t W e

W W e

W eW

Solution 14.26

A Multiplication Rules

As n goes to infinity, we can replace the summation with an integral:

4 4

1 0

lim [ ] [( 1) ] [ ( )]Tn

n j

Z jh Z j h dZ t

We can now use the multiplication rules to evaluate the integral:

4 2 2

0 0 0 0

[ ( )] [ ( )] [ ( )] 0 0T T T T

dZ t dZ t dZ t dt dt

The expected value and variance of zero are both zero:

(0,0)N

Solution 14.27

E Multiplication Rules

As n goes to infinity, we can replace the summation with an integral:

2 3

1

2 3 2 3

0 0 0 0

lim [ ] [( 1) ] [ ] [( 1) ] [ ] [( 1) ]

( ) [ ( )] [ ( )] [ ( )] [ ( )] [ ( )]

n

n j

T T T T

X Z jh Z j h Z jh Z j h Z jh Z j h

dZ t dZ t dZ t dZ t dZ t dZ t

We can use the multiplication rules to evaluate the second and third integrals:

0 0 0

( ) [ ( )] ( ) 0 ( )T T T

X dZ t dt dt dZ t Z T T Z T T

The expected value of X is:

[ ]= + = + =[ ] ( ) 0E X E Z T T T T

Page 344: ActuarialBrew.com Exam MFE / 3F Actuarial Models …actuarialbrew.com/data/documents/MFE-Solutions-2014-1st-ed.pdf · Exam MFE / 3F Actuarial Models Financial Economics Segment Solutions

Exam MFE/3F Solutions Chapter 14 – Brownian Motion

© ActuarialBrew.com 2014 Page 14.18

The variance of X is:

[ ]= + = + =[ ] ( ) 0Var X Var Z T T T T

Since the expected value and the variance are both equal to T, the distribution is:

( , )N T T

Solution 14.28

E Product Rule for Stochastic Differential Equations

To simplify the notation, we use r for r(t) and 0r for r(0).

The first two terms of ( )r t do not contain random variables, so their differentials are easy to find. The third term will be more difficult:

( ) s

s

- - -

- - -

= + - +

È ˘= - + + Í ˙Î ˚

Ú

Ú

( )0 0

( )0 0

1 ( ) ( )

( ) ( )

tt t s t

tt t s t

r r e b e e r s dZ s

dr r e dt be dt d e r s dZ s

a a a

a a aa a

The third term has a function of t in the integral. We can pull the t-dependent portion out of the integral, so that we are finding the differential of a product. We then use the following version of the product rule to find the differential:

[ ] = +( ) ( ) ( ) ( ) ( ) ( )d U t V t dU t V t U t dV t

The differential of the third term is:

s s

s s

s s

- -

- -

-

È ˘ È ˘=Í ˙ Í ˙Î ˚ Î ˚

= - +

= - +

Ú Ú

ÚÚ

( )0 0

0

0

( ) ( ) ( ) ( )

( ) ( ) ( ) ( )

( ) ( ) ( ) ( )

t ts t t s

tt s t t

tt s

d e r s dZ s d e e r s dZ s

e dt e r s dZ s e e r t dZ t

e dt e r s dZ s r t dZ t

a a a

a a a a

a a

a

a

Putting all three terms together, we have:

0 0

0 0

0 0

0 0

( ) ( ) ( )

( ) ( ) ( )

( ) ( ) ( )

( ) ( )

tt t t s

tt t t s

tt t t s

tt t t s

dr r e dt be dt e dt e r s dZ s rdZ t

r e be e e r s dZ s dt rdZ t

r e b b be e e r s dZ s dt rdZ t

r e b be e e r s dZ s

a a a a

a a a a

a a a a

a a a a

a a a

a +

a +

a +

( )0 0

( )

(1 ) ( ) ( ) ( )tt t s t

dt bdt rdZ t

r e b e e r s dZ s dt bdt rdZ t

a a a

a

a + a

Page 345: ActuarialBrew.com Exam MFE / 3F Actuarial Models …actuarialbrew.com/data/documents/MFE-Solutions-2014-1st-ed.pdf · Exam MFE / 3F Actuarial Models Financial Economics Segment Solutions

Exam MFE/3F Solutions Chapter 14 – Brownian Motion

© ActuarialBrew.com 2014 Page 14.19

The portion in the brackets above is equal to the expression for r(t) provided in the question:

( )

( )

( ) ( )

dr rdt bdt rdZ t

bdt rdt rdZ t

b r dt rdZ t

a a

a a

a

Putting the functional relationship back in, we see that this differential equation matches Choice E:

[ ] s= - +( ) ( ) ( )dr b r t dt r t dZ ta

The equation above is the Cox-Ingersoll-Ross Model for the short rate, which appears in Chapter 19 of the Study Manual.

Solution 14.29

E Geometric Brownian Motion Equivalencies

We make use of the following equivalency:

( )( )

a d s a d s s+È ˘= - + ¤ - -Í ˙Î ˚+È ˘= + ¤ -Í ˙Î ˚

2 2( ) ( )( ) ( ) ln ( 0.5 ) ,

( ) ( )

( ) ( )0.04 0.3 ( ) ln 0.005 , 0.09

( ) ( )

dS t S t hdt dZ t N h h

S t S t

dS t S t hdt dZ t N h h

S t S t

To avoid a cluttered appearance, we use tS to represent ( )S t in the solution below.

We can rewrite the expression for G , so that it is the product of independent random variables:

2 13 3

1 / 31 / 3 31 1 2 1 2

1 2 3 0 0 00 0 1 0 1 2

1 / 33 23 3 31 2 1 2

0 03 2 2 0 1 20 1

SS S S S SG S S S S S S

S S S S S S

S SS S S SS S

S S S SS S

Taking the natural log, we have:

31 20

0 1 2

2 1ln ln ln ln ln

3 3SS S

G SS S S

The variance is:

2 22 1

ln 0 0.09 (0.09) (0.09) 0.143 3

Var G

Page 346: ActuarialBrew.com Exam MFE / 3F Actuarial Models …actuarialbrew.com/data/documents/MFE-Solutions-2014-1st-ed.pdf · Exam MFE / 3F Actuarial Models Financial Economics Segment Solutions

Exam MFE/3F Solutions Chapter 14 – Brownian Motion

© ActuarialBrew.com 2014 Page 14.20

Solution 14.30

A Geometric Brownian Motion Equivalencies

We make use of the following equivalency:

( )( )

a d s a d s s+È ˘= - + ¤ - -Í ˙Î ˚+È ˘= + ¤ -Í ˙Î ˚

2 2( ) ( )( ) ( ) ln ( 0.5 ) ,

( ) ( )

( ) ( )0.04 0.3 ( ) ln 0.005 , 0.09

( ) ( )

dS t S t hdt dZ t N h h

S t S t

dS t S t hdt dZ t N h h

S t S t

To avoid a cluttered appearance, we use tS to represent ( )S t in the solution below.

We can rewrite the expression for G , so that it is the product of independent random variables:

2 13 3

1 / 31 / 3 31 1 2 1 2

1 2 3 0 0 00 0 1 0 1 2

1 / 33 23 3 31 2 1 2

0 03 2 2 0 1 20 1

SS S S S SG S S S S S S

S S S S S S

S SS S S SS S

S S S SS S

Taking the natural log, we have:

31 20

0 1 2

2 1ln ln ln ln ln

3 3SS S

G SS S S

The expected value is:

2 1ln ln(2) ( 0.005) ( 0.005) ( 0.005) 0.6831

3 3E G

Solution 14.31

C Geometric Brownian Motion Equivalencies

We make use of the following equivalency:

( )( )

a d s a d s s+È ˘= - + ¤ - -Í ˙Î ˚+È ˘= + ¤ -Í ˙Î ˚

2 2( ) ( )( ) ( ) ln ( 0.5 ) ,

( ) ( )

( ) ( )0.04 0.3 ( ) ln 0.005 , 0.09

( ) ( )

dS t S t hdt dZ t N h h

S t S t

dS t S t hdt dZ t N h h

S t S t

To avoid a cluttered appearance, we use tS to represent ( )S t in the solution below.

Page 347: ActuarialBrew.com Exam MFE / 3F Actuarial Models …actuarialbrew.com/data/documents/MFE-Solutions-2014-1st-ed.pdf · Exam MFE / 3F Actuarial Models Financial Economics Segment Solutions

Exam MFE/3F Solutions Chapter 14 – Brownian Motion

© ActuarialBrew.com 2014 Page 14.21

We can rewrite the expression for G , so that it is the product of independent random variables:

2 13 3

1 / 31 / 3 31 1 2 1 2

1 2 3 0 0 00 0 1 0 1 2

1 / 33 23 3 31 2 1 2

0 03 2 2 0 1 20 1

SS S S S SG S S S S S S

S S S S S S

S SS S S SS S

S S S SS S

Taking the natural log, we have:

31 20

0 1 2

2 1ln ln ln ln ln

3 3SS S

G SS S S

The expected value and variance are:

2 2

2 1ln ln(2) ( 0.005) ( 0.005) ( 0.005) 0.6831

3 3

2 1ln 0 0.09 (0.09) (0.09) 0.14

3 3

E G

Var G

Therefore, the distribution of ln[G] is normal with the following parameters:

ln (0.6831, 0.14)G N

The expected value of G is:

0.6831 0.5(0.14)[ ] 2.1237E G e

Solution 14.32

E Geometric Brownian Motion Equivalencies

We make use of the following equivalency, substituting x and y for S as needed:

[ ] ( )a d s a d s s= - + ¤ = - - +2( )( ) ( ) ln ( ) 0.5 ( )

( )dS t

dt dZ t d S t dt dZ tS t

Since ( )y t is the exchange rate denominated in euros per dollar, we have:

1

2

2

1( ) [ ( )]

( )ln ( ) ln ( )

[ln ( )] [ln ( )] (0.03 0.5 0.1 ) 0.10 ( )

[ln ( )] 0.025 0.10 ( )( )

0.025 0.5 ( 0.10) 0.10 ( ) 0.02 0.10 ( )( )

y t x tx t

y t x t

d y t d x t dt dZ t

d y t dt dZ tdy t

dt dZ t dt dZ ty t

-= =

= -

È ˘= - = - - ¥ +Î ˚= - -

È ˘= - + ¥ - - = - -Î ˚

Page 348: ActuarialBrew.com Exam MFE / 3F Actuarial Models …actuarialbrew.com/data/documents/MFE-Solutions-2014-1st-ed.pdf · Exam MFE / 3F Actuarial Models Financial Economics Segment Solutions

Exam MFE/3F Solutions Chapter 14 – Brownian Motion

© ActuarialBrew.com 2014 Page 14.22

Solution 14.33

B Geometric Brownian Motion Equivalencies

We make use of the following equivalency, substituting x and y for S as needed:

[ ] ( )a d s a d s s= - + ¤ = - - +2( )( ) ( ) ln ( ) 0.5 ( )

( )dS t

dt dZ t d S t dt dZ tS t

Since ( )x t is the exchange rate denominated in dollars per euro, we have:

( )( )

s s s

s s

s s s s

-= =

= -

È ˘= - = - - + - ¥ - -Í ˙Î ˚

= - - +

È ˘= - - + + = - +Î ˚

1

2 2€

2€

2 2€ €

1( ) [ ( )]

( )ln ( ) ln ( )

[ln ( )] [ln ( )] 0.5 ( ) ( )

[ln ( )] 0.5 ( )

( )0.5 0.5 ( ) ( ) ( )

( )

x t y ty t

x t y t

d x t d y t r r dt dZ t

d x t r r dt dZ t

dx tr r dt dZ t r r dt dZ t

x t

Solution 14.34

E Geometric Brownian Motion

From the process for the stock price, we observe that:

s = 0.30

The volatility is the annualized standard deviation of the natural log of the prepaid forward price:

( )

( )

( )

sÈ ˘Í ˙Î ˚ =

È ˘Í ˙Î ˚ =

È ˘ =Í ˙Î ˚

,

,3

,3

ln ( )

ln ( )0.30

ln ( ) 0.09

Pt T

Pt

Pt

Var F S

t

Var F S

t

Var F S t

Alternative Solution

The prepaid forward price is:

d- - - - - ¥ + -= =2(3 ) 0.05(3 ) (0.09 0.5 0.30 ) 0.30[ ( ) (0)]

,3 ( ) ( ) (0)P t t t Z t ZtF S e S t e S e

The natural log of the prepaid forward price is:

È ˘ = - - + + - ¥ + -Î ˚2

,3ln ( ) 0.05(3 ) ln (0) (0.09 0.5 0.30 ) 0.30[ ( ) (0)]PtF S t S t Z t Z

Page 349: ActuarialBrew.com Exam MFE / 3F Actuarial Models …actuarialbrew.com/data/documents/MFE-Solutions-2014-1st-ed.pdf · Exam MFE / 3F Actuarial Models Financial Economics Segment Solutions

Exam MFE/3F Solutions Chapter 14 – Brownian Motion

© ActuarialBrew.com 2014 Page 14.23

The first three terms above are not random, so their variances are zero:

{ } { }È ˘ = + + + - = =Î ˚2

,3ln ( ) 0 0 0 0.30[ ( ) (0)] 0.30 0.09PtVar F S Var Z t Z t t

Solution 14.35

C Geometric Brownian Motion

From the process for the stock price, we observe that:

s = 0.30

The volatility is the annualized standard deviation of the natural log of the forward price:

s=,[ln ]t TVar F

t

The question uses t instead of T to represent the maturity of the contract, but we can make the appropriate substitutions:

=

= ¥ =

3,

23,

[ln ]0.30

3

[ln ] (0.30) 3 0.27

t

t

Var F

Var F

Alternative Solution

The forward price is:

d- - - - - ¥ + -= =2( )( 3) 0.01( 3) (0.09 0.5 0.30 )3 0.30[ (3) (0)]

3, ( ) (3) (0)r t t Z ZtF S e S e S e

The natural log of the prepaid forward price is:

È ˘ = - - + + - ¥ + -Î ˚2

3,ln ( ) 0.01( 3) ln (0) (0.09 0.5 0.30 )3 0.30[ (3) (0)]tF S t S Z Z

The first three terms above are not random, so their variances are zero:

{ } { }È ˘ = + + + - = ¥ =Î ˚2

3,ln ( ) 0 0 0 0.30[ (3) (0)] 0.30 3 0.27tVar F S Var Z Z

Solution 14.36

B Geometric Brownian Motion Equivalencies

The forward price at time t is:

d- - - - -= = = =( )( ) (0.06 0.03)( ) 0.03( ),( ) ( ) ( ) ( )r T t T t T t

t TF t F S t e S t e S t e

Page 350: ActuarialBrew.com Exam MFE / 3F Actuarial Models …actuarialbrew.com/data/documents/MFE-Solutions-2014-1st-ed.pdf · Exam MFE / 3F Actuarial Models Financial Economics Segment Solutions

Exam MFE/3F Solutions Chapter 14 – Brownian Motion

© ActuarialBrew.com 2014 Page 14.24

We can determine the natural log of the forward price and the differential of the natural log. Then we convert the differential of the natural log of the forward price into the differential of the forward price:

[ ]

-== + -

= + - = - ¥ + -

= - ¥ += + = +

0.03( )

2

2

( ) ( )ln ( ) ln ( ) 0.03 0.03

[ln ( )] [ln ( )] 0 0.03 (0.16 0.5 0.2 ) 0.2 ( ) 0.03

(0.13 0.5 0.2 ) 0.2 ( )

( ) 0.13 ( ) 0.2 ( ) ( ) ( ) 0.13 0.2 ( )

T tF t S t eF t S t T t

d F t d S t dt dt dZ t dt

dt dZ t

dF t F t dt F t dZ t F t dt dZ t

Alternative Solution

The partial derivatives are:

-

-- -

==

È ˘∂ Î ˚= = - = -∂

0.03( )

0.03 0.030.03 0.03 0.03( )

0

( )( ) ( 0.03) ( )( 0.03)

T tS

SS

T tT t T t

t

F e

F

S t e eF S t e e S t e

t

From Itô’s Lemma, we have:

[ ][ ]

- -

-

-

= + +

= + + -

= + -

= + + -

= + + -

=

2

0.03( ) 2 0.03( )

0.03( )

0.03( )

1( ) ( ) [ ( )]

21

( ) (0)[ ( )] ( )( 0.03)2

( ) ( )( 0.03)

( ) 0.16 0.2 ( ) ( )( 0.03)

( ) 0.16 0.2 ( ) ( )( 0.03)

( ) 0

S SS t

T t T t

T t

T t

dF t F dS t F dS t F dt

e dS t dS t S t e dt

e dS t F t dt

e S t dt dZ t F t dt

F t dt dZ t F t dt

F t [ ]+.13 0.2 ( )dt dZ t

Solution 14.37

E Geometric Brownian Motion Equivalencies

The prepaid forward price at time t is:

d- - - -= = =( ) 0.03( ),( ) ( ) ( ) ( )P T t T t

t TG t F S S t e S t e

Page 351: ActuarialBrew.com Exam MFE / 3F Actuarial Models …actuarialbrew.com/data/documents/MFE-Solutions-2014-1st-ed.pdf · Exam MFE / 3F Actuarial Models Financial Economics Segment Solutions

Exam MFE/3F Solutions Chapter 14 – Brownian Motion

© ActuarialBrew.com 2014 Page 14.25

We can determine the natural log of the prepaid forward price and the differential of the natural log. Then we convert the differential of the natural log of the prepaid forward price into the differential of the prepaid forward price:

[ ]

- -== - +

= - + = - ¥ + +

= - ¥ += + = +

0.03( )

2

2

( ) ( )ln ( ) ln ( ) 0.03 0.03

[ln ( )] [ln ( )] 0 0.03 (0.16 0.5 0.2 ) 0.2 ( ) 0.03

(0.19 0.5 0.2 ) 0.2 ( )

( ) 0.19 ( ) 0.2 ( ) ( ) ( ) 0.19 0.2 ( )

T tG t S t eG t S t T t

d G t d S t dt dt dZ t dt

dt dZ t

dG t G t dt G t dZ t G t dt dZ t

Solution 14.38

D Multiplication Rules

Statement (i) is true, because when the Black-Scholes framework applies, the natural log of stock prices follows arithmetic Brownian motion.

Statement (ii) is false, because when the Black-Scholes framework applies, stock prices are lognormally distributed as described below:

( )a d s s+ - - - 2 2ln ( ) ln ( ) ( 0.5 ) , S t h S t N h h

Since the stock prices are lognormally distributed, we have:

a d s+ - = + - = - - 2[ ( ) ( )] [ln ( ) ln ( )] ( 0.5 )E X t h X t E S t h S t h

Now consider Statement (iii). Small increments in a Brownian motion are represented by the differential as the time increment becomes increasingly small. For example, in the

study manual’s discussion of quadratic variation, -Ê ˆ Ê ˆ-Á ˜ Á ˜Ë ¯ Ë ¯

( 1)jT j TZ Z

n n in the summation

becomes ( )dZ t in the integral form:

Æ• =

È ˘-Ê ˆ Ê ˆ= - = =Í ˙Á ˜ Á ˜Ë ¯ Ë ¯Î ˚ Ú

2 2

1 0

( 1)Quadratic Variation lim [ ( )]

Tn

n j

jT j TZ Z dZ t T

n n

Page 352: ActuarialBrew.com Exam MFE / 3F Actuarial Models …actuarialbrew.com/data/documents/MFE-Solutions-2014-1st-ed.pdf · Exam MFE / 3F Actuarial Models Financial Economics Segment Solutions

Exam MFE/3F Solutions Chapter 14 – Brownian Motion

© ActuarialBrew.com 2014 Page 14.26

Likewise, -Ê ˆ Ê ˆ-Á ˜ Á ˜Ë ¯ Ë ¯

( 1)jT j TX X

n n in the summation can be replaced by ( )dX t in the

integral form. Below we use the multiplication rules to simplify the solution:

a d s s

a d s a d s s s

a d s a d s s s

s

Æ• =

È ˘-Ê ˆ Ê ˆ È ˘- = = - - +Í ˙Á ˜ Á ˜ Î ˚Ë ¯ Ë ¯Î ˚

È ˘= - - + - - +Î ˚

È ˘= - - ¥ + - - ¥ +Î ˚

È ˘= + +Î ˚

Â Ú Ú

Ú

Ú

Ú

2 22 2

1 0 0

2 2 2 2 2 2

0

2 2 2 2

0

2

0

( 1)lim [ ( )] ( 0.5 ) ( )

( 0.5 ) ( ) 2( 0.5 ) ( ) [ ( )]

( 0.5 ) 0 2( 0.5 ) 0

0 0

T Tn

n j

T

T

T

jT j TX X dX t dt dZ t

n n

dt dt dZ t dZ t

dt

dt s s= =Ú2 2

0

T

dt T

Therefore, Statement (iii) is true.

Solution 14.39

E Synthetic Risk-free Asset from 2 Risky Assets

Let X be the percentage invested in Asset 1. Over the next instant, the percentage return on the investment is:

( ) ( )+ + - -0.08 0.2 (1 ) 0.0925 0.3X dt dZ X dt dZ

We can determine X so that the dZ terms cancel:

- - =- - =- + ==

=

0.2 0.3(1 ) 00.2 0.3(1 ) 00.2 0.3 0.3 00.5 0.3

0.60

XdZ X dZX XX XX

X

The portion of the $1,000 to be invested in Asset 1 is:

¥ = ¥ =1,000 1,000 0.60 600X

Solution 14.40

C Geometric Brownian Motion

This question is based on the “Remarks” that appear after the official solution to Question #19 on the Spring 2009 MFE Exam.

Page 353: ActuarialBrew.com Exam MFE / 3F Actuarial Models …actuarialbrew.com/data/documents/MFE-Solutions-2014-1st-ed.pdf · Exam MFE / 3F Actuarial Models Financial Economics Segment Solutions

Exam MFE/3F Solutions Chapter 14 – Brownian Motion

© ActuarialBrew.com 2014 Page 14.27

Statement I is false. The stock price consists of 2 components: the prepaid forward price and the present value of the dividends, and the stock price’s volatility is an average of the volatilities of the two components. The volatility of the dividends is zero, so as the dividends are paid, the average approaches the volatility of the prepaid forward. Geometric Brownian motion calls for constant volatility, so the stock price process cannot be a geometric Brownian motion.

Statement II is true. Let’s use ,P

t TF to denote the prepaid forward price. Since the

prepaid forward price follows geometric Brownian motion, we have:

m s sm s - += + ¤ = ¥2, ( 0.5 ) ( )

, 0,,

( )P

t T P P t Z tt T TP

t T

dFdt dZ t F F e

F

The forward price process is:

m s s

m s s m s s

- - + -

- - + - - +

= ¥ = ¥

= ¥ =

2

2 2

( ) ( 0.5 ) ( ) ( ), , 0,

( 0.5 ) ( ) ( 0.5 ) ( )0,0,

P r T t P t Z t r T tt T t T T

P rT r t Z t r t Z tTT

F F e F e e

F e e F e

This can be recognized as geometric Brownian motion:

m s sm s - - += - + ¤ = ¥2, ( 0.5 ) ( )

, 0,,

( ) ( )t T r t Z tt T T

t T

dFr dt dZ t F F e

F

Statement III is true. Since the prepaid forward price follows geometric Brownian motion, we again have:

m s sm s - += + ¤ = ¥2, ( 0.5 ) ( )

, 0,,

( )P

t T P P t Z tt T TP

t T

dFdt dZ t F F e

F

The stock price can be written in terms of the prepaid forward price:

d d- - -= fi = ¥( ) ( ), ,P T t P T t

t tt T t TF S e S F e

The stock price process is:

d m s s d

d m d s s m d s s

- - + -

- - + - - +

= ¥ = ¥

= ¥ =

2

2 2

( ) ( 0.5 ) ( ) ( ), 0,

( 0.5 ) ( ) ( 0.5 ) ( )00,

P T t P t Z t T tt t T T

P T t Z t t Z tT

S F e F e e

F e e S e

This can be recognized as geometric Brownian motion:

m d s sm d s - - += - + ¤ = ¥2( 0.5 ) ( )

0( ) ( ) t Z ttt

t

dSdt dZ t S S e

S

Page 354: ActuarialBrew.com Exam MFE / 3F Actuarial Models …actuarialbrew.com/data/documents/MFE-Solutions-2014-1st-ed.pdf · Exam MFE / 3F Actuarial Models Financial Economics Segment Solutions

Exam MFE/3F Solutions Chapter 14 – Brownian Motion

© ActuarialBrew.com 2014 Page 14.28

Solution 14.41

D Black-Scholes Formula

The prepaid forward volatility is 20%:

,0.5

,0.5

( )0.20 ( ) 0.20

( )

Pt

PFPt

dF Sdt dZ t

F S

The prepaid forward prices of the stock and the strike price are:

-

-

= - =

= =

0.07(4 /12)0,

0.07(0.5)0,

( ) 70 4.00 66.0923

( ) 75 72.4204

PT

PT

F S e

F K e

We use the prepaid forward volatility in the Black-Scholes Formula:

0, 2 20,

1

2 1

( ) 66.0923ln 0.5 ln 0.5 0.20 0.5( ) 72.42040.20 0.5

0.57584

0.57584 0.20 0.5 0.71726

PT

PFPT

PF

PF

F ST

F Kd

T

d d T

s

s

s

Ê ˆÊ ˆ+Á ˜ + ¥ ¥Á ˜ Á ˜Ë ¯ Ë ¯

= =

= -

= - = - - = -

We have:

1( ) (0.57584) 0.71764N d N

2( ) (0.71726) 0.76339N d N

The value of the European put option is:

0, 0, 2 0 0, 1( ), ( ), , ( ) ( ) ( )

72.4204 0.76339 66.0923 0.717647.8546

P P rTEur T T TP F S F K T Ke N d S PV Div N d

Solution 14.42

A Volatility of Prepaid Forward

The prepaid forward price at time t is equal to the stock price minus the present value of the dividend:

( )- -= -

4120.07

,0.5( ) ( ) 4tP

tF S S t e

Page 355: ActuarialBrew.com Exam MFE / 3F Actuarial Models …actuarialbrew.com/data/documents/MFE-Solutions-2014-1st-ed.pdf · Exam MFE / 3F Actuarial Models Financial Economics Segment Solutions

Exam MFE/3F Solutions Chapter 14 – Brownian Motion

© ActuarialBrew.com 2014 Page 14.29

We can take the differential of both sides. For £ < 40

12t , we have:

( )

[ ] [ ] ( )( )

412

412

412

0.07,0.5

0.07,0.5

0.07,0.5 ,0.5

,0.5

( ) ( ) 0.28

( ) ( ) ( ) ( ) ( ) ( ) 0.28

( ) ( ) ( ) ( ) ( ) ( ) ( ) ( ) ( ) 0.28

( )

tPt

tPt PF PF

tP PPF t PF t

PPF t PF t

dF S dS t e dt

F S dt dZ t S t t dt t dZ t e dt

F S dt F S dZ t S t t S t dt t S t dZ t e dt

F S dt F

a s a s

a s a s

a s

- -

- -

- -

= -

+ = + -

+ = + -

+ ( )4120.07

,0.5 ( ) ( ) ( ) ( ) ( ) 0.28 ( ) ( ) ( )tP S dZ t S t t S t e dt t S t dZ ta s

- -È ˘= - +Í ˙Î ˚

The coefficients to the dZ(t) terms above are equal for £ < 40

12t :

,0.5

,0.5

( ) ( ) ( )

( ) ( )

( )

PPF t

PF Pt

F S t S t

t S t

F S

s sss

=

=

Since we know the time-0 values on the right side of the equation, we can find sPF :

( )4120.07 0,0.5

( ) ( ) (0) 700.189 1.059126 0.20017

( ) 70 4PF P

t

t S t

F S e

s ss- -

¥= = = ¥ =-

The prepaid forward prices of the stock and the strike price are:

-

-

= - =

= =

0.07(4 /12)0,

0.07(0.5)0,

( ) 70 4.00 66.0923

( ) 75 72.4204

PT

PT

F S e

F K e

We use the prepaid forward volatility in the Black-Scholes Formula:

0, 2 20,

1

2 1

( ) 66.0923ln 0.5 ln 0.5 0.20017 0.5( ) 72.42040.20017 0.5

0.57522

0.57522 0.20017 0.5 0.71676

PT

PFPT

PF

PF

F ST

F Kd

T

d d T

s

s

s

Ê ˆÊ ˆ+Á ˜ + ¥ ¥Á ˜ Á ˜Ë ¯ Ë ¯

= =

= -

= - = - - = -

We have:

1( ) ( 0.57522) 0.28257N d N

2( ) ( 0.71676) 0.23676N d N

Page 356: ActuarialBrew.com Exam MFE / 3F Actuarial Models …actuarialbrew.com/data/documents/MFE-Solutions-2014-1st-ed.pdf · Exam MFE / 3F Actuarial Models Financial Economics Segment Solutions

Exam MFE/3F Solutions Chapter 14 – Brownian Motion

© ActuarialBrew.com 2014 Page 14.30

The value of the European call option is:

0, 0, 0 0, 1 2( ), ( ), , ( ) ( ) ( )

66.0923 0.28257 72.4204 0.236761.5294

P P rTEur T T TC F S F K T S PV Div N d Ke N d

Solution 14.43

B Volatility of Prepaid Forward

The prepaid forward price at time t follows geometric Brownian motion:

a s s a s- -= ¤ = +2 ,0.5( 0.5 ) ( )

,0.5 0,0.5,0.5

( )( ) ( ) ( )

( )PF PF PF

Ptt Z tP P

t PF PFPt

dF SF S F S e dt dZ t

F S

From (v), we know that the expected return is a = 0.12PF . From statement (iv) and the

exponent in the first expression above, we have:

a s

ss

- =

- =

=

2

2

0.5 0.04

0.12 0.5 0.04

0.40

PF PF

PF

PF

The prepaid forward prices of the stock and the strike price are:

-

-

= - =

= =

0.07(4 /12)0,

0.07(0.5)0,

( ) 70 4.00 66.0923

( ) 75 72.4204

PT

PT

F S e

F K e

We use the prepaid forward volatility in the Black-Scholes Formula:

0, 2 20,

1

2 1

( ) 66.0923ln 0.5 ln 0.5 0.4 0.5( ) 72.42040.18186

0.4 0.5

0.18186 0.4 0.5 0.46470

PT

PFPT

PF

PF

F ST

F Kd

T

d d T

s

s

s

Ê ˆÊ ˆ+Á ˜ + ¥ ¥Á ˜ Á ˜Ë ¯ Ë ¯

= = = -

= - = - - = -

We have:

1( ) ( 0.18186) 0.42785N d N

2( ) ( 0.46470) 0.32107N d N

The value of the European call option is:

0, 0, 0 0, 1 2( ), ( ), , ( ) ( ) ( )

66.0923 0.42785 72.4204 0.321075.0256

P P rTEur T T TC F S F K T S PV Div N d Ke N d

Page 357: ActuarialBrew.com Exam MFE / 3F Actuarial Models …actuarialbrew.com/data/documents/MFE-Solutions-2014-1st-ed.pdf · Exam MFE / 3F Actuarial Models Financial Economics Segment Solutions

Exam MFE/3F Solutions Chapter 14 – Brownian Motion

© ActuarialBrew.com 2014 Page 14.31

Solution 14.44

C Forward Exchange Contract

When the contract is initiated, the prepaid forward prices of the payoffs to each party must be equal. Therefore, at time t:

b- - - -=0.02(5 ) 0.06(5 )( ) ( ) ( )t te A t e B t t

We can rearrange the equation above to solve for b ( )t :

b- -

- -=

0.02(5 )

0.06(5 )( )

( )( )

t

te A t

te B t

We make use of the following geometric Brownian motion equivalency:

a d s sa d s - - += - + ¤ =2( 0.5 ) ( )( ) ( ) ( ) ( ) ( ) ( ) (0) t Z tdS t S t dt S t dZ t S t S e

Substituting ( )A t and ( )B t for ( )S t as needed, we have:

b

- - - +-

- - - +

- - + -

= =

= =

2

2

0.02(5 ) [0.08 0.5(0.20) ] 0.2 ( )0.04(5 )

0.06(5 ) [0.09 0.5(0.40) ] 0.4 ( )

0.20 0.04 0.05 0.2 ( ) 0.20 0.01 0.2 ( )

( ) (0)( )

( ) (0)5

1.254

t t Z tt

t t Z t

t t Z t t Z t

e A t A et e

e B t B e

e e e e

Solution 14.45

D Ornstein-Uhlenbeck Process

The key to this question is recognizing that the process is an Ornstein-Uhlenbeck process:

( )l l ll a s a s- - - -= ¥ - + ¤ = + - + Ú ( )

0

( ) [ ( )] ( ) ( ) (0) 1 ( )t

t t t sdX t X t dt dZ t X t X e e e dZ s

The parameters of the process are:

l a s= = =4 0 3

Therefore the solution is:

( )- - - - -È ˘Í ˙= + ¥ - + = +Í ˙Î ˚

Ú Ú4 4 4( ) 4 4

0 0

( ) (0) 0 1 3 ( ) (0) 3 ( )t t

t t t s t sX t X e e e dZ s e X e dZ s

In this form, we can observe that:

= = = = = -4 (0) 5 3 4A B X C D

The sum of A, B, C, and D is:

4 5 3 4 8A B C D+ + + = + + - =

Page 358: ActuarialBrew.com Exam MFE / 3F Actuarial Models …actuarialbrew.com/data/documents/MFE-Solutions-2014-1st-ed.pdf · Exam MFE / 3F Actuarial Models Financial Economics Segment Solutions

Exam MFE/3F Solutions Chapter 14 – Brownian Motion

© ActuarialBrew.com 2014 Page 14.32

Solution 14.46

B Portfolio Returns

The instantaneous percentage increase of the fund is the weighted average of the return on the stock (including its dividend yield) and the return on the risk-free asset, minus the asset fee:

[ ]

dÈ ˘= + + -Í ˙Î ˚= + + + -= +

( ) ( )0.80 0.20 0.01

( ) ( )

0.80 0.09 0.25 ( ) 0.03 0.20(0.07) 0.01

0.10 0.20 ( )

dW t dS tdt rdt dt

W t S t

dt dZ t dt dt dt

dt dZ t

Since the fund’s value follows geometric Brownian motion, we can convert the partial differential equation into an expression for the value of the fund:

a d s sa d s - - +

- ¥ +

= - + ¤ =

= + ¤ =

2

2

( 0.5 ) ( )

(0.10 0.5 0.20 ) 0.20 ( )

( )( ) ( ) W( ) (0)

( )( )

0.10 0.20 ( ) W( ) (0)( )

W W W Wt Z tW W W

t Z t

dW tdt dZ t t W e

W tdW t

dt dZ t t W eW t

Simplifying the lower right-hand expression above, we have:

+= 0.08 0.20 ( )W( ) (0) t Z tt W e

Solution 14.47

A Standard Brownian Motion

The page references below refer to the Second Edition of the Derivatives Markets textbook.

Brownian motion is continuous and has the Markov property, so ( )Z t is a diffusion process, and statement B is true [Second sentence on page 651 of the Derivatives Markets textbook].

Brownian motion is continuous, so statement C is true [Final bullet point on page 650 of the Derivatives Markets textbook].

Statement D is true, because =(0) 0Z and Brownian motion exhibits the infinite crossings property [Final sentence of the section preceding “Arithmetic Brownian Motion” on page 653 of the Derivatives Markets textbook].

Statement E is true, because + -{ ( ) ( )}Z t s Z t has a normal distribution described by

(0, )N s [Second bullet point on page 650 of the Derivatives Markets textbook].

Since the other statements are true, Statement A must be the false statement.

Page 359: ActuarialBrew.com Exam MFE / 3F Actuarial Models …actuarialbrew.com/data/documents/MFE-Solutions-2014-1st-ed.pdf · Exam MFE / 3F Actuarial Models Financial Economics Segment Solutions

Exam MFE/3F Solutions Chapter 14 – Brownian Motion

© ActuarialBrew.com 2014 Page 14.33

Solution 14.48

A Geometric Brownian Motion Equivalencies

We make use of the following equivalency:

( )a d s a d s s+È ˘= - + ¤ - -Í ˙Î ˚ 2 2( ) ( )

( ) ( ) ln ( 0.5 ) , ( ) ( )

dS t S t hdt dZ t N h h

S t S t

To avoid a cluttered appearance, we use tS to represent ( )S t in the solution below. We

can rewrite the expression for G , so that it is the product of independent random variables:

2

2

( 1)

2

( 1)

11

1

1

0 0 00 0 0

( 1)

00

T TN N

T T T TN N N N

T N TN N

T TN N

T N TN N

N NN

Tj

NT

NN

T

jTG S S S S

N

S S S S SS S S

S S S S S

S S SS

S S S

2

( 1)

1

1( 1)

00

T TN N

T N TN N

N

NNN

TS S S

SS S S

Taking the natural log, we have:

2

( 1)0

0

( 1) 1ln ln ln ln ln

T TN N

T N TN N

TS S SN

G SS N S N S

Page 360: ActuarialBrew.com Exam MFE / 3F Actuarial Models …actuarialbrew.com/data/documents/MFE-Solutions-2014-1st-ed.pdf · Exam MFE / 3F Actuarial Models Financial Economics Segment Solutions

Exam MFE/3F Solutions Chapter 14 – Brownian Motion

© ActuarialBrew.com 2014 Page 14.34

The variance, lnVar G , is:

2

( 1)

2 2

00

2 2 2 22 2 2 2

1 1ln ln ln ln

1 1 1 10

T TN N

T N TN N

TS S SN

Var S Var Var VarS N S N S

T N T T T N NN N N N N N N N N

2

2 22 2 2

3 3

2

2

( 1)(2 1)( 1) 1

6

( 1)(2 1)

6

T T N N NN N

N N

N N T

N

Solution 14.49

C Pure Brownian Motion

The expected value of the product is:

{ }{ }{ }

È ˘ È ˘= - +Î ˚ Î ˚È ˘= - +Î ˚È ˘ È ˘= - +Î ˚ Î ˚

È ˘È ˘ È ˘= - +Î ˚ Î ˚ Î ˚È ˘= ¥ + Î ˚

=

2

2

(5) (10) (1) (5) (10) (5) (5) (1)

(5) (10) (5) (5) (5) (1)

(5) (10) (5) (1) (5) (5) (1)

(5) (1) (10) (5) (1) [ (5)] (1)

8 0 [ (5)] (1)

[ (5)

E Z Z Z E Z Z Z Z Z

E Z Z Z Z Z Z

E Z Z Z Z E Z Z Z

E Z Z E Z Z Z E Z Z

E Z Z

E ZÈ ˘Î ˚

2] (1)Z

The variance of (5)Z is equal to the amount of time elapsed from time 1 to time 5:

È ˘ = - =Î ˚[ (5)] (1) 5 1 4Var Z Z

We can use the following expression for variance to solve for the expected value of the square:

[ ] [ ]( )( )

( )

È ˘= -Î ˚

È ˘È ˘ È ˘= -Î ˚ Î ˚Î ˚È ˘= -Î ˚

È ˘ =Î ˚

22

22

22

2

[ (5)] (1) [ (5)] (1) [ (5)] (1)

4 [ (5)] (1) 8

[ (5)] (1) 68

Var X E X E X

Var Z Z E Z Z E Z Z

E Z Z

E Z Z

Since the expected value of the product is equal to the expected value of the square of (5)Z , we have:

È ˘È ˘ = =Î ˚ Î ˚2(5) (10) (1) [ (5)] (1) 68E Z Z Z E Z Z

Page 361: ActuarialBrew.com Exam MFE / 3F Actuarial Models …actuarialbrew.com/data/documents/MFE-Solutions-2014-1st-ed.pdf · Exam MFE / 3F Actuarial Models Financial Economics Segment Solutions

Exam MFE/3F Solutions Chapter 15 – The Sharpe Ratio and Itô’s Lemma

© ActuarialBrew.com 2014 Page 15.01

Chapter 15 – Solutions

Solution 15.01

C Sharpe Ratio

Stock X and Stock Q are perfectly positively correlated since each one is driven by the same stochastic process, so they must have the same Sharpe ratio:

0.05 0.080.11 0.31

(0.31)(0.05) 0.31 (0.11)(0.08) 0.110.20 (0.31)(0.05) (0.11)(0.08)

0.0335

QX

X Q

rr

r r

r rr

r

aas s

--=

- -=

- = -= -

=

Solution 15.02

D Confidence Intervals

( )Y t follows geometric Brownian motion with the following parameters:

0.66 0.6 0.6 0.6Y Y Y Y Ya d s s s- = = - = = - =

The upper z-value for the 90% confidence interval is found below:

( ) ( ) ( )1 0.90 0.05000

2 2

1.64485

U U U

U

pP z z P z z P z z

z

-> = fi > = fi > =

fi =

The upper limit of the confidence interval for (4)Y is:

2 2( 0.5 ) (0.66 0.5 0.6 )(4) 0.6 1.64485 4(0) 27 1,325.6483

UY Y Y YT z TY e ea d s s- - + - ¥ + ¥ ¥= =

The corresponding upper limit of the confidence interval for (4)S is the cubic root of the upper limit for (4)Y :

131,325.6483 10.9852U = =

Solution 15.03

D Sharpe Ratio and Arbitrage

The Sharpe ratio of Asset 1 is greater than that of Asset 2:

0.08 0.04 0.07 0.04

0.23 0.250.1739 0.12

Page 362: ActuarialBrew.com Exam MFE / 3F Actuarial Models …actuarialbrew.com/data/documents/MFE-Solutions-2014-1st-ed.pdf · Exam MFE / 3F Actuarial Models Financial Economics Segment Solutions

Exam MFE/3F Solutions Chapter 15 – The Sharpe Ratio and Itô’s Lemma

© ActuarialBrew.com 2014 Page 15.02

Since both assets follow geometric Brownian motion, a strategy that involves purchasing s1 11/( )S shares of Asset 1 results in an arbitrage profit of:

a as s

È ˘- - - -È ˘- = - =Í ˙ Í ˙Î ˚Î ˚1 2

1 2

0.08 0.04 0.07 0.040.053913

0.23 0.25r r

dt dt dt

Therefore a strategy that involves purchasing 1 share of Asset 1 results in an arbitrage profit of:

s ¥ = =1 1 0.053913 (0.23)(20)(0.053913) 0.248S dt dt dt

Solution 15.04

E Sharpe Ratio and Arbitrage

The Sharpe ratio of Stock Q is greater than that of Stock X:

0.07 0.03 0.05 0.03

0.12 0.110.3333 0.1818

Since both assets follow geometric Brownian motion, a strategy that involves purchasing s1/( )Q QS shares of Stock Q and selling s1/( )X XS shares of Stock X results an arbitrage

profit of:

a as s

È ˘- - - -È ˘- = - =Í ˙ Í ˙Î ˚Í ˙Î ˚

0.07 0.03 0.05 0.030.1515

0.12 0.11Q X

Q X

r rdt dt dt

Therefore a strategy that involves selling 1 share of Stock X results in an arbitrage profit of:

0.1515 (0.11)(182)(0.1515) 3.033X XS dt dt dt

Solution 15.05

B Sharpe Ratio

Stock C has a risk-free return of aC . Therefore, the continuously compounded risk-free rate of return is aC . So, the Sharpe ratio of Asset A is:

a a

a- -= =

3 3 10.333

6 6C C

C

Asset B is perfectly positively correlated with Asset A, so it has the same Sharpe ratio as Asset A.

Page 363: ActuarialBrew.com Exam MFE / 3F Actuarial Models …actuarialbrew.com/data/documents/MFE-Solutions-2014-1st-ed.pdf · Exam MFE / 3F Actuarial Models Financial Economics Segment Solutions

Exam MFE/3F Solutions Chapter 15 – The Sharpe Ratio and Itô’s Lemma

© ActuarialBrew.com 2014 Page 15.03

Solution 15.06

C Sharpe Ratio and Arbitrage

The Sharpe ratio of Stock Q is greater than that of Stock X:

0.09 0.06 0.08 0.06

0.25 0.200.12 0.10

Since both assets follow geometric Brownian motion, a strategy of purchasing s1/( )Q QS

shares of Stock Q and selling s1/( )X XS shares of Stock X results an arbitrage profit of:

a as s

È ˘- - - -È ˘- = - =Í ˙ Í ˙Î ˚Í ˙Î ˚

0.09 0.06 0.08 0.060.02

0.25 0.20Q X

Q X

r rdt dt dt

The strategy also calls for lending $1:

( ) ( )s s- = - =1/ 1 / 1 / 0.20 1/ 0.25 1X Q

The investor is allowed to lend $200, so the arbitrage can be scaled up by a factor of:

=200200

1

Therefore, the arbitrage profit is:

¥ =200 0.02 4dt dt

Solution 15.07

A Market Price of Risk and Arbitrage

The market prices of risk for Stock X and Stock Q must be equal or arbitrage would be possible:

a

aa

-- =-

- - = -

=

0.060.08 0.060.20 0.25

0.25(0.08 0.06) 0.20( 0.06)

0.035

Q

Q

Q

Notice that Qa is less than the risk-free rate of return. From Asset X we can see that

assets that are positively related to Z have a positive risk premium. This implies that assets such as Asset Q that are negatively related to Z must have a negative risk premium.

Page 364: ActuarialBrew.com Exam MFE / 3F Actuarial Models …actuarialbrew.com/data/documents/MFE-Solutions-2014-1st-ed.pdf · Exam MFE / 3F Actuarial Models Financial Economics Segment Solutions

Exam MFE/3F Solutions Chapter 15 – The Sharpe Ratio and Itô’s Lemma

© ActuarialBrew.com 2014 Page 15.04

Solution 15.08

D Market Price of Risk and Arbitrage

The market prices of risk for Stock X and Stock Q must be equal or arbitrage would be possible:

- -=-

- - = -+ = +

=

0.08 0.040.20 0.250.25(0.08 ) 0.20(0.04 )

0.25 0.20 0.02 0.0080.0622

r r

r rr r

r

Solution 15.09

B Sharpe Ratio

Since the derivative is perfectly positively correlated with the stock, it has the same Sharpe ratio as the stock:

as- -= = =0.07 0.02 0.05

0.250.20 0.20

r

Solution 15.10

C Valuing a Claim on Sa

We have:

0.5

0.5 1.50.5 0.25 0S SS t

V S

V S V S V

From Itô's Lemma and the multiplication rules:

2

0.5 1.5 2

20.5 1.5

0.5 0.5 1.5 2

0.5 0.5 0.5

0.5

1( )

2

0.5 0.5( 0.25) ( ) 0

0.5 0.07 0.20 0.125 0.07 0.20

0.035 0.10 0.125 0.04

0.035 0.10 0.005

0.03

S SS tdG V dS V dS V dt

S dS S dS dt

S Sdt SdZ S Sdt SdZ

S dt S dZ S S dt

S dt S dZ S dt

S

0.50.100.03 0.10

dt S dZGdt GdZ

The drift term is the first term:

0.03Gdt

Page 365: ActuarialBrew.com Exam MFE / 3F Actuarial Models …actuarialbrew.com/data/documents/MFE-Solutions-2014-1st-ed.pdf · Exam MFE / 3F Actuarial Models Financial Economics Segment Solutions

Exam MFE/3F Solutions Chapter 15 – The Sharpe Ratio and Itô’s Lemma

© ActuarialBrew.com 2014 Page 15.05

Solution 15.11

E Delta and Sa

The dividend yield of [ ]2(2)S is:

d d s= - - - - = - - - ¥ ¥ ¥ = -2 2* ( ) 0.5 ( 1) 0.05 2(0.05 0) 0.5 2 1 0.2 0.09r ra a a

The market-maker sold a prepaid forward of the 2S asset. The prepaid forward price is:

( *)( ) 0.09 2 2 0.18 2, ( ) ( )P T t

t TF S T e S t e S e S a a

The partial derivative of the prepaid forward price with respect to the stock price is:

( )∂

= = ¥ =∂

0.18 20.18 0.182 2 3 7.1833

e Se S e

S

The delta of the contingent claim is 7.1833. Since the market-maker sold the contingent claim, the delta of the market-maker’s un-hedged position is –7.1833. To bring the position’s delta to zero, the market-maker purchases 7.1833 shares of stock.

Solution 15.12

E Power Option

The dividend yield of [ ]2(1)S is:

d d s= - - - - = - - - ¥ ¥ ¥ = -2 2* ( ) 0.5 ( 1) 0.05 2(0.05 0) 0.5 2 1 0.2 0.09r ra a a

The usual expression for put-call parity is:

-+ = +0, ( )rT PTC Ke F S P

In this case, the underlying asset is 2S and the strike asset is 2K :

d

-

-

- -

+ = +

- = -

- = -

2 20,

2 20,

( *) 2 2

( )

( )

rT PT

P rTT

T rT

C K e F S P

C P F S K e

C P e S K e

To find the delta of the position consisting of a long call and a short put, we take the partial derivative of both sides with respect to S:

( ) ( )d

d- -

- ¥∂ ¥ -∂ -

= = ¥ + = ¥ ¥ =∂ ∂

( *) 2 2( *) 0.09 22 0 2 3 7.1833

T rTT

e S K eC PS e e

S S

Since the delta of the un-hedged position is 7.1833, the market-maker brings the delta to zero by selling 7.1833 shares of stock.

Page 366: ActuarialBrew.com Exam MFE / 3F Actuarial Models …actuarialbrew.com/data/documents/MFE-Solutions-2014-1st-ed.pdf · Exam MFE / 3F Actuarial Models Financial Economics Segment Solutions

Exam MFE/3F Solutions Chapter 15 – The Sharpe Ratio and Itô’s Lemma

© ActuarialBrew.com 2014 Page 15.06

Solution 15.13

A Valuing a Claim on Sa

Since the stock is governed by geometric Brownian motion, we can use the following formula:

2( ) ( ) 0.5 ( 1)d S S dt S dZ a a aa a a a

We have:

a d s= = = =0.5 0.09 0.02 0.20a

Putting these values into the formula gives us:

( ) ( )( )

= - + - +

= - +

= +

0.5 2 0.5 0.50.5(0.09 0.02) 0.5(0.5)( 0.5)(0.20) (0.5)(0.20)

0.035 0.005 0.10

0.03 0.10

d S S dt S dZ

Sdt SdZ

Sdt SdZ

Since the expected change in ( )Z t is zero, the expected change in the function is:

= =0.03 0.03 60 0.232Sdt dt dt

Solution 15.14

A Risk-Neutral Process

The risk-neutral price process is:

d s= - + ( )( ) ( )

( )dS t

r dt dZ tS t

We observe that:

m d= -r and s = 0.2

The dividend yield of 3S is:

d d s d d= - - - - = - - - ¥ ¥ ¥ = - - -2 2* ( ) 0.5 ( 1) 3( ) 0.5 3 2 0.2 3( ) 0.12r r r r r ra a a

The formula for the forward price of 3S is:

( *)( ), ( ) ( )r T t

t TF S T e S ta a

Setting this expression equal to the forward price provided in statement (ii), we have:

[ 3( ) 0.12]( ) 3 3 0.36( )( ) ( )3( ) 0.12 0.36

0.080.08

r r r T t T te S t S t er

r

Page 367: ActuarialBrew.com Exam MFE / 3F Actuarial Models …actuarialbrew.com/data/documents/MFE-Solutions-2014-1st-ed.pdf · Exam MFE / 3F Actuarial Models Financial Economics Segment Solutions

Exam MFE/3F Solutions Chapter 15 – The Sharpe Ratio and Itô’s Lemma

© ActuarialBrew.com 2014 Page 15.07

Solution 15.15

E Itô’s Lemma

We have:

- -

=

= ¥ = = - =

2

1 22

ln

12 2 2 0S SS t

V S

V S S V S VS

From Itô’s Lemma:

2

1 2 2

1 2 2

1 2

1 2 1

1ln ( )

21

2 2 ( ) 02

2 (0.09 0.25 ) (0.25)

(0.18 0.50 ) 0.0625

0.18 0.0625 0.50

S SS tdV d S V dS V dS V dt

S dS S dS dt

S dt dZ S dt

S dt dZ S dt

S S dt S dZ

Solution 15.16

B Itô’s Lemma

We have:

- -

=

= ¥ = = - =

2

1 22

ln1

2 2 2 0S SS t

V S

V S S V S VS

From Itô’s Lemma and the multiplication rules:

2

1 2 2

1 1 2 2

2

2

1[ln ( )] ( )

21

2 2 ( ) 02

2 0.4[0.12 ] 2 0.33 (0.33)

0.096 0.1089 0.660.8

0.096 0.1089 0.660.8

S SS td S t V dS V dS V dt

S dS S dS

S S dt S dZ S dt

dt dZS SS

dt dZS SS

Page 368: ActuarialBrew.com Exam MFE / 3F Actuarial Models …actuarialbrew.com/data/documents/MFE-Solutions-2014-1st-ed.pdf · Exam MFE / 3F Actuarial Models Financial Economics Segment Solutions

Exam MFE/3F Solutions Chapter 15 – The Sharpe Ratio and Itô’s Lemma

© ActuarialBrew.com 2014 Page 15.08

Solution 15.17

D Geometric BM Equivalencies and the Sharpe Ratio

A lognormal stock price implies that changes in the stock price follow geometric Brownian motion, and vice versa:

a d s s a d s- - += ¤ = - +2( 0.5 ) ( )( ) (0) ( ) ( ) ( ) ( )X X X Xt Z t

X X XX t X e dX t X t dt X t dZ

We can use the equation provided in the question to find values for aX and sX in the price process for Stock X:

s

a d s

a

a

+ += fi =

==

= - -

= - -

= + =

0.14 0.07 0.42 ( ) 0.14 0.07 0.42 ( )

0.14

2

2

2

( ) 15 ( ) 15

(0) 15

0.42

0.07 0.5

0.07 0 0.5(0.42 )

0.07 0.5(0.42 ) 0.1582

t Z t t Z t

X

X X X

X

X

X t e e X t e e

X e

The price process for Stock X is therefore:

( ) 0.1582 ( ) 0.42 ( ) ( )dX t X t dt X t dZ t

From the price process for Stock Q, we observe that:

as

=

=

0.08

0.13Q

Q

Stock X and Stock Q are perfectly correlated since they are driven by the same stochastic process, so they must have the same Sharpe ratio:

- -=

- = -= -

=

0.1582 0.080.42 0.13

0.13(0.1582) 0.13 0.42(0.08) 0.420.29 0.42(0.08) 0.13(0.1582)

0.04494

r r

r rr

r

Solution 15.18

A Itô’s Lemma

We have:

== = =

2

2 2 0S SS t

V SV S V V

Page 369: ActuarialBrew.com Exam MFE / 3F Actuarial Models …actuarialbrew.com/data/documents/MFE-Solutions-2014-1st-ed.pdf · Exam MFE / 3F Actuarial Models Financial Economics Segment Solutions

Exam MFE/3F Solutions Chapter 15 – The Sharpe Ratio and Itô’s Lemma

© ActuarialBrew.com 2014 Page 15.09

From Itô’s Lemma and the multiplication rules:

[ ]

= + + = + ¥ +

= + + = + +

2 2 2

2

1 1[ ( ) ] ( ) 2 2( ) 0

2 2

2 0.09 0.25 0.25 [0.18 0.0625] 0.5

S SS td S t V dS V dS V dt SdS dS

S dt dZ dt S dt SdZ

Solution 15.19

E Risk-Neutral Process

The risk-neutral price process is of the form:

d s= - + ( )( ) ( )

( )dS t

r dt dZ tS t

Therefore, we have:

d

dd

- =- =

=

0.050.08 0.05

0.03

r

The true price process is of the form:

a d s= - +( )( ) ( )

( )dS t

dt dZ tS t

Therefore, we have:

a da- == + =

0.090.09 0.03 0.12

Solution 15.20

C Risk-Neutral Process

The true price process is of the form:

a d s= - +( )( ) ( )

( )dS t

dt dZ tS t

Therefore, we have:

a dd- == - =

0.130.15 0.13 0.02

The risk-neutral price process is of the form:

d s= - + ( )( ) ( )

( )dS t

r dt dZ tS t

Therefore, we have:

d- =

= + =0.08

0.08 0.02 0.10rr

Page 370: ActuarialBrew.com Exam MFE / 3F Actuarial Models …actuarialbrew.com/data/documents/MFE-Solutions-2014-1st-ed.pdf · Exam MFE / 3F Actuarial Models Financial Economics Segment Solutions

Exam MFE/3F Solutions Chapter 15 – The Sharpe Ratio and Itô’s Lemma

© ActuarialBrew.com 2014 Page 15.10

Solution 15.21

B Risk-Neutral Process and Sharpe Ratio

The risk-neutral price process is of the form:

d s= - + ( )( ) ( )

( )dS t

r dt dZ tS t

Since the dividend rate is 0:

d- =- ==

0.07

0 0.070.07

r

rr

The volatility is:

s = 0.30

The Sharpe ratio is:

as- -= =0.13 0.07

0.200.30

r

Solution 15.22

E Itô’s Lemma

Let’s call the expression in the first given statement ( )V t :

[ ]a b= = -Ú 2

0( ) ( ) ( ) ( )

tV t Z s dZ s Z t t

By definition, we have:

È ˘= =Í ˙Î ˚Ú0( ) ( ) ( ) ( ) ( )t

dV t d Z s dZ s Z t dZ t

We can find another expression for ( )dV t using Itô’s Lemma and [ ]a b= -2( ) ( )V t Z t t .

The partial derivatives are:

a a b= = = -2 2 Z ZZ tV Z V V

From Itô’s Lemma, we have:

a a ba a b

= + += + -= + -

2( ) ( ) 0.5 [ ( )]

2 ( ) ( ) 0.5(2 )

2 ( ) ( ) ( )

Z ZZ tdV t V dZ t V dZ t V dt

Z t dZ t dt dt

Z t dZ t dt

Since we previously saw that =( ) ( ) ( )dV t Z t dZ t , we can find the values of a and b by

noting that the coefficient to ( ) ( )Z t dZ t is 1 and that the coefficient to dt is zero:

a aa b b

= fi =- = fi =

2 1 0.50 0.5

Page 371: ActuarialBrew.com Exam MFE / 3F Actuarial Models …actuarialbrew.com/data/documents/MFE-Solutions-2014-1st-ed.pdf · Exam MFE / 3F Actuarial Models Financial Economics Segment Solutions

Exam MFE/3F Solutions Chapter 15 – The Sharpe Ratio and Itô’s Lemma

© ActuarialBrew.com 2014 Page 15.11

The value of a b+ is:

a b+ = + =0.5 0.5 1

Solution 15.23

B Risk-Neutral Process

In the Black-Scholes framework:

a d s= - +( )( ) ( )

( )dS t

dt dZ tS t

We can use statements (ii) and (iii) in the question to find a and s :

a d a as- = fi - = fi ==

0.08 0.04 0.08 0.120.30

The relationship between ( )Z T and ( )Z T is described by:

as-= + ( ) ( )

rZ T Z T T

Under the risk-neutral probability measure ( )Z T is a pure Brownian motion, so:

[ ]

[ ]

as

as

as

-È ˘È ˘ = +Í ˙Î ˚ Î ˚-= +

-=

* *

*

*

( ) ( )

0 ( )

( )

rE Z T E Z T T

rE Z T T

rE Z T T

Since the expectation of (0.25)Z is -0.05 , we have:

[ ] as-= ¥

-- = ¥

=

* (0.25) 0.25

0.120.05 0.25

0.300.06

rE Z

r

r

Solution 15.24

C Market Price of Risk

When the price follows geometric Brownian motion, the natural log of the price follows arithmetic Brownian motion:

[ ] ( )a s a s s= + ¤ = - +2( ) ( ) ( ) ( ) ln ( ) 0.5dS t S t dt S t dZ t d S t dt dZ

Page 372: ActuarialBrew.com Exam MFE / 3F Actuarial Models …actuarialbrew.com/data/documents/MFE-Solutions-2014-1st-ed.pdf · Exam MFE / 3F Actuarial Models Financial Economics Segment Solutions

Exam MFE/3F Solutions Chapter 15 – The Sharpe Ratio and Itô’s Lemma

© ActuarialBrew.com 2014 Page 15.12

Therefore:

[ ] ( )= + ¤ = - +2( )( ) ln ( ) 0.5

( )dY t

Gdt HdZ t d Y t G H dt HdZY t

The arithmetic Brownian motion provided in the question for [ ]ln ( )d Y t allows us to find

an expression for H and G:

[ ] [ ] ( )s

s

= + = - +

=

- =

= +

2

2

2

ln ( ) 0.07 ( ) and ln ( ) 0.5

0.5 0.07

0.07 0.5

d Y t dt dZ t d Y t G H dt HdZ

H

G H

G H

Since X and Y have the same source of randomness, ( )dZ t , they must have the same market price of risk:

- -=

-=

+ -=

= +

- + =

2

2

2

0.09 0.05 0.050.16

0.050.25

0.07 0.5 0.050.25

0.25 0.02 0.5

0.5 0.25 0.02 0

GH

GH

HH

H H

H H

We use the quadratic formula to solve for H:

20.25 ( 0.25) 4(0.5)(0.02)

0.1 or 0.42(0.5)

H

Since we are given that s < 0.30 and we know that s=H , it must be the case that:

= 0.1H

We can now find the value of G:

0.05 0.05

0.250.1

0.075

G GH

G

Page 373: ActuarialBrew.com Exam MFE / 3F Actuarial Models …actuarialbrew.com/data/documents/MFE-Solutions-2014-1st-ed.pdf · Exam MFE / 3F Actuarial Models Financial Economics Segment Solutions

Exam MFE/3F Solutions Chapter 15 – The Sharpe Ratio and Itô’s Lemma

© ActuarialBrew.com 2014 Page 15.13

Solution 15.25

C Valuing a Claim on Sa

The stock’s price follows geometric Brownian motion, and the claim pays (5)S a , where = -1a . Therefore, the forward price is:

2

2

( ) 0.5 ( 1)( *)( )0,

1(0.07 0.02) 0.5( 1)( 2)0.25 5 1 0.0125 5

( ) (0) (0)

2 0.5 0.5322

r Tr TTF S T e S e S

e e

a a aa a a

Solution 15.26

B Valuing a Claim on Sa

The drift term of the Itô process can be expressed in terms of the expected return on the stock and the dividend yield of the stock:

a d

aa

= -= -

=

0.08 ( )

0.08 0.020.10

dt dt

The expected return on the stock is 10%.

The stock’s price follows geometric Brownian motion, and the claim pays ( )S T a , where = -1a . The expected return on the claim is therefore:

g a= - + = - - + =( ) 1(0.10 0.07) 0.07 0.04r ra

The ratio of the expected return on the claim to the expected return on the stock is:

ga

= =0.040.40

0.10

Solution 15.27

B Valuing a Claim on Sa

The stock price follows geometric Brownian motion, and the claim pays ( )S T a , where = 0.5a . The prepaid forward price is therefore:

2

2

( ) 0.5 ( 1)( *)( )0,

[0.08 0.5(0.08 0.03) 0.5(0.5)( 0.5)0.3 ]2

( ) (0) (0)

100 8.7590

r r TP TTF S T e S e S

e

a a aa a a

Page 374: ActuarialBrew.com Exam MFE / 3F Actuarial Models …actuarialbrew.com/data/documents/MFE-Solutions-2014-1st-ed.pdf · Exam MFE / 3F Actuarial Models Financial Economics Segment Solutions

Exam MFE/3F Solutions Chapter 15 – The Sharpe Ratio and Itô’s Lemma

© ActuarialBrew.com 2014 Page 15.14

Solution 15.28

A Valuing a Claim on Sa

The stock price follows geometric Brownian motion, and the claim pays ( )S T a , where = 2a . The dividend rate on the claim is therefore:

d d s= - - - - = - - - - = -2 2* ( ) 0.5 ( 1) 0.04 2(0.04 0) 0.5(2)(2 1)(0.2) 0.08r ra a a

Solution 15.29

D Valuing a Claim on Sa

The stock price follows geometric Brownian motion, and the claim pays ( )S T a , where = 2a . The forward price on the claim is therefore:

2

2 2

( ) 0.5 ( 1)( *)( )0,

( ) 0.5 ( 1) 2 0.5(2)(2 1)(0.25) (0.5)

( ) (0) (0)

(0) (10.305)

109.56

r Tr TT

r T T

F S T e S e S

S e e e

a a aa a a

a a a

Solution 15.30

D Valuing a Claim on Sa

The forward price of an asset is its current price accumulated at the risk-free rate minus the asset’s dividend rate:

2 2 ( *)( ), ( ) ( ) r T t

t TF S T S t e

The forward price is equal to the current price of the derivative:

2 2,

2 ( *)( ) 2

( *)( )

( ) ( )

( ) ( )

1

( *)( ) 0

*

* 0.10

t T

r T t

r T t

F S T S t

S t e S t

e

r T t

r

Solution 15.31

E Valuing a Claim on Sa

The forward price of the derivative is:

2( ) 0.5 ( 1) ( )( *)( )

, ( ) (0) ( )r T tr T

t TF S T S e S t ea a aa a a

Page 375: ActuarialBrew.com Exam MFE / 3F Actuarial Models …actuarialbrew.com/data/documents/MFE-Solutions-2014-1st-ed.pdf · Exam MFE / 3F Actuarial Models Financial Economics Segment Solutions

Exam MFE/3F Solutions Chapter 15 – The Sharpe Ratio and Itô’s Lemma

© ActuarialBrew.com 2014 Page 15.15

The forward price is equal to the current price of the derivative:

2

2

2 2,

( ) 0.5 ( 1) ( ) 2

2(0.10 ) 0.5(2)(2 1)(0.40) ( )2 2

0.20 2 0.16 ( )

( ) ( )

( ) ( )

( ) ( )

1

(0.36 2 )( ) 0

0.36 2 0

0.18

t T

r T t

T t

T t

F S T S t

S t e S t

S t e S t

e

T t

a a aa

Solution 15.32

A Prepaid Forward Price of Sa

We make use of the following expression for the prepaid forward price of the derivative:

( *)( ), ( ) ( )P T t

t TF S T e S ta a

We can substitute this expression for the prepaid forward price in the equation provided in the question:

2

,

( *)( )

( ) 0.5 ( 1) ( )

2

2

2 2 2

2

2

( ) ( )

( ) ( )

1

( ) 0.5 ( 1) ( ) 0

( ) 0.5 ( 1) 0

0.16 (0.16 0.00) 0.5 (0.10) 0.5 (0.10) 0

0.005 0.155 0.16 0

5

P x xt T

T t x x

r x r x x T t

F S T S t

e S t S t

e

r x r x x T t

r x r x x

x x x

x x

x

2

155 160 0

155 155 4(5)( 160)2(5)

1 or 32

x

x

x x

We were told in the question that 1 is a solution. The other solution is -32.

Page 376: ActuarialBrew.com Exam MFE / 3F Actuarial Models …actuarialbrew.com/data/documents/MFE-Solutions-2014-1st-ed.pdf · Exam MFE / 3F Actuarial Models Financial Economics Segment Solutions

Exam MFE/3F Solutions Chapter 15 – The Sharpe Ratio and Itô’s Lemma

© ActuarialBrew.com 2014 Page 15.16

Solution 15.33

E Risk-Neutral Process

The risk-neutral price process for ( )S t is:

d s= - + ( )( ) ( )

( )dS t

r dt dZ tS t

We are given that d = 0.03 and:

= + ( )0.04 0.12 ( )

( )dS t

dt dZ tS t

We can solve for r:

d- =

- ==

0.04

0.03 0.040.07

r

rr

The expected return is twice the risk-free rate of return:

a = ¥ = ¥ =2 2 0.07 0.14r

The expected value of the stock at the end of two years is:

a d- -È ˘+ = = =Î ˚( )2 (0.14 0.03)2( 2) ( ) ( ) 100 124.61E S t S t S t e e

Solution 15.34

C Valuing a Claim on Sa

Since the payoff consists of S(2) shares of stock at the end of 2 years, the value of this payoff at time 2 is:

2(2) (2) (2)S S S

The value of the derivative is therefore the prepaid forward price of ( )S T a , where = 2a and = 2T . The prepaid forward price is:

2

2

( ) 0.5 ( 1)( *)( )0,

[ 0.04 2(0.04 0.05) 0.5(2)(2 1)0.3 ]2 2

( ) (0) (0)

(12) 152.9045

r r TP TTF S T e S e S

e

a a aa a a

Solution 15.35

D Power Option

Since ( )S t is a geometric Brownian motion, so is [ ( )]S t a :

a d s a d s sÈ ˘= - + fi = - + - +Î ˚2( ) ( )

( ) ( ) ( ) 0.5 ( 1) ( )( )

dS t d Sdt dZ t dt dZ t

S t S

a

aa a a a

Page 377: ActuarialBrew.com Exam MFE / 3F Actuarial Models …actuarialbrew.com/data/documents/MFE-Solutions-2014-1st-ed.pdf · Exam MFE / 3F Actuarial Models Financial Economics Segment Solutions

Exam MFE/3F Solutions Chapter 15 – The Sharpe Ratio and Itô’s Lemma

© ActuarialBrew.com 2014 Page 15.17

The coefficient of ( )dZ t is the asset’s volatility. For 3[ ( )]S t , let’s call this volatility s * :

s s= = ¥ =* 3 0.3 0.9a

The dividend yield on 3[ ( )]S t is:

d d s= - - - - = - - - - =2 2* ( ) 0.5 ( 1) 0.15 3(0.15 0.24) 0.5(3)(3 1)(0.3) 0.15r ra a a

The initial price of 3[ ( )]S t is:

= =3 3[ (0)] 10 1,000S

We can use the Black-Scholes formula to find the value of a 1-year European call option with 3[ ( )]S t as the underlying asset and $900 as the strike price. This option has a payoff

of -3[ (1)] 900S when >3[ (1)] 900S and zero otherwise.

The first step is to calculate 1d and 2d :

( )3 2

1

2

2 1

ln( / ) ( *) 0.5( *)

( *)

ln(1,000 / 900) (0.15 0.15 0.5 0.9 ) 10.56707

0.9 1

( *) 0.56707 0.9 1 0.33293

S K r Td

T

d d T

d s

s

s

+ - +=

+ - + ¥ ¥= =

= - = - = -

We have:

1

2

( ) (0.56707) 0.71467

( ) ( 0.33293) 0.36959

N d N

N d N

The value of the European power call option is:

3 ( *)1 2

0.15 1 0.15 1

( ) ( )

1,000 0.71467 900 0.36959 328.8240

T rTEurC S e N d Ke N d

e e

Solution 15.36

E Market Price of Risk

The market price of risk of Asset 1 must be equal to that of Asset 2:

0.08 0.04 0.02 0.04

0.250.125

Page 378: ActuarialBrew.com Exam MFE / 3F Actuarial Models …actuarialbrew.com/data/documents/MFE-Solutions-2014-1st-ed.pdf · Exam MFE / 3F Actuarial Models Financial Economics Segment Solutions

Exam MFE/3F Solutions Chapter 15 – The Sharpe Ratio and Itô’s Lemma

© ActuarialBrew.com 2014 Page 15.18

Over the next instant, the returns on Stock 1 and Stock 2 are:

= + = + = += - = - = -

1 1 1

2 2 2

Return on S 0.08 0.250 0.08(75) 0.250(75) 6.0 18.75

Return on S 0.02 0.125 0.02(25) 0.125(25) 0.5 3.125

S dt S dZ dt dZ dt dZ

S dt S dZ dt dZ dt dZ

Since the investor purchases 1 share of Stock 1, the amount of Stock 2 that must be purchased to remove the random term (i.e., the dZ term) is:

= =18.756

3.125N

When the return on Stock 1 is added to the return on 6 shares of Stock 2, there is no random term:

= += -=

1

2

Return on S 6.0 18.75

Return on 6 shares of S 3.0 18.75

9.0

dt dZ

dt dZ

dt

Solution 15.37

E Market Price of Risk

The market price of risk of Asset 1 must be equal to that of Asset 2:

0.08 0.04 0.02 0.04

0.250.125

Over the next instant, the returns on Stock 1 and Stock 2 are:

= + = + = += - = - = -

1 1 1

2 2 2

Return on S 0.08 0.250 0.08(75) 0.250(75) 6.0 18.75

Return on S 0.02 0.125 0.02(25) 0.125(25) 0.5 3.125

S dt S dZ dt dZ dt dZ

S dt S dZ dt dZ dt dZ

Since the investor purchases 1 share of Stock 1, the amount of Stock 2 that must be purchased to remove the random term (i.e., the dZ term) is:

= =18.756

3.125N

The cost of purchasing 1 share of Stock 1 and 6 shares of Stock 2 is:

¥ + ¥ =1 75 6 25 225

Since the portfolio is a zero-investment portfolio, $225 is borrowed.

Solution 15.38

B Itô’s Lemma

The expression for ( )G t is:

= 2G t S

Page 379: ActuarialBrew.com Exam MFE / 3F Actuarial Models …actuarialbrew.com/data/documents/MFE-Solutions-2014-1st-ed.pdf · Exam MFE / 3F Actuarial Models Financial Economics Segment Solutions

Exam MFE/3F Solutions Chapter 15 – The Sharpe Ratio and Itô’s Lemma

© ActuarialBrew.com 2014 Page 15.19

The partial derivatives are:

==

=

2

0

2

S

SS

t

G t

G

G tS

From Itô’s Lemma, we have:

= + +

= + ¥ ¥ +

= + +

= + +

2

2 2

2

2 2

1( ) ( ) [ ( )]

2

( ) 0.5 0 [ ( )] 2

[0.5 0.4 ] 22

[0.5 0.4 ]

S SS tdG t G dS t G dS t G dt

t dS t dS t tSdt

t Sdt SdZ tSdt

t S dt dZ t S dtt

Since 2G t S , let’s divide both sides by 2t S and reorganize the expression:

= + +

Ê ˆ= + +Á ˜Ë ¯

+Ê ˆ= +Á ˜Ë ¯

( ) 2[0.5 0.4 ]

( )

20.5 0.4

0.5 20.4

dG tdt dZ dt

G t t

dt dZt

tdt dZ

t

Solution 15.39

B Itô’s Lemma

The expression for ( )G t is:

= tG S

The partial derivatives are:

-

-

=

= -

=

1

2( 1)

ln

tS

tSS

tt

G tS

G t t S

G S S

From Itô’s Lemma, we have:

[ ]- -

- -

= + +

= + - +

= - + + - +

= - + + - +

2

1 2 2

1 2 2

1( ) ( ) [ ( )]

2

( ) 0.5 ( 1) [ ( )] ln

0.5 0.5 ( 1) ln

0.5 0.5 ( 1) ln

S SS t

t t t

t t t

t t t t

dG t G dS t G dS t G dt

tS dS t t t S dS t S Sdt

tS Sdt SdZ t t S S dt S Sdt

tS dt tS dZ t t S dt S Sdt

Page 380: ActuarialBrew.com Exam MFE / 3F Actuarial Models …actuarialbrew.com/data/documents/MFE-Solutions-2014-1st-ed.pdf · Exam MFE / 3F Actuarial Models Financial Economics Segment Solutions

Exam MFE/3F Solutions Chapter 15 – The Sharpe Ratio and Itô’s Lemma

© ActuarialBrew.com 2014 Page 15.20

Since tG S , let’s divide both sides by tS and reorganize the expression:

( )

= - + + - +

= - + - + +

= - + +

2

2

( )0.5 0.5 ( 1) ln

( )

0.5 0.5 0.5 ln

0.5 ln

dG ttdt tdZ t t dt Sdt

G t

tdt t dt tdt Sdt tdZ

t t S dt tdZ

We can find an expression for the natural log of S:

==

=

ln lnln

ln

tG SG t S

GS

t

Substituting this in for the natural log of S, we have:

Ê ˆ= - + +Á ˜Ë ¯2( ) ln ( )

0.5( )

dG t G tt t dt tdZ

G t t

Solution 15.40

D Valuing a Claim on Sa

The derivative is of the form Sa , and the derivative has a dividend yield of zero:

d =* 0

We can use the formula for the dividend yield of the claim to solve for a:

2

2

2 2 2

2 2 2

* ( ) 0.5 ( 1)

0 0.07 (0.07 0.00) 0.5 ( 1)

0 0.07 0.07 0.5 0.5

0 0.5 (0.5 0.07) 0.07

r r

a a a

a a a

a a a

a a

We make use of the quadratic formula:

2 4 2 2

2

2 4 2

2

2 2

2

2

0.07 0.5 0.25 0.07 0.0049 4( 0.5 )(0.07)

0.5 0.07 0.25 0.07 0.0049

0.5 0.07 (0.5 0.07)

0.14or 1

a

a

a

a a

Page 381: ActuarialBrew.com Exam MFE / 3F Actuarial Models …actuarialbrew.com/data/documents/MFE-Solutions-2014-1st-ed.pdf · Exam MFE / 3F Actuarial Models Financial Economics Segment Solutions

Exam MFE/3F Solutions Chapter 15 – The Sharpe Ratio and Itô’s Lemma

© ActuarialBrew.com 2014 Page 15.21

The question specifies that a is negative, so a cannot be equal to 1:

2

2 2

0.14

0.14

0.14

k

k

a

Solution 15.41

D Valuing a Claim on Sa

The derivative is of the form Sa , and the derivative has a dividend yield of equal the dividend yield of the stock:

d d= =* 0.03

We can use the formula for the dividend yield of the claim to solve for a:

2

2

2 2 2

2 2 2

* ( ) 0.5 ( 1)

0.03 0.09 (0.09 0.03) 0.5 ( 1)

0 0.06 0.06 0.5 0.5

0 0.5 (0.5 0.06) 0.06

r r

a a a

a a a

a a a

a a

We make use of the quadratic formula:

2 2 2

2 4 2 2

2

2 4 2 2

2

2 4 2

2

2 2 2

2

0 0.5 (0.5 0.06) 0.06

0.5 0.06 [0.25 0.06 0.0036] 4( 0.5 )(0.06)

0.5 0.06 0.25 0.06 0.0036 0.12

0.5 0.06 0.25 0.06 0.0036

0.5 0.06 (0.5 0.06)

0.

a a

a

a

a

a

a2 2

2

2

5 0.06 (0.5 0.06)

0.12or 1

a a

Page 382: ActuarialBrew.com Exam MFE / 3F Actuarial Models …actuarialbrew.com/data/documents/MFE-Solutions-2014-1st-ed.pdf · Exam MFE / 3F Actuarial Models Financial Economics Segment Solutions

Exam MFE/3F Solutions Chapter 15 – The Sharpe Ratio and Itô’s Lemma

© ActuarialBrew.com 2014 Page 15.22

The question specifies that a is negative, so a cannot be equal to 1:

2

2 2

0.12

0.12

0.12

k

k

a

Solution 15.42

D Market Price of Risk

The market price of risk of Stock 1 must be equal to that of Stock 2:

0.10 0.08 0.06 0.08

0.050.05

kk

Over the next instant, the returns on Stock 1 and Stock 2 are:

1 1 1

2 2 2

Return on S 0.10 0.05 0.10(80) 0.05(80) 8.0 4.0

Return on S 0.06 0.05 0.06(40) 0.05(40) 2.4 2.0

S dt S dZ dt dZ dt dZ

S dt S dZ dt dZ dt dZ

Since the investor purchases 1 share of Stock 1, the amount of Stock 2 that must be purchased to remove the random term (i.e., the dZ term) is:

4

22

When the return on Stock 1 is added to the return on 2 shares of Stock 2, there is no random term:

1

2

Return on 8.0 4.0

Return on 2 shares of 2 (2.4 2.0 )

12.8

S dt dZ

S dt dZ

dt

Solution 15.43

B Market Price of Risk

We provide 2 solutions to this question. The first solution does not require us to determine k.

Stock 1 has an expected return of 12% and Stock 2 has an expected return of 8.4%. Since the portfolio earns the risk-free rate of 10%, we have:

+ - =+ - ==

=

0.12 % 0.084(1 %) 0.100.12 % 0.084 0.084 % 0.100.036 % 0.016

% 0.4444

x xx x

xx

Page 383: ActuarialBrew.com Exam MFE / 3F Actuarial Models …actuarialbrew.com/data/documents/MFE-Solutions-2014-1st-ed.pdf · Exam MFE / 3F Actuarial Models Financial Economics Segment Solutions

Exam MFE/3F Solutions Chapter 15 – The Sharpe Ratio and Itô’s Lemma

© ActuarialBrew.com 2014 Page 15.23

Alternative Solution

The market price of risk of Stock 1 must be equal to that of Stock 2:

0.12 0.10 0.084 0.10

0.050.04

kk

The resulting differential equations are:

1 1 1

2 2 2

( ) 0.12 ( ) 0.05 ( ) ( )

( ) 0.084 ( ) 0.04 ( ) ( )

dS t S t dt S t dZ t

dS t S t dt S t dZ t

The quantities of Stock 1 and Stock 2 that are purchased are:

¥ ¥ -= =1 2

1 2

1,000 % 1,000 (1 %)( ) ( )

x xQ Q

S t S t

The partial differential equation describing the return on the portfolio is:

1 1 2 2

1 1 1 2 2 2

1 1 2 2 1 1 2 2

( ) ( )

0.12 ( ) 0.05 ( ) ( ) 0.084 ( ) 0.04 ( ) ( )

0.12 ( ) 0.084 ( ) 0.05 ( ) 0.04 ( ) ( )

Q dS t Q dS t

Q S t dt S t dZ t Q S t dt S t dZ t

S t Q S t Q dt S t Q S t Q dZ t

Since the portfolio earns the risk-free rate, the coefficient to the random term ( )dZ t must be zero:

1 1 2 2

1 1 2 2

1 21 2

0.05 ( ) 0.04 ( ) 0

0.05 ( ) 0.04 ( )

1,000 % 1,000 (1 %)0.05 ( ) 0.04 ( )

( ) ( )

0.05 % 0.04(1 %)0.09 % 0.04

% 0.4444

S t Q S t Q

S t Q S t Q

x xS t S t

S t S t

x xx

x

Solution 15.44

D Drift and Itô’s Lemma

The first equation and its partial derivatives are:

= -= = =

( ) 3 ( ) 53 U 0 0Z ZZ t

U t Z tU U

This results in:

21( , ) ( ) 3 0 0 3

2Z ZZ tdU Z t U dZ U dZ U dt dZ dZ

Since there is no dt term, the drift is zero for dU.

Page 384: ActuarialBrew.com Exam MFE / 3F Actuarial Models …actuarialbrew.com/data/documents/MFE-Solutions-2014-1st-ed.pdf · Exam MFE / 3F Actuarial Models Financial Economics Segment Solutions

Exam MFE/3F Solutions Chapter 15 – The Sharpe Ratio and Itô’s Lemma

© ActuarialBrew.com 2014 Page 15.24

The second equation and its the partial derivatives are:

[ ]= -

= = = -

3

2

( ) 2 ( ) 3

6 V 12 3Z ZZ t

V t Z t t

V Z Z V

This results in:

= + + = + ¥ -

= + - = + -

2 2 2

2 2

1 1( , ) ( ) 6[ ( )] 12 ( )( ) 3

2 2

6[ ( )] 6 ( ) 3 6[ ( )] [6 ( ) 3]

Z ZZ tdV Z t V dZ V dZ V dt Z t dZ Z t dZ dt

Z t dZ Z t dt dt Z t dZ Z t dt

Since the dt term is nonzero, the drift is nonzero for dV.

For the third equation, we are given:

= - Ú3 2

0

( ) ( ) 3 ( )t

W t t Z t s Z s ds

Let’s deal with the second term first. We have:

È ˘ È ˘Í ˙ Í ˙= =Í ˙ Í ˙Î ˚ Î ˚Ú Ú2 2 2

0 0

3 ( ) 3 ( ) 3 ( )t t

d s Z s ds d s Z s ds t Z t dt

We can now write the differential equation as:

È ˘= -Î ˚3 2( ) ( ) 3 ( )dW t d t Z t t Z t dt

For the first term, we must use Itô’s Lemma. Let’s define:

= 3( ) ( )X t t Z t

The partial derivatives are:

= = =3 2 0 3Z ZZ tX t X X t Z

This results in:

= + + = + ¥ ¥ +

= +

2 3 2 2

3 2

1 1( ) ( ) 0 ( ) 3

2 2

( ) 3 ( )

Z ZZ tdX t X dZ X dZ X dt t dZ dZ t Zdt

t dZ t t Z t dt

Therefore:

È ˘= - = - = + -Î ˚

=

3 2 2 3 2 2

3

( ) ( ) 3 ( ) ( ) 3 ( ) ( ) 3 ( ) 3 ( )

( )

dW t d t Z t t Z t dt dX t t Z t dt t dZ t t Z t dt t Z t dt

t dZ t

Since there is no dt term, the drift is zero for dW.

Page 385: ActuarialBrew.com Exam MFE / 3F Actuarial Models …actuarialbrew.com/data/documents/MFE-Solutions-2014-1st-ed.pdf · Exam MFE / 3F Actuarial Models Financial Economics Segment Solutions

Exam MFE/3F Solutions Chapter 15 – The Sharpe Ratio and Itô’s Lemma

© ActuarialBrew.com 2014 Page 15.25

Solution 15.45

B Itô’s Lemma & Ornstein-Uhlenbeck Process

Notice that the stochastic process for ( )R t is an Ornstein-Uhlenbeck process with 1, 0.05, and 0.10 :

( )( )

l l la s l a s- - - -

- - -

= + - + ¤ = ¥ - +

= + - + ¤ = - +

Ú

Ú

( )

0

0

( ) (0) 1 ( ) ( ) [ ( )] ( )

( ) (0) 0.05 1 0.10 ( ) ( ) [0.05 ( )] 0.10 ( )

tt t t s

tt t s t

X t X e e e dZ s dX t X t dt dZ t

R t R e e e dZ s dR t R t dt dZ t

The equation above for dR(t) describes the Vasicek Model for the short rate, which appears in Chapter 19 of the ActuarialBrew.com Study Manual.

Since 2X R , the partial derivatives of ( )X t are:

==

=

2

2

0

R

RR

t

X R

X

X

We can use Itô’s Lemma to find an expression for the differential of ( )X t :

= + +21( ) ( ) [ ( )]

2R RR tdX t X dR t X dR t X dt

Making use of the multiplication rules, we see that:

{ }= - + =22[ ( )] [0.05 ( )] 0.10 ( ) 0.01dR t R t dt dZ t dt

We can now find the differential of ( )X t . To simplify the notation, we use R for ( )R t and X for ( )X t below:

( )

= + +

= + ¥ +

= += - + +

= - + +

= - + +

2

2

2

2

2

1( )

21

2 2( ) 02

2 ( )

2 [0.05 ] 0.1 0.01

(0.10 2 ) 0.2 0.01

(0.10 2 0.01) 0.2

R RR tdX X dR X dR X dt

RdR dR dt

RdR dR

R R dt dZ dt

R R dt RdZ dt

R R dt RdZ

We are given that X is equal to 2R :

= fi =2X R X R

Page 386: ActuarialBrew.com Exam MFE / 3F Actuarial Models …actuarialbrew.com/data/documents/MFE-Solutions-2014-1st-ed.pdf · Exam MFE / 3F Actuarial Models Financial Economics Segment Solutions

Exam MFE/3F Solutions Chapter 15 – The Sharpe Ratio and Itô’s Lemma

© ActuarialBrew.com 2014 Page 15.26

Substituting for R, we have:

= - + +

= - + +

2(0.10 2 0.01) 0.2

(0.10 2 0.01) 0.2

dX R R dt RdZ

X X dt XdZ

Solution 15.46

E Itô’s Lemma

We have:

[ ]-= 1( ) ( )Y t X t

The partial derivatives are:

- -= - = =2 3 2 0X XX tY X Y X Y

From Itô’s Lemma and the multiplication rules:

[ ][ ]

- -

- -

- - - -

= + +

= - - + + ¥ ¥ +

= - - + +

È ˘= - + -Î ˚È ˘= - + -Î ˚

2

2 3

2 3

1 2 3 2

2 3 2

1( ) ( )

21

4(5 ) 6 2 36 02

20 4 6 36

4 20 36 6

4 20 36 6

X XX tdY t Y dX Y dX Y dt

X X dt dZ X dt

X dt Xdt dZ X dt

X X X dt X dZ

Y Y Y dt Y dZ

From the expression above, we can determine a ( )Y and b ( )Y :

a a

b b

= - + fi = - + =

= - fi = - = -

2 3 2 3

2 2

( ) 4 20 36 (0.5) 4(0.5) 20(0.5) 36(0.5) 1.5

( ) 6 (1) 6(1) 6

Y Y Y Y

Y Y

The solution is:

a b- = - - =(0.5) (1) 1.5 ( 6) 7.5

Page 387: ActuarialBrew.com Exam MFE / 3F Actuarial Models …actuarialbrew.com/data/documents/MFE-Solutions-2014-1st-ed.pdf · Exam MFE / 3F Actuarial Models Financial Economics Segment Solutions

Exam MFE/3F Solutions Chapter 15 – The Sharpe Ratio and Itô’s Lemma

© ActuarialBrew.com 2014 Page 15.27

Solution 15.47

C Valuing a Claim on Sa

The expected value can be used to solve for a:

2

2

[ ( ) 0.5 ( 1) ]( )

[ (0.08) 0.5 ( 1)0.4 ](1 0)

2

2

2

( ) ( )

1.7 0.6ln(1.7) ln(0.6) (0.08) 0.08 ( 1)

ln(1.7) ln(0.6) (0.08) 0.08 0.08

0 0.08 ln(0.6) ln(1.7)

ln(0.6) ln(0.6) 4(0.

T tE S T S t e

e

a a a a a

a a a a

a a a a

a a a a

a a

a 08) ln1.7)

2(0.08)0.90928 or 7.2946

a a

Since a is negative, its value must be –0.90928.

We can now calculate the expected return for the contingent claim:

g a= - + = - - + =( ) 0.90928(0.08 0.05) 0.05 0.02272r ra

Since we are given the expected value of the contingent claim at time 1, we can make use of it to find the time-0 price of the contingent claim:

0.022720, ( ) ( ) 1.7 1.662P T

TF S T e E S T e a a

Solution 15.48

B Sharpe Ratio

The price follows geometric Brownian motion:

a d s sa d s - - += - + ¤ =2( 0.5 ) ( )( )

( ) ( ) ( ) (0)( )

t Z tdS tdt dZ t S t S e

S t

We have:

a s s a

a s s a

- - = = fi = + =

- - = = fi = + =

2 21 1 1 1

2 22 2 2 2

0 0.5 0.05 & 0.2 0.05 0.5(0.2) 0.07

0 0.5 0.04 & 0.3 0.04 0.5(0.3) 0.085

Page 388: ActuarialBrew.com Exam MFE / 3F Actuarial Models …actuarialbrew.com/data/documents/MFE-Solutions-2014-1st-ed.pdf · Exam MFE / 3F Actuarial Models Financial Economics Segment Solutions

Exam MFE/3F Solutions Chapter 15 – The Sharpe Ratio and Itô’s Lemma

© ActuarialBrew.com 2014 Page 15.28

Since the two assets are perfectly positively correlated, they must have the same Sharpe ratio:

a as s- -

=

- -=

- = -=

=

1 2

1 2

0.07 0.0850.2 0.3

0.021 0.3 0.017 0.20.004 0.1

0.04

r r

r r

r rr

r

Solution 15.49

C Sharpe Ratio

The price follows geometric Brownian motion:

a d s sa d s - - += - + ¤ =2( 0.5 ) ( )( )

( ) ( ) ( ) (0)( )

t Z tdS tdt dZ t S t S e

S t

We have:

a s s a

a s s a

- - = = fi = + + =

- - = = fi = + =

2 21 1 1 1

2 22 2 2 2

0.02 0.5 0.05 & 0.2 0.05 0.02 0.5(0.2) 0.09

0 0.5 0.04 & 0.4 0.04 0.5(0.4) 0.12

Since the two assets are perfectly positively correlated, they must have the same Sharpe ratio:

a as s- -

=

- -=

- = -=

=

1 2

1 2

0.09 0.120.2 0.4

0.036 0.4 0.024 0.20.012 0.2

0.06

r r

r r

r rr

r

Solution 15.50

B Sharpe Ratio

The price follows geometric Brownian motion:

a d s sa d s - - += - + ¤ =2( 0.5 ) ( )( )

( ) ( ) ( ) (0)( )

t Z tdS tdt dZ t S t S e

S t

Page 389: ActuarialBrew.com Exam MFE / 3F Actuarial Models …actuarialbrew.com/data/documents/MFE-Solutions-2014-1st-ed.pdf · Exam MFE / 3F Actuarial Models Financial Economics Segment Solutions

Exam MFE/3F Solutions Chapter 15 – The Sharpe Ratio and Itô’s Lemma

© ActuarialBrew.com 2014 Page 15.29

We have:

a s s a

a s s a

- - = = fi = + + =

- - = = fi = + =

2 21 1 1 1

2 22 2 2 2

0.04 0.5 0.16 & 0.2 0.16 0.04 0.5(0.2) 0.22

0 0.5 0.31 & 0.4 0.31 0.5(0.4) 0.39

Since the two assets are perfectly positively correlated, they must have the same Sharpe ratio:

a as s- -

=

- -=

- = -=

=

1 2

1 2

0.22 0.390.2 0.4

0.088 0.4 0.078 0.20.010 0.2

0.05

r r

r r

r rr

r

Solution 15.51

E Claim on Sa

The delta and gamma of the claim are:

( )∂∂ ∂DD = = G = = =

∂ ∂ ∂

343 2

44 12

SSS S

S S S

At time 3, the value of =(3) 2S , so the value of gamma is:

G = = ¥ =2 212 12 2 48S

Solution 15.52

D Claim on Sa

The delta and gamma of the claim are:

( )-

- -∂∂ ∂DD = = G = = = -

∂ ∂ ∂

11 2( 1)

kkk k

kSSkS k k S

S S S

At time 3, the value of =(3) 2S , so:

-

-

G = -

= -

2

2

( 1)

48 ( 1)2

k

k

k k S

k k

Since we are given that k is a positive integer between 0 and 6, let’s try a few integers to see which one works. For = 0k , the right side of the equation above is 0. For = 1k , the right side is again 0. For = 2k , the right side is 2. For = 3k , the right side is 12. For = 4k , the right side is 48, so the correct value of k is 4.

Page 390: ActuarialBrew.com Exam MFE / 3F Actuarial Models …actuarialbrew.com/data/documents/MFE-Solutions-2014-1st-ed.pdf · Exam MFE / 3F Actuarial Models Financial Economics Segment Solutions

Exam MFE/3F Solutions Chapter 15 – The Sharpe Ratio and Itô’s Lemma

© ActuarialBrew.com 2014 Page 15.30

From the process for the stock, we observe that the expected rate of price increase is:

a d- = 0.14

Since we are given that the dividend yield is 6%, we can find the stock’s expected return:

a a- = fi =0.06 0.14 0.20

We can now obtain the expected return on the derivative:

g a= - + = - + =( ) 4(0.20 0.12) 0.12 0.44k r r

Solution 15.53

B Claim on Sa

The usual relationship is:

2( ) ( )( ) ( ) ( ) 0.5 ( 1) ( )

( )dS t d S

dt dZ t dt dZ tS t S

a d s a d s sÈ ˘= - + fi = - + - +Î ˚

a

aa a a a

In this case, 0.25a d- = and s s= - , so we have:

2( ) ( )0.25 ( ) 0.25 0.5 ( 1) ( )

( )dS t d S

dt dZ t dt dZ tS t S

s s sÈ ˘= - fi = + - -Î ˚

a

aa a a a

From statement (ii), we have the following 2 equations and 2 unknowns:

20.25 0.5 ( 1) 0.02

0.8s

s

ÏÔ + - = -Ì- =ÔÓ

a a a

a

The second equation can be written as 0.8s-=a , and we can substitute this value of a

into the first equation:

[ ][ ]

2

2

2

2

2

0.25 0.5 ( 1) 0.02

0.8 0.8 0.80.25 0.5 1 0.02

0.20.4 0.8 0.02

0.2 0.4 0.8 0.02

0.2 0.32 0.4 0.02

0.4 0.34 0.2 0

0.34 ( 0.34) 4( 0.4)(0.2)2( 0.04)

12.5

s

ss s s

ss

s s s

s s s

s s

s

s

+ - = -- - -Ê ˆ Ê ˆ È ˘+ - = -Á ˜ Á ˜ Í ˙Ë ¯ Ë ¯ Î ˚

- - - - = -

+ - - =

- - =

- - + =

± - - -=

-= -

a a a

or 0.4s =

Since we are told that s is a positive constant, we conclude that s = 0.4 .

Page 391: ActuarialBrew.com Exam MFE / 3F Actuarial Models …actuarialbrew.com/data/documents/MFE-Solutions-2014-1st-ed.pdf · Exam MFE / 3F Actuarial Models Financial Economics Segment Solutions

Exam MFE/3F Solutions Chapter 15 – The Sharpe Ratio and Itô’s Lemma

© ActuarialBrew.com 2014 Page 15.31

Solution 15.54

B Market Price of Risk and Arbitrage

The market price of risk of Stock X is greater than that of Stock Q:

0.03 0.05 0.07 0.05

0.20 0.400.10 0.05

An arbitrage strategy can be constructed with the following 3 steps:

1. Buy the following quantity of shares of Stock X:

s

= = -- ¥

1 10.25

0.20 20X XS

Since the quantity to buy is negative, we sell 0.25 shares of Stock X.

2. Sell the following quantity of Stock Q:

s= =

¥1 1

0.50.4 5Q QS

We sell 0.5 shares of Stock Q.

3. Lend the following quantity of dollars at the risk-free rate:

s s

- = - =-

1 1 1 17.50

0.40 0.20Q X

Since the quantity to lend is positive, we lend $7.50 at the risk-free rate.

Since arbitrage produces riskless profits, the arbitrageur would like to implement the strategy above on as large a scale as possible, but lending is limited to $60. The strategy described above requires lending $7.50, so the strategy can be scaled up by a factor of:

=608

7.50

Scaling the strategy by a scale of 8 means that instead of purchasing –0.25 shares of Stock X, the number of shares purchased is:

¥ - = -8 ( 0.25) 2

Solution 15.55

E Market Price of Risk and Arbitrage

The market price of risk of Stock 1 is greater than that of Stock 2:

0.03 0.05 0.07 0.05

0.20 0.400.10 0.05

Page 392: ActuarialBrew.com Exam MFE / 3F Actuarial Models …actuarialbrew.com/data/documents/MFE-Solutions-2014-1st-ed.pdf · Exam MFE / 3F Actuarial Models Financial Economics Segment Solutions

Exam MFE/3F Solutions Chapter 15 – The Sharpe Ratio and Itô’s Lemma

© ActuarialBrew.com 2014 Page 15.32

Since both assets follow geometric Brownian motion, a strategy that involves purchasing s1 11/( )S shares of Stock 1 and selling 2 21/( )S shares of Stock 2 results in an arbitrage

profit of:

a as s

È ˘- - - -È ˘- = - =Í ˙ Í ˙-Î ˚Î ˚1 2

1 2

0.03 0.05 0.07 0.050.05

0.20 0.40r r

dt dt dt

Since s1 is negative, the amount of Stock 1 to purchase is negative:

s= = -- ¥1 1

1 10.25

0.20 20S

Therefore the strategy calls for selling Stock 1.

Since s2 is positive, the amount of Stock 2 to sell is positive:

s= =

¥2 2

1 10.5

0.40 5S

Therefore, the strategy calls for selling Stock 2.

The proceeds from selling Stock 1 and Stock 2 are positive, and these funds are lent at the risk-free rate. Selling Stock 1, selling Stock 2, and lending at the risk-free rate is described by the strategy in Choice E.

Solution 15.56

C Arbitrage

To keep the notation from becoming cluttered, we’ve dropped (t) from the processes below. For example, we write Q(t) as Q.

The incremental change in Asset Q is:

a= + +1 22QdQ Qdt QdZ QdZ

We can create a zero-cost, risk-free portfolio by taking the following steps:

1. Purchase 1 unit of Asset Q.

2. Sell 10.20

QS

units of Stock 1. [To remove the 1QdZ term]

3. Sell 2

20.40

QS

units of Stock 2. [To remove the 22QdZ term]

4. Lend Ê ˆ+ -Á ˜Ë ¯2

0.20 0.40Q Q

Q at the risk-free rate. [To invest the net funds from the

three steps above]

Page 393: ActuarialBrew.com Exam MFE / 3F Actuarial Models …actuarialbrew.com/data/documents/MFE-Solutions-2014-1st-ed.pdf · Exam MFE / 3F Actuarial Models Financial Economics Segment Solutions

Exam MFE/3F Solutions Chapter 15 – The Sharpe Ratio and Itô’s Lemma

© ActuarialBrew.com 2014 Page 15.33

The resulting portfolio has the following incremental return:

1 2 1 2

1 2 1 2

0.12 0.10 22 2

0.20 0.40

20.08

0.2 0.40.12 0.10 2

2 20.20 0.40

20.08

0.2 0.4

0.12 0.100.20

Q

Q

Q

Q QdQ Qdt QdZ QdZ dt QdZ dt QdZ

Q QQ dt

Q QQdt QdZ QdZ dt QdZ dt QdZ

Q QQ dt

QQdt dt

a

a

a

¥È ˘ È ˘È ˘= + + - + - +Í ˙ Í ˙Î ˚ Î ˚ Î ˚Ê ˆ+ + -Á ˜Ë ¯

¥= + + - - - -

Ê ˆ+ + -Á ˜Ë ¯

= + - -

( )

2 20.08

0.40 0.2 0.4

0.38Q

Q Q Qdt Q dt

Qdta

¥ Ê ˆ+ + -Á ˜Ë ¯

= -

Since the portfolio is a risk-free, zero-cost portfolio, its return must be zero. Therefore, its drift must be zero:

( )a

aa

- =

- =

=

0.38 0

0.38 0

0.38

Q

Q

Q

Q

Solution 15.57

C Itô’s Lemma

From Itô’s Lemma, we have:

20.5 ( )

0.5

0.5

Z ZZ t

Z ZZ t

ZZ t Z

dS S dZ S dZ S dt

ZdZ S dZ S dt S dt

ZdZ S S dt S dZ

The factors in front of the terms containing dt must be equal, and the factors in front of the terms containing dZ must also be equal, so:

0 0.5 and ZZ t ZS S Z S

Let's use the second equation above to find ZZS :

1Z

ZZ

S Z

S

We can now use the first equation to solve for tS :

0 0.5

0 0.5

0.5

ZZ t

t

t

S S

S

S

Page 394: ActuarialBrew.com Exam MFE / 3F Actuarial Models …actuarialbrew.com/data/documents/MFE-Solutions-2014-1st-ed.pdf · Exam MFE / 3F Actuarial Models Financial Economics Segment Solutions

Exam MFE/3F Solutions Chapter 15 – The Sharpe Ratio and Itô’s Lemma

© ActuarialBrew.com 2014 Page 15.34

Integrating ZS with respect to Z and tS with respect to t and adding them together, we see that the following is a solution:

20.5 0.5S Z t

This matches the solution in Choice C.

Solution 15.58

A Quadratic Variation

The quadratic variation is the sum of the squared increments of the process.

(i) W is not stochastic, and its differential is found below:

= - = -

= -

2 3

2

( ) ( 1)

(3 1)

W t t t t t

dW t dt

Making use of the multiplication rule, ¥ = 0dt dt , we obtain the quadratic variation:

= = - = - ¥ =Ú Ú Ú3.7 3.7 3.7

2 2 2 2 2 23.7

0 0 0

( ) [ ] [(3 1) ] (3 1) ( )] 0V W dW t dt t dt dt

(ii) Let’s consider the values of X(t) as t increases from 0 to 3.7:

t X(t) Change in X(t)

0 0

(0, 1] 1 1

(1, 2] 2 1

(2, 3] 3 1

(3, 3.7] 4 1

The increments to X are 0, except just after time 0, time 1, time 2, and time 3. At those 4 times, the increments are 1. An infinite number of zeros is summed, and their sum is zero. The four non-zero increments (of 1 each) sum to 4:

Æ• =

Ï ¸Ê ˆ Ê ˆ= = ¥ - - ¥Ì ˝Á ˜ Á ˜Ë ¯ Ë ¯Ó ˛

= + + + + + + + + + + + + + + + =

ÂÚ

23.72 23.7

102 2 2 2 2 2 2 2 2 2 2 2

3.7 3.7( ) [ ] lim [ 1]

1 0 0 1 0 0 1 0 0 1 0 0 4

n

n i

V X dX X i X in n

(iii) We can find dY by finding the partial differential of each of the terms:

= += +4 0.8

4 0.8Y t ZdY dt dZ

Making use of the multiplication rules, we have:

= + = =2 2 2( ) (4 0.8 ) 0.64( ) 0.64dY dt dZ dZ dt

Page 395: ActuarialBrew.com Exam MFE / 3F Actuarial Models …actuarialbrew.com/data/documents/MFE-Solutions-2014-1st-ed.pdf · Exam MFE / 3F Actuarial Models Financial Economics Segment Solutions

Exam MFE/3F Solutions Chapter 15 – The Sharpe Ratio and Itô’s Lemma

© ActuarialBrew.com 2014 Page 15.35

The quadratic variation is:

= = = ¥ - =Ú Ú3.7 3.7

2 23.7

0 0

( ) [ ] 0.64 0.64 (3.7 0) 2.368V Y dY dt

Since < <0 2.368 4 , we have:

< <2 2 23.7 3.7 3.7( ) ( ) ( )V W V Y V X

Solution 15.59

D Claim on Sa

The question asks for the ratio of the expected value to the prepaid forward price:

( *)( )

( ) (2 0) 2( *)( )

,

( ) ( ) ( )

( )( )

T tT t

T tPt T

E S T S tX S t ee e e

Y e S tF S T

aa

aa

The ratio of the expected value of the stock to the prepaid forward price of the stock is:

[ ]

a da

d

-

- ¥

È ˘Î ˚ = =( )2

22

0,2

(2) (0) (0)

(2) (0)P

E S S S ee

F S S e

We can now use (i) and (ii) to solve for a :

a a= fi =20.78663 0.10

0.64404e

We can use (ii) and (iii) to solve for r:

[ ][ ]

d

d

-

-= =

= fi =

2( )0,2 2

20,2

2

(2) (0)

(2) (0)

0.71177 0.05

0.64404

rr

P

r

F S S ee

F S S e

e r

We can now find the expected return on the contingent claim:

g a= - + = - + =( ) 3(0.10 0.05) 0.05 0.20r ra

The ratio is:

2 2 0.20 1.4918X

e eY

Page 396: ActuarialBrew.com Exam MFE / 3F Actuarial Models …actuarialbrew.com/data/documents/MFE-Solutions-2014-1st-ed.pdf · Exam MFE / 3F Actuarial Models Financial Economics Segment Solutions

Exam MFE/3F Solutions Chapter 15 – The Sharpe Ratio and Itô’s Lemma

© ActuarialBrew.com 2014 Page 15.36

Solution 15.60

A Market Price of Risk

From (i), we have:

d d= =0.05 0.02X Y

From (ii), we have:

a d a ds

- = fi = + = + ==

0.03 0.03 0.03 0.05 0.08

0.10X X X X

X

From (iii), we have:

m a d ss= - -

= -

20.5

0.20Y Y Y

Y

Stocks X and Y must have the same market price of risk:

a as s

a

a

- -=

-- =-

=

0.070.08 0.070.10 0.20

0.05

X Y

X Y

Y

Y

r r

We can now find m :

m a d s= - - = - - ¥ - =2 20.5 0.05 0.02 0.5 ( 0.20) 0.01Y Y Y

Solution 15.61

B Market Price of Risk

Stock 1 does not pay dividends, so its parameters are:

a m s m= =1 1 and 10

Stock 2 has a dividend yield of 0.03, so its parameters are:

a d s s

aa

- - = =

- - ¥ ==

22 2 2 2

22

2

0.5 0.02 0.10

0.03 0.5 (0.10) 0.02

0.055

Page 397: ActuarialBrew.com Exam MFE / 3F Actuarial Models …actuarialbrew.com/data/documents/MFE-Solutions-2014-1st-ed.pdf · Exam MFE / 3F Actuarial Models Financial Economics Segment Solutions

Exam MFE/3F Solutions Chapter 15 – The Sharpe Ratio and Itô’s Lemma

© ActuarialBrew.com 2014 Page 15.37

Since the two stocks have the same source of uncertainty, they must have the same market price of risk:

a as s

mm

m mm

- -=

- -=

- == -

1 2

1 2

0.04 0.055 0.0410 0.10

0.04 1.50.08

r r

Solution 15.62

A Risk-Neutral Pricing

The risk-neutral price process is:

d s= - + ( )( ) ( )

( )dS t

r dt dZ tS t

From statement (iii), we can obtain the risk-free rate and the volatility parameter:

d d

s- = fi = + = + ==

0.045 0.045 0.045 0.05 0.0950.30

r r

We can use the following equivalency:

d s sd s - - +

- ¥ +

= - + ¤ =

= + ¤ = =

2

2

( 0.5 ) ( )

(0.045 0.5 0.3 ) 0.3 ( ) 0.3 ( )

( )( ) ( ) ( ) (0)

( )( )

0.045 0.30 ( ) ( ) (0)( )

r t Z t

t Z t Z t

dS tr dt dZ t S t S e

S tdS t

dt dZ t S t S e eS t

The expected value of the derivative under the risk-neutral probability measure is:

{ } { }{ } { }

{ }

È ˘È ˘ = ¥Í ˙Í ˙Î ˚ Í ˙Î ˚È ˘ È ˘= ¥ = ¥Í ˙ Í ˙Î ˚ Î ˚

È ˘= ¥Í ˙Î ˚

22 0.3 (2) 0.3 (2)

2 20.3 (2) 0.3 (2)

2 0.3 (2)

(2) ln (2) ln

0.3 (2) 0.09 (2)

0.09 (2)

Z Z

Z Z

Z

E S S E e e

E e Z E e Z

E Z e

Page 398: ActuarialBrew.com Exam MFE / 3F Actuarial Models …actuarialbrew.com/data/documents/MFE-Solutions-2014-1st-ed.pdf · Exam MFE / 3F Actuarial Models Financial Economics Segment Solutions

Exam MFE/3F Solutions Chapter 15 – The Sharpe Ratio and Itô’s Lemma

© ActuarialBrew.com 2014 Page 15.38

The value of (2)Z has a risk-neutral distribution that is the same as the distribution of

¥ 2Z , for some standard normal random variable Z. Therefore, we have:

{ } { } { }

[ ]

¥ ¥

¥

È ˘È ˘È ˘ = ¥ = ¥Í ˙Í ˙Í ˙Î ˚ Î ˚ Î ˚È ˘ È ˘= ¥ = ¥Í ˙ Í ˙Î ˚ Î ˚È ˘= ¥ = +Í ˙Î ˚

=

222 0.3 (2) 0.3 2

2 0.3 2 2 0.3 2

2 0.18 0.5 0.18

(2) ln (2) 0.09 (2) 0.09 2)

0.09 2 0.18

0.18 0.18 1 0.18

0.23240

Z Z

Z Z

Z

E S S E Z e E Z e

E Z e E Z e

E Z e e

Although the amount of the payment becomes known at time 2, the payment is not actually made until time 3, so the time-0 price is obtained by discounting the expected value at the risk-free rate for 3 years:

- - ¥¥ = ¥ =3 3 0.0950.23240 0.23240 0.17477re e

Solution 15.63

B Gap Power Option

The usual put-call parity expression for gap calls and gap puts is:

d- -+ = +1rT TGapCall K e Se GapPut

In this case, the underlying asset is 4S , so we replace S with 4S and we replace d with

the dividend yield of 4S , which we denote d * :

4 *1

0.08(0.5) 4 * 0.525.90 850 5 106.91* 0.1800

rT TGapCall K e S e GapPut

e e

d

d

d

- - ¥

- - ¥

+ = +

- + = += -

We have another formula for d * , and we can use it to solve for d :

d d s

dd

= - - - -

- = - - - -=

2

2

* ( ) 0.5 ( 1)

0.1800 0.08 4(0.08 ) 0.5(4)(4 1)(0.10)0.0300

r ra a a

Page 399: ActuarialBrew.com Exam MFE / 3F Actuarial Models …actuarialbrew.com/data/documents/MFE-Solutions-2014-1st-ed.pdf · Exam MFE / 3F Actuarial Models Financial Economics Segment Solutions

Exam MFE/3F Solutions Chapter 16 – The Black-Scholes Equation

© ActuarialBrew.com 2014 Page 16.01

Chapter 16 – Solutions

Solution 16.01

A Black-Scholes Equation

The expressions for the value of the derivative and the partial derivatives are:

[ ]

{ }

-

-

= - +

=

= -

= - + -

0.07

0.07 1

0.07 2

0.07 0.07

ln 0.02 0.08

0.07 ln 0.02 0.08 0.02

t

tS

tSS

t tt

V e S t

V e S

V e S

V e S t e

From the Black-Scholes equation, we have:

s d

b

b

b

+ - + =

- -+ + =

- + + - + - =

È ˘- + + - + = - +Î ˚

2 2

2 2 0.07 0.07

2

2 0.07 0.07 0.07 0.07

0.07 2 0.07

0.5 ( )

0.5 (0.07 0.03)0.07

0.5 0.04 0.07 (ln 0.02 0.08) 0.02 0.07

0.5 0.02 0.07(ln 0.02 0.08) 0.07 ln 0.02

SS S tt t

t

t t t t

t t

S V r SV V rV

S e SeV V

SS

e e e S t e V

e S t e S t[ ][ ] [ ]b

b

b

bb b

- + + - + = - +

- + =

=

== = -

2

2

2

2

0.08

0.5 0.02 0.07 ln 0.02 0.08 0.07 ln 0.02 0.08

0.5 0.02 0

0.5 0.02

0.040.2 or 0.2

S t S t

Since 0.2 is not one of the available choices, it must be the case that b = -0.2 .

Solution 16.02

D The Black-Scholes Equation

We can use the Black-Scholes equation to find the value of the European option:

s d+ - + =

+ - - ==

2 2

2 2

0.5 ( )

0.5(0.32) (40) (0.008) (0.095 0.00)(40)(0.869) 2.238 0.09518.1006

SS S tS V r SV V rV

VV

The value now of continuous payments of $5 per year for 4 years is:

-- =0.095(4)1

5 16.63890.095e

Page 400: ActuarialBrew.com Exam MFE / 3F Actuarial Models …actuarialbrew.com/data/documents/MFE-Solutions-2014-1st-ed.pdf · Exam MFE / 3F Actuarial Models Financial Economics Segment Solutions

Exam MFE/3F Solutions Chapter 16 – The Black-Scholes Equation

© ActuarialBrew.com 2014 Page 16.02

Adding the two components of the derivative together, we have:

+ =18.1006 16.6389 34.7395

Solution 16.03

D B-S Equation and Expected Return on Option

We can use the Black-Scholes equation to find the value of the option:

s d+ - + =

+ - - ==

2 2

2 2

0.5 ( )

0.5(0.30) (50) (0.020) (0.08 0.02)(50)(0.743) 3.730 0.089.3625

SS S tS V r SV V rV

VV

Since 0.13 , the instantaneous expected return is:

a d sa

- + +=

+ -=

=

2 2

2 2

( ) 0.5

0.13(50)(0.743) 0.5(0.30) (50) (0.020) 3.7309.3625

0.3578

S SS tOption

SV S V VV

Solution 16.04

B Black-Scholes Equation

Since European options do not normally pay dividends, we assume that the option does not pay dividends:

=( ) 0D t

We can use the Black-Scholes equation to find rV :

s d+ - + + =

+ - - + ==

2 2

2 2

0.5 ( ) ( )

0.5(0.30) (50) (0.020) (0.08 0.02)(50)(0.743) 3.730 0

0.749

SS S tS V r SV V D t rV

rVrV

The expected change per unit of time under the risk-neutral distribution is:

[ ]

[ ]

[ ]

+=

+=

=

*

*

*

( )

00.749

0.749

E dV D t dtrV

dt

E dV

dt

E dV

dt

Page 401: ActuarialBrew.com Exam MFE / 3F Actuarial Models …actuarialbrew.com/data/documents/MFE-Solutions-2014-1st-ed.pdf · Exam MFE / 3F Actuarial Models Financial Economics Segment Solutions

Exam MFE/3F Solutions Chapter 16 – The Black-Scholes Equation

© ActuarialBrew.com 2014 Page 16.03

Solution 16.05

E Black-Scholes Equation

The expressions for the value of the derivative and the partial derivatives are:

-

-

=

=

= -

=

1

2

ln

ln

rt

rtS

rtSS

rtt

V e S

V e S

V e S

V re S

From the Black-Scholes equation, we have:

( )s d

d

d

d

dd

d

- -

- -

+ - + =

- + - + =

- + - =

- + - =

- + - =- =

=

2 2

2 2 2 1

2 2 2 1

2

2

0.5 ( )

0.5(0.2) ( ) (0.09 ) ( ) ln ln

0.5(0.2) ( ) (0.09 ) ( ) 0

0.5(0.2) (0.09 ) 0

0.5(0.2) (0.09 ) 00.09 0.5(0.04)

0.07

SS S t

rt rt rt rt

rt rt

rt rt

S V r SV V rV

S e S S e S re S r e S

S e S S e S

e e

Solution 16.06

E Black-Scholes Equation

For the derivative, we have:

d= =( ) ( *) ( ) 0.05 ( )D t V t V t

The expressions for the value of the derivative and the partial derivatives are:

-

-

=

=

= -

=

0.5

0.5 1

0.5 2

0.5

2 ln

2

2

ln

rt

rtS

rtSS

rtt

V e S

V e S

V e S

V re S

Page 402: ActuarialBrew.com Exam MFE / 3F Actuarial Models …actuarialbrew.com/data/documents/MFE-Solutions-2014-1st-ed.pdf · Exam MFE / 3F Actuarial Models Financial Economics Segment Solutions

Exam MFE/3F Solutions Chapter 16 – The Black-Scholes Equation

© ActuarialBrew.com 2014 Page 16.04

From the Black-Scholes equation, we have:

2 2

0.05 0.052 2 0.05 0.05

2

2 0.05 0.05 0.05 0.05 0.05

2 0.05

0.5 ( ) ( )

2 20.5 (0.1 0.055) 0.1 ln 0.05 0.1 2 ln

0.5 2 0.045 2 0.1 ln 0.05 2 ln 2 0.1 ln

0.

SS S t

t tt t

t t t t t

t

S V r SV V D t rV

e eS S e S V e S

SS

e e e S e S e S

e

0.05

2

2

09 0

0.09 0

0.090.30

te

Solution 16.07

D Sharpe Ratio

The stock and the call option must have the same Sharpe ratio at all times, including time 2:

g

s s-- =

0.0950.12 0.095

C

The cost of the shares required to delta-hedge the call option is the number of shares required, D , times the cost of each share, S. Therefore, based on statement (iv) in the question, we have:

D ¥ = 78S

We can find the volatility of the call option in terms of the volatility of the underlying stock:

s s s s s¥ D= ¥ W = ¥ = ¥ =783.9

20C CS

V

We can now solve for g :

g g

s s-- = fi =0.0950.12 0.095

0.19253.9

Solution 16.08

B Sharpe Ratio

The stock and the call option must have the same Sharpe ratio at all times, including time 3:

a

s s- -=0.095 0.26 0.095

C

Page 403: ActuarialBrew.com Exam MFE / 3F Actuarial Models …actuarialbrew.com/data/documents/MFE-Solutions-2014-1st-ed.pdf · Exam MFE / 3F Actuarial Models Financial Economics Segment Solutions

Exam MFE/3F Solutions Chapter 16 – The Black-Scholes Equation

© ActuarialBrew.com 2014 Page 16.05

The cost of the shares required to delta-hedge the call option is the number of shares required, D , times the cost of each share, S. Therefore, based on statement (iv) in the question, we have:

D ¥ = 78S

We can find the volatility of the call option in terms of the volatility of the underlying stock:

s s s s s¥ D= ¥ W = ¥ = ¥ =783.9

20C CS

V

We can now solve for a :

a a

s s- -= fi =0.095 0.26 0.095

0.13733.9

Solution 16.09

E Sharpe Ratio

The stock and the call option must have the same Sharpe ratio at all times, including time 3:

a

s s- -=0.095 0.36 0.095

C

The cost of the shares required to delta-hedge the call option is the number of shares required, D , times the cost of each share, S. Therefore, based on statement (iv) in the question, we have:

D ¥ = 82S

We can find the volatility of the call option in terms of the volatility of the underlying stock:

s s s s s¥ D= ¥ W = ¥ = ¥ =824.1

20C CS

V

We can now solve for a :

a a

s s- -= fi =0.095 0.36 0.095

0.15964.1

The general form for a dividend-paying stock that follows geometric Brownian motion is:

a d s= - +( )( ) ( )

( )dS t

dt dZ tS t

Page 404: ActuarialBrew.com Exam MFE / 3F Actuarial Models …actuarialbrew.com/data/documents/MFE-Solutions-2014-1st-ed.pdf · Exam MFE / 3F Actuarial Models Financial Economics Segment Solutions

Exam MFE/3F Solutions Chapter 16 – The Black-Scholes Equation

© ActuarialBrew.com 2014 Page 16.06

We can use the differential equation provided in the question to find the dividend yield:

s a d

dd

= + fi = -

= -=

( )0.12 ( ) 0.12

( )0.12 0.1596

0.0396

dS tdt dZ t

S t

Page 405: ActuarialBrew.com Exam MFE / 3F Actuarial Models …actuarialbrew.com/data/documents/MFE-Solutions-2014-1st-ed.pdf · Exam MFE / 3F Actuarial Models Financial Economics Segment Solutions

Exam MFE/3F Solutions Chapter 17 – The Black Model for Options on Bonds

© ActuarialBrew.com 2014 Page 17.01

Chapter 17 – Solutions

Solution 17.01

C Forward Prices

We wish to find the price, agreed upon now to be paid in 2 years, for a 1-year bond:

0 0,2(2,3) [ (2,3)]P F P=

This price is the ratio of the 3-year bond price to the 2-year bond price:

0

0

(0, )( , )

(0, )(0,3) 0.7722

(2,3) 0.9007(0,2) 0.8573

P T sP T T s

P TP

PP

++ =

= = =

Solution 17.02

C Black Model

The method of presenting the forward price volatilities in this question is the same as the method that appears in Problems 24.1, 24.2, and 24.3 of the second edition of the Derivatives Markets textbook.

The forward price volatilities are:

[ ]{ } [ ]{ } [ ]{ }

= = =ln (1,2) ln (2,3) ln (3,4)

0.100 0.105 0.120t t tVar P Var P Var P

t t t

The appropriate volatility for an option that expires in 2 years on a bond that matures in 3 years is:

0.105s =

The bond forward price is:

0(0, ) (0,3)

( , ) 0.9007(0, ) (0,2)

P T s PF P T T s

P T P+= + = = =

The values of 1d and 2d are:

2 2

1

2 1

0.9007ln 0.5 ln 0.5(0.105) (2)

0.920.06827

0.105 2

0.06827 0.105 2 0.21676

FT

Kd

T

d d T

s

ss

Ê ˆ Ê ˆ+ +Á ˜ Á ˜Ë ¯ Ë ¯= = = -

= - = - - = -

We have:

1( ) ( 0.06827) 0.47279N d N

2( ) ( 0.21676) 0.41420N d N

Page 406: ActuarialBrew.com Exam MFE / 3F Actuarial Models …actuarialbrew.com/data/documents/MFE-Solutions-2014-1st-ed.pdf · Exam MFE / 3F Actuarial Models Financial Economics Segment Solutions

Exam MFE/3F Solutions Chapter 17 – The Black Model for Options on Bonds

© ActuarialBrew.com 2014 Page 17.02

The Black formula for the call price is:

1 2(0, ) ( ) ( )

0.8573[0.9007 0.47279 0.92 0.41420]0.03840

C P T F N d K N d

Solution 17.03

B Black Model

The forward price volatilities are:

[ ]{ } [ ]{ } [ ]{ }

= = =ln (1,2) ln (2,3) ln (3,4)

0.100 0.105 0.120t t tVar P Var P Var P

t t t

The appropriate volatility for an option that expires in 2 years on a bond that matures in 3 years is:

0.105s =

The bond forward price is:

0(0, ) (0,3)

( , ) 0.9007(0, ) (0,2)

P T s PF P T T s

P T P+= + = = =

The values of 1d and 2d are:

2 2

1

2 1

0.9007ln 0.5 ln 0.5(0.105) (2)

0.920.06827

0.105 2

0.06827 0.105 2 0.21676

FT

Kd

T

d d T

s

ss

Ê ˆ Ê ˆ+ +Á ˜ Á ˜Ë ¯ Ë ¯= = = -

= - = - - = -

We have:

1( ) (0.06827) 0.52721N d N

2( ) (0.21676) 0.58580N d N

The Black formula for the put price is:

2 1(0, ) ( ) ( )

0.8573[0.92 0.58580 0.9007 0.52721]0.05492

P P T K N d F N d

Page 407: ActuarialBrew.com Exam MFE / 3F Actuarial Models …actuarialbrew.com/data/documents/MFE-Solutions-2014-1st-ed.pdf · Exam MFE / 3F Actuarial Models Financial Economics Segment Solutions

Exam MFE/3F Solutions Chapter 17 – The Black Model for Options on Bonds

© ActuarialBrew.com 2014 Page 17.03

Solution 17.04

D Black Model

The price of the caplet is:

Caplet price (1.1075) (Put Option Price)= ¥

The put option:

has a zero-coupon bond that expires at time 3 as its underlying asset expires at time 2 has a strike price of:

1 1

0.90291 1.1075R

KK

= = =+

The forward price volatilities are:

[ ]{ } [ ]{ } [ ]{ }

= = =ln (1,2) ln (2,3) ln (3,4)

0.100 0.105 0.120t t tVar P Var P Var P

t t t

The appropriate volatility for an option that expires in 2 years on a bond that matures in 3 years is:

0.105s =

The bond forward price is:

0(0, ) (0,3)

( , ) 0.9007(0, ) (0,2)

P T s PF P T T s

P T P+= + = = =

The values of 1d and 2d are:

2 2

1

2 1

0.9007ln 0.5 ln 0.5(0.105) (2)

0.90290.05782

0.105 2

0.05782 0.105 2 0.09067

FT

Kd

T

d d T

s

ss

Ê ˆ Ê ˆ+ +Á ˜ Á ˜Ë ¯ Ë ¯= = =

= - = - = -

We have:

1( ) ( 0.05782) 0.47695N d N

2( ) (0.09067) 0.53612N d N

The Black formula for the put price is:

2 1(0, ) ( ) ( )

0.8573[0.9029 0.53612 0.9007 0.47695]0.04670

P P T K N d F N d

Page 408: ActuarialBrew.com Exam MFE / 3F Actuarial Models …actuarialbrew.com/data/documents/MFE-Solutions-2014-1st-ed.pdf · Exam MFE / 3F Actuarial Models Financial Economics Segment Solutions

Exam MFE/3F Solutions Chapter 17 – The Black Model for Options on Bonds

© ActuarialBrew.com 2014 Page 17.04

The price of the caplet is:

Caplet price (1.1075) (Put Option Price) 1.1075 0.04670 0.05172

Solution 17.05

C Black Model

The price of the caplet is:

Caplet price (1.12) (Put Option Price)= ¥

The put option:

has a zero-coupon bond that expires at time 4 as its underlying asset expires at time 3 has a strike price of:

1 1

0.89291 1.12R

KK

= = =+

The forward price volatilities are:

[ ]{ } [ ]{ } [ ]{ }

= = =ln (1,2) ln (2,3) ln (3,4)

0.100 0.105 0.120t t tVar P Var P Var P

t t t

The appropriate volatility for an option that expires in 3 years on a bond that matures in 4 years is:

0.12s =

The bond forward price is:

0(0, ) (0,4) 0.6956

( , ) 0.9008(0, ) (0,3) 0.7722

P T s PF P T T s

P T P

The values of 1d and 2d are:

2 2

1

2 1

0.9008ln 0.5 ln 0.5(0.12) (3)

0.89290.14655

0.12 3

0.14655 0.12 3 0.06130

FT

Kd

T

d d T

s

ss

Ê ˆ Ê ˆ+ +Á ˜ Á ˜Ë ¯ Ë ¯= = =

= - = - = -

We have:

1( ) ( 0.14655) 0.44174N d N

2( ) (0.06130) 0.52444N d N

Page 409: ActuarialBrew.com Exam MFE / 3F Actuarial Models …actuarialbrew.com/data/documents/MFE-Solutions-2014-1st-ed.pdf · Exam MFE / 3F Actuarial Models Financial Economics Segment Solutions

Exam MFE/3F Solutions Chapter 17 – The Black Model for Options on Bonds

© ActuarialBrew.com 2014 Page 17.05

The Black formula for the put price is:

2 1(0, ) ( ) ( )

0.7722[0.8929 0.52444 0.9008 0.44174]0.054308

P P T K N d F N d

The price of the caplet is:

Caplet price (1.12) (Put Option Price) 1.12 0.054308 0.060825

Solution 17.06

E Floorlet in the Black Model

The price of the floorlet is:

= ¥Floorlet price (1.12) (Call Option Price)

The call option:

has a zero-coupon bond that expires at time 4 as its underlying asset expires at time 3 has a strike price of:

1 1

0.89291 1.12R

KK

= = =+

The forward price volatilities are:

[ ]{ } [ ]{ } [ ]{ }

= = =ln (1,2) ln (2,3) ln (3,4)

0.100 0.105 0.120t t tVar P Var P Var P

t t t

The appropriate volatility for an option that expires in 3 years on a bond that matures in 4 years is:

0.12s =

The bond forward price is:

0(0, ) (0,4) 0.6956

( , ) 0.9008(0, ) (0,3) 0.7722

P T s PF P T T s

P T P

The values of 1d and 2d are:

2 2

1

2 1

0.9008ln 0.5 ln 0.5(0.12) (3)

0.89290.14655

0.12 3

0.14655 0.12 3 0.06130

FT

Kd

T

d d T

s

ss

Ê ˆ Ê ˆ+ +Á ˜ Á ˜Ë ¯ Ë ¯= = =

= - = - = -

Page 410: ActuarialBrew.com Exam MFE / 3F Actuarial Models …actuarialbrew.com/data/documents/MFE-Solutions-2014-1st-ed.pdf · Exam MFE / 3F Actuarial Models Financial Economics Segment Solutions

Exam MFE/3F Solutions Chapter 17 – The Black Model for Options on Bonds

© ActuarialBrew.com 2014 Page 17.06

We have:

1( ) (0.14655) 0.55826N d N

2( ) ( 0.06130) 0.47556N d N

The Black formula for the call price is:

1 2(0, ) ( ) ( )

0.7722[0.9008 0.55826 0.8929 0.47556]0.060444

C P T F N d K N d

The price of a floorlet with a notional amount of $1 is:

Floorlet price (1.12) (Call Option Price) 1.12 0.060444 0.067697

Since the notional amount is $1,000, we multiply by $1,000 to obtain the answer:

$1,000 0.067697 67.697¥ =

Solution 17.07

E Forward Rate Agreements

By purchasing the caplet and selling the floorlet, the investor has created a forward rate agreement that locks in the rate RK from time 2 to time 3. The investor’s time 3 cash

flow is the payment from the caplet minus the payment from the floorlet:

[ ] [ ]- - - = -2 2 20, 0,R R RMax R K Max K R R K

A forward rate agreement, which can be entered into at zero cost, provides a cash flow at time 3 of:

-2 0(2,3)R R

Both 0 (2,3)R and RK are known values at time 0. Since the cost of both strategies is the

same (zero), to preclude arbitrage, we must have:

= 0(2,3)RK R

We can find the forward rate that applies from time 2 to time 3:

= = - = - =0(0,2) 0.8573

(2,3) 1 1 0.1102(0,3) 0.7722R

PK R

P

Solution 17.08

D Black Formula

The option expires in 1 year, so = 1T . The underlying bond matures 1 year after the option expires, so = 1s .

Page 411: ActuarialBrew.com Exam MFE / 3F Actuarial Models …actuarialbrew.com/data/documents/MFE-Solutions-2014-1st-ed.pdf · Exam MFE / 3F Actuarial Models Financial Economics Segment Solutions

Exam MFE/3F Solutions Chapter 17 – The Black Model for Options on Bonds

© ActuarialBrew.com 2014 Page 17.07

The bond forward price is:

0(0, ) 0.8799

( , ) 0.92088(0, ) 0.9555

P T sF P T T s

P T+= + = = =

The volatility of the forward price is:

0.07s =

We have:

2 2

1

2 1

1

2

0.92088ln 0.5 ln 0.5(0.07) (1)

0.91110.18752

0.07 1

0.18752 0.07 1 0.11752

( ) (0.18752) 0.57437

( ) (0.11752) 0.54678

FT

Kd

T

d d T

N d N

N d N

s

ss

Ê ˆ Ê ˆ+ +Á ˜ Á ˜Ë ¯ Ë ¯= = =

= - = - == == =

The Black formula for the call price is:

[ ][ ]

1 2(0, ) ( ) ( )

0.9555 0.92088 0.57437 0.9111 0.54678

0.02939

C P T F N d K N d= ¥ - ¥

= ¥ - ¥=

Solution 17.09

C Black Model

We need the volatility of a forward that matures at time 2 and has a bond maturing at time 4 as its underlying asset:

[ ]{ } [ ]{ }

s+

= = = =ln ( , ) ln (2,4) 0.0289

0.17t tVar P T T s Var P tt t t

The bond forward price is:

+= + = = = =0

(0, ) (0,4) 0.6956( , ) 0.81138

(0, ) (0,2) 0.8573P T s P

F P T T sP T P

The values of 1d and 2d are:

2 2

1

2 1

0.81138ln 0.5 ln 0.5(0.17) (2)

0.920.40235

0.17 2

0.40235 0.17 2 0.64277

FT

Kd

T

d d T

s

ss

Ê ˆ Ê ˆ+ +Á ˜ Á ˜Ë ¯ Ë ¯= = = -

= - = - - = -

Page 412: ActuarialBrew.com Exam MFE / 3F Actuarial Models …actuarialbrew.com/data/documents/MFE-Solutions-2014-1st-ed.pdf · Exam MFE / 3F Actuarial Models Financial Economics Segment Solutions

Exam MFE/3F Solutions Chapter 17 – The Black Model for Options on Bonds

© ActuarialBrew.com 2014 Page 17.08

We have:

1( ) ( 0.40235) 0.34371N d N

2( ) ( 0.64277) 0.26019N d N

The Black formula for the call price is:

1 2(0, ) ( ) ( )

0.8573[0.81138 0.34371 0.92 0.26019]0.03387

C P T F N d K N d

Solution 17.10

A Black Model

The bond forward price is:

0(0, ) (0,2) 0.8417

( , ) 0.90906(0, ) (0,1) 0.9259

P T s PF P T T s

P T P+= + = = = =

The values of 1d and 2d are:

2 2

1

2 1

0.90906ln 0.5 ln 0.5(0.10) (1)

0.900090.14918

0.10 1

0.14918 0.10 1 0.04918

FT

Kd

T

d d T

s

ss

Ê ˆ Ê ˆ+ +Á ˜ Á ˜Ë ¯ Ë ¯= = =

= - = - =

We have:

1( ) ( 0.14918) 0.44071N d N

2( ) ( 0.04918) 0.48039N d N

The Black formula for the put price is:

2 1(0, ) ( ) ( )

0.9259[0.90009 0.48039 0.90906 0.44071]0.029408

P P T K N d F N d

Solution 17.11

E Black Model

The price of the caplet is:

Caplet price (1.111) (Put Option Price)= ¥

Page 413: ActuarialBrew.com Exam MFE / 3F Actuarial Models …actuarialbrew.com/data/documents/MFE-Solutions-2014-1st-ed.pdf · Exam MFE / 3F Actuarial Models Financial Economics Segment Solutions

Exam MFE/3F Solutions Chapter 17 – The Black Model for Options on Bonds

© ActuarialBrew.com 2014 Page 17.09

The put option:

has a zero-coupon bond that expires at time 2 as its underlying asset expires at time 1 has a strike price of:

1 1

0.900091 1.111R

KK

= = =+

The bond forward price is:

0(0, ) (0,2) 0.8417

( , ) 0.90906(0, ) (0,1) 0.9259

P T s PF P T T s

P T P+= + = = = =

The volatility of the bond forward price is:

[ ]{ } [ ]{ }

s

Ï ¸È ˘Ì ˝Í ˙+ Î ˚Ó ˛= = =

= = =

( ,2)ln

ln ( , ) ln (1,2) ( ,1)

0.010.01 0.1

t t

P tVar

Var P T T s Var P P t

t t t

tt

The values of 1d and 2d are:

2 2

1

2 1

0.90906ln 0.5 ln 0.5(0.10) (1)

0.900090.14918

0.10 1

0.14918 0.10 1 0.04918

FT

Kd

T

d d T

s

ss

Ê ˆ Ê ˆ+ +Á ˜ Á ˜Ë ¯ Ë ¯= = =

= - = - =

We have:

1( ) ( 0.14918) 0.44071N d N

2( ) ( 0.04918) 0.48039N d N

The Black formula for the put price is:

2 1(0, ) ( ) ( )

0.9259[0.90009 0.48039 0.90906 0.44071]0.029408

P P T K N d F N d

The price of the caplet is:

Caplet price (1.111) (Put Option Price) 1.111 0.029408 0.03267

Page 414: ActuarialBrew.com Exam MFE / 3F Actuarial Models …actuarialbrew.com/data/documents/MFE-Solutions-2014-1st-ed.pdf · Exam MFE / 3F Actuarial Models Financial Economics Segment Solutions

Exam MFE/3F Solutions Chapter 17 – The Black Model for Options on Bonds

© ActuarialBrew.com 2014 Page 17.10

This page is intentionally blank.

Page 415: ActuarialBrew.com Exam MFE / 3F Actuarial Models …actuarialbrew.com/data/documents/MFE-Solutions-2014-1st-ed.pdf · Exam MFE / 3F Actuarial Models Financial Economics Segment Solutions

Exam MFE/3F Solutions Chapter 18 – Binomial Short Rate Models

© ActuarialBrew.com 2014 Page 18.01

Chapter 18 – Solutions

Solution 18.01

C Binomial Interest Rate Model

The price of the 2-year zero-coupon bond is:

1

0* (0.12 0.16) (0.12 0.08)(0,2) 0.5 0.5 0.7873ii rP E e e e=- - + - +È ˘Â= = ¥ + ¥ =Í ˙Î ˚

Solution 18.02

A Binomial Interest Rate Model

The price of the 3-year zero-coupon bond is:

20*

0

(0.12 0.16 0.20)

(0.12 0.16 0.12)

(0.12 0.08 0.12)

(0.12 0.08 0.04)

(0,3)

0.25

0.25

0.25

0.250.7005

ii rP E e

e

e

e

e

=-

- + +

- + +

- + +

- + +

È ˘Â= Í ˙Î ˚

= ¥

+ ¥

+ ¥

+ ¥=

Solution 18.03

B Binomial Interest Rate Model

The terminology can be a bit confusing. This same option could also be described as a 2-year put on a 3-year zero-coupon bond, with the understanding that the 3-year bond will be a 1-year bond at the end of 2 years. In this case, since a 1-year bond will have matured by the end of 2 years, it is apparent that the 1-year bond in the question refers to a 1-year bond at the end of 2 years.

The tree of prices for the 3-year bond is:

0.8187 1.0000 0.7267 0.8869 1.0000

0.7005 0.8869 1.0000 0.8528 0.9608 1.0000

There is no need to calculate the first two columns of the tree to answer this question, but they are shown above for illustrative purposes.

Page 416: ActuarialBrew.com Exam MFE / 3F Actuarial Models …actuarialbrew.com/data/documents/MFE-Solutions-2014-1st-ed.pdf · Exam MFE / 3F Actuarial Models Financial Economics Segment Solutions

Exam MFE/3F Solutions Chapter 18 – Binomial Short Rate Models

© ActuarialBrew.com 2014 Page 18.02

The third column is easily obtained by discounting a 1-year bond at the possible interest rates at time 2:

0.20

0.12

0.04

0.8187

0.8869

0.9608

e

e

e

-

-

-

=

=

=

The put option pays off only if the price falls to $0.8187. If this occurs, the option payoff is:

2 0.85 0.8187 0.0313V = - =

There is a 0.25 risk-neutral probability of this occurring, so the price of the put option is:

2 1

0* (0.12 0.16)0 2 0.25(0.0313) 0.0059ii rV E V e e

-=- - +È ˘Â= ¥ = =Í ˙Î ˚

Solution 18.04

C Binomial Interest Rate Model

The tree of prices for the 3-year bond is:

0.9048 1.0000 0.8523 0.9418 1.0000

0.8361 0.9418 1.0000 0.9233 0.9802 1.0000

There is no need to calculate the first two columns of the tree to answer this question, but they are shown above for illustrative purposes.

The third column is easily obtained by discounting a 1-year bond at the possible interest rates at time 2:

0.10

0.06

0.02

0.9048

0.9418

0.9802

e

e

e

-

-

-

=

=

=

The call option pays off if the final interest rate is 0.06 or lower. The possible payoffs are:

2 2

2 2

2 2

10% 0.0000

6% 0.9418 0.92 0.0218

2% 0.9802 0.92 0.0602

r V

r V

r V

= fi == fi = - == fi = - =

Page 417: ActuarialBrew.com Exam MFE / 3F Actuarial Models …actuarialbrew.com/data/documents/MFE-Solutions-2014-1st-ed.pdf · Exam MFE / 3F Actuarial Models Financial Economics Segment Solutions

Exam MFE/3F Solutions Chapter 18 – Binomial Short Rate Models

© ActuarialBrew.com 2014 Page 18.03

Each of the 4 paths has a 0.25 risk-neutral probability of occurring. This leads to the following call price:

2 10*

0 2

(0.06 0.08)

(0.06 0.08)

(0.06 0.04)

(0.06 0.04)

0.25(0.0000)

0.25(0.0218)

0.25(0.0218)

0.25(0.0602)

0.02327

ii rV E V e

e

e

e

e

-=-

- +

- +

- +

- +

È ˘Â= ¥Í ˙Î ˚

=

+

+

+=

Solution 18.05

B Black-Derman-Toy Model

The amount of the loan does not affect the forward rate.

The price of a 3-year zero-coupon bond is:

2*

00

1(0,3)

(1 )

1 1 10.25

1.10 1.1517 1.18891 1 1

0.251.10 1.1517 1.1489

1 1 10.25

1.10 1.1293 1.14891 1 1

0.251.10 1.1293 1.1173

0.6930

ii

P Er=

È ˘Í ˙=

+Í ˙Î ˚

= ¥ ¥ ¥

+ ¥ ¥ ¥

+ ¥ ¥ ¥

+ ¥ ¥ ¥

=

The price of a 2-year zero-coupon bond is:

1*

00

1(0,2)

(1 )

1 1 1 10.5 0.5

1.10 1.1517 1.10 1.12930.7972

ii

P Er=

È ˘Í ˙=

+Í ˙Î ˚

= ¥ ¥ + ¥ ¥

=

The contract rate on the forward rate agreement is the forward rate:

= - = - =0(0,2) 0.7972

(2,3) 1 1 0.1503(0,3) 0.6930

PR

P

Page 418: ActuarialBrew.com Exam MFE / 3F Actuarial Models …actuarialbrew.com/data/documents/MFE-Solutions-2014-1st-ed.pdf · Exam MFE / 3F Actuarial Models Financial Economics Segment Solutions

Exam MFE/3F Solutions Chapter 18 – Binomial Short Rate Models

© ActuarialBrew.com 2014 Page 18.04

Solution 18.06

B Black-Derman-Toy Model

Although the cap payments are made at the end of each year, we will value them at the beginning of each year using the following formula:

1

1

0,-year caplet payoff at time ( 1)

1T R

T

Max R KT T Notional

R-

-

-È ˘Î ˚- = ¥+

A 2-year cap consists of a 1-year caplet and a 2-year caplet.

The payoff for the 1-year caplet is zero since 10% is less than 14%.

The payoff for the 2-year caplet is positive only when the short-term interest rate increases to 15.17%:

1

1

0, 0.1517 0.142-year caplet payoff at time 1 100 100

1 1.1517

1.0159

RMax R K

R

-È ˘ -Î ˚= ¥ = ¥+

=

This payment is made at time 1 and it has a 50% probability of occurring, so the value of the 2-year cap is:

0.5 1.0159

0.4621.10¥ =

Solution 18.07

C Black-Derman-Toy Model

Although the cap payments are made at the end of each year, we will value them at the beginning of each year using the following formula:

1

1

0,-year caplet payoff at time ( 1)

1T R

T

Max R KT T Notional

R-

-

-È ˘Î ˚- = ¥+

A 3-year cap consists of a 1-year caplet, a 2-year caplet, and a 3-year caplet.

The payoff for the 1-year caplet is zero since 10% is less than 14%.

The payoff for the 2-year caplet is positive only when the short-term interest rate increases to 15.17%:

10.1517 0.14

0.1517 100 1.01591.1517

R-= fi ¥ =

The payoff for the 3-year caplet is positive when the short-term rate exceeds 14%:

2

2

0.1889 0.140.1889 100 4.1130

1.18890.1489 0.14

0.1489 100 0.77471.1489

R

R

-= fi ¥ =

-= fi ¥ =

Page 419: ActuarialBrew.com Exam MFE / 3F Actuarial Models …actuarialbrew.com/data/documents/MFE-Solutions-2014-1st-ed.pdf · Exam MFE / 3F Actuarial Models Financial Economics Segment Solutions

Exam MFE/3F Solutions Chapter 18 – Binomial Short Rate Models

© ActuarialBrew.com 2014 Page 18.05

The possible payments from the cap are illustrated in the tree below:

4.1130 1.0159

0.0000 0.7747 0.0000

0.0000

The value of the 2-year caplet is:

0.5 1.0159

Value of 2-year caplet 0.46181.10¥= =

The value of the 3-year caplet is:

1 1Value of the 3-year caplet 0.25 4.1130

1.10 1.15171 1

0.25 0.77471.10 1.1517

1 10.25 0.7747

1.10 1.12931.1204

= ¥ ¥ ¥

+ ¥ ¥ ¥

+ ¥ ¥ ¥

=

The value of the 3-year cap is:

Value of 3-year cap (1-year caplet) + (2-year caplet) + (3-year caplet)0.000 0.4618 1.1204 1.5822

Solution 18.08

A Black-Derman-Toy Model

There are two ways to answer this question.

Method 1

First, we consider the quick way. The short-term rates in one period are 1R and

121

he Rs :

1

1

21

0.1293

0.1517h

R

e Rs

=

=

We can solve for 1s , which is the yield volatility for the 2-year bond:

121

1

1

0.15170.1293

0.1517ln 0.5 0.0799

0.1293

he RR

s

s

=

Ê ˆ= ¥ =Á ˜Ë ¯

Page 420: ActuarialBrew.com Exam MFE / 3F Actuarial Models …actuarialbrew.com/data/documents/MFE-Solutions-2014-1st-ed.pdf · Exam MFE / 3F Actuarial Models Financial Economics Segment Solutions

Exam MFE/3F Solutions Chapter 18 – Binomial Short Rate Models

© ActuarialBrew.com 2014 Page 18.06

This method only works for the 2-year bond! For longer term bonds, we must use the method shown below.

Method 2

The second method takes a bit longer. First, we create the tree of prices for the 2-year bond:

0.8683 0.7972

0.8855

The calculations to obtain these prices are:

1(1,2, ) 0.8683

1.15171

(1,2, ) 0.88551.1293

1(0,2) 0.5(0.8683 0.8855) 0.7972

1.10

u

d

P r

P r

P

= =

= =

= ¥ + =

We don’t need to calculate P(0,2) to answer this question, but we included it for completeness.

The formula for the one-year yield volatility for a T-year zero-coupon bond is:

1 /( 1)

1 /( 1)

(1, , ) 1Yield volatility 0.5 ln

(1, , ) 1

Tu

Td

P T r

P T r

- -

- -

È ˘-Í ˙= ¥Í ˙-Î ˚

The yield volatility of the 2-year bond is:

1/(2 1)

1 /(2 1)0.8683 1 0.1517

Yield volatility 0.5 ln 0.5 ln 0.07990.12930.8855 1

Solution 18.09

B Black-Derman-Toy Model

The tree of prices for the 3-year bond is:

0.8411 0.7430

0.6930 0.8704 0.7816 0.8950

Page 421: ActuarialBrew.com Exam MFE / 3F Actuarial Models …actuarialbrew.com/data/documents/MFE-Solutions-2014-1st-ed.pdf · Exam MFE / 3F Actuarial Models Financial Economics Segment Solutions

Exam MFE/3F Solutions Chapter 18 – Binomial Short Rate Models

© ActuarialBrew.com 2014 Page 18.07

The calculations to obtain these prices are:

1(2,3, ) 0.8411

1.18891

(2,3, ) (2,3, ) 0.87041.1489

1(2,3, ) 0.8950

1.11730.5(0.8411 0.8704)

(1,3, ) 0.74301.1517

0.5(0.8704 0.8950)(1,3, ) 0.7816

1.12930.5(0.7430 0.7816)

(0,3) 0.1.10

uu

ud du

dd

u

d

P r

P r P r

P r

P r

P r

P

= =

= = =

= =

+= =

+= =

+= = 6930

We don’t need to calculate P(0,3) to answer this question, but we included it for completeness.

The formula for the one-year yield volatility for a T-year zero-coupon bond is:

1 /( 1)

1 /( 1)

(1, , ) 1Yield volatility 0.5 ln

(1, , ) 1

Tu

Td

P T r

P T r

- -

- -

È ˘-Í ˙= ¥Í ˙-Î ˚

The yield volatility of the 3-year bond is:

1 /(3 1)

1 /(3 1)0.7430 1

Yield volatility 0.5 ln 0.1000.7816 1

- -

- -

È ˘-= ¥ =Í ˙-Í ˙Î ˚

Solution 18.10

D Black-Derman-Toy Model

The value of 0r is the same as the yield on the 1-year zero-coupon bond, and the yield

volatility of the 2-year bond is 1s :

0

1

8%

12%

r

s==

The first part of the interest rate tree is:

121ur R e s= 0.24

1R e

0r fi 8%

1dr R= 1R

Page 422: ActuarialBrew.com Exam MFE / 3F Actuarial Models …actuarialbrew.com/data/documents/MFE-Solutions-2014-1st-ed.pdf · Exam MFE / 3F Actuarial Models Financial Economics Segment Solutions

Exam MFE/3F Solutions Chapter 18 – Binomial Short Rate Models

© ActuarialBrew.com 2014 Page 18.08

To obtain the value of ur , we need to determine the value of 1R . The tree must correctly

price the 2-year zero-coupon bond, so:

0.2411

2

0.2411

0.2411

1 1 1(0,2) (0.5)

1.08 11

1 1 1 1(0.5)

1.09 1.08 11

1 11.818028

11

PRR e

RR e

RR e

Ê ˆ= +Á ˜++Ë ¯

Ê ˆÊ ˆ = +Á ˜Á ˜Ë ¯ ++Ë ¯

= +++

There are 2 ways to answer this question.

Method 1

Since we are provided with the possible answers, a perfectly valid approach during the exam is to use trial & error to see which one works. We use the following relationships:

0.24 0.241 1u uR e r R r e

Let’s try Choice C first:

0.24

1 11.833614

1.10171 0.1017e

The value 1.833614 is greater than the correct answer of 1.818028, so let’s try a higher interest rate. Let’s try Choice D:

0.24

1 11.818017

1.11221 0.1122e

The value of 1.818017 is within rounding tolerance to the correct answer of 1.818028, so Choice D is the correct answer.

Method 2

The second method is more time consuming:

( ) ( )

( )

0.2411

0.24 0.241 1 1 1

0.24 2 0.241 1 1 1

21 1 1

21 1

1 11.818028

11

1.818028 1 1 1 1

1.818028 1 2 2.271249

1.818028 4.129196 2.311168 2 2.271249

2.311168 1.857947 0.181971 0

RR e

R e R R R e

R e R R e R

R R R

R R

= +++

+ + = + + +

+ + + = +

+ + = +

+ - =

Page 423: ActuarialBrew.com Exam MFE / 3F Actuarial Models …actuarialbrew.com/data/documents/MFE-Solutions-2014-1st-ed.pdf · Exam MFE / 3F Actuarial Models Financial Economics Segment Solutions

Exam MFE/3F Solutions Chapter 18 – Binomial Short Rate Models

© ActuarialBrew.com 2014 Page 18.09

We can use the quadratic formula to find 1R :

2

1

1 1

1.857947 1.857947 4(2.311168)( 0.181971)2(2.311168)

0.088253 or 0.892153

R

R R

- ± - -=

= = -

We use the positive value of 1R since the negative value doesn’t make sense for interest

rates. The value of ur is:

12 2 0.121 0.088253 0.112192ur R e es ¥= = =

Solution 18.11

A Black-Derman-Toy Model

The value of 0r is the same as the yield on the 1-year zero-coupon bond, and the yield

volatility of the 2-year bond is 1s :

0

1

8%

12%

r

s==

The first part of the interest rate tree is:

121ur R e s= 0.24

1R e

0r fi 8%

1dr R= 1R

To obtain the value of ur , we need to determine the value of 1R . The tree must correctly

price the 2-year zero-coupon bond, so:

0.2411

2

0.2411

0.2411

1 1 1(0,2) (0.5)

1.08 11

1 1 1 1(0.5)

1.09 1.08 11

1 11.818028

11

PRR e

RR e

RR e

Ê ˆ= +Á ˜++Ë ¯

Ê ˆÊ ˆ = +Á ˜Á ˜Ë ¯ ++Ë ¯

= +++

( ) ( )

( )0.24 0.24

1 1 1 1

0.24 2 0.241 1 1 1

21 1 1

21 1

1.818028 1 1 1 1

1.818028 1 2 2.271249

1.818028 4.129196 2.311168 2 2.271249

2.311168 1.857947 0.181971 0

R e R R R e

R e R R e R

R R R

R R

+ + = + + +

+ + + = +

+ + = +

+ - =

Page 424: ActuarialBrew.com Exam MFE / 3F Actuarial Models …actuarialbrew.com/data/documents/MFE-Solutions-2014-1st-ed.pdf · Exam MFE / 3F Actuarial Models Financial Economics Segment Solutions

Exam MFE/3F Solutions Chapter 18 – Binomial Short Rate Models

© ActuarialBrew.com 2014 Page 18.10

We can use the quadratic formula to find 1R :

2

1

1 1

1.857947 1.857947 4(2.311168)( 0.181971)2(2.311168)

0.088253 or 0.892153

R

R R

- ± - -=

= = -

We use the positive value of 1R . The values of ur and dr are:

12 2 0.12

1

1

0.088253 0.112192

0.088253u

d

r R e e

r R

s ¥= = == =

The first part of the interest rate tree is:

11.2192% 8.0000%

8.8253%

The cap pays off only if the interest rate reaches 11.2192%. There is a 50% probability of this occurring, so the value of the interest rate cap is:

100 (0.112192 0.1)

0.5 0.50751.08 1.112192¥ -¥ =

¥

Solution 18.12

B Black-Derman-Toy Model

The value of 0r is the same as the yield on the 1-year zero-coupon bond, so 0 7%r = . The

first part of the interest rate tree is:

120.076821ur e

7% 0.076821dr =

The value of 1s must correctly price the 2-year zero-coupon bond, so:

1

1

1 1

1

2

2

2

2 2

2

1 1 1(0,2) (0.5)

1.07 1 0.0768211 0.076821

1 1 1 1(0.5)

1.08 1.07 1 0.0768211 0.076821

1.834705(1 0.076821 )(1 0.076821) 1 0.076821 1 0.076821

1.834705(1.076821 0.082722 )

Pe

e

e e

e

1

1

2

2

1

1

2.076821 0.076821

0.074950 0.1011722 ln(1.349855)

0.149998

e

e

Page 425: ActuarialBrew.com Exam MFE / 3F Actuarial Models …actuarialbrew.com/data/documents/MFE-Solutions-2014-1st-ed.pdf · Exam MFE / 3F Actuarial Models Financial Economics Segment Solutions

Exam MFE/3F Solutions Chapter 18 – Binomial Short Rate Models

© ActuarialBrew.com 2014 Page 18.11

The yield volatility of the 2-year zero-coupon bond is:

1 0.149998s =

Solution 18.13

D Black-Derman-Toy Model

Each node is 2 i he s times the one below it. This means that 13.51% is 24e s times as large as 7.17%:

24

2

0.1351 0.0717

0.1351 1ln 0.15838

0.0717 4

e s

s

=

Ê ˆ= ¥ =Á ˜Ë ¯

The interest rate for the node above 7.17% is:

22 2 0.158380.0717 0.0717 0.09842X e es ¥= = =

Solution 18.14

D Binomial Interest Rate Model

Since interest rates can move up or down by 300 basis points each period, the interest-rate tree is:

0.17 0.14 0.11

0.11 0.11 0.08 0.05

The price of the 3-year zero-coupon bond is:

20*

0

(0.11 0.14 0.17)

(0.11 0.14 0.11)

(0.11 0.08 0.11)

(0.11 0.08 0.05)

(0,3)

0.25

0.25

0.25

0.250.72054

ii rP E e

e

e

e

e

=-

- + +

- + +

- + +

- + +

È ˘Â= Í ˙Î ˚

= ¥

+ ¥

+ ¥

+ ¥=

The yield of the 3-year zero-coupon bond is:

[ ]ln (0, ) ln(0.72054)

0.109253

P Tyield

T= - = - =

Page 426: ActuarialBrew.com Exam MFE / 3F Actuarial Models …actuarialbrew.com/data/documents/MFE-Solutions-2014-1st-ed.pdf · Exam MFE / 3F Actuarial Models Financial Economics Segment Solutions

Exam MFE/3F Solutions Chapter 18 – Binomial Short Rate Models

© ActuarialBrew.com 2014 Page 18.12

Solution 18.15

E Black-Derman-Toy Model

The tree of prices for the 3-year bond is:

0.7717 0.6648

0.6086 0.8366 0.7351 0.8858

The calculations to obtain these prices are:

1(2,3, ) 0.7717

1.29591

(2,3, ) (2,3, ) 0.83661.1953

1(2,3, ) 0.8858

1.12890.5(0.7717 0.8366)

(1,3, ) 0.66481.2097

0.5(0.8366 0.8858)(1,3, ) 0.7351

1.17160.5(0.6648 0.7351)

(0,3) 0.1.15

uu

ud du

dd

u

d

P r

P r P r

P r

P r

P r

P

= =

= = =

= =

+= =

+= =

+= = 6086

We don’t need to calculate P(0,3) to answer this question, but we included it for completeness.

The formula for the one-year yield volatility for a T-year zero-coupon bond is:

1 /( 1)

1 /( 1)

(1, , ) 1Yield volatility 0.5 ln

(1, , ) 1

Tu

Td

P T r

P T r

- -

- -

È ˘-Í ˙= ¥Í ˙-Î ˚

The yield volatility of the 3-year bond is:

1 /(3 1)

1 /(3 1)0.6648 1

Yield volatility 0.5 ln 0.15420.7351 1

- -

- -

È ˘-= ¥ =Í ˙-Í ˙Î ˚

Solution 18.16

A Black-Derman-Toy Model

Although the cap payments are made at the end of each year, we will value them at the beginning of each year using the following formula:

1

1

0,-year caplet payoff at time ( 1)

1T R

T

Max R KT T Notional

R-

-

-È ˘Î ˚- = ¥+

Page 427: ActuarialBrew.com Exam MFE / 3F Actuarial Models …actuarialbrew.com/data/documents/MFE-Solutions-2014-1st-ed.pdf · Exam MFE / 3F Actuarial Models Financial Economics Segment Solutions

Exam MFE/3F Solutions Chapter 18 – Binomial Short Rate Models

© ActuarialBrew.com 2014 Page 18.13

A 4-year cap consists of a 1-year caplet, a 2-year caplet, a 3-year caplet, and a 4-year caplet.

The payoff for the 1-year caplet is zero since 15% is less than 21%.

The payoff for the 2-year caplet is also zero since 20.96% and 17.16% are both less than 21%.

The payoff for the 3-year caplet is positive only when the short-term interest rate increases to 29.59%:

20.2959 0.21

0.2959 100 6.62861.2959

R-= fi ¥ =

The payoff for the 4-year caplet is positive only when the short-term rate increases to 25.03%:

30.2503 0.21

0.2503 100 3.22321.2503

R

The possible payments from the cap are illustrated in the tree below:

3.2232 6.6286 0.0000 0.0000

0.0000 0.0000 0.0000 0.0000

0.0000 0.0000

The value of the 3-year caplet is:

2 6.6286Value of 3-year caplet (0.5) 1.1913

1.15 1.2096

The value of the 4-year caplet is:

3 3.2232Value of the 4-year caplet (0.5) 0.2235

1.15 1.2096 1.2959

The value of the 4-year cap is:

(1-year caplet) (2-year caplet) (3-year caplet) (4-year caplet)

0.000 0.0000 1.1913 0.2235 1.4148

Page 428: ActuarialBrew.com Exam MFE / 3F Actuarial Models …actuarialbrew.com/data/documents/MFE-Solutions-2014-1st-ed.pdf · Exam MFE / 3F Actuarial Models Financial Economics Segment Solutions

Exam MFE/3F Solutions Chapter 18 – Binomial Short Rate Models

© ActuarialBrew.com 2014 Page 18.14

Solution 18.17

C Cox-Ingersoll-Ross Model

We begin with the Sharpe ratio and parameterize it for the CIR model:

af

afs s

af

-=

-=

-=

( , , )( , )

( , , )

( , , )( , ) ( )

( , , )( , )

r t T rr t

q r t T

r r t T rB t T rr t T r

rB t T

We use the value of a (0.08,0,3) provided in the question:

f

f

-¥ =

=

0.105324 0.080.08

(0,3)

(0,3) 0.31655

B

B

Making use of the fact that =(0,3) (2,5)B B , we have:

af

a f

-¥ =

= += +=

(0.09,2,5) 0.090.09

(2,5)

(0.09,2,5) 0.09 (2,5) (0.09)0.09 0.31655(0.09)0.1184895

B

B

Solution 18.18

D Interest Rate Cap

The tree of interest rates is:

13.991% 12.038%

7.000% 10.328% 10.055% 7.623%

Although the cap payments are made at the end of each year, we will follow the convention of valuing them at the beginning of each year using the following formula:

1

1

0,-year caplet payoff at time ( 1)

1T R

T

Max R KT T Notional

R-

-

-È ˘Î ˚- = ¥+

The interest rate cap consists of a 1-year caplet, a 2-year caplet, and a 3-year caplet.

The payoff for the 1-year caplet is zero since 7% is less than 11.0%.

Page 429: ActuarialBrew.com Exam MFE / 3F Actuarial Models …actuarialbrew.com/data/documents/MFE-Solutions-2014-1st-ed.pdf · Exam MFE / 3F Actuarial Models Financial Economics Segment Solutions

Exam MFE/3F Solutions Chapter 18 – Binomial Short Rate Models

© ActuarialBrew.com 2014 Page 18.15

The payoff for the 2-year caplet is positive only when the short-term interest rate increases to 12.038%, since 10.055% is less than 11%:

10.12038 0.11

0.12038 100 0.92651.12038

R-= fi ¥ =

The payoff for the 3-year caplet is positive only when the short-term rate increases to 13.991%, since the other short-term interest rates at that time are less than 11%:

20.13991 0.11

0.13991 100 2.62391.13991

R

The payoff table is illustrated in the tree below:

2.6239 0.9265

0.0000 0.0000 0.0000 0.0000

The value of the 2-year caplet is:

0.9265

Value of 2-year caplet 0.5 0.43291.07

The value of the 3-year caplet is:

2 2.6239Value of the 3-year caplet (0.5) 0.5472

1.07 1.12038

The value of the 3-year cap is:

(1-year caplet) (2-year caplet) (3-year caplet)

0.0000 0.4329 0.5472 0.9801

Solution 18.19

E Black-Derman-Toy Model

Each node is 2 i he s times the one below it. This means that 38.75% is 24e s times as large as 10.08%:

s

s

=

Ê ˆ= ¥ =Á ˜Ë ¯

24

2

0.3875 0.1008

0.3875 1ln 0.33664

0.1008 4

e

The interest rate for the node above 10.08% is:

s ¥= =22 2 0.336640.1008 0.1008 0.1976e e

Page 430: ActuarialBrew.com Exam MFE / 3F Actuarial Models …actuarialbrew.com/data/documents/MFE-Solutions-2014-1st-ed.pdf · Exam MFE / 3F Actuarial Models Financial Economics Segment Solutions

Exam MFE/3F Solutions Chapter 18 – Binomial Short Rate Models

© ActuarialBrew.com 2014 Page 18.16

The tree of prices for the 3-year bond is:

0.7207 0.6567

? 0.8350 0.7669 0.9084

The calculations to obtain these prices are:

= =

= = =

= =

+= =

+= =

1(2,3, ) 0.7207

1.38751

(2,3, ) (2,3, ) 0.83501.1976

1(2,3, ) 0.9084

1.10080.5(0.7207 0.8350)

(1,3, ) 0.65671.1845

0.5(0.8350 0.9084)(1,3, ) 0.7669

1.1367

uu

ud du

dd

u

d

P r

P r P r

P r

P r

P r

We cannot calculate the current price of the bond, P(0,3) because we do not know the value of 0r , but we do not need P(0,3) to answer this question.

The formula for the one-year yield volatility for a T-year zero-coupon bond is:

1 /( 1)

1 /( 1)

(1, , ) 1Yield volatility 0.5 ln

(1, , ) 1

Tu

Td

P T r

P T r

- -

- -

È ˘-Í ˙= ¥Í ˙-Î ˚

The yield volatility of the 3-year bond is:

- -

- -

È ˘-= ¥ =Í ˙-Í ˙Î ˚

1 /(3 1)

1 /(3 1)0.6567 1

Yield volatility 0.5 ln 0.25010.7669 1

Solution 18.20

D Black-Derman-Toy Model

The value of 0r is the same as the yield on the 1-year zero-coupon bond, and the yield

volatility of the 2-year bond is 1s :

s

= fi =+=

00

1

10.9346 7%

1

9%

rr

Page 431: ActuarialBrew.com Exam MFE / 3F Actuarial Models …actuarialbrew.com/data/documents/MFE-Solutions-2014-1st-ed.pdf · Exam MFE / 3F Actuarial Models Financial Economics Segment Solutions

Exam MFE/3F Solutions Chapter 18 – Binomial Short Rate Models

© ActuarialBrew.com 2014 Page 18.17

The first part of the interest rate tree is:

121ur R e s= 0.18

dr e

0r fi 7%

1dr R= dr

We need to determine the value of dr . The tree must correctly price the 2-year zero-

coupon bond, so:

Ê ˆ= +Á ˜++Ë ¯

Ê ˆ= +Á ˜++Ë ¯

0.18

0.18

1 1 1(0,2) (0.5)

1.07 11

1 10.8264 0.9346(0.5)

11

dd

dd

Prr e

rr e

The quickest way to answer this question is to use trial and error.

Let’s try Choice C first:

0.18

1 10.9346(0.5) 0.8422

1 0.101 (0.10)e

Since the 0.8422 is greater than 0.8264, let’s try a higher short rate.

Let’s try Choice D:

0.18

1 10.9346(0.5) 0.8264

1 0.11931 (0.1193)e

Choice D is the correct answer.

Solution 18.21

A Risk-Neutral Probability

Let’s use P(0,2) to denote the price of a 2-year zero-coupon bond that matures for $1.

We can make use of put-call parity:

0(69) 69 (0,2) (69)

(69) (69) 69 (0,2) 75

C P S P

P C P

We can use the stock prices to determine the risk-neutral probability that the up state of the world occurs:

( ) 1

0(1 ) (1.06) 50 /75 0.3933* 0.59

100 /75 50 /75 0.6667

hr h r de dp

u d u d

Page 432: ActuarialBrew.com Exam MFE / 3F Actuarial Models …actuarialbrew.com/data/documents/MFE-Solutions-2014-1st-ed.pdf · Exam MFE / 3F Actuarial Models Financial Economics Segment Solutions

Exam MFE/3F Solutions Chapter 18 – Binomial Short Rate Models

© ActuarialBrew.com 2014 Page 18.18

If the up state occurs, then the zero-coupon bond will have a value of 1

1.075 at time 1, and

if the down state occurs, then the zero-coupon bond will have a value of 1

1.045 at time 1.

The time 0 value is found using the risk-neutral probabilities and the risk-free rate at time 0:

1 1 1

(0,2) 0.59 0.41 0.887911.06 1.075 1.045

P

We can now use the equation for put-call parity described above to find the solution:

(69) (69) 69 (0,2) 75 69 0.88791 75 13.7344P C P

Solution 18.22

B Caplet in Black-Derman-Toy Model

In each column of rates, each rate is greater than the rate below it by a factor of:

s2 i he

Therefore, the missing rate in the third column is:

s- = ¥ =2 1 0.22900.2290 0.2290 0.1747

0.3001ie

The missing rate in the fourth column is:

s- = ¥ =2 1 0.14510.3582 0.3582 0.2649

0.1962ie

The tree of short-term rates is:

35.82% 30.01% 21.16% 26.49%

15.00% 22.90% 16.98% 19.62% 17.47% 14.51%

The caplet pays off only if the interest rate at the end of the third year is greater than 20%. The payoff table is:

Time 0 Time 1 Time 2 Time 3 11.6478 0.0000 0.0000 5.1314

0.0000 0.0000 0.0000 0.0000 0.0000 0.0000

Page 433: ActuarialBrew.com Exam MFE / 3F Actuarial Models …actuarialbrew.com/data/documents/MFE-Solutions-2014-1st-ed.pdf · Exam MFE / 3F Actuarial Models Financial Economics Segment Solutions

Exam MFE/3F Solutions Chapter 18 – Binomial Short Rate Models

© ActuarialBrew.com 2014 Page 18.19

The payments have been converted to their equivalents payable at the end of 3 years. The calculations for the tree above are shown below:

¥ - =

¥ - =

100 (0.3582 0.20)11.6478

1.3582100 (0.2649 0.20)

5.13141.2649

We work recursively to calculate the value of the caplet. For example, if the interest rate increases at the end of the first year and at the end of the second year, then its value at the end of the second year is:

¥ + ¥ =0.5 11.6478 0.5 5.1314

6.45301.3001

The completed tree is:

Time 0 Time 1 Time 2 Time 3 11.6478 6.4530 3.5245 5.1314

1.9204 2.0876 0.8923 0.0000 0.0000 0.0000

The value of the year-4 caplet is $1.9204.

Solution 18.23

D Binomial Interest Rate Model

A 1-year bond in 2 years is a 3-year bond now. The tree of prices for the 3-year bond is:

0.8353 1.0000 0.7278 0.8869 1.0000

0.6658 0.9048 1.0000 0.8421

0.9418 1.0000

There is no need to calculate the first two columns of the tree to answer this question, but they are shown above for illustrative purposes.

Page 434: ActuarialBrew.com Exam MFE / 3F Actuarial Models …actuarialbrew.com/data/documents/MFE-Solutions-2014-1st-ed.pdf · Exam MFE / 3F Actuarial Models Financial Economics Segment Solutions

Exam MFE/3F Solutions Chapter 18 – Binomial Short Rate Models

© ActuarialBrew.com 2014 Page 18.20

The third column is easily obtained by discounting a 1-year bond at the possible interest rates at time 2:

-

-

-

-

=

=

=

=

0.18

0.12

0.10

0.06

0.8353

0.8869

0.9048

0.9418

e

e

e

e

The put option pays off only if the time-2 price falls below $0.90. This occurs at 2 nodes, and the payoffs are:

= - == - =

2

2

up-up: 0.90 0.8353 0.0647

up-down: 0.90 0.8869 0.0131

V

V

The price of the put option is:

-=-

+ +È ˘Â= ¥ = + =Í ˙Î ˚

2 10

2*

0 2 (0.12 0.15) (0.12 0.15)(0.8) 0.0647 (0.8)(0.2)0.0131

0.0332ii rV E V ee e

Solution 18.24

B Black-Derman-Toy Model

The ratio of the short rate to the rate below it is constant within a column, so:

= fi = fi =0.450.30

0.20uu ud ud

udud dd ud

r r rr

r r r

The value of a 2-year bond is:

0 0

0 0

1 1 1 1(0,2) 0.5

1 1 1 1

1 1 1 0.757140.5

1 1.40 1.25 1

u dP

r r r r

r r

The value of a 3-year bond is:

ÈÊ ˆ Ê ˆ Ê ˆ Ê ˆ Ê ˆ Ê ˆ= +ÍÁ ˜ Á ˜ Á ˜ Á ˜ Á ˜ Á ˜+ + + + + +Ë ¯ Ë ¯ Ë ¯ Ë ¯ Ë ¯ Ë ¯ÍÎ

˘Ê ˆ Ê ˆ Ê ˆ Ê ˆ Ê ˆ Ê ˆ+ + ˙Á ˜ Á ˜ Á ˜ Á ˜ Á ˜ Á ˜+ + + + + +Ë ¯ Ë ¯ Ë ¯ Ë ¯ Ë ¯ Ë ¯ ˙

Ê ˆ Ê ˆ Ê ˆ= +Á ˜ Á ˜ ÁË ¯ Ë ¯ Ë+

0 0

0 0

0

1 1 1 1 1 1(0,3) 0.25

1 1 1 1 1 1

1 1 1 1 1 11 1 1 1 1 1

1 1 1 10.25

1 1.40 1.45 1.40

u uu u ud

d ud d dd

Pr r r r r r

r r r r r r

rÈ ˘Ê ˆ Ê ˆ Ê ˆ Ê ˆ Ê ˆ+ +Í ˙˜ Á ˜ Á ˜ Á ˜ Á ˜ Á ˜¯ Ë ¯ Ë ¯ Ë ¯ Ë ¯ Ë ¯Î ˚

=+ 0

1 1 1 1 11.30 1.25 1.30 1.25 1.20

0.581031 r

Page 435: ActuarialBrew.com Exam MFE / 3F Actuarial Models …actuarialbrew.com/data/documents/MFE-Solutions-2014-1st-ed.pdf · Exam MFE / 3F Actuarial Models Financial Economics Segment Solutions

Exam MFE/3F Solutions Chapter 18 – Binomial Short Rate Models

© ActuarialBrew.com 2014 Page 18.21

The forward price is:

0

0

0.581031

0,2 0 0.757141

(0,3)(2,3) (2,3) 0.76740

(0,2)r

r

PF P P

P

Multiplying by 1,000, we have:

[ ]¥ = ¥ =0,21,000 (2,3) 1,000 0.76740 767.40F P

Page 436: ActuarialBrew.com Exam MFE / 3F Actuarial Models …actuarialbrew.com/data/documents/MFE-Solutions-2014-1st-ed.pdf · Exam MFE / 3F Actuarial Models Financial Economics Segment Solutions

Exam MFE/3F Solutions Chapter 18 – Binomial Short Rate Models

© ActuarialBrew.com 2014 Page 18.22

This page is intentionally blank.

Page 437: ActuarialBrew.com Exam MFE / 3F Actuarial Models …actuarialbrew.com/data/documents/MFE-Solutions-2014-1st-ed.pdf · Exam MFE / 3F Actuarial Models Financial Economics Segment Solutions

Exam MFE/3F Solutions Chapter 19 – Continuous-Time Models of Interest Rates

© ActuarialBrew.com 2014 Page 19.01

Chapter 19 – Solutions

Solution 19.01

A Duration-Hedging

The number of 10-year bonds that must be purchased to duration-hedge the position is:

2 2

1 1

( ) ( , ) (3 0) 85.400.349

( ) ( , ) (10 0) 73.50T t P t T

NT t P t T

- ¥ - ¥= - = - = -- ¥ - ¥

Purchasing –0.349 of the 10-year bonds is equivalent to selling 0.349 of the 10-year bonds.

Solution 19.02

C Delta-Hedging

The number of 10-year bonds that must be purchased to delta-hedge the position is:

2

1

( , , ) 1.930.607

( , , ) 3.18r

r

P r t TN

P r t T-= - = - = --

Purchasing –0.607 of the 10-year bonds is equivalent to selling 0.607 of the 10-year bonds.

Solution 19.03

E Rendleman-Bartter Model

In the Rendleman-Bartter model, the short rate follows geometric Brownian motion:

s s s= + ¤ = - +2 [ln ] ( 0.5 )dr rdt rdZ d r dt dZa a

Choice E is of this form with:

s s- = =20.5 0.01 and 0.02a

Solution 19.04

C Vasicek Model

The process describes the Vasicek model with:

0.20.10.02

bs

===

a

The Sharpe ratio is zero, so:

0f =

Page 438: ActuarialBrew.com Exam MFE / 3F Actuarial Models …actuarialbrew.com/data/documents/MFE-Solutions-2014-1st-ed.pdf · Exam MFE / 3F Actuarial Models Financial Economics Segment Solutions

Exam MFE/3F Solutions Chapter 19 – Continuous-Time Models of Interest Rates

© ActuarialBrew.com 2014 Page 19.02

We can use these values to find the value that the yield approaches as the time to maturity goes to infinity:

s sf Ê ˆ= + - = + - =Á ˜Ë ¯

2 2

2 20.02 0.02

0.5 0.1 0 0.5 0.0950.2 0.2

r ba a

Solution 19.05

D Vasicek Model & Forward Interest Rates

The process describes the Vasicek model with:

0.20.10.02

bs

===

a

We can use these values to find the price of a 10-year zero-coupon bond:

- - -

- - ¥

- -= = =

=

= = =

) 0.2(10)

(0,10) 4.32332 0.08

1 1(0,10) 4.32332

0.2(0,10) 0.57774

( ,0,10) (0,10) 0.57774 0.40881

T t

B r

e eB

A

P r A e e

a(

a

We can also find the price of a 9-year zero-coupon bond:

) 0.2(9)

(0,9) 4.17351 0.08

1 1(0,9) 4.17351

0.2(0,9) 0.62674

( ,0,9) (0,9) 0.62674 0.44883

T t

B r

e eB

A

P r A e e

a(

a

The forward interest rate, agreed upon now, that applies from time 9 to time 10 can be found with the following formula:

0

0

0

1 ( ,0, )( , ) 1 1

( , ) ( ,0, )

( ,0,9) 0.44883(9,10) 1 1 0.0979

( ,0,10) 0.40881

P r TR T T s

P T T s P r T s

P rR

P r

+ = - = -+ +

= - = - =

Since the amount of the loan is $100, the payment at the end of 10 years is:

100 (1 0.0979) 109.79¥ + =

Solution 19.06

B Rendleman-Bartter Model

In the Rendleman-Bartter Model, the short-term rate follows geometric Brownian motion.

Page 439: ActuarialBrew.com Exam MFE / 3F Actuarial Models …actuarialbrew.com/data/documents/MFE-Solutions-2014-1st-ed.pdf · Exam MFE / 3F Actuarial Models Financial Economics Segment Solutions

Exam MFE/3F Solutions Chapter 19 – Continuous-Time Models of Interest Rates

© ActuarialBrew.com 2014 Page 19.03

From Chapter 5 of the ActuarialBrew.com Study Manual, we know that the probability that the short rate is greater than 10% at the end of 3 months can be found as:

[ ]> = 2ˆProb ( ) 0.10 ( )r t N d

Although we have -0.25 for the volatility parameter in the geometric Brownian motion,

we use the absolute value of -0.25 in the formula for 2d , because the s in the formula

for 2d is the standard deviation of the stock’s return, and it must therefore be positive:

2 2

2

( ) 0.09ln ( 0.5 )( ) ln (0.10 0.5 0.25 ) 0.25

0.10ˆ0.25 0.25

0.70538

r tT t

Kd

T t

a d s

s

Ê ˆ Ê ˆ+ - - - + - ¥ ¥Á ˜ Á ˜Ë ¯ Ë ¯= =

-= -

The value of 2ˆ( )N d is:

2ˆ( ) ( 0.70538) 0.24029N d N= - =

Solution 19.07

A Cox-Ingersoll-Ross Model

The process describes the Cox-Ingersoll-Ross model with:

0.18

0.10.17

bs

===

a

The Sharpe ratio is zero, so f is zero:

( , ) 0

0

0

r t

r

ffs

f

=

=

=

We can use these values to find r , the yield as the maturity goes to infinity:

g f s

f g

= - + = - + ¥ =

= = =- +- +

2 2 2 2( ) 2 (0.18 0) 2 0.17 0.30033

2 2(0.18)(0.10)0.0749

(0.18 0 0.30033)( )b

r

a

a

a

Page 440: ActuarialBrew.com Exam MFE / 3F Actuarial Models …actuarialbrew.com/data/documents/MFE-Solutions-2014-1st-ed.pdf · Exam MFE / 3F Actuarial Models Financial Economics Segment Solutions

Exam MFE/3F Solutions Chapter 19 – Continuous-Time Models of Interest Rates

© ActuarialBrew.com 2014 Page 19.04

Solution 19.08

A Risk-Neutral Vasicek Model

Greg uses the Vasicek model for the short rate. The risk-neutral version of Greg’s model is:

[ ][ ][ ]

s f s

sf s

= + +

= - + +

= - + ¥ +

= - +

( ) ( ) ( , ) ( )

( )

0.2(0.1 ) 0.05 0.08 0.05

0.2(0.12 ) 0.05

dr r r r t dt r dZ

b r dt dZ

r dt dZ

r dt dZ

a

a

Colleen’s model will produce the same price as Greg’s if the risk-neutral version of her model is the same as the risk-neutral version of Greg’s model.

The risk-neutral version of the model shown in Choice A is:

[ ] [ ]sf s= - + + = - + ¥ +

= - +

( ) 0.2(0.08 ) 0.05 0.16 0.05

0.2(0.12 ) 0.05

dr b r dt dZ r dt dZ

r dt dZ

a

Therefore, the risk-neutral version of Choice A is the same as the risk-neutral version of Greg’s model, and Choice A is correct.

For the sake of thoroughness, let’s consider the other choices as well. As shown below, the risk-neutral version of Choice B does not match the risk-neutral version of Greg’s model:

[ ] [ ]sf s= - + + = - + ¥ +

= - +

( ) 0.2(0.08 ) 0.05 0.32 0.05

0.2(0.16 ) 0.05

dr b r dt dZ r dt dZ

r dt dZ

a

As shown below, the risk-neutral version of Choice C does not match the risk-neutral version of Greg’s model:

[ ] [ ]sf s= - + + = - + ¥ +

= - +

( ) 0.2(0.12 ) 0.05 0.08 0.05

0.2(0.14 ) 0.05

dr b r dt dZ r dt dZ

r dt dZ

a

Choices D and E are unlikely to be correct, because they are based on the Cox-Ingersoll-Ross model. Since we have already shown that Choice A is correct, and the question tells us that only one model produces the same price as Greg’s model, Choices D and E must be incorrect.

Solution 19.09

D Vasicek Model

This question is based on Table 24.1 in the second edition of the Derivatives Markets textbook.

Page 441: ActuarialBrew.com Exam MFE / 3F Actuarial Models …actuarialbrew.com/data/documents/MFE-Solutions-2014-1st-ed.pdf · Exam MFE / 3F Actuarial Models Financial Economics Segment Solutions

Exam MFE/3F Solutions Chapter 19 – Continuous-Time Models of Interest Rates

© ActuarialBrew.com 2014 Page 19.05

The process can be written as:

0.25(0.10 ) 0.01dr r dt dZ= - +

This is the Vasicek model with:

0.25

0.100.01

bs

===

a

The gap between r and b is expected to close at a rate equal to a times the gap:

0.25(0.10 0.05) 0.0125X

Solution 19.10

E Continuous-Time Interest Rate Models

The page numbers below refer to the Second Edition of Derivatives Markets.

Choice A is true, because the Rendleman-Bartter model assumes that the short-rate follows geometric Brownian motion. See the sentence preceding Equation 24.24 on page 785.

Choice B is true, because a shortcoming of the Vasicek model is that interest rates can be negative. See the last sentence of the third paragraph on page 786.

Choice C is true, because a shortcoming of the Vasicek model is that the variance does not change with the short-rate. See the last sentence of the third paragraph on page 786.

Choice D is true, because in the CIR model the variance increases with the short-rate. See the second bullet point on the lower half of page 787.

Choice E is false, because the Rendleman-Bartter model is not mean-reverting. See the last paragraph on page 785.

Solution 19.11

E Duration-Hedging

This question is similar to Problem 24.4 at the end of Derivatives Markets (second edition) Chapter 24.

Let’s assume that the bonds both mature for $1. We’ll scale up the result at the end.

The prices of the bonds are:

2(0.08)

7(0.08)

1(0,2) 0.85214379

1(0,7) 0.57120906

Pe

Pe

= =

= =

Page 442: ActuarialBrew.com Exam MFE / 3F Actuarial Models …actuarialbrew.com/data/documents/MFE-Solutions-2014-1st-ed.pdf · Exam MFE / 3F Actuarial Models Financial Economics Segment Solutions

Exam MFE/3F Solutions Chapter 19 – Continuous-Time Models of Interest Rates

© ActuarialBrew.com 2014 Page 19.06

The quantity of 7-year bonds that must be purchased is:

2(0.08)

2 27(0.08)

1 1

( ) ( , ) 2 (0,2) 20.426236

( ) ( , ) 7 (0,7) 7

T t P t T P eN

T t P t T P e

-

-- ¥ ¥ ¥= - = - = - = -- ¥ ¥ ¥

Since N is negative, 0.426236 7-year bonds are sold.

The amount that is lent is:

2 1( , ) ( , ) 0.85214379 0.426236 0.57120906

0.60867413

W P t T N P t T= - - ¥ = - + ¥= -

Since W is negative, 0.60867413 is borrowed at the short-term rate of 8%.

This hedged position has an initial value of zero. After 1 day, the short-term rate increases to 8.5%, and the new value of the position is:

( ) ( ) ( )1 1 1365 365 365

2 0.085 7 0.085 0.080.426236 0.60867413 0.00009662e e e

- - - -- - = -

Let’s scale this up by $1,000,000:

1,000,000 ( 0.00009662) 96.62¥ - = -

The investor loses $96.62 on the hedge.

Solution 19.12

C Risk-Neutral Vasicek Model

The short-rate follows the Vasicek model:

s= - +( )dr b r dt dZa

From the realistic process provided in the question, we observe that = 0.12b :

s= - + fi =(0.12 ) 0.12dr r dt dZ ba

The risk-neutral process is given by:

[ ] [ ]s f s sf s= + + = - + + ( ) ( ) ( , ) ( ) ( )dr r r r t dt r dZ b r dt dZa a

From risk-neutral process provided in the question, we observe that s = 0.03 and = 0.3a :

s= - + fi = =0.3(0.125 ) 0.03 0.03 and 0.3dr r dt dZ a

Page 443: ActuarialBrew.com Exam MFE / 3F Actuarial Models …actuarialbrew.com/data/documents/MFE-Solutions-2014-1st-ed.pdf · Exam MFE / 3F Actuarial Models Financial Economics Segment Solutions

Exam MFE/3F Solutions Chapter 19 – Continuous-Time Models of Interest Rates

© ActuarialBrew.com 2014 Page 19.07

We use the coefficient of dt in the risk-neutral process to determine the Sharpe ratio f :

sff

ff

f

f

- + = -- + = -

- + = -+ =

-=

=

( ) 0.3(0.125 )0.3(0.12 ) 0.03 0.3(0.125 )0.12 0.1 0.125

0.12 0.1 0.1250.125 0.12

0.10.05

b r rr r

r r

a

The values of (0,10)B and (0.08,0,10)q are:

) 0.3 101 1( , ) 3.16738

0.3(0.08,0,10) (0,10) ( ) (0,10) 3.16738 0.03 0.09502

T te eB t T

q B r B

a(

a

We can now determine the expected return on the bond:

af

a

a

-=

-=

=

( , , )( , )

( , , )(0.08,0,10) 0.08

0.050.09502

(0.08,0,10) 0.08475

r t T rr t

q r t T

Solution 19.13

D Risk-Neutral Cox-Ingersoll-Ross Model

The short-rate follows the Cox-Ingersoll-Ross model:

s= - +( )dr b r dt rdZa

From the realistic process provided in the question, we observe that = 0.08b :

s= - + fi =(0.08 ) 0.08dr r dt rdZ ba

The risk-neutral process is given by:

[ ]s f s

s f ss

f s

= + +

È ˘= - + +Í ˙Í ˙Î ˚È ˘= - + +Î ˚

( ) ( ) ( , ) ( )

( )

( )

dr r r r t dt r dZ

rb r r dt rdZ

b r r dt rdZ

a

a

a

From risk-neutral process provided in the question, we observe that s = 0.04 :

s= - + fi =0.2(0.096 ) 0.04 0.04 dr r dt rdZ

Page 444: ActuarialBrew.com Exam MFE / 3F Actuarial Models …actuarialbrew.com/data/documents/MFE-Solutions-2014-1st-ed.pdf · Exam MFE / 3F Actuarial Models Financial Economics Segment Solutions

Exam MFE/3F Solutions Chapter 19 – Continuous-Time Models of Interest Rates

© ActuarialBrew.com 2014 Page 19.08

We use the coefficient of dt in the risk-neutral process to determine a and f :

ff

f

- + = -- - = -

- - = -

( ) 0.2(0.096 )

( ) 0.0192 0.2

0.08 ( ) 0.0192 0.2

b r r r

b r r

r r

a

a a

a a

From the equation above, we observe that:

f f

= fi =- = fi = - = - =

0.08 0.0192 0.24

0.2 0.2 0.24 0.2 0.04

a a

a a

In the CIR model, the Sharpe ratio for a bond of any maturity is:

f fs

= = =0.07( , ) 0.04 0.2646

0.04r

r t

All bonds have the same Sharpe ratio, so the Sharpe ratio of the 9-year bond is 0.2646.

Solution 19.14

A Delta-Gamma Approximation

The process describes the Vasicek model with:

0.3, 0.1, 0.12b a

The short-term interest rate increases by one standard deviation, so under the Vasicek model it increases by:

s sª = =10.12 0.00628

365dt h

Itô’s Lemma is the basis of the delta-gamma-theta approximation:

= + +21( )

2r rr tdP P dr P dr P dt

Since we are using only the delta-gamma portion of the approximation, we can set the final term to 0. Let’s write the delta-gamma approximation in its discrete form:

- ª + 21(0.08628,0,10) (0.08,0,10) (0.00628) (0.00628)

2r rrP P P P

Page 445: ActuarialBrew.com Exam MFE / 3F Actuarial Models …actuarialbrew.com/data/documents/MFE-Solutions-2014-1st-ed.pdf · Exam MFE / 3F Actuarial Models Financial Economics Segment Solutions

Exam MFE/3F Solutions Chapter 19 – Continuous-Time Models of Interest Rates

© ActuarialBrew.com 2014 Page 19.09

Delta is rP , and gamma is rrP . Both are found below:

( , )

2

0.3(10)

2

( , , ) ( , )( , ) ( , , )

[ ( , )] ( , , )

1(0,10) 3.16738

0.3(0.08,0,10) 3.16738 0.60024 1.90119

(0.08,0,10) 3.16738 0.60024 6.02177

B t T r

r

rr

r

rr

P r t T A t T eP B t T P r t T

P B t T P r t T

eB

P

P

-

-

== -

=

-= =

= - ¥ = -

= ¥ =

The delta-gamma approximation of the new price can now be calculated:

= + ¥ + ¥

= + ¥ - + ¥ ¥

=

2

2

1(0.08628,0,10) (0.08,0,10) 0.00628 0.00628

21

0.60024 0.00628 ( 1.90119) 0.00628 6.021772

0.588417

r rrP P P P

Since the bond matures for $1,000, its new price is:

¥ =1,000 0.588417 588.417

Solution 19.15

A Delta-Gamma-Approximation

The process describes the Cox-Ingersoll-Ross model with:

0.3, 0.1, 0.4b a

The Sharpe ratio is zero, so f is zero:

ff fs

= fi = fi =( , ) 0 0 0r

r t

The short-term interest rate increases by one standard deviation, so under the CIR model it increases by:

s s¥ ª ¥ = =0.080.4 0.00592

365r dt r h

Itô’s Lemma is the basis of the delta-gamma-theta approximation:

= + +21( )

2r rr tdP P dr P dr P dt

Page 446: ActuarialBrew.com Exam MFE / 3F Actuarial Models …actuarialbrew.com/data/documents/MFE-Solutions-2014-1st-ed.pdf · Exam MFE / 3F Actuarial Models Financial Economics Segment Solutions

Exam MFE/3F Solutions Chapter 19 – Continuous-Time Models of Interest Rates

© ActuarialBrew.com 2014 Page 19.10

Since we are using only the delta-gamma portion of the approximation, we can set the final term to 0. Let’s write the delta-gamma approximation in its discrete form:

- ª + 21(0.08592,0,10) (0.08,0,10) (0.00592) (0.00592)

2r rrP P P P

We must calculate the value of g in order to obtain (0,10)B :

2 2 2 2

( )

( )

0.64031 10

0.64031 10

( ) 2 (0.3 0) 2 0.4 0.64031

2( 1)(0,10)

( )( 1) 2

2 1

(0.3 0 0.64031) 1 2(0.64031)

1,205.456572.12216

568.03352

T t

T te

Be

e

e

a

a

Now we can solve for delta and gamma:

-== -

== - ¥ = -

= ¥ =

( , )

2

2

( , , ) ( , )

( , ) ( , , )

[ ( , )] ( , , )

(0.08,0,10) 2.12216 0.50045 1.06203

(0.08,0,10) 2.12216 0.50045 2.25380

B t T r

r

rr

r

rr

P r t T A t T e

P B t T P r t T

P B t T P r t T

P

P

The delta-gamma approximation of the new price can now be calculated:

= + +

= + ¥ - + ¥ =

2

2

1(0.08592,0,10) (0.08,0,10) (0.00592) (0.00592)

21

0.50045 0.00592 ( 1.06203) (0.00592) 2.25380 0.4942002

r rrP P P P

Since the bond matures for $1000, its new price is:

¥ =1,000 0.494200 494.20

Solution 19.16

B Vasicek Model

In the Vasicek Model, we have:

-= ( , )( , , ) ( , ) B t T rP r t T A t T e

Page 447: ActuarialBrew.com Exam MFE / 3F Actuarial Models …actuarialbrew.com/data/documents/MFE-Solutions-2014-1st-ed.pdf · Exam MFE / 3F Actuarial Models Financial Economics Segment Solutions

Exam MFE/3F Solutions Chapter 19 – Continuous-Time Models of Interest Rates

© ActuarialBrew.com 2014 Page 19.11

We use the following two facts about the Vasicek model:

( , )A t T and ( , )B t T do not depend on r.

= -( , ) (0, )A t T A T t and = -( , ) (0, )B t T B T t . This implies:

(0,3) (2,5) (3,6)(0,3) (2,5) (3,6)

A A AB B B

= == =

Since (0,3) (2,5)A A , we have two equations and two unknowns:

(0,3)(0.08)

(0,3)(0.10)

(0,3) 0.8782

(0,3) 0.8395

B

B

A e

A e

-

-

=

=

Dividing the second equation into the first equation allows us to find (0,3)B :

(0,3)(0.08) (0,3)(0.10) 0.87820.8395

0.8782(0,3)(0.02) ln

0.8395(0,3) 2.25339

B Be

B

B

- + =

Ê ˆ= Á ˜Ë ¯=

We can now solve for the value of (0,3)A :

(0,3)(0.08)

(0,3)(0.08)

2.25339(0.08)

(0,3) 0.8782

(0,3) 0.8782

(0,3) 0.8782(0,3) 1.05168

B

B

A e

A e

A eA

- =

=

==

We can now solve for *r :

(0,3)( *)

2.25339 *

(0,3) 0.8586

1.05168 0.8586

0.85862.25339 * ln

1.05168

* 0.0900

B r

r

A e

e

r

r

-

-

=

=

Ê ˆ- = Á ˜Ë ¯=

Solution 19.17

B Vasicek Model

The process describes the Vasicek model with:

0.2

0.1

0.02

b

s

===

a

Page 448: ActuarialBrew.com Exam MFE / 3F Actuarial Models …actuarialbrew.com/data/documents/MFE-Solutions-2014-1st-ed.pdf · Exam MFE / 3F Actuarial Models Financial Economics Segment Solutions

Exam MFE/3F Solutions Chapter 19 – Continuous-Time Models of Interest Rates

© ActuarialBrew.com 2014 Page 19.12

The Sharpe ratio is:

f = 0.05

We can use these values to find the price of the 10-year zero-coupon bond:

[ ] s

s sf

¥ ¥¥ - - ¥

- - -

- - -

-

Ê ˆ= + - = + - =Á ˜Ë ¯

- -= = =

= = =

=

2 2 2 2[ ( , )] 4.32332 0.02( , ) ( )4 4 0.2

2 2

2 2

) 0.2(10)

0.1000(4.32332 10)

0.02 0.020.5 0.1 0.05 0.5 0.1000

0.2 0.2

1 1( , ) 4.32332

0.2

( , ) 0.56157

( , , ) ( , )

B t Tr B t T T t

T t

r b

e eB t T

A t T e e

P r t T A t T e

a

a(

a a

a

- ¥= =( , ) 4.32332 0.080.56157 0.39737B t T r e

Since the bond matures for $100, its price is:

¥ =100 0.39737 39.737

Solution 19.18

C Vasicek Model

The process describes the Vasicek model with:

sf

====

0.150.120.030.07

ba

Since the current value of the short-term interest rate is 8%:

s s

= ¥ - = ¥ - == =

( ) ( ) 0.15 (0.12 0.08) 0.006( ) 0.03r b rr

a a

We can use the following relationship to find the value of theta, tP :

21[ ( )] ( ) ( ) ( , )

2 rr r trP r P r r r t P Pa

We can use the formula for the price of a zero-coupon bond to find the first and second derivative with respect to the short-term interest rate:

-

-

-

=

= - = -

= =

( , )

( , )

2 ( , ) 2

( , , ) ( , )

( , ) ( , ) ( , ) ( , , )

[ ( , )] ( , ) [ ( , )] ( , , )

B t T r

B t T rr

B t T rrr

P r t T A t T e

P B t T A t T e B t T P r t T

P B t T A t T e B t T P r t T

The value of (0,10)B is:

) 0.15(10 0)1 1

(0,10) 5.179130.15

T te eB

a(

a

Page 449: ActuarialBrew.com Exam MFE / 3F Actuarial Models …actuarialbrew.com/data/documents/MFE-Solutions-2014-1st-ed.pdf · Exam MFE / 3F Actuarial Models Financial Economics Segment Solutions

Exam MFE/3F Solutions Chapter 19 – Continuous-Time Models of Interest Rates

© ActuarialBrew.com 2014 Page 19.13

We have:

=

= - = - ¥ = -

= = ¥ =2 2

(0.08,0,10) 0.36636

( , ) ( , , ) 5.17913 0.36636 1.89743

[ ( , )] ( , , ) 5.17913 0.36636 9.82702

r

rr

P

P B t T P r t T

P B t T P r t T

We can now find theta:

2

2

1[ ( )] ( ) ( ) ( , )

21

0.08(0.36636) [0.03] (9.82702) 0.006 (0.03)(0.07) ( 1.89743)2

0.0403

rr r t

t

t

rP r P r r r t P P

P

P

a

Solution 19.19

A Risk-Neutral Interest Rate Models

Zachary is using the Vasicek model:

s= - +

= ¥ - +=

( ) ( )0 (0.10 ) 0.05 ( )0.05 ( )

dr b r dt dZ tdr r dt dZ tdr dZ t

a

Although Zachary’s model resembles Charlie’s model, Zachary’s model uses a non-zero Sharpe ratio:

f = 0.20

Therefore, Zachary’s model is not risk-neutral.

Charlie’s model is a risk-neutral version of the Vasicek model, so Charlie uses a Sharpe ratio of zero:

f = 0.00

Therefore, Charlie’s model is different from Zachary’s model.

The risk-neutral version of Zachary’s model is:

[ ][ ][ ]

s f s

sf s

= + +

= - + +

= ¥ - + ¥ +

= +

( ) ( ) ( , ) ( ) ( )

( ) ( )

0 (0.10 ) 0.05 0.20 0.05 ( )

0.01 0.05 ( )

dr r r r t dt r dZ t

b r dt dZ t

r dt dZ t

dt dZ t

a

a

This risk-neutral process is the same as the risk-neutral process used by Ann. Therefore, Choice (A) is the correct answer.

Page 450: ActuarialBrew.com Exam MFE / 3F Actuarial Models …actuarialbrew.com/data/documents/MFE-Solutions-2014-1st-ed.pdf · Exam MFE / 3F Actuarial Models Financial Economics Segment Solutions

Exam MFE/3F Solutions Chapter 19 – Continuous-Time Models of Interest Rates

© ActuarialBrew.com 2014 Page 19.14

Solution 19.20

A Delta-Gamma-Theta Approximation

The interest rate process describes the Vasicek model. In this model, we have:

[ ]

-

-

-

=

= -

=

( , )

( , )

2 ( , )

( , , ) ( , )

( , , ) ( , ) ( , )

( , , ) ( , ) ( , )

B t T r

B t T rr

B t T rrr

P r t T A t T e

P r t T B t T A t T e

P r t T B t T A t T e

We can calculate the value of ( , )B t T :

) 0.25(5 0)1 1

(0,5) 2.853980.25

T te eB

a(

a

We are given tP in the question, and we can calculate rP and rrP :

[ ]

-

-

= - = - ¥ = -

= = ¥ ==

(0,5)

2 (0,5) 2

(0.07,0,5) (0,5) (0,5) 2.85398 0.5961 1.70126

(0.07,0,5) (0,5) (0,5) 2.85398 0.5961 4.85536

(0.07,0,5) 0.0685

B rr

B rrr

t

P B A e

P B A e

P

A one standard deviation movement in the short-term interest rate over one day is:

s sª = ¥ =10.10 0.0052342

365dt h

We can use the delta-gamma-theta approximation to estimate the resulting change in the price:

= + +

Ê ˆª - + + = -Á ˜Ë ¯

2

2

1( )

21 1

1.70126(0.0052342) (4.85536)(0.0052342) 0.0685 0.0086512 365

r rr tdP P dr P dr P dt

The new price is:

- =0.5961 0.008651 0.5874

Page 451: ActuarialBrew.com Exam MFE / 3F Actuarial Models …actuarialbrew.com/data/documents/MFE-Solutions-2014-1st-ed.pdf · Exam MFE / 3F Actuarial Models Financial Economics Segment Solutions

Exam MFE/3F Solutions Chapter 19 – Continuous-Time Models of Interest Rates

© ActuarialBrew.com 2014 Page 19.15

Solution 19.21

E Cox-Ingersoll-Ross Model

We begin with the Sharpe ratio and parameterize it for the Cox-Ingersoll-Ross model:

af

afs s

af

-=

-=

-=

( , , )( , )

( , , )

( , , )( , ) ( )

( , , )( , )

r t T rr t

q r t T

r r t T rB t T rr t T r

rB t T

We can substitute 7.0% for a (0.05,0,10) :

f

ff

-¥ =

= +=

0.07 0.050.05

(0,10)

0.07 0.05 (0,10) (0.05)

(0,10) 0.40

B

B

B

Making use of the fact that =(0,10) (5,15)B B , we have:

af

a f

-¥ =

= += +=

(0.06,5,15) 0.060.06

(5,15)

(0.06,5,15) 0.06 (5,15) (0.06)

0.06 0.40(0.06)

0.084

B

B

Solution 19.22

C Vasicek Model

The Vasicek model of short-term interest rates is:

( )dr b r dt dZs= - +a

Therefore, we can determine the value of a:

s= - + fi =0.35( ) 0.35dr b r dt dZ a

In the Vasicek model, the Sharpe ratio is constant:

( , )r tf f=

Therefore, for any r, t, and T, we have:

( , , )

( , , )r t T rq r t T

af -=

Page 452: ActuarialBrew.com Exam MFE / 3F Actuarial Models …actuarialbrew.com/data/documents/MFE-Solutions-2014-1st-ed.pdf · Exam MFE / 3F Actuarial Models Financial Economics Segment Solutions

Exam MFE/3F Solutions Chapter 19 – Continuous-Time Models of Interest Rates

© ActuarialBrew.com 2014 Page 19.16

Since the Sharpe ratio is constant:

a a- -=(0.06,0,5) 0.06 (0.03,2,8) 0.03

(0.06,0,5) (0.03,2,8)q q

We now make use of the following formula for ( , , )q r t T :

s s= =( , , ) ( , ) ( ) ( , )q r t T B t T r B t T

Substituting this expression for ( , , )q r t T into the preceding equation allows us to solve for

(0.03,2,8) :

0.35(5 0) 0.35(8 2)

(0.06,0,5) 0.06 (0.03,2,8) 0.03(0.06,0,5) (0.03,2,8)

(0.06,0,5) 0.06 (0.03,2,8) 0.03(0,5) (2,8)

0.073 0.06 (0.03,2,8) 0.03

1 10.35 0.35

0.013 (0.03,2,8) 0.032.3606 2.5073

(0

q q

B B

e e

.03,2,8) 0.0438

Solution 19.23

D Interest Rate Derivative

The realistic process for the short rate follows:

( ) ( ) where: ( ) 0.04 0.25 & ( ) 0.4dr r dt r dZ r r r a a

The risk-neutral process follows:

[ ]s f s= + + ( ) ( ) ( , ) ( )dr r r r t dt r dZa

We can use the coefficient of the first term of the risk-neutral process to solve for the Sharpe ratio, f( , )r t :

s f

ff

- = +- = - +=

0.10 0.25 ( ) ( ) ( , )0.10 0.25 0.04 0.25 0.4 ( , )

( , ) 0.15

r r r r tr r r t

r t

a

The derivative, like all interest-rate dependent assets, must have a Sharpe ratio of 0.15. Let’s rearrange the differential equation for g, so that we can more easily observe its Sharpe ratio:

b fb b

+ -= + - fi = =( 0.09) 0.09( 0.09) ( , )

dg r rr dt dZ r t

g

Page 453: ActuarialBrew.com Exam MFE / 3F Actuarial Models …actuarialbrew.com/data/documents/MFE-Solutions-2014-1st-ed.pdf · Exam MFE / 3F Actuarial Models Financial Economics Segment Solutions

Exam MFE/3F Solutions Chapter 19 – Continuous-Time Models of Interest Rates

© ActuarialBrew.com 2014 Page 19.17

Since the Sharpe ratio is 0.15:

bb

=

=

0.090.15

0.6

Solution 19.24

E Interest Rate Derivative

Let’s rearrange the differential equation for g, so that we can more easily observe its Sharpe ratio:

f + -= + - fi = =( 0.02)( 0.02) 0.08 ( , ) 0.25

0.08dg r r

r dt dZ r tg

The realistic process for the short rate follows:

( ) ( ) where: ( ) 0.04 0.25 & ( ) 0.4dr r dt r dZ r r r a a

The risk-neutral process follows:

[ ]s f s= + + ( ) ( ) ( , ) ( )dr r r r t dt r dZa

We can now find the coefficient of the first term of the risk-neutral process:

m s f= +

= - + ¥= -

( ) ( ) ( ) ( , )0.04 0.25 0.4 0.250.14 0.25

r r r r trr

a

Solution 19.25

E CIR Model

Let’s find the differential form of each choice.

Choice A does not contain a random variable, so its differential form is easy to find:

s- -È ˘= - +Î ˚( ) (0) t tdr t br e e dta aa a

Choice B is easily recognized as an arithmetic Brownian motion:

s= +( ) ( )dr t bdt dZ t

Choice C is easily recognized as a geometric Brownian motion:

s= +( ) ( ) ( ) ( )dr t br t dt r t dZ t

Choice D is (perhaps not so easily) recognized as an Ornstein-Uhlenbeck process:

s= ¥ - +( ) [ ( )] ( )dr t b r t dt dZ ta

Choice D therefore describes the Vasicek Model.

Page 454: ActuarialBrew.com Exam MFE / 3F Actuarial Models …actuarialbrew.com/data/documents/MFE-Solutions-2014-1st-ed.pdf · Exam MFE / 3F Actuarial Models Financial Economics Segment Solutions

Exam MFE/3F Solutions Chapter 19 – Continuous-Time Models of Interest Rates

© ActuarialBrew.com 2014 Page 19.18

By the process of elimination, we see that Choice E must be the correct answer. But for the sake of thoroughness, let’s find the differential of Choice E as well.

The first two terms of Choice E do not contain random variables, so their differentials are easy to find. The third term will be more difficult:

( ) s

s

- - -

- - -

= + - +

È ˘= - + + Í ˙Î ˚

Ú

Ú

( )0

( )0

( ) (0) 1 ( ) ( )

( ) (0) ( ) ( )

tt t s t

tt t s t

r t r e b e e r s dZ s

dr t r e dt be dt d e r s dZ s

a a a

a a aa a

The third term has a function of t in the integral. We can pull the t-dependent portion out of the integral, so that we are finding the differential of a product. We then use the following version of the product rule to find the differential:

[ ] = +( ) ( ) ( ) ( ) ( ) ( )d U t V t dU t V t U t dV t

The differential of the third term is:

s s

s s

s s

- -

- -

-

È ˘ È ˘=Í ˙ Í ˙Î ˚ Î ˚

= - +

= - +

Ú Ú

ÚÚ

( )0 0

0

0

( ) ( ) ( ) ( )

( ) ( ) ( ) ( )

( ) ( ) ( ) ( )

t ts t t s

tt s t t

tt s

d e r s dZ s d e e r s dZ s

e dt e r s dZ s e e r t dZ t

e dt e r s dZ s r t dZ t

a a a

a a a a

a a

a

a

Putting all three terms together, we have:

0

0

0

( ) (0) ( ) ( ) ( ) ( )

(0) ( ) ( ) ( ) ( )

(0) ( ) ( ) ( ) ( )

(0)

tt t t s

tt t t s

tt t t s

t t

dr t r e dt be dt e dt e r s dZ s r t dZ t

r e be e e r s dZ s dt r t dZ t

r e b b be e e r s dZ s dt r t dZ t

r e b be

a a a a

a a a a

a a a a

a a

a a a

a +

a +

a +0

( )0

( ) ( ) ( ) ( )

(0) (1 ) ( ) ( ) ( ) ( )

tt s

tt t s t

e e r s dZ s dt bdt r t dZ t

r e b e e r s dZ s dt bdt r t dZ t

a a

a a a

a

a + a

The portion in the brackets above is equal to the expression for r(t) provided in the Choice E:

( ) ( ) ( ) ( )

( ) ( ) ( )

( ) ( ) ( )

dr t r t dt bdt r t dZ t

bdt r t dt r t dZ t

b r t dt r t dZ t

a a

a a

a

Page 455: ActuarialBrew.com Exam MFE / 3F Actuarial Models …actuarialbrew.com/data/documents/MFE-Solutions-2014-1st-ed.pdf · Exam MFE / 3F Actuarial Models Financial Economics Segment Solutions

Exam MFE/3F Solutions Chapter 19 – Continuous-Time Models of Interest Rates

© ActuarialBrew.com 2014 Page 19.19

Solution 19.26

C Delta-Gamma Approximation for Bonds

In the model described in the question, we have:

[ ] [ ]

-

-

-

=

= - = -

= =

( , )

( , )

2 2( , )

( , , ) ( , )

( , , ) ( , ) ( , ) ( , ) ( , , )

( , , ) ( , ) ( , ) ( , ) ( , , )

B t T r

B t T rr

B t T rrr

P r t T A t T e

P r t T B t T A t T e B t T P r t T

P r t T B t T A t T e B t T P r t T

The delta-gamma-theta approximation is:

[ ] [ ] [ ] [ ]+ + - ª + - + + - +2( ), , ( ), , ( ) ( ) 0.5 ( ) ( )r rr tP r t h t h T P r t t T r t h r t P r t h r t P P h

This question asks us to use only the delta and gamma portions of the approximation, so we make the following adjustments to the formula above:

The theta term, tP h , is removed from the expression above.

We replace ( )r t with 0.06.

We replace +( )r t h with 0.02.

The delta-gamma approximation is therefore:

2

2

2 2

(0.02,0,2) (0.06,0,2) 0.04 (0.06,0,2) 0.5 0.04 (0.06,0,2)

(0.02,0,2) (0.06,0,2) 0.04 (0.06,0,2) 0.5 0.04 (0.06,0,2)

(0.02,0,2) (0.06,0,2)

0.04 (0,2) (0.06,0,2) 0.5 0.04 (0,2)

Est r rr

Est r rr

Est

P P P P

P P P P

P P

B P B

2 2

(0.06,0,2)

(0.02,0,2) (0.06,0,2) 1 0.04 3 0.5 ( 0.04) 3

(0.02,0,2) (0.06,0,2) 1.1272

Est

Est

P

P P

P P

Therefore, we have:

(0.02,0,2) (0.06,0,2) 1.1272

1.1272(0.06,0,2) (0.06,0,2)

EstP PP P

Solution 19.27

D Theta in the Cox-Ingersoll-Ross Model

The process describes the Cox-Ingersoll-Ross-Model with:

s s

= ¥ - = -

= =

( ) ( ) 0.09(0.11 )

( ) 0.22

r b r r

r r r

a a

Page 456: ActuarialBrew.com Exam MFE / 3F Actuarial Models …actuarialbrew.com/data/documents/MFE-Solutions-2014-1st-ed.pdf · Exam MFE / 3F Actuarial Models Financial Economics Segment Solutions

Exam MFE/3F Solutions Chapter 19 – Continuous-Time Models of Interest Rates

© ActuarialBrew.com 2014 Page 19.20

The price of a 5-year bond is:

- - ¥= = =( , ) 3.4854 0.06( , , ) ( , ) 0.9054 0.7345B t T rP r t T A t T e e

The formula for the price of a bond must satisfy the following partial differential equation:

[ ]s s f= + + +21[ ( )] ( ) ( ) ( , )

2 rr r trP r P r r r t P Pa

Delta and gamma for the bond are:

-== - = - ¥ = -

= = ¥ =

( , )

2 2

( , , ) ( , )Delta: ( , ) ( , , ) 3.4854 0.7345 2.5602

Gamma: [ ( , )] ( , , ) (3.4854) 0.7345 8.9233

B t T r

r

rr

P r t T A t T eP B t T P r t T

P B t T P r t T

We can now solve for theta:

[ ]

( )

s s f= + + +

¥

È ˘= ¥ + - + ¥ - +Î ˚=

2

2

1[ ( )] ( ) ( ) ( , )

20.06 0.7345

10.22 0.06 8.9233 0.09(0.11 0.06) 0.22 0.06 (0.00) ( 2.5602)

20.0426

rr r t

t

t

rP r P r r r t P P

P

P

a

Solution 19.28

C Cox-Ingersoll-Ross Model

The CIR model is:

s= - +( )dr b r dt rdZa

From the partial differential equation provided in the question, we can obtain the following parameters for the CIR model:

s= = = =0.00960.12 0.08 0.10

0.12ba

We can define c in terms of f :

ff f f

s s= = fi =( , ) & ( , )

rr t r t c r c

In the CIR model, as the maturity of a zero-coupon bond approaches infinity, its yield approaches:

2 ln[ ( , , )]

Lim( ) T

b P r t Tr

T ta

a

Page 457: ActuarialBrew.com Exam MFE / 3F Actuarial Models …actuarialbrew.com/data/documents/MFE-Solutions-2014-1st-ed.pdf · Exam MFE / 3F Actuarial Models Financial Economics Segment Solutions

Exam MFE/3F Solutions Chapter 19 – Continuous-Time Models of Interest Rates

© ActuarialBrew.com 2014 Page 19.21

From the information provided in part (ii) of the question, we know that r is equal to 0.0725. Therefore:

20.0725

( )2 0.12 0.08

0.0725(0.12 )

0.0192 0.0725(0.12 )

0.26483 0.12

0.14483

ba

a

We can substitute this value into the formula for g :

g f s

f f

f f f

f f f ff

f

= - +

+ = - +

+ + = - +

+ + = - + +=

=

2 2

2 2

2 2 2

2 2

( ) 2

0.14483 (0.12 ) 2(0.1)

0.28966 0.020975 (0.12 ) 2(0.10)

0.28966 0.020975 (0.0144 0.24 ) 0.02

0.52966 0.013425

0.025347

a

The value of c is:

0.025347

0.253470.10

c

Solution 19.29

A Vasicek Model

The Vasicek model is:

s= - +( )dr b r dt dZa

From the partial differential equation provided in the question, we can obtain the following parameters for the Vasicek model:

0.03

0.30 0.10 0.120.3

b a

In the Vasicek model, as the maturity of a zero-coupon bond approaches infinity, its yield approaches:

2

2ln[ ( , , )]

0.5 LimT

P r t Tr b

T ta a

Page 458: ActuarialBrew.com Exam MFE / 3F Actuarial Models …actuarialbrew.com/data/documents/MFE-Solutions-2014-1st-ed.pdf · Exam MFE / 3F Actuarial Models Financial Economics Segment Solutions

Exam MFE/3F Solutions Chapter 19 – Continuous-Time Models of Interest Rates

© ActuarialBrew.com 2014 Page 19.22

From the information provided in the question, we know that r is equal to 0.04. Therefore:

2

2

2

2

0.5 0.04

0.12 0.120.1 0.5 0.04

0.30 0.300.05

ba a

Solution 19.30

C Risk-Neutral Version of the Vasicek Model

The process for the short rate follows the Vasicek Model. The true process can be rewritten in the familiar form:

0.08(0.10 ) 0.04 0.08, 0.10, 0.04dr r dt dZ ba

The risk-neutral process can be written as:

[ ] [ ][ ]

s f s sf s

f

= + + = - + +

= - + +

( ) ( ) ( , ) ( ) ( )

0.08(0.10 ) 0.04 0.04

dr r r r t dt r dZ b r dt dZ

r dt dZ

a a

We can set the bracketed expression above equal to the bracketed expression in (ii):

f

ff

- + = -+ - = -

=

0.08(0.10 ) 0.04 0.0128 0.080.008 0.04 0.08 0.0128 0.08

0.12

r rr r

The parameter (2,6)B is:

)

0.08(6 2)

1( , )

1(2,6) 3.4231

0.08

T teB t T

eB

a(

a

We can use the Sharpe ratio to determine a (0.04,2,6) :

af

afs

a sfa

-=

-=

= += + ¥ ¥ =

( , , )( , )

( , , )( , , )

( , )( , , ) ( , )(0.04,2,6) 0.04 3.4231 0.04 0.12 0.05643

r t T rr t

q r t Tr t T rB t T

r t T r B t T

Page 459: ActuarialBrew.com Exam MFE / 3F Actuarial Models …actuarialbrew.com/data/documents/MFE-Solutions-2014-1st-ed.pdf · Exam MFE / 3F Actuarial Models Financial Economics Segment Solutions

Exam MFE/3F Solutions Chapter 19 – Continuous-Time Models of Interest Rates

© ActuarialBrew.com 2014 Page 19.23

Solution 19.31

B Risk-Neutral Version of the Vasicek Model

From (i), we observe that:

s s

= - ¥= =

( ) ( )( ) 0.04r x y r tr

a

The risk-neutral version of the Vasicek model is:

[ ] [ ]s f s f= + + = - ¥ + + ( ) ( ) ( , ) ( ) ( ) 0.04 0.04dr r r r t dt r dZ x y r t dt dZa

Setting the above expression in brackets equal to the expression in brackets from (ii), we have:

ff

ff

- ¥ + = + - ¥+ = +

==

( ) 0.04 0.0048 ( )

0.04 0.00480.04 0.0048

0.12

x y r t x y r t

x x

The parameter (3,7)B is:

)

(7 3) 4

4

1( , )

1 1(3,7) 3.42314

T t

y

eB t T

e eB yy

a(

a

a

aa

We can use the Sharpe ratio to determine a (0.06,3,7) :

af

afs

a sfa

-=

-=

= += + ¥ ¥ =

( , , )( , )

( , , )( , , )

( , )( , , ) ( , )(0.06,3,7) 0.06 3.42314 0.04 0.12 0.07643

r t T rr t

q r t Tr t T rB t T

r t T r B t T

Solution 19.32

C Risk-Neutral Version of the Vasicek Model

The process for the short rate follows the Vasicek Model. The true process can be rewritten in the familiar form:

0.08(0.10 ) 0.04 0.08, 0.10, 0.04dr r dt dZ b a

Page 460: ActuarialBrew.com Exam MFE / 3F Actuarial Models …actuarialbrew.com/data/documents/MFE-Solutions-2014-1st-ed.pdf · Exam MFE / 3F Actuarial Models Financial Economics Segment Solutions

Exam MFE/3F Solutions Chapter 19 – Continuous-Time Models of Interest Rates

© ActuarialBrew.com 2014 Page 19.24

We can use (iii) to obtain the Sharpe ratio:

fff

f

= -

= -= -

=

( ) ( )

(4) (4) 40.02 0.22 4

0.05

Z t Z t t

Z Z

The parameter (4,8)B is:

)

(8 4) 0.08 4

1( , )

1 1(4,8) 3.42314

0.08

T teB t T

e eB

a(

a

a

a

We can use the Sharpe ratio to determine a (0.07,4,8) :

af

afs

a sfa

-=

-=

= += + ¥ ¥ =

( , , )( , )

( , , )( , , )

( , )( , , ) ( , )(0.07,4,8) 0.07 3.42314 0.04 0.05 0.0768

r t T rr t

q r t Tr t T rB t T

r t T r B t T

Solution 19.33

E Cox-Ingersoll-Ross Model

The CIR model is:

s= - +( )dr b r dt rdZa

From the partial differential equation provided in the question, we can obtain the following parameters for the CIR model:

s= = = =0.0140.14 0.10 0.12

0.14ba

We can define c in terms of f :

ff f f

s s= = fi =( , ) & ( , )

rr t r t c r c

In the CIR model, as the maturity of a zero-coupon bond approaches infinity, its yield approaches:

2 ln[ ( , , )]

Lim( ) T

b P r t Tr

T ta

a

Page 461: ActuarialBrew.com Exam MFE / 3F Actuarial Models …actuarialbrew.com/data/documents/MFE-Solutions-2014-1st-ed.pdf · Exam MFE / 3F Actuarial Models Financial Economics Segment Solutions

Exam MFE/3F Solutions Chapter 19 – Continuous-Time Models of Interest Rates

© ActuarialBrew.com 2014 Page 19.25

From the information provided in part (ii) of the question, we know that the rightmost portion is equal to 0.14. Therefore:

20.14

( )2 0.14 0.10

0.14(0.14 )

0.028 0.14(0.14 )

0.2 0.14

0.06

b

a

a

We can substitute this value into the formula for g :

g f s

f f

f f f

f f f ff

f

= - +

+ = - +

+ + = - +

+ + = - + +=

=

2 2

2 2

2 2 2

2 2

( ) 2

0.06 (0.14 ) 2(0.12)

0.12 0.0036 (0.14 ) 2(0.12)

0.12 0.0036 (0.0196 0.28 ) 0.0288

0.4 0.0448

0.112

a

The value of c is:

0.112

0.93330.12

c

Solution 19.34

C Vasicek Model

The Vasicek model allows negative yields.

The Vasicek model is:

s= - +( )dr b r dt dZa

From the partial differential equation provided in the question, we can obtain the following parameters for the Vasicek model:

0.01

0.20 0.05 0.140.2

b a

In the Vasicek model, as the maturity of a zero-coupon bond approaches infinity, its yield approaches:

2

2ln[ ( , , )]

0.5 LimT

P r t Tr b

T ta a

Page 462: ActuarialBrew.com Exam MFE / 3F Actuarial Models …actuarialbrew.com/data/documents/MFE-Solutions-2014-1st-ed.pdf · Exam MFE / 3F Actuarial Models Financial Economics Segment Solutions

Exam MFE/3F Solutions Chapter 19 – Continuous-Time Models of Interest Rates

© ActuarialBrew.com 2014 Page 19.26

From the information provided in the question, we know that r is equal to –0.083. Therefore:

2

2

2

2

0.5 0.083

0.14 0.140.05 0.5 0.083

0.20 0.200.16

b

a a

Solution 19.35

A Rendleman-Bartter Model

In the Rendleman-Bartter Model, the short-term rate follows geometric Brownian motion. From Chapter 5 of the ActuarialBrew.com Study Manual, the payoff of a call option can be found as:

[ ] a d- -= - ¥( )( )1 2ˆ ˆCall Payoff ( ) ( )T t

tE S e N d K N d

We can consider the derivative to be 1,000 call options with the short rate replacing the stock price and 0.10 replacing the strike price. Although we have -0.25 for the volatility parameter in the geometric Brownian motion, we use the absolute value of -0.25 in the

formulas for 1d and 2d , because the s in the formulas is the standard deviation of the

stock’s return, and it must therefore be positive:

2 2

1

2 2

2

( ) 0.09ln ( 0.5 )( ) ln (0.10 0.5 0.25 ) 0.25

0.10ˆ 0.580380.25 0.25

( ) 0.09ln ( 0.5 )( ) ln (0.10 0.5 0.25 ) 0.25

0.10ˆ 0.705380.25 0.25

r tT t

Kd

T tr t

T tK

dT t

a d s

s

a d s

s

Ê ˆ Ê ˆ+ - + - + + ¥ ¥Á ˜ Á ˜Ë ¯ Ë ¯= = = -

-Ê ˆ Ê ˆ+ - - - + - ¥ ¥Á ˜ Á ˜Ë ¯ Ë ¯

= = = --

The values of 1( )N d and 2ˆ( )N d are:

1

2

ˆ( ) ( 0.58038) 0.28083ˆ( ) ( 0.70538) 0.24029

N d N

N d N

= - =

= - =

The expected value of the payoff is:

[ ] ( )( )

1 2

0.10(0.25)

ˆ ˆPayoff 1,000 ( ) ( )

1,000 0.09 0.28083 0.10 0.24029 1.8855

T ttE S e N d K N d

e

a d- -È ˘= ¥ - ¥Î ˚È ˘= ¥ ¥ - ¥ =Î ˚

Page 463: ActuarialBrew.com Exam MFE / 3F Actuarial Models …actuarialbrew.com/data/documents/MFE-Solutions-2014-1st-ed.pdf · Exam MFE / 3F Actuarial Models Financial Economics Segment Solutions

Exam MFE/3F Solutions Chapter 19 – Continuous-Time Models of Interest Rates

© ActuarialBrew.com 2014 Page 19.27

Solution 19.36

D Cox-Ingersoll-Ross Model

In the CIR Model, we have:

-= ( , )( , , ) ( , ) B t T rP r t T A t T e

In the CIR model, we have:

= -( , ) (0, )A t T A T t and = -( , ) (0, )B t T B T t .

This implies that we can define A and B as shown below:

= = == = =

(0,8) (1,9) (3,11)(0,8) (1,9) (3,11)

A A A AB B B B

We can obtain the ratio of the second price to the first price:

-

--

¸= = Ô fi =˝= = Ô

0.251.25

( ,0,8) 0.4005 0.3581

0.4005(1.25 ,1,9) 0.3581

BrBr

Br

P r Aee

P r Ae

The ratio obtained above can be used to find the bond price needed to answer the question:

1.5 1.25 0.25 0.3581(1.5 ,3,11) 0.3581 0.3202

0.4005Br Br BrP r Ae Ae e- - -= = ¥ = ¥ =

Page 464: ActuarialBrew.com Exam MFE / 3F Actuarial Models …actuarialbrew.com/data/documents/MFE-Solutions-2014-1st-ed.pdf · Exam MFE / 3F Actuarial Models Financial Economics Segment Solutions

Exam MFE/3F Solutions Chapter 19 – Continuous-Time Models of Interest Rates

© ActuarialBrew.com 2014 Page 19.28

This page is intentionally blank.